Está en la página 1de 204

1.

Cuál de las siguientes es una causa de


cianosis central?. Señale la respuesta 6) Un paciente que presenta HBsAg +, Anti
correcta: HBs negativo, Anti HBc IgM, HBeAg + y
a-Menor gasto cardiaco sin fallo retrógrado. Anti HBe negativo. ¿Cuál es la
b-Hipoventilación alveolar con bradipnea. interpretación?.
c-Exposición al frío con vasoconstricción en Señale la respuesta correcta:
dermis. a- crónica de alta infectividad
d- Fenómeno de Raynaud secundario b- aguda de alta infectividad
2) ¿Cuál de los siguientes medicamentos c- crónica de baja infectividad
es el de mayor efectividad en vómitos y d- aguda de baja infectividad
náuseas inducidas por quimioterapia,
siendo su mecanismo de acción el 7) ¿Cuál de los siguientes son factores
antagonismo de los receptores 5 – predisponentes para los cálculos de
HT3?. colesterol?:
Señale la respuesta correcta: a- Fibrosis quística
a- Dimenhidrinato. b- Ayuno
b- Escopolamina. c- Hemólisis crónica
c- Ondasentron d- Cirrosis alcohólica
d- Domperidona.
8) ¿Cuál de las siguientes es causa de
3) ¿Cuál de las siguientes anemia con un índice de producción de
es causa de reticulocitos mayor o igual a 2,5?
hipopigmentación?. Señale la a- Talasemia
respuesta correcta: b- Nefropatía
a- Pelagra. c-Efectos tóxicos de fármacos
b- Tiña versicolor d-Hemólisis intravascular
c- Deficiencia de vitamina B12.
d- Queratosis seborreica, 9) ¿Cuál de los siguientes tumores se
asocia a eritrocitosis como síndrome
4) En un paciente de 30 años con paraneoplásico?
diagnóstico de neumonía a- Cáncer de pulmón
extrahospitalaria, que se encontraba b- Hepatocarcinoma
sano previamente y no había recibido c- Cáncer de próstata
antibióticoterapia en los últimos 90 d- Cáncer de mama
días. ¿Qué antibiótico indica
inicialmente ?. Señale la respuesta 10) ¿Cuál de los siguientes es un criterio
correcta: mayor en la endocarditis infecciosa?
a- Trimetroprima sulfametoxazol EV a- Fiebre mayor a 38°
b- Ceftriaxona endovenoso (EV) b- Nueva insuficiencia valvular
c-Claritromicina vía oral. c- Embolia arterial recidivante
d-Piperacilina tazobactan (EV) d- Cardiopatía predisponente

5)¿Cuál de los siguientes es un criterio 11) ¿Cuál es el tratamiento para una


diagnóstico del colon irritable?. Señale gangrena gaseosa?. Señale la respuesta
la respuesta correcta: correcta:
a- El dolor abdominal mejora con la a- Nafcilina u oxacilina 2gr ev cada 4 a 6 hs
defecación b- Clindamicina 660 a 900mg ev cada 6 a
b- La presencia de dolor abdominal sin 8/hs más penicilina G 4 millones de U
relación con la frecuencia de ev cada 4-6hs
evacuaciones c- Aciclovir 800 mg PO 5 veces al día
c- Presencia de dolor abdominal sin durante siete a diez días más
relación con la frecuencia de las eritromicina 500 mg PO cada 6 hs.
evacuaciones y con los cambios en la d- Vancomicina 1 gr ev cada 12hs
forma de las heces
d- Presencia de dolor abdominal, dos 12) ¿Cuál de las siguientes es una
veces por día en los últimos seis meses. característica del vértigo laberíntico?.
Señale la respuesta correcta:
a- Nistagmo horizontal de complejos inmunitarios patógenos?.
b- Acúfenos o hipoacusia Señale la respuesta correcta:
c- Romberg negativo a- Poliangeítis microscópica
d- Nistagmo vertical b- Arteritis de células gigantes
c- Crioglobulinemia mixta, secundaria a
13) ¿Cuál de las siguientes patologías se hepatitis C
asocia con enfermedad intersticial d- Síndrome de Churg Strauss
difusa en la radiografía de tórax?.
Señale la respuesta correcta: 18) ¿Cuál de las siguientes es una
a- Absceso bacteriano característica de la hiponatremia
b- Infarto pulmonar hipervolémica?. Señale la respuesta
c- Fibrosis pulmonar idiopática correcta:
d- Malformación vascular a- Deficiencia de ACTH
b- Edema generalizado y ascitis
14) En un paciente con riesgo alto de c- Sodio urinario alto
síndrome de lisis tumoral, ¿Cuándo d- Ácido úrico bajo
tiene indicación de comenzar la
hemodiálisis?. Señale la respuesta 19) ¿Cuál de las siguientes es causa de
correcta: hipertiroidismo secundario?. Señale la
a- Potasio sérico mayor a 4,8 meq/l respuesta correcta:
b- Ácido úrico sérico mayor a 10 a- Enfermedad de Graves
mg/100ml b- Adenoma tóxico
c- Creatinina sérica de 5 mg/dl c- Adenoma hipofisario secretor de TSH
d- Fosfato sérico mayor a 6 mg/100ml d- Ingesta excesiva de hormona tiroidea

15) Un paciente de 16 años de edad con 20) ¿En cuál de las siguientes patologías
antecedente reciente de infección pulmonares se observan granulomas
gastrointestinal por E.coli, consulta por de células epitelioides semejantes a
fiebre, cefalea intensa, y en el sarcoides no caseificantes?
laboratorio se constatan: anemia, a- La silicosis
trombocitopenia y aumento de LDH. En b- La asbestosis
el frotis de sangre periférica c- La neumoconiosis de los trabajadores
esquistocitos. Los estudios de orina de las minas de carbón
revelan hematuria y leve proteinuria. d- La beriliosis crónica
¿Cuál es el diagnóstico presuntivo más
probable? 21) Dentro de las manifestaciones
a- Necrosis tubular aguda isquémica musculoesqueléticas de la artritis
secundaria a las enterotoxinas. inducida por depósito de cristales,
b- Síndrome urémico ¿Cuál de las siguientes afecciones
hemolítico/trombótico considera la correcta?.
trombocitopénico
c- Pielonefritis bilateral aguda con a-Síndrome del túnel carpiano
bicitopenia autoinmune. b-Epicondilitis
d- Nefritis intersticial alérgica y c- Epitrocleítis
enfermedad tubular con bicitopenia . d- Fascitis

16) ¿Cuál de las siguientes es causa de 22) En una paciente con probable
enfermedad tubulointersticial aguda?. prolactinoma y elevación de la
Señale la respuesta correcta: prolactina, ¿Cuál de los siguientes
a- Nefropatía por células de mieloma. estudios solicita?. Señale la respuesta
b- Nefritis por radiación correcta:
c- Nefritis por citomegalovirus a-Mamografía
d- Reflujo vesicoureteral b-Resonancia magnética nuclear de silla
turca
17) ¿Cuál de los siguientes síndromes c- Tomografía computada con contraste de
vasculíticos es producido por depósito ambas carótidas
d- Angio TC de circulación vertebral
enfermedad cerebrovascular
23) Paciente de 25 años de edad, que 15 isquémica?:
días antes presentó una infección a- Glucemia de 130 mg%.
respiratoria alta, continuando en la b- Tensión arterial diastólica > 110 mmHg.
actualidad con tos seca. Es llevada a la c- Plaquetas < 130.000/mm3.
guardia con cuadro de desorientación y d- Cirugía Mayor en el último año.
fiebre. Se interna para optimizar
diagnóstico y terapéutica. Se constata 29) ¿Cuál de las siguientes es indicación de
que en 24 hs tiene una diuresis de 7 l/d. tratamiento con insulina en un
acompañada de polidipsia. paciente diabético tipo 2?:
Osmolaridad urinaria de 200 mosmol/l. a- Aquellos pacientes con normopeso
Restringiendo la vía oral, la paciente b- Pacientes con glucemia en ayunas de
continúa poliúrica. 200mg%
¿Cuál de los siguientes diagnósticos es el c- Diabético tipo 2 que fracasa al
más probable?: tratamiento con dosis máximas de
a- Polidipsia primaria drogas orales y se mantiene
b- Diabetes insípida central hiperglucémico.
c- Diabetes insípida central secundaria a d- Paciente con infección urinaria.
encefalitis viral
d- Diabetes insípida nefrogénica 30) Ante la sospecha de estenosis
secundaria a necrosis tubular. hipertrófica de píloro, ¿cómo iniciaría
los exámenes complementarios?
24) ¿Cuál es la deformidad valvular a- Transito digestivo
observada más frecuentemente en la b- Radiología simple
cardiopatía reumática crónica? c- Ecografía.
a- La estenosis mitral d- pHmetría gástrica.
b- La estenosis tricúspidea aislada
c- La insuficiencia mitral aislada 31) ¿Cuál de las siguientes cefalosporinas
d- La asociación de estenosis mitral con no requiere ajuste de dosis con Filtrado
estenosis aórtica Glomerular entre 25-10 ml/min?
a- Ceftazidima
25) ¿Cuál de las siguientes es causa de b- Cefepime
acortamiento del intervalo QT en el c-Ceftriaxona
electrocardiograma? d-Cefuroxima
a- Hipocalcemia
b- Hipopotasemia 32) ¿Cuál de los siguientes fármacos
c- Hipercalcemia antirretrovirales produce depresión de
d- Hiponatremia la Medula Ósea (medulotóxico)?
a- Efavirenz
26) ¿Cuál de las siguientes es causa de b- Raltegravir
insuficiencia cardiaca con fracción de c -Atazanavir
eyección mayor a 50%?. Señale la d- Zidovudina
respuesta correcta:
a- Obstruccion moderada de la 33) ¿Cuál de los siguientes no se incluye
descendente posterior como un grupo de riesgo para recibir la
b- Miocardiopatía hipertrófica vacuna antineumocócica?
c- Miocardiopatía dilatada no isquémica a- EPOC
d- Bradiarritmias crónicas b- Fisura etmoidal con fístula de LCR y
27) La causa más frecuente de vértigo es: rinorraquia
a- Síndrome de Menière. c- Enfermedad de Whipple
b- Síndrome de Wallenberg. d- Asplenia funcional o anatómica
c- Neuronitis vestibular.
d- Vértigo posicional benigno. 34) Paciente de 53 años con antecedente
de reemplazo de válvula aórtica hace 3
28) ¿Cuál de los siguientes, es considerado años. Consulta por síndrome febril de
Criterio de Exclusión de infusión de dos semanas de evolución asociado a
fibrinolíticos en pacientes con astenia. Refiere un episodio de
hematuria hace 5 días que cedió 39) En un cáncer de recto localizado a 9 cm
espontáneamente. Al examen físico del margen anal y que abarca 1/3 de su
presenta buen estado general, circunferencia,
febrícula (T° axilar: 37.4°), hemorragias ¿cuál es la intervención quirúrgica de
en astilla en pulgar derecho. Señale la elección?
conducta más adecuada: a- Proctocolectomía izquierda
a- Descarta diagnóstico de endocarditis b- Amputación abdominoperineal u
por ser un paciente de bajo riesgo. operación de Miles
b- Realiza hemocultivos, de ser negativos c- Resección local endoanal
descarta el diagnóstico de endocarditis. d- Resección anterior u operación de Dixon
c- Realiza hemocultivos y ecocardiograma
transtorácico. De ser negativo realiza 40) En un paciente de 70 años de edad,
ecocardiograma transesofágico. ¿cuál de las siguientes causas es la más
d- Realiza hemocultivos y ecocardiograma frecuente de isquemia colónica aguda?
transtorácico. De ser negativo descarta a- Embolia arterial
el diagnóstico de endocarditis b- Trombosis arterial
35) ¿Cuál es el signo o síntoma más c- Vólvulo sigmoide
frecuente en un carcinoma papilar de d- Episodio de bajo volumen minuto
mama? cardíaco
a- Nódulo palpable subareolar
b- Derrame sanguinolento por el pezón 41) ¿Cuál es el tumor más frecuente del
c- Adenopatía axilar y nódulo mamario mediastino anterior?
reconocido semiológicamente a- Timoma.
d- Retracción del pezón b- Quiste broncogénico. c- Quiste
esofágico.
36) ¿En qué casos está indicada una biopsia d- Tumor neurogénico.
pleural con aguja?
a- Paciente con caverna tuberculosa que 42) Paciente con trauma torácico
desarrolla un derrame pleural penetrante en cara lateral del
b- Paciente con un nódulo pulmonar hiliar hemitórax derecho. Signos clínicos de
y atelectasia segmentaria del lóbulo derrame pleural masivo y radiografía
superior de tórax con velamiento completo y
c- Paciente que desarrolla un derrame descompensación hemodinámica.
pleural de origen desconocido ¿Qué conducta adoptaría?
d- Recidiva de un derrame pleural en un a- Arteriografía de vasos centrales
paciente tratado por cáncer de pulmón. b- Tomografía computada
c-Toracocentesis
37) ¿Cuál es la localización más frecuente, d-Toracotomía exploradora inmediata
dentro de las enunciadas, de una
angiodisplasia? 43) En un herido de bala cuyo trayecto
a- Colon descendente perforó el mediastino de lado a lado, y
b- Colon ascendente la Rx. presenta neumotórax y
c- Colon sigmoide neumomediastino leve: ¿Qué estudio
d- Recto diagnóstico indica como
38) En una hemorragia digestiva baja, imprescindible?
intermitente, con pérdida de sangre a
razón de 0.1 ml/ min, ¿cuál de los a- Broncofibroscopia para confirmar
siguientes métodos diagnósticos tiene perforación de tráquea y/o grandes
mayor sensibilidad? bronquios.
a- Arteriografía selectiva b- TAC de tórax.
b- Colonoscopía c- Punción pleural. d- Esofagograma.
c-Glóbulos rojos marcados con 99mTc 44) ¿Las hernias inguinales de tipo directas,
d-Centellografía con sulfuro de tecnesio se presentan por?
coloidal, marcado con 99mTC a- Dilatación del orificio inguinal
profundo.
b- Persistencia del conducto peritoneo
vaginal.
c- Debilidad de la pared posterior. b- Artritis Séptica bilateral.
d- Dilatación del orificio del orificio c- Esclerodermia.
inguinal superficial. d- Lupus Eritematoso Sistémico.

45) Un paciente con cefaleas, náuseas, 51) En una paciente de 80 años que se
diplopía y edema de papila se presenta en la guardia con dolor,
encuentra en: rotación externa y acortamiento del
a- La etapa descompensada del miembro inferior, Ud. sospecha una:
síndrome de a- Fractura de cadera.
hipertensión intracraneano b- Luxación de cadera.
b- La etapa final del síndrome de c- Fractura de fémur.
hipertensión intracraneano d- Luxación de rodilla.
c- La etapa compensada del
síndrome de 52) Un joven de 22 años es llevado a la
hipertensión intracraneano guardia. Sus padres informan que
d- La etapa de las hernias encefálicas desde hace varios meses se ha ido
aislando de su entorno social, y hace
46) Los tumores intracraneales intraxiales: unas semanas tuvo un enfrentamiento
a- Son extracerebrales en su trabajo. En la entrevista se
b- Se separan fácilmente del parénquima muestra poco colaborador pero
cerebral orientado en tiempo y espacio. Durante
c- Suelen ser gliomas el interrogatorio confiesa que sus
d- Suelen ser meningiomas compañeros están conspirando con su
jefe para expulsarlo de su puesto
47) ¿En qué tipo de hidrocefalia está laboral. No se infiere riesgo inminente
contraindicada la punción lumbar? para sí o para terceros. Antecedentes
a- En el pseudotumor cerebral de importancia: los padres refieren que
b- En la hidrocefalia comunicante de chico era solitario, y su rendimiento
c- En la hidrocefalia no comunicante escolar fue bajo. Tuvo algunas novias
d- En la hidrocefalia normotensiva con las que se peleaba por celos.
¿Cuál es el diagnóstico más probable y la
48) Durante el ayuno, el acetil-CoA conducta más adecuada?
producido en la oxidación de ácidos a- Trastorno Bipolar. Interconsulta con
grasos es utilizado para la síntesis de: psiquiatra de guardia para iniciar
tratamiento
a- Glucosa y glucógeno en hígado b- Trastorno de personalidad paranoide.
b- Acetoacetato e hidroxibutirato en Derivación a psiquiatría para
tejido hepático seguimiento por consultorios externos.
c- Piruvato y glucosa en hígado c- Esquizofrenia paranoide. Interconsulta
d- Hidroximetilglutaril CoA y colesterol en con psiquiatra de guardia para iniciar
tejidos extrahepáticos tratamiento
d- Trastorno delirante de tipo
49) En un paciente con maniobras de persecutorio. Derivación a psiquiatría
Barlow y Ortolani positivas Ud. para seguimiento por consultorios
sospecha: externos
a- Epifisiolisis de cadera.
b- Necrosis ósea avascular. 53) Una mujer de 70 años, ama de casa,
c- Coxa vara. acude a la consulta acompañada por su
d- Displasia congénita de cadera. esposo que informa que hace unos
meses se ha vuelto retraída, apática e
50) En una paciente de 45 años de edad irritable. Presenta dificultad para
que presenta sinovitis en ambas acordarse de cosas que habían
muñecas, evidenciándose en las sucedido el día anterior y problemas
radiografías disminución del espacio para calcular el dinero al ir de compras.
articular y osteopenia subcondral, A la evaluación está desorientada en
¿Qué patología sospecha? tiempo y espacio, desprolija en su
a- Artritis Reumatoidea. vestimenta, lúcida pero notoriamente
ansiosa y con dificultad para encontrar c- Mononucleosis infecciosa.
las palabras para expresarse. No se d- Toxoplasmosis.
detectaron alteraciones en el examen
físico. Todos los análisis de laboratorio 57) En un lactante de 6 meses con una
dieron negativos, pero una tomografía tumoración intrarrenal
computada mostró una marcada macroscópicamente sólida y
atrofia cortical y un Mini Mental Test de amarillenta, constituida
Folstein arrojó un puntaje de 19. ¿Cuál histológicamente por células
es el diagnóstico más probable y la fusiformes, acompañadas por islotes de
conducta más adecuada? cartílago y focos de hemopoyesis. ¿Cuál
a- Deterioro cognitivo normal para la es el diagnóstico?
edad, seguimiento y reevaluación a-Tumor rabdoide.
periódica para monitorear la aparición b-Sarcoma de células claras.
de algún síntoma de enfermedad c- Tumor de Wilms.
b- Síndrome confusional agudo, d- Nefroma mesoblástico
tratamiento de su causa. Generar un
ambiente familiar para el paciente. 58) ¿Qué virus se encuentra involucrado en
c- Síndrome demencial (compatible con etiología de la papilomatosis laríngea
enfermedad de Alzheimer), inicio de juvenil?
tratamiento de los síntomas cognitivos a- HPV 11 y 6.
y no cognitivos. b-Virus de Epstein Barr.
d- Cuadro depresivo, inicio de tratamiento c- Herpes Virus 11.
con fármacos antidepresivos. d- Virus Sincicial Respiratorio.
54) ¿Cuál de las siguientes opciones es una
complicación posible ante una 59) En un niño de diez años que presenta a
infección por virus de la hepatitis A? nivel de miembro superior derecho
a- Evolución a la cronicidad. pápulas de hasta 3 mm planas de color
b- Evolución a la cirrosis. carne agrupadas e histológicamente
c-Falla hepática fulminante. constituidas por infiltrado compuesto
d-Hepatocarcinoma. por histiocitos y células gigantes
aisladas, circunscripto a una papila
55) ¿Qué diagnóstico hace en un niño de dérmica. ¿Cuál es el diagnóstico más
tres años de edad que comienza con probable?
dolor en sus tobillos al cual se agrega a- Liquen nítido.
dolor abdominal y un exantema b- Liquen plano.
purpúrico en cintura y miembros c- Xantogranuloma juvenil.
inferiores?. En los exámenes d- Pitiriasis liquenoide aguda
complementarios se observa
proteinuria, hematuria, y en la biopsia 60) En una biopsia de piel que muestra una
renal se muestra proliferación lesión vesiculoampollar intracórnea o
mesangial y depósitos de Ig A por subcórnea con escasas células
inmunofluorescencia. inflamatorias, ¿qué diagnóstico
a-Púrpura de Schonlein Henoch. realizaría?
b-Granulomatosis de Wegener. a- Impétigo.
c- Displasia renal. b- Eritema tóxico neonatal.
d- Sindrome de Alport. c- Sindrome de la piel escaldada
estafilocóccica.
56) En un niño de tres años con síndrome d- Enfermedad de Hailey.
febril de etiología desconocida,
linfadenopatía cervical, rash cutáneo 61) ¿Cuál de las siguientes dermatosis
eritematoso e inyección conjuntival, nutricionales se caracteriza por la
cuya biopsia ganglionar evidenció malabsorción de Zinc?
infartos en parches asociados a a- Acrodermatitis enteropática.
trombos de fibrina en pequeños vasos b- Pelagra.
¿Qué diagnóstico sospecha? c- Enfermedad de Hartnup.
a- Lupus eritematoso diseminado. d- Frinodermia.
b- Enfermedad de Kawasaki.
62) ¿Cuál es la complicación digestiva más consumiendo suplementos vitamínicos
frecuente en un Recién Nacido diariamente durante los últimos siete
pretérmino de bajo peso al nacer, que meses, como parte de un programa de
presenta letargia y manifestaciones de entrenamiento físico intensivo. ¿Al
acidosis metabólica? exceso de qué vitaminas se pueden
a- Atrofia vellositaria. atribuir estos hallazgos?
b- Vólvulo intestinal. a- A
c-Enterocolitis necrotizante. b- C
d-Reflujo gastroesofágico. c- D
d- B1
63) Un niño de 2 años, quien es portador de
anemia falciforme presenta 67) El Screening para Streptococo beta
tumefacción simétrica de las manos. El agalactiae
diagnóstico más probable es: a- Se realiza en forma universal entre las 35-
a- Osteomielitis 36 semanas mediante un hisopado
b- Artritis reumatoide pauciarticular vaginal y perianal.
c- Síndrome mano pie (Dactilitis) b- Se realiza en forma universal entre las 35-
d- Artritis juvenil 36 semanas mediante un hisopado de
la cavidad oral
64) Usted asiste a un niño de 6 años por c- Se utiliza para prevenir una infección en
presentar desde hace 24 horas un la madre.
cuadro caracterizado por dolor d- Se efectúa mediante un hisopado de la
periumbilical. Posteriormente se cavidad oral, para iniciar antibiótico
agregan náuseas y anorexia y el dolor, terapia antes del trabajo de parto.
que se ha reagudizado, se localiza
ahora en el cuadrante inferior derecho. 68) La maduración pulmonar se realiza:
Al examen físico el abdomen se halla a- Entre la semana 24 y 34
blando, depresible y doloroso en dicho b- Después de las 34 semanas
cuadrante, con dolor a la c- Antes de las 24 semanas
descompresión y ruidos hidroaéreos d- Entre las 22 y 24 semanas
presentes. El diagnóstico más probable
es: 69) El diagnostico de enfermedad de
a- Apendicitis aguda Chagas
b- Estreñimiento a- Se realiza mediante la dupla serológica
c- Gastroenteritis vírica a saber HAI/IFI O HAI/ELISA
d- Infección urinaria b- Se realiza mediante la serología por el
ELISA
65) Usted recibe un neonato con ictericia, c- Se realiza por la presencia del signo de
dificultad para alimentarse, romania
hipoglucemia y cataratas. El d- Se realiza por la presencia se
diagnóstico más probable es: manifestaciones cardiacas en pacientes
a- Deficit de galactosinasa infectadas.
b- Galactosemia
c- Deficit de piruvato dehidrogenasa 70) En la Enfermedad hemolítica perinatal,
d- Enfermedad de Hurler la inmuno profilaxis se realiza en
puérperas:
66) Un joven de 16 años llega a la consulta a- RH negativa que han tenido un recién
porque presenta vómitos, anorexia, nacido RH positivo
astenia, constipación y poliuria de b- RH negativa que han tenido un recién
instalación progresiva en las últimas nacido RH negativo
semanas. En los exámenes de c- RH negativa que han tenido un recién
laboratorio solicitados se destacan: nacido RH positivo con prueba de
calcio sérico 13 mg/dl; concentraciones Coombs directa positiva.
elevadas de 25-D < a 150ng/ml) e d- RH negativa con recién nacido RH
hiperfosfatemia. Una ecografía renal positivo y prueba de Coombs directa
muestra signos de hipercalcinosis. negativa.
Surge del interrogatorio que ha estado
71) Señale la opción correcta en el manejo b-Antes de la relación sexual y luego se lo
del hipertiroidismo durante el retira c- Durante 12 días, debiéndose
embarazo: reinsertar luego un nuevo anillo.
a- Seguimiento de concentración de T4 d- Durante 7 días, debiéndose reinsertar
cada 2 a 4 semanas. luego un nuevo anillo.
b- Realización de ecografía durante el
primer trimestre del embarazo para 77) Indique la opción correcta respecto de
evaluar respuesta del tratamiento. la mamografía:
c- Se considera rango normal de T4 tres a-Se debe realizar en el control del segundo
veces por encima del valor de T4 de la trimestre del embarazo.
no embarazada. b-Sirve para diferenciar imágenes sólidas de
d- Dosaje de TSH cada 2 semanas. quísticas.
c- Es de gran utilidad en las pacientes más
72) Si se confirma sífilis en una embarazada jóvenes.
por las pruebas serológicas pero nunca d- Se debe realizar entre los 35 y 40 años de
se evidenció la clínica de la enfermedad edad una mamografía de base.
y es de evolución desconocida
¿Cuántas dosis Ud. Le indicaría de Penicilina 78)Las fístulas rectovaginales, son las más
Benzatinica? frecuentes de este tipo de fístulas e
a- Una dosis intervienen varios factores.
b- Dos dosis ¿Cuál es la correcta?
c- Tres dosis a- Las de origen actínico en general son
d- Ninguna pues debe ser un falso positivo. tardías por necrosis del tabique
rectovaginal.
73) ¿Cuál es el tratamiento de elección b- Las de causa obstétrica se deben a una
para la prevención de recurrencia de lesión quirúrgica.
convulsiones para una paciente que c- Son sumamente raras y son casi
presenta Eclampsia? siempre de naturaleza traumática.
a- Fenitoina d- Su localización en general es alta en la
b- Coctel lítico vagina o en la cúpula vaginal.
c- Sulfato de Magnesio
d- Diazepam 79) Se considera que el tratamiento inicial
de una paciente con diagnóstico de
74) El sangrado masivo en Obstetricia se cáncer de mama es quirúrgico cuando:
define como: a-No tiene metástasis a distancia
a-Descenso del hematocrito más de 10 b-Tiene compromiso de piel
puntos. c- Tiene úlcera y eritema
b-Descenso de la hemoglobina mayor a 8 d- Posee infiltración cutánea
gramos/dl. c- Pérdida brusca de 500 ml.
d- Pérdida de más del 30% de la volemia 80) La Neoplasia intraepitelial vaginal
(VAIN) se caracteriza por cambios
75) ¿Cual de los siguientes gestágenos, progresivos intraepiteleales.
usados en anticonceptivos orales, tiene ¿Cuál considera correcta?.
menor efecto sobre el metabolismo a- Presenta una incidencia de 0.2 al 0.3
lipídico? por 100000 mujeres
a) Medroxiprogesterona b- Representa el 1% de las neoplasias
b) Desogestrel intraepiteliales del tracto genital
c) Clomifeno inferior
d) Noretisterona c- El 30% se encuentra asociado con otras
neoplasias intraepiteliales
76) Como nuevas tecnologías se encuentra d- El 40% se relaciona con el papiloma
el anillo vaginal en anticoncepción. virus humano (HPV)
Considere la correcta: Este anillo se
coloca: 81) Señale la opción correcta con respecto
a-Durante 21 días y se retira una semana. a la sífilis:
a- El periodo de incubación es de 9 a 120
días (promedio 31 días)
b- El periodo de incubación es de 9 a 60 c- Se caracteriza por presentar FSH
días (promedio 21 días) basales elevadas (mayor a 15 UI/ML) o
c- La vía de contagio es exclusivamente estradiol superior a 75 pg/ml.
sexual d- Hay una producción inadecuada de
d- La sífilis latente aquella que se progesterona por parte del cuerpo
manifiesta por reacciones serológicas y amarillo, posiblemente como
por la prueba de campo oscuro consecuencia directa de una génesis
folicular anormal
82) El CA 19.9 es un marcador tumoral que
se encuentra generalmente elevado 86) Los estrógenos son hormonas capaces
en: de producir y mantener las
a- Carcinoma seroso del ovario. características femeninas en la mujer.
b- Carcinoma de endometrio. Su administración produce:
c-En cáncer digestivo y en menor medida en a- Aumento de HDL y LDL colesterol
mama b- Aumento de HDL y disminución del LDL
d-Tumores de las células germinales. colesterol
c- Disminución del HDL y aumento del LDL
83) El cáncer de vagina es un tumor poco colesterol
frecuente, con una incidencia del 2% de d- Disminución de los triglicéridos y del
los tumores ginecológicos. Señale la HDL colesterol
correcta:
a- Epidemiológicamente son tumores que 87) Las vías genitales se forman en la mujer
afectan a una edad entre 25 y 55 años. a partir de los conductos de Muller.
b- Estadio I, carcinoma limitado a la pared Considere la más correcta:
vaginal. a- Entre la 6º y 9º semana transcurre la
c- Estadio III, carcinoma que compromete el etapa de la formación tubárica
tejido subvaginal y no se extiende a la exclusivamente.
pared pelviana. b- Entre la 6º y 9º semana transcurre la
d- El más frecuente es el adeno carcinoma etapa de la formación tubárica, uterina
de células claras. y cérvico vaginal.
c- En la 8º semana se unen los conductos
84) Los tumores trofoblásticos de Muller
gestacionales son raros en el mundo d- En la 5º semana transcurre la etapa de
occidental, pero altamente curables. la formación tubárica.
Elija la opción correcta.
El Corion carcinoma es la forma histológica: 88) Se define a la endometriosis (EDT)
a- Maligna más frecuente, es muy como la presencia de glándulas
quimiosensible. endometriales y estroma funcionante.
b- Maligna menos frecuente y quimio- Considere la correcta:
resistente. a- Las células T citotóxicas y las natural
c- Maligna más frecuente y es quimio- killer (NK, asesinas) actúan como
resistente. segunda línea de defensa del sistema
d- Más rara. inmune.
b- Las mujeres con EDT tienen una
85) La falla Ovárica Oculta afecta a un alteración en la actividad de los
grupo de pacientes con insuficiente linfocitos B y anticuerpos anormales.
dotación folicular ovárica, en presencia c- Las células T citotóxicas no están
de ciclos menstruales regulares. ¿Cuál involucradas en la histólisis directa de
es la correcta?: las células foráneas.
a- Se caracteriza por presentar FSH d- La reabsorción del tejido endometrial
basales elevadas (mayor a 20 UI/ML) o deportado por el flujo retrógrado no
estradiol superior a 75 pg/ml. esta a cargo del sistema inmune.
b- Se caracteriza por presentar FSH
basales elevadas (mayor a 10 UI/ML) o 89) ¿Cuál es el tipo de defecto atribuible a
estradiol de 30 pg/ml. glaucoma que se encuentra en el
campo visual?
a- Cuadrantanopsia.
b- Escotoma arcuato. b-Unicamente del mesodermo lateral
c- Escotoma central. somático.
d- Cuadrantopsia. c- Del mesodermo lateral somático y
septum transverso.
90) ¿Cuál es la causa de muerte en un d- Del mesodermo lateral, crestas neurales
cuadro de picadura por escorpión? y mesénquima extracardiaco del
a- Deshidratación severa septum transverso.
b- Insuficiencia hepática
c- Insuficiencia renal 96) ¿Cómo será la descendencia de una
d- Shock cardiogénico mujer afectada de enfermedad
fibroquística del páncreas (autosómica
91) Si en el electrocardiograma de un recesiva) con un varón sano no
paciente se observa un QRS isodifasico portador?:
en D1, el eje cardiaco en el plano a- El 100% de sus hijas mujeres serán
frontal se encuentra en: portadoras
a- 0° b- El 100% de sus nietas serán portadoras
b- 30° c- El 25% de su descendencia estará
c- 60° afectada
d- 90° d- El 25% de su descendencia será
homocigota dominante
92) Si un enfermo presenta una deficiencia
de los factores de la coagulación 97) El principal componente de la barrera
dependientes de la vitamina K hemato- testicular está constituido por:
presentará: a- Endotelio capilar peritubular
a- Tiempo de sangría prolongado b- Células de Sertoli
b- Tiempo de sangría y tiempo de c- Membrana basal del tubo seminífero
trombina prolongados d- Células mioides
c- Tiempo de tromboplastina parcial
activada (APTT o KPTT) y tiempo de 98) El nucleosoma está compuesto por:
protrombina (de Quick) prolongados a- Una parte fibrilar densa y una parte
d- Tiempo de sangría, KPTT y tiempo de granular periférica.
Quick prolongados b- Ribonucleoproteínas ricas en uridina.
c- Por un octámero de histonas (H2A,
93) Las lesiones talámicas se caracterizan H2B, H3 y H4) más ADN.
por: d- Un octámero de proteínas ácidas más
a- Afectación de la artería estriada lateral. ADN
b- Producen un impacto importante en el
hemicuerpo contralateral 99) La muestra más apropiada para el
principalmente por afectación de vías diagnóstico de la brucelosis es:
sensitivas a- Orina para cultivo.
c- No afectar los sistemas motores ni b- Sangre para hemocultivos y pruebas
circuitos cognitivos. serológicas.
d- No comprometer las vías nociceptivas. c- Materia fecal para cultivo y detección
de toxinas.
94) ¿De qué segmentos medulares se d- Orina para detección de antígenos por
originan las raíces nerviosas motoras aglutinación.
del nervio femoral?
a- L1, L2 Y L3. 100) ¿Cuál de los siguientes
b- L2, L3 Y L4. patógenos puede atravesar la placenta
c- L3, L4 Y S1. y causar infección en el feto?
d- L4, S1 Y S2. a- Listeria monocytogenes.
b- Streptococcus pneumoniae.
95) Indique cuál es el origen de los tejidos c- Bacillus cereus.
embrionarios que forman parte del d- Bordetella parapertussis.
corazón:
a-Del mesodermo lateral de la placa
cardiogénica y endodermo.
1.B 11.B 21.A 31.C 41.A 51.A 61.A 71.A 81.B 91.D
2.C 12.B 22.B 32.D 42.D 52.B 62.C 72.C 82.C 92.C
3.B 13.C 23.C 33.C 43.A 53.C 63.C 73.C 83.B 93.B
4.C 14.B 24.A 34.D 44.C 54.C 64.A 74.A 84.A 94.B
5.A 15.B 25.C 35.B 45.C 55.A 65.B 75.B 85.A 95.D
6.B 16.C 26.B 36.C 46.C 56.B 66.C 76.A 86.B 96.A
7.B 17.C 27.D 37.B 47.C 57.D 67.A 77.D 87.B 97.B
8.D 18.B 28.B 38.C 48.B 58.A 68.A 78.A 88.B 98.C
9.B 19.C 29.C 39.D 49.D 59.A 69.A 79.A 89.B 99.B
10.B 20.D 30.C 40.D 50.A 60.C 70.D 80.A 90.D 100.A
b) Solubilización del ácido úrico
1.Paciente que acude por intento de suicidio tras dejar a c) Reactividad con hipoxantina
su novia. Tenía problemas con el consumo de alcohol. d) Efecto antiinflamatorio sobre el tejido articular
¿Qué tipo de personalidad tiene? e) Secreción tubular aumentada de ácido úrico
a) Doble
b) Neurasténica 6.El cuadro más frecuente en el adulto mayor es:
c) Adictivo
d) Histérica a) Psicosis aranoide
e) Paranoica b) Neurosis de ansiedad
c) Depresión
2.¿Cuál es la probable causa de presbiacusia o sordera d) Alzheimer
gradual que acompaña al envejecimiento? e) Ninguna de las alternativas es correcta.

a) Inflamación crónica del oído externo 7.Según el proceso de Reforma en Salud del MINSA. ¿Qué
b) Acumulación de líquido en el oído medio se prioriza en la organización de los servicios de
c) Perforación de la membrana timpánica salud?
d) Pérdida acumulada gradual de células ciliares
e) Otoesclerosis a) Asistencia y recuperación
b) Detección y tratamiento precoz
3.Una paciente de 34 años de edad presenta un cuadro de c) Promoción y prevención
cirrosis descompensada con ascitis y encefalopatía. d) Rehabilitación
Se detecta virus de la hepatitis C y un e) Recuperación y rehabilitación
hepatocarcinoma de 3 cm en el lóbulo hepático
derecho. ¿Cuál es el tratamiento de elección? 8.Escolar de 6 años presenta hace dos días fiebre de
38.5°C, hiporexia, malestar general, dolor en
a) Resección del lóbulo hepático erecho hipocondrio derecho y náuseas. Se ha reportado 4
b) Resección limitada del tumor casos similares en su aula. No tiene antecedentes
c) Interferón patológicos de importancia. Al examen: activo,
d) Trasplante hepático hemodinárnicamente estable, hígado a 4cm DRCD y
e) Derivación porto-sistémica doloroso a la palpación. Resto del examen sin
alteración. ¿Cuál es el diagnóstico más probable?
4.Una mujer de 23 años acude a consulta por
irregularidades menstruales e infertilidad. Su a) Hepatitis C
evaluación clínica es completamente normal. Sus b) Colecistitis Aguda
exámenes bioquímicos muestran una reducción en c) Hepatitis B
los niveles de estradiol y hormona folículo d) Pancreatitís
estimulante (FSH). Se le diagnostica un e) Hepatitis A
hipogonadismo central. ¿Cuál de las siguientes es la
función más importante de la FSH? 9.En relación a la infección por VIH, marque la respuesta
CORRECTA:
a) Provocar ovulación
b) Estimular la secreción de progesterona c) Provocar la a) En el periodo asintomático no hay producción viral.
fase secretoria urinaria b) La carga viral indica el pronóstico del paciente y el
d) Inhibir la secreción de estrógenos e) Estimular la recuento de CD4 es el marcador inmunológico.
maduración del folículo c) La quimioprofil axis con cotrimoxazol protege contra
Cryptosporidium y Pneumocystis carinii Isoniazida se
indica como profilaxis siempre y cuando el paciente
5.Un varón de 54 años acude a urgencias por dolor severo tenga CD4 menor de 200 x mm3
en el dedo gordo del pie derecho. En el pasado había d) Las infecciones oportunistas se presentan
tenido episodios similares de menor gravedad, que e)solamente con CD4 menor de 200 x mm3
siempre trataba con analgésicos. El dedo esta
enrojecido, inflamado y muy sensible a cualquier 10.Lactante de 11 meses, inicia con vómitos, fiebre de
movimiento. Una aspiración mediante aguja confirma 38,2°C e irritabilidad. Se agregan deposiciones
la presencia de cristales de ácido úrico y se le trata líquidas sin moco ni sangre. Al examen: mucosas
con indometacina (AINE) oral. Un mes más tarde secas, ojos hundidos y llenado capilar de 2 segundos
continua libre de síntomas y se le recomienda ¿Cuál es el estado de hidratación?
alopurinol como tratamiento
preventivo ¿Cuál de los siguientes mecanismos de acción a) Hidratado
corresponde con el del alopurinol? b) Deshidratación leve
c) Deshidratación severa sin shock
a) Inhibición de la xantina oxidasa d) Deshidratación moderada.
e) Deshidratación severa con shock
18.Lo primero que debe realizarse en un paciente con
11.¿Qué vacuna está contraindicada en un niño alérgico a sospecha de cáncer del tercio medio recto es:
la ovoalbúmina?
a)Tacto rectal
a) Vacuna pentavalente b)Proctoscopia
b) BCG c)Proctosigmoidoscopia
c) Vacuna triple SPR d) Vacuna HVB d)Colonoscopia
e) Vacuna contra el neumococo e)Rx contrastada de colon

12.Niño de 9 meses de edad que tiene un peso de 10kg 19.El tumor más frecuente del epiplón es:
,cabello despigmentado, edema en miembros,
queilitis y eccema ¿Cuál es el diagnóstico más a)Carcinoma metastásico b)Fibroma
probable? c)Lipoma d)Leiomiosarcoma e)Neurofibroma

a)Marasmo 20.La operación de elección para la estenosis pilórica


b) Eutrófico congénita es:
c) Desnutrición calórica-proteica
d) Kwashiorkor a)Gastroyeyunostomía b)Gastroduodenostomía
e) Con bajo peso c)Piloroplastía d)Pilorotomía e)Antrectomía

13.Adolescente de 12 años, acude a consulta por 21.Adolescente de 17 años, acude a consulta 3 meses
desplazamiento del pezón y la areola y ausencia del después de sufrir abuso sexual y robo de sus
pectoral mayor y menor. ¿Cuál es el diagnóstico más pertenencias, por recuerdos recurrentes del episodio
probable? referido, le produce angustia y depresión. ¿Cuál es
tipo de trastorno que presenta?
a)Distrofia muscular b)Miopatia tiroidea c)Escápula alada
d)Síndrome de Poland e)Síndrome de Turner a)Estrés post traumático agudo b)Pánico
c)Mixto ansioso depresivo d)Obsesivo compulsivo
14.¿Cómo se denomina el enanismo en niños originado e)Estrés Post traumatico crónico
por el maltrato y el descuido crónico?
22.Un centro de salud recibe un ecógrafo remitido por la
a)Psico-social b)Síndrome de Laron c)Hipofisiario Red de Salud. ¿En qué documento de gestión debe
d)Acondroplásíco e)Cretinismo incluirse la actividad para poner en funcionamiento el
ecógrafo?
15.Si al evaluar a un neonato durante la atención
inmediata encuentra: respiraciones espontáneas, a)Plan de mantenimiento b)Plan operativo institucional
frecuencia cardiaca de 80 por minuto y acrocianosis. c)Plan anual de adquisiciones
Luego de los pasos iniciales, usted ¿qué indicaría?: d)Reglamento de organización y funciones e)Manual de
organización y funciones
a)Bolseo (VVP), masaje cardiaco e intubación
b)Adrenalina, expansores de volumen y bicarbonato. 23.Varón de 30 años sufre accidente de tránsito. Al
C)Bolseo (VVP) examen: herida
d)Intubación y masaje cardiaco e)Masaje cardiac contuso cortante en región parietal derecha,
escoriaciones en tórax y abdomen, equimosis en
16.¿Cuál es el tratamiento más adecuado en un paciente flanco derecho, fractura de clavícula derecha y fémur
con hipokalemia severa?: izquierdo. Flexión patológica del cuello, apertura
ocular al provocarle dolor y no emite sonidos,
a)Administrar K por vía oral murmullo vesicular no pasa en HTD, matidez a la
b)Reposición de K alrededor de 20-40 mEq/por hora percusión; abdomen: distendido, RHA presentes.
infusión por vía central Utilizar c)cloruro de potasio al ¿Cuál es la conducta a seguir?
2% en forma rápida
d)Tratar la causa que originó la hipokalemia e)Administrar a)Dos vías endovenosas gruesas y cortas b)Colocar tubo
bicarbonato de sodio al 8.4% de drenaje torácico c)Colocación de tubo
endotraqueal d)Lavado peritoneai diagnóstico
17.El signo de Grey Turner que aparece en pancreatitis e)Tomografía cerebral
aguda hemorrágica consiste en equimosis en:

a)Fosas iliacas 24.Varón de 58 años, acude por presentar ortópnea,


b)Ángulos costovertebrales c)Zona periumbilical d)Cuello edema de miembros inferiores y disnea. Se hace el
e)Apéndice xifoides diagnostico de falla cardíaca crónica descompensada.
¿Cuál de los siguientes fármacos utilizados en el 32.¿En cuál de los siguientes casos está indicada la
tratamiento, tiene como mecanismo de acción ser profilaxis antibiótica?
antagonista aldosterona?
a)Apendicitis aguda con peritonitis localizada
a)Espironolactona b)Valsartan c)Bisoprolol d)Enalapril b)Apendicitis aguda con peritonitis generalizada
e)Furosemida c)Plastrón apendicular
d)Apendicitis aguda no complicada e)Absceso apendicular
25.Fractura de la tibia con dolor y disminución de pulso
pedio ¿Cuál es el diagnóstico más probable? 33.Ictericia neonatal a predominio directo progresiva, BT:
31 mg/ dl ¿Cuál es el diagnóstico más probable?
a)Síndrome compartimental b)Hematoma de pierna
c)Lesión de nervio tibial a)Atresia biliar Anemia hemolítica
d)Embolia grasa b)Hipoplasia de vías biliares c)Quiste del colédoco
e)Trombosis venosa profunda d)Estenosis de los conductos biliares

26.¿Qué vacuna está contraindicada en gestantes? 34.Para lograr que se consolide una fractura, debemos
lograr la:
a)Hepatitis B b)Influenza c)Paperas d)Meningocócica a)Analgesia b)Estabilidad c)Inestabilidad
e)Rabia d)Todas las alternativas. E)Ninguna de las alternativas.

27.¿Cuál es la reacción adversa más frecuente en el 35.Varón de 60 años, sufre atropello hace 15 minutos. Al
tratamiento con heparina? examen: FC: 140 lat/min PA: 60/40 mm Hg, frialdad
distal y dolor en hemiabdomen inferior. Ecografía:
a)Leucopenia b)Agranulocitosis c)Trombocitopenia liquido libre en regular cantidad en fondo de saco
d)Anemia aplásica e)Reacción leucemoide rectovesical, ¿Cuál es la conducta a seguir?

a)Laparotomía exploratoria b)TAC de abdomen


c)Ecografía doppler
d)Lavado peritoneal e)Paracentesis

28.Paciente con dolor abdominal crónico tipo cólico e 36.Lactante que no quiere su papilla ¿cuál debe ser la
ictericia . Exámenes de laboratorio: BT: 10mg/ dl conducta?
¿Cuál es el diagnóstico más probable?
a)Utilizar endulzantes artificiales b)Agregarle sal a la
a)Pancreatitis aguda b)Colelitiasis papilla
c)Cáncer de páncreas d)Coledocolitiasis e)Tumor de c)Darle solo puré de fruta d)Fraccionar las tomas de
Klatskin papillas. E)Agregar condimentos naturales

29.En hiperkalemia con cambios electrocardiográficos 37.Pelvis con conjugado diagonal de 10.5 o sea el
¿Cuál es el tratamiento inicial? obstétrico queda en 9cm.
¿De qué tipo de pelvis se trata?
a)Bicarbonato IV b)Hemodiálisis c)Gluconato de calcio
d)Insulina a)Pelvis normal
e)B-2 agonistas b)Estrechez pélvica
c)Pelvis estrecha no viable d)Pelvis ginecoide
30.Señale el factor de riesgo predisponente para IMA e)Pelvis platipeloide
según el Estudio Framingham:

a)Obesidad 38.¿Cuál de las siguientes medidas, produce el mayor


b)Sexo masculino impacto en la disminución de las infecciones
c)Historia familiar de IMA d)HTA sistémica e)Dislipidemia intrahospitalarias?

31.La evaluación de la permeabilidad de la trompa de a)Uso de alcohol gel antes de examinar al paciente
Eustaquio, se realiza mediante: b)Profilaxis antibiótica a los pacientes
c)Lavado de manos del personal de salud d)Uso de
a)Maniobra de Valsalva. B)Maniobra de Toynbee máscara N - 95
c)Catecismo con sonda de YTAR e)Uso de mascarilla quirúrgica
d)Todas las alternativas son correctas e)Ninguna de las
alternativas 39.A qué edad recomienda la American Academy of
Pediatrics que se detecte la hipoacusia?
a)Al nacer 46.En salud comunitaria, ¿cuál es la estrategia para el
b)3 meses trabajo en zonas urbanas y rurales?
c)2 años a)Reunión vecinal
d)4-5 años b)Asamblea de padres de familia c)Visita domiciliaría
e)12 años d)Reunión de junta directiva e)Feria comunal

40.Niño de 3 años, afebril, con lesiones 47.¿Cuál de los siguientes signos es un hallazgo tardío en
papuloequimoticas en miembros superiores e el cáncer de mama?
inferiores, simétricas, edema, miembros inferiores y a)Tumor palpable
dolor abdominal, no visceromegalia, ¿cuál sería su b)Retracción reciente del pezón c)Oscurecimiento de la
diagnóstico clínico? areola d)Asimetría del tamaño de las mamas
e)Secreción sanguinolenta por el pezón
a)Purpura trombocitopénica b)Púrpura de Henoch-
Schönlein c)Leucemia infantil 48.Gestante de 32 semanas, preeclámptica con manejo
d)Linfoma expectante, presenta convulsiones. Luego de
e)Ninguna de las anteriores tratamiento con sulfato de magnesio presenta paro
cardiorespiratorio. ¿Cuál es el antídoto?
41.De las siguientes afirmaciones, una no es cierta con a)Bicarbonato de sodio b)Permanganato de potasio
respecto a infección urinaria en niños: c)Nitroprusiato de sodio d)Gluconato de calcio
a)El cuadro clínico depende en parte de la edad, e)Carbonato sódico
localización e intensidad de la infección
b) La E. coli es el agente etiológico más frecuente 49.¿Qué vía tiene menor biodisponibilidad? :
c)Las recurrencias son raras a)Oral b)Intramuscular
d)A menor edad mayor tendencia a formar cicatrices c)Subcutánea d)Intradérmica e)Endovenosa
renales
e) Ninguna alternativa es correcta. 50.El embarazo molar ocurre por la unión de :
a)Espermatozoide anucleado y óvulo normal
42.¿Cuál es la anomalía del aparato genital femenino b)Espermatozoide bicéfalo y óvulo normal
causada por el progreso caudal insuficiente de los c)Espermatozoide normal y ovulo normal
conductos paramesonéfrlicos? d)Espermatozoide sin cola y óvulo anucleado
e)Espermatozoide ndmial y óvulo anucleado
a) Hipertrofia de clítoris
b) Tabique vaginal 51.El prolapso uterino de II grado según POPQ es cuando
c) Agenesia de la vagina d) Himen perforado e) Aplasia el punto más declive se encuentra:
uterina a)A dos centímetros distales de la carúncula hymeneal
b) Más de un centímetro por debajo del nivel del himen
43.La bronquitis en los primeros meses de vida puede c)A dos centímetros proximales de la carúncula himenea!
dejar secuelas. Marque usted lo correcto: d)Entre el centímetro p revio al nivel himeneal y el
centimetro distal a este
a)Bronquiectasia b)Sobreinfección bacteriana e)Sobrepasando a 3 centímetros del introito
c)Desarrollo de asma en algunos niños d)Todas de las
alternativas son correctas. E)Ninguna de las 52.Mujer con antecedente de hemorragia obstétrica que
alternativas es correcta. no pudo dar de lactar, actualmente con astenia e
hipotensión ¿Cuál es el diagnóstico más probable?
a)Hipotiroidismo
b)Hipogonadismo
c)Hipopituitarismo
44.La meningoencefalitis bacteriana en recién nacido es d)Insuficiencia suprarrenal
más frecuentemente producida por: e)Hipoparatiroidismo
a)Haemophilus influenzae b)Bacterias gram negativas
c)Streptococcus pneumoniae 53.Paciente con pulso arrítmico y deficitario. ¿A qué
d)Todas de las alternativas son correctas. e)Ninguna de las enfermedad corresponde?
alternativas es correcta. a)Taquicardia supraventricular
b)Fibrilación auricular
45.Pre-escolar de 3 años, con calendario de vacunación c)Taquicardia ventricular
incompleto. Presenta rinorrea blanquecina. Al d)Fibrilación ventricular
examen: T»: 37.8°C, erupción macular generalizada e)Bloqueo AV
con adenopatías retroauriculares y occipitales. ¿Cuál
es el diagnóstico? 54.En relación a las hemorragias del puerperio inmediato,
a)Rubeola b)Urticaria c)Poséola d)Varicela e)Sarampión marque la combinación de respuestas verdaderas (V)
o falsas (F) que corresponda:
1. La hemorragia del puerperio inmediato es la principal Laboratorio: leucocitos 23,000 x campo y PCR: 1.5
causa de muerte materna en el Perú. mg/L, ¿Cuál es la conducta a seguir?
2. El pujo antes de la dilatación completa puede a)Repetir el hemograma antes de iniciar
condicionar desgarros cervicales antibióticoterapia b)Tomar hemocultivo y estudio de
3. La edad muy joven es la principal causa de Placenta líquido cefalorraquídeo c)Tomar hemocultivo e iniciar
Previa antibióticoterapia
3. El parto instrumentado puede condicionar d)Repetir el hemograma y PCR e iniciar antibióticoterapia.
laceraciones perineo vaginales e)Estudio del líquido cefalorraquídeo e inicio de
4. La versión interna y extracción podálica pueden antibióticoterapia
condicionar ruptura uterina

F,V,F,V,V
V,F,F,V,V 60.Se denomina afaquia a:
F,V,V,F,F a)Presencia de pus en cámara anterior
V,V,V,V,V b)Estado normal de refracción
V,V,F,V,V c)Presencia de dos cristalinos
d)Ausencia de cristalino
e)Sinequia posterior del iris
55.Una medida que puede prevenir la hemorragia y
formación de hematomas al suturar un desgarro 61.Preescolar de 2 años previamente sano, presenta
perineo vaginal, cervical o episiotomía es la siguiente. puntitos morados en cara, miembros
a)Poner suturas interrumpidas e intervalos de 0.3 cms inferiores tronco, niega otras molestias, Al examen:
b)Usar sutura crómico 1 petequias diseminadas.
c)Usar aguja redonda en vez de cortante ¿Cuál es el diagnóstico más probable?
d)Hacer sutura continua a)Anemia aplásica
e)Poner el primer punto a 0.5 a 1 cm, por encima del b)Púrpura de Henoch-Schónlein
borde superior del desgarro o episiotomía c)Púrpura trombocitopenica idiopática
d)Síndrome de Kasabach-Merritt.
56.Pre-escolar de 3 años es llevado a consulta con e)Leucemia linfoblástlea.
dermatitis, enteritis, alopecia y retraso del
crecimiento. ¿Cuál es la probable deficiencia de 62.Varón de 60 años, fuma 15 cigarrillos al día desde los
vitamina que presenta? 25 arios de edad. Hace un mes presenta
a)Ácido antoténico expectoración hernoptoica escasa. Radiografía de
b)Tiamina tórax:
c)Cianocobalamina nódulo paratraqueal derecho de 3cm de diámetro. Biopsia
d)Ácido fálico por broncoscopia: Ca broncogénico, Estadiaje: T1,
e)Niacina NO; MO. ¿Cuál es el tratamiento?
a)Radioterapia
57.Mujer de 32 años, que hace 2 días presenta malestar b)Quimioterapia
general y sensación febril. Se agrega cefalea intensa, c)Radioterapia y Quimioterapia
vómitos, compromiso de sensorio y 3 episodios de d)Cirugía y radioterapia
convulsiones. Al examen: Glasgow 10, rigidez de e)Cirugía
nuca, T°: 38,9 °C ¿Cuál es el diagnóstico más
probable? 63.Mujer de 25 años sexualmente activa que acude por
a)Meningoencefalitis aguda bacteriana b)Meningitis vira! disuria, polaquiuria y tenesmo vesical. Al examen: T°:
c)Meningoencefalitis tuberculosa d)Meningoencefalitis 38.5°C dolor en el flanco derecho y PPL derecho
herpética e)Sindrome convulsivo refractario positivo. ¿Cuál es el germen más probable de
infección?
58.Mujer de 42 años, desde hace 2 meses presenta tos, a)Proteus mirabilis
episodios de disnea y pérdida de 5 Kg. Al examen: b)Klebsielia sp
38°C, adelgazada, crepitantes en vértice derecho y c)Enterococo sp
algunos roncantes en ambos campos pulmonares, d)Estreptococo
¿Cuál es el diagnóstico más probable? e)Escherichia coli
a)Neumonía crónica b)Bronquitis crónica c)Tuberculosis
pulmonar d)Asma bronquial 64.A los niños de un colegio pre-escolar se les realiza un
e)Enfermedad pulmonar intersticial test de Graharn para detectar oxiurasis. Si el
resultado es negativo, al día siguiente se les repite la
59.Recién nacido de 33 semanas, hospitalizado en la UCI prueba. ¿Qué ocurre en esta secuencia?
neonata] por 1 semana, las últimas 24 horas presenta a)Disminuye la sensibilidad y aumenta la especificidad
hipotermia, distensión abdominal, hipoactividad b)Aumenta la sensibilidad y aumenta la especificidad
marcada y disminución de la presión arterial. c)Disminuye la sensibilidad y disminuye la
especificidad d)Aumenta la sensibilidad y disminuye
la especificidad 71.Una mujer de 69 años presenta una pigmentación
e)No hay variación de la sensibilidad ni de la especificidad oscura aterciopelada en las axilas. Había perdido
unos 5 kg de peso en los últimos tres meses y se
65.Gestante multípara que en las últimas cinco semanas quejaba de acidez y saciedad precoz. No tenía otros
se despierta y encuentra que se ha producido una síntomas. ¿Cuál de las siguientes enfermedades debe
hemorragia vaginal, su parto anterior fue normal, no investigarse?
presenta dolor ni contracciones. Con el síndrome a)Carcinoma en algún órgano
expuesto, el diagnostico que usted haría en este caso b)Linfoma
es: c)Diabetes mellitus
a)Ruptura del seno marginal d)Sarcoidosis
b)Desprendimiento de placenta e)Alergia
c)Placenta previa
d)Trabajo de parto inminente 72.La detección de la proporción de obesos en una fecha
e)Carcinoma cervical determinada (18-11-2012) es:
a)Prevalencia
66.La incontinencia urinaria de urgencia en la mujer post- b)Incidencia
menopausica se debe: c)Incidencia acumulada
a)Pérdida del ángulo uretra vesical posterior d)Riesgo atribuible
b)Aumento del ángulo uretra vesical posterior e)Riesgo relativo
c)Trigonitis senil
d)Infecciones crónicas inevitables de vías urinarias que 73.¿Cuál es la situación de la famiIia que con relativa
ocurren en mujeres post -menopaúsicas frecuencia se da en la etapa del adulto mayor?
e)Pérdida de la inervación del cuello vesical a)Formación
b)Expansión
67.Mujer de 33 años presenta lesiones dérmicas en rostro c)Contracción
que aumentan por exposición al sol, dolor articular d)Disolución
migratorio y convulsiones. Antecedente de 2 abortos. e)Comprensión
Laboratorio: anemia, plaquetopenia, ANA (+), Rx. De
tórax: derrame pleural leve bilateral, ¿Cuál es el 74.Gestante de 8 semanas, acude a su control prenatal.
diagnóstico? No refiere síntomas o problemas al orinar. Trae
a)Espóndiloartropatía seronegativa resultado: urocuitivo: coli + de 100,000 UFC/ml,
b)Lupus eritematoso sistémico sensible a todos los antibióticos. ¿Cuál de los
c)Esclerosis sistémica progresiva siguientes indicaría?
d)Polimiositis a)Doxiciclina b)Rifampicina
e)Artritis reumatoide c)Trimetropin y Sulfametoxasol d)Cefuroxima
e)Ciprofloxacina
68.Gestante adolescente de 32 semanas por FUR, acude a
Emergencia por cefalea frontal persistente desde 75.Varón de 33 años, presenta cefalea intensa luego de
hace 3 días. Al Examen: PA: 160/110 mm Hg. esfuerzo físico, casi inmediatamente pierde el
Anasarca e hiperreflexia. ¿Cuál es el tratamiento sensorio, cayendo al suelo. Al examen: PA: 120/80
indicado para el control de la hipertensión?: mm Hg. Glasgow 8, sin compromiso motor, rigidez de
a)Nifedipino nuca (+), signo de Kerning (+), ¿Cuál es el diagnóstico?
b)Furosemida a)Hematoma subdural agudo b)Hemorragia
c)Clorotiazida subaracnoidea c)DCV isquemico embólico d)DCV
d)Captopril isquémico gigante e)Neurocisticercosis
e)Verapamilo
76.Postmenopáusica con sangrado genital de útero. ¿Cuál
69.Paciente gestante de 22 semanas, presento 3 abortos es la causa más frecuente ?:
a las 12, 13, 14 semanas, en la gestación actual se a)Miomatosis uterina b)Cáncer endometrial c)Cáncer de
procedió a realizar una operación cervical con el ovario d)Quiste ovárico e)Endometriosis
objeto de llevar adelante el embarazo:
a)Amenaza de aborto b)Aborto incompleto c)Aborto 77.Posta con mal saneamiento y no hay protocolo para
frustro d)Aborto inevitable e)Aborto habitual atención , en el análisis FODA tenemos:
a)Debilidad más amenaza b)Solo amenaza
70.La gestante presenta: c)Solo debilidad d)Oportunidad más amenaza e)Debilidad
a)Falso trabajo de parto más oportunidad
b)Trabajo de parto en fase activa
c)Trabajo de parto en fase de desaceleración 78.La amenorrea primaria es característica
d)Trabajo de parto en fase latente de todas las siguientes condiciones,
e)Trabajo de parto en fase acelerada EXCEPTO:
a)Resistencia a las Gonadotropinas b)Síndrome de Turner a)Hiperhemesis gravídica b)Sinequias intrauterinas
c)Disgenesia Gonadal Pura d)Síndrome de Ovario c)Coagulopatía por consumo d)Endometritis
Poliquístico e)Síndrome de Rokitansky e)Ninguno de los anteriores

79.Gestante que presenta cefalea, PA >160/ 105mmHg y 88.Lactante de 18 meses de edad, peso 11.8 kg,
albuminuria +++ ¿Cuál es el diagnóstico más asintomático, hace 24 días que recibe la visita de su
probable? tío, sintomático respiratorio, evaluado en el hospital
a)Preeclampsia severa b)Preeclampsia leve con DX TBC pulmonar BK ++. ¿Cuál es la conducta con
c)Hipertensión gestacional severa d)Hipertensión el lactante?
gestacional leve e)Preeclampsia sobreagregada a)PPD
b)BK con sonda nasogástrica c)INH medio comprimido
80.Niña que no menstrua, presenta dolor abdominal diario d)Rx tórax
cíclico. ¿Cuál es el diagnóstico? e)Todas las alternativas son correctas.
a)Hipogonadismo b)Hipotiroidismo c)Síndrome de Turner
d)Himen imperforado e)Ovario poliquístico 89.Varón de 29 años acude con múltiples ganglios
cervicales, fiebre y dolor de garganta. Por factores de
81.Mujer de 45 años presenta dolor abdominal episódico riesgo que mencionó el paciente se sospecha de
y refiere, que el dolor aumenta posterior mente a la infección por el VIH.
ingestión de comidas ricas en grasas. ¿La acción de ¿Qué examen es útil si pensamos que presenta un
cuál de las siguientes hormonas es responsable de la síndrome retrovirai agudo?
intensificación de sus síntomas?: a)Elisa-VIH b)Western Blot c)Recuento de CD4 d)IFI-VIH
a)Gastrina b)Secretina e)Carga Viral
c)Colecistocinina (CCK) d)Pepsina
e)Somatostat ina 90.A cualquier atributo o circunstancia de una persona o
de la comunidad asociado al aumento de la
82.¿Cuál de los siguientes neurotransmisores produce el probabilidad de que desarrolle un proceso mórbido,
ingreso de los iones cloro, en la membrana post se le denomina:
sináptica?: a)Riesgo atribuible b)Riesgo relativo c)Prevalencia
a)Glutamato b)GABA d)Factor de e)riesgo Incidencia
c)Aspartato d)Serotonina e)Noradrenalin

83.En la fórmula del perfil vaginal la 3era cifra 91.Todo fenómeno de salud de poblaciones
corresponde a: humanas está biológica y ……………….. determinado:
a)Uretrocele b)Enterocele c)Rectocele d)Histerocele a)Genéticamente b)Culturalmente c)Hereditariamente
e)Cistocele d)Socialmente
e)Ninguna de las alternativas.
84.En la mujer climatérica la dislipidemia más frecuente
es: 92.Varón de 19 años acude por dolor abdominal en
a)Aumento colesterol total b)Aumento de HDL c)Aumento rnesogastrio que luego se localiza en FiD, náuseas y
de LDL d)Descanso de triglicéridos e)Todas las vómitos en 2 oportunidades e hiporexia A la
anteriores palpación dolor en FID, este signo se denomina
a)Dunphy b) Mc Burney C)Rovsing d)Blumbere)Rebote
85.En el manejo de la hemorragia post parto se
consideran necesario las siguientes medidas 93.Escolar de 6 años, presenta desde los 11 meses,
generales, excepto: episodios recurrentes de erupción papulovesicular,
a)Abrir vía endovenosa con bránula N° 18. b)Administrar eritematosa y pruriginosa en cuero cabelludo y región
cloruro de sodio + oxitocina c)Laparotomía retroauricular, Antecedente de rinitis intermitente. Al
exploratoria inmediat a d)Mantener vías aéreas examen: piel xerótica con zonas de liquenificación.
permeables e)Cateterizar vejiga ¿Qué tratamiento le indicaría?
a)Jabón de glicerina y antihistamínicos b)Corticoide
86.El diagnostico de trabajo de parto incluye todo, sistémico y antihistamínícos c)Crema humectante y
Excepto: antibió ticos d)CorticoIde tópico y antihistamínicos
a)Confirmación del trabajo de parto b)Periodo y fase del e)Tacrolimus y ciclosporina A
trabajo de parto c)Encajamiento y descensos
d)Identificación de la presentación y posición de la cabeza 94.La oferta eficaz y sistematizada de servicios básicos de
fetal e)Tiempo transcurrido entre el inicio y la hora de salud que satisfagan las necesidades de la población
la evolución se denomina:
a)Cobertura b)Seguridad social c)Demanda
87.Un aborto frustro, la complicación que se asocia con d)Servicios básicos de salud (SBS) e)Necesidad de salud
mayor frecuencia es:
95.Un niño presenta meningoencefalitis por a)Antidiurética b)Aldosterona c)Angiotensina II
meningococo, él estuvo en contacto íntimo con sus 2 d)Factor natriurético atrial e)Ninguna
hermanitos a quienes se les tuvo que dar
medicamentos para evitar la presencia de la 104.La articulación radiocarpiana es una artculación del
enfermedad, esta acción a qué medida de prevención tipo:
corresponde: a)Enartrosis
a)Promoción b)Primordial c)Protección b)Artrodia c)Condilea d)Tróclea e)Trocoide
d)Tratamiento oportuno e)Diagnóstico precoz
105.La vena que pasa por el canal deltopectoral es:
96.Un estudio observacional en el que se comparan 110 a)V. Humeral
individuos con cáncer de senos paranasales con 220 b)V.Intercostac)l V. Cefálica
individuos sin la enfermedad, corresponde a un d)V. Basílica
estudio: Longitudinal e)V. Circunfleja
a)Caso-control b)Cohortes concurrente c)Transversal
d)Cohortes no concurrente 106.Es el sustrato utilizado para sintetizar serotonina:
a)Acetil CoA b)Serina c)Tirosina d)Fenilalanina
97.La presencia continua de una enfermedad causada por e)Triptófano
un grande infeccioso en una zona geográfica
determinada se denomina: 107.Las glándulas que destruyen parte de su citoplasma
a)Holoendemia b)Epidemia c)Brote d)Hiperendemia para formar la secreción glandular se llaman:
e)Endemia a)Holocrinab)Apocrinac)Merócrinad)Endocrinae)Africrina
s
98.En la prueba de hipótesis, el investigador comete un
error tipo II o beta cuando: 108.Fractura de maxilar y unguis con ojo rotado en medial
a)No establece el nivel de significancia y aducido. Indique cuál es la lesión:
b)No rechaza la hipótesis nula, siendo falsa c)Rechaza la a)Lesión de músculo recto interno b)Lesión de músculo
hipótesis alterna, siendo falsa d)Rechaza la hipótesis recto superior c)Lesión de músculo recto externo
alterna, siendo verdadera e)Rechaza la hipótesis nula, d)Lesión de músculo recto inferior e)Lesión de
siendo verdadera músculo oblícuo menor

99.La medida de validez de una prueba diagnóstica que 109.Niño con fiebre, inyección conjuntival, lesiones
evalúa “la probabilidad de que el paciente tenga la descamativas en las manos y adenopatías ¿Cuál es el
enfermedad con su resultado positivo de la prueba diagnóstico más probable?
evaluada”, se denomina: a)Celulitis
a)Eficacia diagnostica b)Especificidad c)Sensibilidad b) Carbunco
d)Valor predictivo negativo e)Valor predictivo positivo c)Pioderma gangrenoso d)Enfermedad de Kawasaki
e)Enfermedad de Lyme
100.Los principios éticos para la investigación en seres
humanos están definidos por la: 110.Mujer con lesiones vesiculares en los labios mayores
a)Declaración de alma – Ata b)Declaración de Helsinki asociadas a adenopatías inguinales. ¿Cuál es el
c)Declaración de OMS diagnóstico más probable?
d)Declaración de las naciones unidas a)Herpes Zoster
e)Declaración universal de los derechos humanos b)Chancroide c)Herpes genital d)Gonorrea
e) Sífilis
101.La medición de asociaciones en epidemiologia utiliza:
a)El riesgo atribuible ocupacional 111.¿Cuál es la hernia que sale por el triángulo de
b)El riesgo atribuible Hasselbach?
c)El riesgo diferencial a)Hernia inguinal indirecta b)Hernia inguinal directa
d)El riesgo relativo c)Hernia de Litre
e)La fracción etiológica de riesgo d)Hernia de Spiegel e)Hernia femoral

102.¿Cuál de los siguientes signos demuestra ovulación en 112.¿Cuál es contraindicación absoluta de


la mujer?: anticonceptivos orales?
a)Aumento de la viscosidad del moco cervical a)Trombosis venosa profunda b)Migraña
b)Desaparición de células cornificadas c)Aumento de c)Hipertensión arterial d)Diabetes mellitus e)Cirugía
1°C de temperatura corporal d)Desaparición del electiva
glucógeno
e)Todas muestran ovulación 113.Paciente con flujo vaginal espeso y cremoso asociado
a prurito ¿cuál es el diagnóstico más probable?
103.En un paciente deshidratado cuál de las siguientes
hormonas esta disminuida:
a)Trichomoniasis vaginal b)Vaginitis por levaduras Calcio: 13 mg/dl, Creatinina: 1.8 mg/dL ¿Cuál es el
c)Vaginosis bacteriana d)Candidiasis vaginal diagnóstico más probable?
e)Vaginitis atrófica a)Garnmapatia monocional de importancia incierta
b)Mieloma múltiple
114.Paciente mujer de 80 años, presenta disnea, además, c)Plasmocitoma solitario
dolor de pecho, con antecedentes de bronquitis d)Síndrome mielodisplásico
crónicas, TBC pulmonar. Al Examen clínico: 2do. e)Insuficiencia renal crónica
Ruido aumentado a predominio pulmonar y signos de
insuficiencia derecha. La Rx muestra derecha, onda P 121.Niño con epistaxis nasal por rinofaringitis ¿Cuál es el
picuda. El diagnóstico del paciente será: tratamiento inmediato?
a)Infarto de Miocardio agudo a)Taponamiento anterior b)Presión interdigital
b)Shock cardiogénico Cor c)Taponamiento posterior
c)pulmonale crónica
d)Diabetes mellitas d)Observación
e)Insuficiencia Cardiaca Izquierda e)Administrar vitamina K

115.Una distribución difusa del infiltrado pulmonar en 122.Paciente con cuadro clínico de obstrucción
una radiografía de pulmones con un patrón en panal intestinal por hernia, el examen auxiliar que ayude al
de abeja sugiere: diagnóstico es:
a)Neumonía por Staphilococus aureus b)Neumonía por a)Tomografía computarizada de abdomen b)Radiografia
Streptococcus pneumoniae c)Neumonía por de abdomen en decúbito c)Radiografia de abdomen
Hemophilus influenzae d)Neumonía por Neumocystis en bipedestación d)Resonancia magnética de
carinii e)Neumonía por Pseudomona aeruginosa abdomen e)Ecografía abdominal

116.Una paciente post menopáusica acude a consulta 123.Niño con fiebre , convulsión tónico clónica
para informarse en la prevención de eventos generalizada , de 3minutos de duración y sin secuelas
vasculares (IMA, ACV) relacionados con la ¿Cuál es el diagnóstico más probable?
arterioesclerosis. Usted aconsejaría: a)Convulsión febril complicada b)Convulsion febril simple
a)Estatinas c)Crisis mioclónica
b)Dosaje de lipoproteína A d)Ausencias e)Crisis atónica
c)Antioxidantes
d)Mayor consumo de ácidos grasos poliinsaturados 124.Niño con contacto de TBC , sin sintomatología . El PPD
e)Todas las opciones mencionadas es 10mm ¿que indicaría?:
a)INH 10mg/ kg por 1 mes b)INH 10mg/ kg por 3 meses
117.Mujer con fiebre, signos meníngeos sin lesiones c)INH 10mg/ kg por 6 meses d)INH 10mg/ kg por 12
focales ni signos de hipertensión endocraneana. meses. E)INH 10mg/ kg por 9 meses.
¿Cuál es el paso inmediato a seguir?
a)Realizar una RM cerebral b)Realizar una TC cerebral 125.Paciente con sarcoma con metástasis pulmonar ,
c)Solicitar hemocultivos d)Realizar fondo de ojo recibe sólo morfina y oxigeno. ¿Qué término
e)Realizar punción lumbar corresponde?
a)Ortot anasia b)Distanasia c)Eutanasia pasiva
118.Traumatismo encéfalo- craneano con intervalo lúcido d)Eutanasia activa
y luego caída del Glasgow ¿Cuál es el diagnóstico más
probable? 126.Varón de 29 años con quemadura del 55% de su
a)Hematoma epidural b)Hematoma subdural agudo superficie corporal por explosión de motor y que
c)Hematoma subdural crónico d)Hemorragia pierde el conocimiento ¿Cuál es el tipo de shock más
subaracnoidea e)Hemorragia intraparenquimal frecuente en este caso?
a)Cardiogénico b)Distributivo c)Hipovolémic
119.¿Cuál es el agente etiológico de la colitis d)Anafiliáctico e)Séptico
pseudomembranosa?
a)Clostridium perfringes 127.Varón de 82 años con neumonía intrahospitalaria.
b)Clostridium botulinum Tiene Pa02/ Fi02 < 250, lacta to en sangre elevado,
c)Clostridium difficcile diuresis en las últimas 24 horas 200 cc con alteración
d)Clostridium tetani del sensorio y que responde a la administración de
e)Clostridium novyi líquidos intravenosos. El paciente se encuentra en:
a)Shock séptico
120.Mujer de 65 años con osteoporosis sin tratamiento, b)Septicemia grave
acude por dorsalgia. Al examen: palidez y dolor en c)Shock séptico resistente
región dorsal. Laboratorio: Hb: 8.5g/dL, leucocitos: d)Septicemia
3900 cel/mm3, plaquetas: 119000 x mm3. Proteínas: e)Sepsis
8.5 g/dL, Albumina: 2,5 g/dL, Globulina: 6 g/dL,
128.Niña con cuadro clínico de hepatitis A. ¿Qué 137.Mujer de 25 años, con quemadura con agua hirviendo
exámenes de laboratorio se debe solicitar? en la totalidad del miembro superior derecho, y cara
a)Transaminasas y anticuerpos IgM antiHVA anterior del miembro inferior del mismo
b)Transaminasas y anticuerpos IgG antiHVA lado. Según la regla de los 9 de Pulaski y Tennisson, ¿cuál
c)Transaminasas es el porcentaje del área afectada?
d)Transaminasa y bilirrubina a)9
e)Transaminasas y anticuerpos IgM e IgG antiHVA b)18
c)27
129.En una neumonitis intersticial asociada a infección d)36
por VIH ¿Cuál es el agente etiológico? e)45
a)Micoplasma pneumoniae
b)Estreptococo pneumoniae 138.Sangrado del tercer trimestre, indoloro, rojo
c)Legionella pneumophila rutilante. ¿Cuál es el diagnóstico más probable?
d)Pneumocistis jiroveci e)Coronavirus a)Rotura uterina
130.Lesión de cargas negativas de la membrana basal y b)Desprendimiento prematuro de placenta c)Placenta
ampliación de los poros de los podocitos. ¿Qué se previa
espera encontrar? d)Rotura prematura de membranas
a)Proteinuria masiva b)Microalbuminuria c)Hematuria e)Miomatosis uterina
d)Macroalbuminuria e)Piuria
139.Adulto mayor con PA 170/70 mmHg . ¿Cuál es el
131.Signo más sugerente de hipotiroidismo primario: tratamiento?
a)Piel seca b)Hipoactividad c)Hiperrreflexi a)IECA b)ARA II
d)Reflejo osteotendinoso lentificado e)Alopecia c)Diurético tiazídico más calcio-antagonista d)Calcio-
antagonista
132.Hipotensión arterial , ruidos cardiacos disminuídos e e)Betabloqueador
ingurgitación yugular. ¿Cuál es el diagnóstico más
probable? 140.Niña con dolor abdominal en la fosa iliaca derecha y
a)Pericarditis constrictiva b)Miocardiopatía restrictiva masa dolorosa al tacto ¿Cuál es el diagnóstico más
c)Infarto agudo de miocardio d)Taponamiento probable?
cardiaco e)Insuficiencia cardiaca derecha a)Apendicitis aguda b)Adenitis mesentérica c)Enfermedad
de Chron d)Quiste a pediculo torcido e)Linfoma
133.Paciente con cuadro clínico de pancreatitis aguda. intestinal
¿Cuál es el examen auxiliar que se debe solicitar
inicialmente? 141.¿Cuál es la acción para prevención en el segundo nivel
a)Procalcitonina b)Amilasa sérica c)TGO en un paciente con TBC 3+++?
d)Proteína C reactiva e)Deshidrogenasa láctica a)Mantener una ventilación adecuada b)Control de
contact os
134.Mujer de 32 años, con dolor abdominal bajo desde c)Mejorar la alimentación del paciente d)Incinerar el
hace 3 meses y secreción vaginal desde hace 6 días. moco y la flema del apaciente e)Formar Comités de
Al tacto vaginal: cérvix doloroso a la movilización, vigilancia comunal
anexos dolorosos a la palpación; se evidencia cérvix
con secreción mucopurulenta ¿Cuál es el diagnóstico 142. Paciente en el sexto mes de vida ¿Qué vacuna que
más probable? debe recibir?:
a)Apendicitis b)Torsión anexial c)Cistitis a)Vacuna contra Rotavirus b)Antihepatítica-HVB
d)Enfermedad inflamatoria pélvica e)Endometriosis c)BCG
d)APO y pentavalente
135.Hematoma organizado en necropsia por fractura de e)Vacuna contra el neumococo
temporal ¿De dónde procede el sangrado?
a)Arteria cerebral media b)Arteria cerebral posterior 143.Mejor terapia para evitar el suicidio :
c)Arteria cerebral anterior d)Arteria meníngea media a)Psicoterapia
e)Arteria cerebelosa b)Tratamientos cognitivos-conductuales
c) Terapia familiar
136.Paciente diagnosticada de varicella en el tercer d)Terapia dialéctico-conductual e)Antidepresivos
trimestre. Nace un niño eutrófico, actualmente tiene
lesiones en costra. ¿Qué recomendaría? 144.Principal diagnóstico diferencial de EPOC con disnea
a)Evitar la lactancia variable:
b)No dar antihistamínicos c)No bañar al niño d)Facilitar la a)Cáncer de pulmón
lactancia. b)Fibrosis pulmonar c)Bronquiectasias
e)Administrar ácido acetil salicílico d)Enfermedad pulmonar intersticial difusa
e)Asma bronquial
145.A los cuantos días se hace el control familiar del RN al 154.Varón con descarga uretral mucosa y luego
dar el alta : purulenta. ¿Cuál es el agente etiológico más
a)24 horas probable?
b)48 horas a)Mycoplasma gentalium b)Haemophilus vaginalis
c)72 horas c)Chlamydia trachomatis d)Gonococo e)Thricomonas
d)96 horas vaginalis
e)5 días
155.Niño que ingirió caustico y la madre le dio leche ¿Cuál
146.Contracciones uterinas cada 10 minutos sin es el tratamiento que debe recibir?
modificación cervical en paciente con 34 semanas de a)Observación
gestación. ¿Cuál es el diagnóstico más probable? b)Endoscopía digestiva alta c)Colocar sonda nasogástrica
a)Amenaza de aborto b)Aborto inminente d)Hacer lavado gástrico e)Hidratación enérgica
c)Amenaza de labor de parto pretérmino d)Amenaza de
labor de parto a término e)Amenaza de labor de parto 156.Factor de riesgo para neumonía aguda:
post-término a)Obesidad b)Estrés c)Alcoholismo
d)Desviación del tabique nasal
147.El divertículo de Meckel, su anatomía se caracteriza e)Talla baja
por:
a)Se localiza dentro del meso intestinal b)Más cerca de la 157.¿En una hernia gigante, que debe hacerse en el
válvula ileocecal c)Más cerca del ángulo de Treitz preoperatorio que facilite la cirugía?
d)Parte media del intenso delgado e)Se localiza en colón a)Profilaxis antibiótica b)Hidratación enérgica
ascendente c)Neumoperitoneo progresivo d)Colocación de sonda
rectal e)Administrar coloides IV
148.¿Cuál de los siguientes tipos de hernias sigue el
trayecto del cordón espermático dentro del músculo 158.Traumatismo torácico que posteriormente produce
cremaster? “pulmón blanco” en la radiografía de tórax ¿Cuál es la
a)Hernia femoral complicación?:
b)Hernia inguinal indirecta c)Hernia inguinal directa a)Atelectasia
d)Hernia Obturatriz e)Hernia de Spiegel. b)Hemorragia alveolar difusa c)SDRA
d)Hemotórax
149.¿Qué cantidad de sodio lleva una solución salina e)Edema pulmonar agudo no cardiogénico
fisiológica normal?
a)124 m Eq/L 159.Centro de Salud con médico general , pediatra ,
b)134 m Eq/L ginecólogo, laboratorio, patología ¿A qué nivel de
c)144 m Eq/L atención pertenece?
d)154 m Eq/L a)I-4
e)164 m Eq/L b)I-3
c)I-2
150.Paciente con FEV 62% que tratamiento debe recibir : d)II-1
a)Corticoide b)Cromoglicato sódico e)II-2
c)B2 agonista
d)Bromuro de ratropio e)Teofilina 160.Niño de 5 años a quien un antihistaminico que le
provoca una torsaide de pointes
151.La duración del TIA es : ¿Cuál es el medicamento?
a)36 horas a)Levocetirizina b)Terfenadina
b)48 horas c)Clorfeniramind)Loratadina e)Desloratadina
c)72 horas
d)24 horas 161.Señora que sufre caída con miembro inferior en
e)7 días flexión, aducción y rotación interna
¿Cuál es el diagnóstico más probable? a)Luxación anterior
152.Niño con pancitopenia y visceromegalia ¿Cuál es el de la cadera b)Luxación posterior de la cadera
diagnóstico más probable? c)Luxación congénita de cadera d)Luxación obturatriz
a)Leucemia mielomielocítica crónica b)Leucemia mieloide de cadera e)Luxofractura de cadera
aguda c)Leucemia linfática aguda d)Leucemia
mielógena crónica e)Leucemia linfocítica crónica 162.Buzo que sufre dolor de rodilla y trastorno del
sensorio ¿Cuál es el diagnóstico probable?
153.Lesión típica de la varicela: a)Obstrucción de vasos pequeños.
a)Vesícula b)Pápula c)Ampolla d)Flictena e)Placas b)Rabdomiólisis c)Esguince d)Encefalopatía hipóxica
e)Intoxicación por oxígeno
163.En un paciente con shock séptico ¿qué se debe indicar
inicialmente? 172.Respecto a la enfermedad de Hansen, marque la
a)Dopamina b)Fluidos EV c)Coloides d)Adrenalina alternativa correcta:
e)Dobutamina a)M. Leprae en un bacilo ácido alcohol resistente, fácil de
cultivar
164.¿Cuál es el parásito que causa anemia b)En África son más prevalentes las formas clínicas de
megaloblástica? menor resistencia inmune c)Baciloscopia se obtiene
a)Tenia solium b)Hymenolepis nana c)Diphylobothrium como mínimo de codos, orejas y rodillas
latum d)Necator americano e)Trichuris trichura d)Test de lepromina en caso de lepra tuberculosis
generalmente es negativo
165.Señale la lesión benigna más frecuente en hallazgo de e)En las formas multibacilares se indica tratamiento
hígado: con dapsona, clofacimina y
a)Adenoma b)Hemangioma c)Angiomiolipoma rifampicina por seis meses
d)Hamartoma
e)Hiperplasia nodular focal 173.Respecto a tuberculosis, marque la alternativa
correcta:
a)Profilaxis se indica en inmunosuprimidos y niños, siendo
166.¿Cuál es el muestro para atención de calidad? su duración seis meses aunque en personas con
a)Aleatorio sin restitución b)Aleatorio sistemático infección por VIH se indica por un año
c)Muestreo estratificado b)Cavidades son frecuentes de observar en las lesiones
d)Muestreo por conglomerados pulmonares inmunosuprimidos c)Tuberculosis
e)Ninguna de las alternativas es correcta. raramente es sistémica
d)Hemoptisis es más frecuente
167.Señale el criterio de severidad de depresión : de observar en
a)Duración menor de una semana b)Delución inmunosuprimidos que inmunocompetentes
c)El episodio es atribuible a abuso de sustancias e)El tratamiento es prolongado por ser una bacteria
psicoactivas de crecimiento rápido
d)No presenta alteraciones del sueño de cualquier tipo
e)No presenta cambios del apetito 174.Lactante de 6 meses, inicia con vómitos y fiebre de
38,2 °C. Horas después se agregan deposiciones
168.¿Cuál de las siguientes enfermedades presenta soplo líquidas abundantes sin moco ni sangre. La madre
pansistólico? decide no darle leche ni papillas. Al continuar con
a)CIA vómitos, tornarse irritable y ojos hundidos, es llevado
b)Estenosis aórtica c)PCA a emergencia ¿Cuál es la conducta a seguir?
d)Insuficiencia aórtica a)Hidratación Parenteral b)Hidratación oral
e)CIV c)Hospitalizar para hidratación oral d)Administrar
antibiótico e)Administrar antiemético
169.¿En cuál de los siguientes órganos se encuentra
células epiteliales estratificadas no queratinizadas? 175.Lactante con diarrea , está irritable, con signo del
a)Estómago b)Ileon c)Esófago d)Colon e)Yeyuno pliegue positivo. ¿Cuál es el grado de deshidratación?
a)No deshidratación b)Deshidratación leve
170.En el llamado pulmón blanco u opacificación c)Deshidratación severa d)Deshidratación moderada
completa de los campos pulmonares, la ausencia de e)Shock
broncograma aéreo con borramiento de la silueta
cardíaca corresponde a que grado de la enfermedad 176.¿Qué nervio se lesiona con la fractura diafisiaria de
de membrana hialina: húmero?
a)Grado i b)GradoII c)GradoIII d)GradoIV e)Grado V a)Cubital b)Mediano c)Radial d)Circunflejo
e)Musculocutáneo
171.Respecto a sepsis y shock séptico, marque la
respuesta falsa: 177.Mujer de 34 años, asintomática sin antecedentes de
a)Sepsis se define como síndrome de respuesta sistémica hepatopatía. Al examen: Coloración amarilla de la
inflamatoria más evidencia clínica de un foco de piel, no hepatornegalia. Perfil hepático normal. La
infección seudoictericia se debe a la cantidad incrementada de:
b)Shock séptico se define como sepsis severa más oliguria a)Caroteno b)Bilirrubina c)Cianocobalamina. D)Melanina
c)Cualquier microorganismos (bacteria, hongo, y virus) e)Fenilalanina
pueden iniciar sepsis
d)En sepsis severa hallazgos clínicos 178.La asociación de fiebre con escalofríos. Dolor en
más importantes son oliguria y alteraciones hipocondrio derecho e ictericia, conocida como triada
agudas del estado mental de charcot, es muy sugerente de:
e)Se ha propuesto al Factor de Necrosis tumoral alfa como a)Colangitis aguda
el mediador central de sepsis b)Colangitis esclerosante primaria c)Colecistitis aguda
d)Pancreatitis aguda e)Colelitiasis 188.Surfista de 30 años de edad presenta una lesión
verrucosa en el labio inferior, única, con eritroplaquia
179.Varón 60 años, acude por dolor agudo en ojo en el paladar ¿Cuál es el diagnóstico más probable?
derecho. Al examen: fotofobia y aumento del tamaño a)Queratosis actínica b)Carcinoma basocelular
del globo ocular ¿Cuál es el diagnóstico más c)Carcinoma espinocelular d)Queratosis seborreica
probable? e)Melanoma maligno
a)Conjuntivitis
b)Retinitis c)Glaucoma d)Cataratas e)Queratitis 189.¿Cuál parásito no se trasmite por via fecal oral ?
a)Ascaris lumbricode b)Enterobio vermicularis c)Necátor
180.Mujer de 45 años. Inicio enfermedad hace 1 semana americano d)Himenolepis diminuta e)Taenia saginata
con dolor intenso en hipocondría derecho, náuseas y
vómitos, ictericia, hace dos días fiebre y malestar 190.Varón de 28 años; con tumoración en región inguinal
general. Impresión diagnóstica: derecha hace 2 años, que aumenta con el esfuerzo
a)Hepatitis viral aguda b)Ampuloma físico y desaparece al acostarse. Desde hace una
c)Adenocarcinoma de vía biliar Colangitis d)esclerosante semana la tumoración no desaparece al decúbito
Coledocolitiasis ¿Cómo se define esta condición de la hernia?
a)Estrangulación
181.Son signos y síntomas de acidosis metabólica. b)Recurrencia c)Coercibilidad. D)Deslizamiento
Excepto: e)Incarceración
a)Dolor abdominal agudo b)Respiración de Kussmaul 191.¿Qué alteración ácido base ocurre en cólera?
c)Tetania A)Acidosis metabólica hiperclorémica con HCH3 bajo
d)Bradicardia e)Desorientación b)Alcalosis metabólica hipoclorémica
c)Acidosis metabólica hipoclorémica con HCO3 bajo
182.La dermatitis seborreica es un proceso de naturaleza: d)Alcalosis metabólica hiperclorémica
a)Infecciosa b)Metabólica c)Constitucional d)Autoinmune e)Acidosis metabólica con AGAP elevado
e)Tumoral
192.¿Cuál de las siguientes alteraciones puede dejar
183.Mejor técnica para enseñar habilidad: lesiones neurológicas?:
a)Observación b)Simulación c)Reforzamiento a)Hiperkalemia b)Hipokalemia c)Hipocalcemia
d)Reestructuración cognitiva e)Tutorías entre iguales d)Hipofosfatemia e)Hipoglicemia

184.Gestante con pelvis estrecha, el ponderado fetal es de 193.Anciana postcolecis- tectomizada presenta dolor y
4.3kg ¿Cuál es el manejo más adecuado? distensión abdominal, además náuseas y vómitos
a)Cesárea de urgencia b)Parto vaginal c)Cesárea electiva ¿Cuál es el diagnóstico más probable?
d)Observación a)Pancreatitis aguda b)Suboclusión intestinal
e)Inducción con oxitocina c)Intusucepción
d)Ileo etabólico e)Ileo biliar
185.Paciente varón de 41 años con
malestar, palpitaciones, con un 194.¿Cuál es el área de superficie corporal de un paciente
patrón electrocardiográfico. Ritmo no quemado a nivel de la cara y 2 manos?
sinusal, no hay ondas P, solo las llamadas f rápidas y a)13%
polimorfas. Las distancias RR son desiguales. La b)10%
morfología del QRS es normal. A cuál de las siguientes c)15%
entidades corresponde. Marque la correcta: d)23%
a)Fibrilación Auricular b)Flúter auricular e)6%
c)Bloqueo auricular – ventrículo (A-V) completo
d)Bradicardia severa 195.¿Cómo se denomina a la protrusión por fondo de saco
e)Taquicardia ventricular de Douglas?
a)Enterocele b)Cistocele c)Rectocele d)Proctocele
186.El Neumotórax a tensión se produce por: e)Quiste de Gardner
a)Bullas Subpleurales b)TBC
c)Respiración mecánica d)Asma bronquial 196.¿Cuál es el objetivo de la maniobra de Pringle?
e)Ninguno de las alternativas. a)Detección de sangrado
b)Prevenir infecciones
187.Reflejo corneal abolido ¿qué nervio está lesionado? c)Control del sangrado
a)VII par craneal b)III par craneal c)II par craneal d)V par d)Prevenir daño del tejido
craneal e)Prevenir del hilio
e)I par craneal
197.Varón de 69 años, que luego de ingesta alcohólica
presenta hematemesis, melena y debilidad. Al
examen: PA: 80/50 mm Hg, FC: 110 x', sudoroso,
pálido y ruidos hidroaéreos aumentados. ¿Cuál es el 3º EXAMEN ENAN
tratamiento inicial?
a)Sucralftato 1¿Cuál es el medicamento inicial de elección en un cuadro
b)Suero fisiológicon EV convulsivo de emergencia?
c)Antiácidos a) Fenobarbital
d)Transfusión de Sangre b) Fenitoina
e)Inhibidores de bomba de protones c) Diazepam
d) Ácido valproico
198.Primigesta de 37 semanas que ingresa por RPM de 12 e) Tiopental
horas de evolución, sin evidencia de contracciones
uterinas, ni signos de infección amniótica. Al examen: 2.Una mujer de 25 años acude a consulta por poliuria y
feto en cefálica, índice de Bishop: 7 puntos, no signos polidipsia. Hasta el momento las investigaciones
de sufrimiento fetal. ¿Cuál es la conducta a seguir? excluyen las causas psicógenas y diabetes. Se realiza
a)Corticoterapia e inducción del parto a las 48 horas prueba de restricción de líquidos. Al final de la misma
b)Antibioticoterapia y esperar el comienzo la osmolalidad urinaria es 240 mosm/kg y el nivel
espontáneo del parto sérico de hormona antidiurética(ADH) se encuentra
c)Cesárea electiva elevado.
d)Esperar el inicio espontáneodel parto ¿Cuál de los siguientes diagnósticos es el más probable?
e)Inducción del parto con oxitócina a) Defecto de la corteza suprarrenal
b) Diabetes insípida nefrógena
199.En niños mayores de 5 años. ¿Cuál el agente c) Diabetes insípida central
etiológico más frecuente de Neumonía adquirida en d) Necrosis tubular aguda
la comunidad? e) Enfermedad de Addison
a)Haernophilus influenza tipo 6
b)Moraxelia catarrhaiis 3.Mujer 54 años, G: 8 P: 8008 con antecedente de partos
c)Staphylococcus aureus domiciliarios. Acude por presentar pérdida de orina
)Streptococcus pneumoniae cuando ríe, tose o al subir gradas. ¿Cuál es el
e)Chlamydia trachomatis diagnóstico más probable?
a) Incontinencia de urgencia
200.Gestante de 34 semanas, hospitalizada 7 días por b) Incontinencia de esfuerzo
RPM, que recibió antibioticoterapia. A la fecha sin c) Incontinencia oculta
signos clínicos de infección. Laboratorio: Proteína C d) Prolapso genital
reactiva 12 mg/L, líquido transcervical: leucocitos 7 x e) Infección del tracto urinario
c y bacterias 6 x c. ¿Cuál es la conducta a seguir?
a) Continuar antibioticoterapia 4.¿Qué hernias tienen mayor riesgo de estrangulamiento?
b) Conducta expectante a) Femorales
c)Administrar corticoides b) Inguinales indirectas
d) Culminar la gestaciòn c) Inguinales directas
e) Realizar amnioinfusión d) Spiegelianas
e) Umbilicales
1.C 21.A 41.C 61.C 81.C 101.D 121.B 141.B 161.B 181.C
2.E 22.B 42.D 62.E 82.B 102.C 122.C 142.D 162.A 182.D
3.D 23.C 43.C 63.E 83.D 103.D 123.B 143.C 163.B 183.B 5.Lactante de un año, con antecedente de prematuridad.
4.E 24.B 44.B 64.D 84.C 104.C 124.C 144.E 164.C 184.C Cursa con palidez de piel y mucosas, inapetencia e
5.A 25.A 45.A 65.C 85.C 105.C 125.A 145.C 165.A 185.A
irritabilidad; no recibió aporte de hierro. Su
6.C 26.C 46.A 66.A 86.E 106.E 126.C 146.C 166.A 186.A
7.C 27.C 47.B 67.B 87.D 107.B 127.B 147.B 167.D 187.D diagnóstico es anemia ferropénica. En el tratamiento.
8.E 28.B 48.D 68.A 88.C 108.C 128.A 148.B 168.E 188.C ¿Cuál es la dosis recomendada de hierro elemental en
9.B 29.C 49.A 69.E 89.E 109.D 129.D 149.D 169.C 189.C
mg/Kg por día?
10.B 30.D 50.C 70.B 90.D 110.C 130.A 150.C 170.D 190.E
11.C 31.D 51.D 71.A 91.B 111.B 131.D 151.D 171.B 191.A a) 1-2
12.D 32.D 52.E 72.A 92.B 112.A 132.D 152.C 172.E 192.E b) 6-8
13.D 33.A 53.B 73.C 93.D 113.D 133.B 153.A 173.A 193.C
c) 11-12
14.A 34.B 54.E 74D 94.A 114.C 134.D 154.D 174.B 194.A
15.C 35.A 55.E 75.B 95.C 115.D 135.D 155.A 175.D 195.A d) 10
16.B 36.D 56.E 76.B 96.A 116.E 136.D 156.C 176.C 196.C e) 15
17.B 37.B 57.A 77.A 97.E 117.E 137.B 157.C 177.A 197.B
18.A 38.C 58.C 78.D 98.B 118.A 138.C 158.B 178.A 198.E
19.A 39.B 59.B 79.A 99.E 119.C 139.C 159.E 179.C 199.D 6.Gestante a término no controlada, VIH+. Al examen:
20.B 40.B 60.D 80.D 100.B 120.B 140.D 160.B 180.A 200.D altura uterina compatible con embarazo de 38
semanas, sin enfermedad oportunista. Se inicia
tratamiento antirretroviral. ¿Cuál sería la forma del
parto y recomendación para la lactancia?
a) Parto vaginal y lactancia materna
b) Cesárea y fórmula maternizada
c) Parto vaginal y fórmula maternizada
d) Cesárea y lactancia materna a) Dos meses
e) Cesárea y lactancia mixta b) Tres meses
7.Los nódulos de Heberden característicos de la c) Cuatro meses
osteoartritis se ubican en las articulaciones... d) Cinco meses
a) Interfalángicas distales. e) Seis meses
b) Radiocarpianas.,
c) Interfalángicas proximales. 14.Paciente con infección por VIH que presenta signos
d) Metacarpofalangicas. meníngeos ¿Cuál es la etiología más frecuente?
E) Metatarsofalángicas a) Toxoplasmosis
b) Histoplasmosis
8.El antibiótico de elección en otitis media aguda es: c) Criptococosis
a) Gentamicina d) Estreptococo pneumoniae
b) Eritromicina e) Listeria monocitógenes
c) Clindamicina
d) Amoxicilina 15.¿Cuál de los siguientes es considerado un efecto
e) Penicilina V colateral de los anticonceptivos orales?
a) Mialgia
9.Varón de 72 años con fibrilación auricular y alcoholismo b) Condritis
crónico. Es traído porque desde hace 1 año presenta c) Hipermenorrea
deterioro cognitivo y algunos déficit focales. Al d) Dismerionea
examen PA: 130/70 mm Hg, FC: 87 F R: 17 x', e) Mastalgia
despierto, orientado parcialmente, Babinski y
Hoffrnan izquierdo positivos. ¿Cuál es la causa de 16.¿Qué elementos debe recibir el lactante como
demencia más probable? suplemento, por no recibir leche materna en
a) Encefalopatia de korsakoff cantidades adecuadas?:
b) Enfermedad de Alzheimer a) Proteínas, calcio, yodo
c) Encefalopatia de wernicke b) Hierro, Vit, D, Vit K
d) Multiinfarto c) Grasas, agua, Vit C
e) Alcoholismo crónico d) Yodo, Zinc, hierro
e) Ninguna de las alternativas es correcta.
10.Varón de 18 años, con herida cortante en muñeca
derecha por asalto con arma blanca. Presenta 17.Varón de 71 años, hipertenso, acude por precordalgia
hipoestesia y postura en hiperextensión intensa, EKG: supradesnivel del 17.segmento ST en D
metacarpofalángica del 4to. y 5to. dedo. ¿Cuál es el II, D III, aVF e infradesnivel ST en V1, ¿Cuál es el
nervio lesionado? diagnóstico más probable?
a) Mediano a) IMA STE diafragmático
b) Radial b) IMA STNE posterior
c) Cubital c) IMA STE posterior
d) Musculocutáneo d) IMA STE inferoposterior
e) Circunflejo e) IMA cara anterior

11.Varón de 53 años, con tuberculosis pulmonar BK 18.Lactante de 9 meses que hace dos días presenta
positivo y en tratamiento con esquema 1, que hiporexia y fiebre. Al examen: T°: 39.3°C, hidratado,
presenta alteraciones de la visión de colores. ¿Cuál de activo, no luce tóxico. Laboratorio: leucocitosis sin
los siguientes fármacos produce con mayor desviación izquierda y sedimento de orina con 30
frecuencia este efecto adverso? leucocitos por campo. ¿Cuál es la conducta a seguir?
a) Pirazinamida a) Repetir sedimento de orina, urocultivo, antibiótico
b) Isoniacida condicional a resultados
c) Rifampicina b) Urocultivo inicial, antibioticoterapia empírica y
d) Estreptomicina reevaluación con resultados
e) Etambutol c) Hospitalización, urocultivo, antibiótico terapia empírica
d) Antibioticoterapia empírica, antipiréticos, urocultivo
12.¿Cuál de las siguientes leucemias es más frecuentes en control en 7
niños? e) días Hemocultivo, urocultivo, antibiótico condicional a
a) Linfoblástica resultados
b) Mieloblásticas 19.Cuando existe una correlación entre dos variables
c) Monoblástica cuantitativas continuas. ¿Cuál es el modelo predictivo
d) Mieloide crónica que debe aplicarse?
e) Todos por igual a) Regresión múltiple
13.El reflejo de moro se observa normalmente hasta la b) Regresión de Cox
edad de: c) De efectos fijos
d) Dispersión e) TAC abdominal
e) Regresión lineal
26.Varón de 60 años, con disnea progresiva desde hace 2
20.¿Cuál es la técnica que se utiliza para el control de horas. Al examen: FR: 34 x', presenta cianosis perioral
sesgo de selección? y distal, sibilantes diseminados en ambos campos
a) Pareamiento pulmonares, escasos roncantes. Laboratorio:
b) Aleatorización Leucocitos 12,368, pH: 7,42, PCO2: 28, P02: 68, HCO3:
c) Estratificación 21. Mejora después de administrar p2 adrenérgicos.
d) Enmascaramiento ¿Cuál es el diagnóstico más probable?
e) Medidas repetidas a) Tromboembolia pulmonar
b) EPOC descompensado
21.En relación con la bilirrubina RN, marque la correcta: c) Insuficiencia respiratoria tipo II
a) No hay riesgo de neuro-toxicidad con d) Crisis asmática
adecuados valores séricos de albúmina e) Alcalosis metabólica e hipoxemia
b) El pH bajo dificulta la precipitación de la bilirrubina a
nivel cerebral. 27.Mujer de 45 años, con esquizofrenia paranoide y HIV
c) Hay más riesgo de Kernicterus positivo, acude al hospital por convulsiones y
en prematuro. trastorno del sensorio. Al examen: Glasgow 9, signos
d) Tiene efectos antioxidantes a niveles altos. meníngeos (+). LCR: levaduras, positivo a tinta china.
e) Hay excelente correlación entre la clínica y los valores ¿Cuál es el tratamiento a seguir?
séricos. a) Fluconazol
22.El puerperio inmediato comprende: Las primeras dos b) Itraconazol
a) horas post parto Las primeras 12 c) Anfotericina B
b) horas post parto d) Ketoconazol
c) Las primeras 24 horas post parto e) Caspofungina
d) Las primeras 48 horas post parto
e) La primera semana post parto 28.Varón de 66 años, diabético, acude por fiebre,
malestar general, tos con expectoración verdosa. Al
23.Mujer de 43 años, acude por convulsiones tónico- examen: PA: 80/50 mm Hg, FC: 110 x', FR: 35 x', MEG,
clónicas generalizadas en 4 oportunidades desde pulsos débiles y oliguria. Laboratorio: leucocitosis
hace 35 minutos, sin recuperación de la conciencia. 16000 x' lactato 1,4 U/L. Se realiza reto de fluidos y se
Durante la evaluación nuevamente se repite el observa estabilización de la PA, mejora el rango
cuadro descrito ¿Cuál es el tratamiento? diurético, ¿cuál es el diagnóstico?
a) Diazepam: 10 mg EV, carga de Fenitoina a 20 mg/k g a) Sepsis grave, foco respiratorio
peso a una infusión máxima de 50 mg/min. b) SIRS e injuria renal aguda
b) Midazoiam 5 mg, fenobarbital EV diluido lento, infusión c) Shock séptico foco respiratorio
más de 50 mg/ d) Sepsis foco urinario
d) Propofol EV, Fenitoina bolo de 25 mg/kg peso, infusión e) Shock séptico foco urinari o
máxima de 75 mg/minuto
c) Diazepam 10 mg EV, seguido de Midazolam en infusión 29.En la fractura de Monteggia hay:
e) Diazepam 5 mg EV, carga de 800 mg de Fenitoina a) Fractura del 1/3 distal + luxación de cabeza de radio
diluida en dextrosa al 5% b) Fractura del 1/3 proximal del cubito + luxación
anterior de cúpula radial
24.Varón de 40 años acude por dolor anal, progresivo e c) Fractura de cubito y radio + luxación de codo
invalidante, de dos días de evolución que se d) Fractura de 1/3 medio del radio + luxación del codo
incrementa al sentarse y caminar. Al examen: e) Ninguna de las alternativas.
tumoración renitente con signos de flogosis en el
margen anal, ¿Cuál es el tratamiento inicial? 30.A una paciente de 70 años de edad se le detecta
a) AINEs irregularidad de la frecuencia cardiaca en una
b) Antibloticoterapia evaluación de rutina. No ha presentado nuevos
c) Baños de asiento síntomas en reposo o asociados a los esfuerzos. En el
d) Corticoterapia ECG, no se documentan ondas P y se encuentra un
e) Drenaje quirúrgico intervalo RR irregular con una frecuencia de 70/ min.
25.Varón de 22 años que en reyerta sufre agresión con En su ECG de hace cuatro años tenía ritmo sinusal
cuchillo. Al examen: PA: 110/70 mm Hg, FC: 80 x', FR. ¿Cuál de las siguientes conductas es la más adecuada
18 x', herida de 4cm en flanco izquierdo, con dolor en el manejo de la paciente?
local y rebote negativo. ¿Cuál es la conducta inicial? a) Cardioversión
a) Laparotornia exploradora b) Antiarrítmicos
b) Exploración de herida c) Bloqueador Beta
c) Laparoscopia exploradora d) Anticoagulación
d) Transfusión urgente e) Ácido acetilsalicílico
¿Cuál de las siguientes características histológicas es
31.A los antioxidantes se les conoce como tales al Prevenir típica de esta condición?
o retardan el envejecimiento: a) Cambios inespecíficos
a) Porque protege el S.N.C. b) Depósitos de inmunoglobulina A (Ig
b) Porque protege al sistema hormonal c) Lesiones dentro de la epidermis (acantólisis)
c) Porque bloquea los electrones impares de los radicales d) Lesiones de la membrana basal
libres e) Depósitos de inmunoglobulina M (IgM)
d) Ninguna de las alternativas.
e) Todas las alternativas son correctas. 37.La ulcera péptica es una enfermedad multifactorial de
curso crónico y periódico. De los siguientes factores
32.¿Cuál es el proceso inicial de la hemostasia? diga ¿cuál es el más importante para su desarrollo?
a) Tapón hemostático temporal a) AINE
b) Vasoconstricción de la arteriola hemostático definitivo b) Infección por helicobacter pylori
c)Tromboplastina hística c)Alcohol
d) Activación de la coagulación d)Tabaco
e)Tapón e)Hiperclorhidria

33.Lactante de 6 meses inicia hace 7 días cuadro de resfrió 38.Una mujer de 68 años con hipertensión arterial y
común, hace dos días se agrega fiebre de 39°C. Hace dislipidemia presenta dolor estroesternal de 30
un día se agrega dificultad respiratoria. Al examen: minutos de duración que se irradia al cuello. Esta
polipnea, crepitantes y roncantes en ambos campos sudorosa y con moderada angustia. El ECG muestra
pulmonares. ¿Cuál es el diagnóstico más probable? una elevación del segmento ST en las derivaciones de
A) Neumonítis viral la cara inferior. ¿Cuál de los siguientes mecanismos es
b) Neumonía bacteriana la causa más probable de esta entidad?
c) Bronquiolitis a) Rotura de una placa coronaria
d) Neumonía complicada b) Aortitis
e) Neumonía atípica c) Inflamación pericárdica
d) Vasculitis
34.Varón de 28 años con cuadro febril de 3 semanas de e) Miocarditis
evolución, sudor nocturno, artralgias y dolor en
cadera bilateral. Refiere comer queso fresco hecho 39.Un varón asintomático de 64 años, ejecutivo de
tradicionalmente. Si se sospecha de Brucellosis, ¿Cuál profesión, que asiste a su chequeo regular anual.
es la prueba de screeníng recomendada? Tuvo un infarto de miocardio anterior hace cuatro
a) 2-Mercaptoetanol años y presenta actualmente en el EKG, supradesnivel
b) Aglutinación en lamina del ST en V3-V6 ¿Cuál es su diagnóstico?
c) Aglutinación en tubo a) Pericarditis calcificada
d) Hemocultivo b) Aneurisma de la pared ventricular
e) Rosa de Bengala c) Quiste hidatídico
d) Pleuropericarditis
35.Multípara de 40 años que una hora después de e) Normal
alumbramiento de su último parto, presenta pérdida
sanguínea vaginal no muy abundante, pero la PA cae 40¿Cuál de los siguientes mecanismos describe mejor el
a 80/50 y el pulso se acelera a 110/m, mostrando efecto de los iones del calcio sobre el miocardio?
sudoración aunque sin perder la conciencia. No a) Inotropismo positivo
refiere dolor al hacer la palpación del abdomen, no se b) Inotropismo negativo
encuentra útero, el examen rápido de la región c) Cronotropismo positivo
vulvoperineal no evidencia desgarros. Cuál de las d) Cronotropismo negativo
siguientes presunciones diagnósticas es la más e) Desacoplamiento de la excitación – contracción
probable:
a) Desgarro perineo vaginal de 4to grado 41¿Cuáles son los componentes de la calidad de los
b) Coagulación intravascular diseminada servicios de salud?
c) Inversión uterina probablemente parcial a) Organizacional, técnico y humano
d) Desgarro cervical b) Científico y técnico
e) Retención total de placenta c) Administrativo y financiero
d) Humano, financiero y gerencia!
36. Una mujer de 64 años notó lesiones del tipo e) Técnico, científico y financiero
ampollosas en sus muslos y axilas. Eran pruriginosas,
pero no dolían. No tenían otros síntomas. En la 42.¿Que significa CREST ?
exploración las lesiones son grandes, a tensión, en a) Calcinosis, Raynaud, endocrinopatía, telagiectasia
forma de ampollas de contenido seroso. La biopsia b) Calcinosis, Raynaud, endocrinopatía y telagiectasia
confirmó el diagnóstico de Penfigoide ampolloso.
c) Calcinosis, Raynaud, alteración del esófago, 48.En qué tipo de episiotomía es más frecuente el
esclerodactilia y telagiectasia desgarro perineo vaginal de 4to grado:
d) Calcinosis, Raynaud, alteración del esófago, a) El medio lateral izquierdo
esclerodermia y telagiectasia b) En la medio lateral derecha
e) Calcinosis, Raynaud, alteración del esófago, c) En la mediana con peritoneo corto
esclerodermia y Tremor d) En la medina con perineo grande y amplio
e) En la cesareada anterior
43.Prematuro de 28 semanas con Displasia
Broncopulmonar en ventilación mecánica por más de 49. Los siguientes son factores de riesgo para la pre
4 semanas con Fi02 > 50%. Para disminuir las eclampsia–eclampsia, con una excepción. Márquela:
complicaciones por el uso de ventilador y retirado a) Historia personal de hipertensión arterial Primigravidez
precozmente, se decide mantener una PaO2 arterial b) Diabetes mellitus
más permisiva. ¿Con qué rango de pCO2 en mm Hg c) Enfermedad trofoblástica gestacional
manejaría este paciente? d) Embarazo prolongado
a) 40 a 49
b) 30 a 39 50.Se considera EHG (Enfermedad hipertensiva de la
c) 76 a 85 gestión) cuando se encuentra:
d) 50 a 75 a) Hipertensión, más proteinuria después de las 20
e) 20 a 29 semana gestacional
b) Hipertensión, edema y aumento exagerado de peso
44.Lactante de 7 meses, con fiebre de 39 °C de 3 días de después de las 20 semana gestacional
evolución, un c) Hipertensión, anasarca y poliuria después de las 20
vómito por día y deposiciones normales. Rechaza semana gestacional
alimentos semisólidos pero lacta bien. Al examen: T: d) Hipertensión, cefalea, tinitus después de la 20 semana
39 °C. Hemodinámicamente estable, mucosa gestacional
orofaringea normal. MV presentes en ambos campos e) Hipertensión, hipoproteinemia y trombocitopenia
pulmonares y piel sin lesiones, ¿Cuál es el diagnóstico después de la 20 semana gestacional
más probable? 51.Neonato de 15 días con lactancia materna exclusiva y
a) Faringitis aguda diagnóstico de galactosemia
b) Fiebre tifoidea ¿Cuál es la conducta a seguir?
c) Infección del tracto urinario a) Puede recibir lactancia mixta con fórmula maternizada
d) Mononucleosis infecciosa b) Puede recibir lactancia materna de manera relativa
e) Diarrea aguda c) Debe recibir fórmula maternizada e iniciar ablactancia
d) Suspender de manera absoluta la lactancia materna
45. En relación a las inmunizaciones. ¿Cuál es la actitud e) Puede recibir una fórmula extensamente hidrolizada.
médica frente a un niño con asplenia funcional o 52.Mujer de 30 años acude por dolor anal intenso al
anatómica? defecar. Antecedente: constipación crónica. Al
a) Sólo puede recibir vacunas inactivadas examen: tacto rectal doloroso, esfínter hipertónico y
b) Al vacunar con agentes atenuados pueden hemorroide centinela. Anoscopia: úlcera lineal media
multiplicarse posterior. ¿Cuál es el tratamiento?
c) Debe evitarse la vacuna antineumocócica a) Esfinterotomía interna lateral
d) Están indicadas las inmunizaciones b) Esfinterotomia externa lateral
e) No debe recibir la vacuna antimeningocócica c) Ligadura de paquete hemorroidal
d) Curetaje de la úlcera
46.Varón de 69 años en UCI, con neumonía, que presenta e) Cauterización de la úlcera
al 7mo día de evolución Murphy (+) ¿Cuál' es el
diagnóstico más probable? 53.El LCR en la meningoencefalitis bacteriana se
a) Coliecistitis aguda iitiasica caracteriza:
b) Coliecistitis aguda reagudizada a) Hipoglucorraquia, hipoproteinor raquia y lactato mayor
c) Colecistitis crónica de 3.9 mEq/L
d) Colecístitis aguda alitiasica b) Hipoglucorraquia, hiperproteinor raquia y lactato
e) Colecistitis crónica reagudizada menor de 1.6 mEq/L
47¿Cuál es la reacción adversa más frecuente en el c) Hipoglucorraquia, hiperproteinorraquia y lactato mayor
tratamiento de la onicomicosis? de 3.9 mEq/L
a) Síndrome de Stevens – Johnson d) Ninguna de las alternativas es correcta .
b) Hepatotoxicidad e) Todas las alternativas son correctas.
c) Nefritis intersticial 54¿En el caso supuesto que su respuesta hubiese sido
d) Hemorragia digestiva cáncer de próstata, cuál de los siguientes análisis de
e) Dermatitis exfoliativa laboratorio hubiese sido de mayor utilidad?
a) Fosfatasa alcalina
b) Fosfatasa acida
c) Antígeno específico prostático 61.Varón de 32 años que ha presentado matarla por
d) Antígeno carcinoembrionario Plasmodium vivax, para prevenir la recaída, ¿cuál es
e) Gonadotropinas coriónicas su indicación terapéutica?
a) Cloroquina 500 mg. X 4 semanas
55.La transición demográfica en el Perú se relaciona con b) Artesunato 1200 a 1600 mg. Vía oral x 3 días
la disminución de la... c) Primaquina 30 mg x día x 14 dlaa
a) Razón de dependencia. d) Quinidina 10 mg/kg, hasta un máximo de 600 mg. En
b) Densidad poblacional solución salina x 2 semanas
c) Tasa global de fecundidad e) Usar mosquiteros
d) Tasa de migración interna.
e) Esperanza de vida al nacer. 62.La presencia de secreción vaginal amarillenta,
espumosa y lesiones eritematosas en cérvix nos da la
56. En una comunidad con servicios de agua a domicilio y sospecha de infección por:
alcantarillado, pero no en el colegio, se ha detectado a) Trichomonas vaginalis
incremento de casos de diarrea aguda en los b) Candida albicans
estudiantes, en este contexto, el personal del Centro c) Gardnerella
de Salud ha planificado realizar las siguientes d) Chlamidya trachomatis
acciones: capacitación a docentes del colegio, e) Neisseria gonorrheae
coordinación con la Municipalidad y el sector
educación para construir servicios sanitarios 63.Mujer de 30 años con G: 3 P: 3003 y FUR hace 1
adecuados (mientras tanto, colocar letrinas). Estas semana, acude a emergencia por dolor de gran
actividades corresponden a: intensidad en bajo vientre y fiebre de 39 °C a más. Al
a) Promoción tacto vaginal: dolor a la movilización de cérvix, masa
b) Prevención palpable en anexo izquierdo y flujo vaginal de regular
c) Gestión volumen. ¿Cuál es el diagnóstico más probable?
d) Organización a) Embarazo ectópico
e) Recuperación b) Absceso pélvico
57¿Cuál de los siguientes tratamientos se considera el c) Apendicitis aguda complicada
primer paso para el manejo inicial de las fracturas d) Pielonefritis aguda
abiertas o expuestas? e) Diverticulitis
a) Fijación con tutores externos
b) Desbridamiento urgente 64.Gestante de 41 semanas G: 2 P: 1001, El embarazo ha
c) Inmovilización endomedular transcurrido normalmente. Al examen: feto en
d) Inmovilización con placas y tornillos cefálico, test no estresante reactivo y Bishop
e) Inmovilización con aparato de yeso favorable. ¿Cuál es la conducta a seguir?
58.Varón de 70 años con dolor de espalda y extremidades a) Esperar hasta que inicie el trab ajo de parto espontaneo
al deambular, astenia de cinco meses de evolución. Al b) Maduración cervical
examen: PA 130/80 mm Hg, FC: 78 x', FR: 20 x', T°: c) Inducir el parto
37.5 °C. Palidez de piel, hepatoesplenomegalia, d) Administrar corticoides e inducir el parto pasadas las 48
Laboratorio: anemia, hipercalcemia, azoemia, horas
¿Cuál es el diagnóstico más probable? e) Hacer cesárea electiva.
a) Leucemia mieloide crónica
b) Leucemia linfática crónica 65.Mujer de 25 años, acude por dolor abdominal en FID,
c) Linterna de células hace 7dias, se auto medica con ciprofloxacino y
d) Mieloma múltiple paracetamol VO. Al examen: PA: 80/50 mm Hg, FC:
e) Leucemia de células plasmáticas 120, FR: 24, T°: 38°C. Piel pálida, sudorosa, escleras
59.¿Cuál de las siguientes enzimas plasmáticas ictéricas Abdomen: distendido, RHA disminuidos, Mc
diagnóstica enfermedad obstructiva biliar? Burney positivo, rebote positivo. El diagnóstico es
a) Gamma-glutamil transferasa apendicitis aguda complicada con peritonitis
b) Transaminasa glutámico-oxalacetica generalizada con indicación quirúrgica. En el post
c) Deshidrogenasa isocítrica operatorio continua ictérica, ¿Cuál de las siguientes
d) Acetilcolinesterasa patologías explica ia ictericia?
e) Leucina aminopeptidasa a) Hepatitis
60.La tuberculosis de primoinfección es más frecuente de b) Pancreatitis
a) Piel c) Pi leflebitis
b) Estómago d) Colangitis
c) Pulmón e) Iatrogenia
d) Ganglio linfático
e) Intestino delgado 66¿Cuál es un principal criterio para endometritis?
a) Fiebre
b) Dolor pélvico
c) Secreción vaginal maloliente b) Arteria aorta
d) Sangrado vaginal c) Arteria renal
e) Estreñimiento d) Arteria colónica
e) Arteria iliaca
67.¿Cómo se denomina la alteración de la fusión de los
pliegues uretrales ? 74. En la enfermedad de Parkinson ¿qué neurotransmisor
a) Hipospadia se altera?
b) Agenesia del genital externo a) Aspartato
c)Pene bífido b) Dopamina
d) Micropene c) Glutamato
e) Hidrocele d) Ácido g-aminobutírico (GABA)
68.Paciente que presenta dolor precordial de 5 minutos e) Serotonina
que cede al reposo .¿Cuál es el diagnóstico más 75.¿Cuál es la mordedura más contaminada?
probable? a) Gato
a) Angina inestable b) Conejo
b) Angina estable c) Hamster
c) Infarto de miocardio d) Humana
d) Angina de Prinzmetal e) Canina
e) Osteocondritis
76.Recién nacido de 32 semanas y 1800 gramos de peso,
69.Después de la penetración de un espermatozoide en el a las 48 horas de vida presenta ictericia,
óvulo, se produce la intensificación del metabolismo hepatoesplenomegalia y lesiones cutáneas de
del huevo. ¿Cuál es la causa del incremento del aspecto petequial en el hemograma se aprecia
metabolismo oxidativo? anemia con eritroblastosis y trombocitopenia, en la
a) Aumento de la necesidad de oxígeno y su consumo radiografía de cráneo se observan calcificaciones
b) Disminución del metabolismo de carbohidratos cerebrales periventriculares. Que diagnostico le
c) Disminución del contenido de aminoácidos libres parece más probable:
d) Disminución de la actividad de fermentos proteolíticos a) Rubeola congénita
e) Aumento de la síntesis de ácidos grasos b) Citomegalovirus congénita
c) Sepsis neonatal
70. Lactante que rueda de prono a supino y se sienta sin d) Enfermedad hemolítica del recién nacido
soporte. ¿Cuál es su edad en meses? e) Sífilis congénita
a) 4
b) 5 77.Varón de 60 años, con masa dolorosa en fosa iliaca
c) 6 derecha y fiebre de 7 días de evolución, ¿Cuál es el
d) 2 diagnóstico más probable?
e) 3 a) NM de ciego
b) Absceso de pared abdominal
71.Recién nacido de 14 días de edad, P: 3700 gr, con c) Hernia de Spiguel Hernia Spiguel
lactancia materna exclusiva, ginecoobstetra, refiere d) Tumor carcinoide
que la madre tiene BK (++). El neonato ha ganado 120 e) Plastrón apendicular
gr de peso, luce activo, al examen físico es normal y
no ha recibido BCG, ¿Cuál es su Dx y conducta con el 78.Mujer de 56 años asintomática acude a su control
neonato? mensual de diabetes mellítus tipo
a) Contacto, aplicar BCG 2. Al examen: no anormalidades. Laboratorio: glucosa:
b) Infección, RX Tórax, Hemograma. 129mg/d1., urocultivo: E. coll
c) Infección, aplicar PPD >100,000 UFC/ml, ante estos resultados. ¿Cuál es la
d) Contacto, INH 18.5 mg/día vo conducta a seguir?
e) Contacto, suspender lactancia materna, dar sucedáneo a) Jugo de Cramberry diario
de la LM 13% 90cc cada 3 horas. b) Observacion
72. La incidencia más alta en que se producen los c) Norfloxacino 400mg / 2 veces al día por 5 días
accidentes por cuerpo extraño en niños es: d) Nitrofurantoína 100mg / d ía como profilaxis por 3
a) A los 4 años de edad. meses
b) A los 36 meses de edad. e) Nitrofurantoina 100 mg cada 6 horas como tratamiento
c) A los 14 meses de edad. por 3 días
d) Cuando el niño diferencia el día y la noche
e) Cuando el niño deja de orinarse en la cama. 79.El fármaco que produce fibrosis pulmonar es:
a) Bleomicina Busulfan
73. Lesión punzocortante a la altura de L1 paravertebral b) Methotrexate
derecha ¿Qué arteria puede lesionarse? c) Ciclofosfamida
a) Arteria hepática d) Todas las alternativas son correctas.
d) Fenobarbital + diazepan de acuerdo a crisis convulsiva
80.¿Cuál de las siguientes drogas en el tratamiento e) Carbamazepina + ácido valpróico por dos años
específico de actúa inhibiendo a la ARN polimerasa? 87.¿Cuál es la causa más frecuente de hipertiroidismo?
a) Rifampicina. a) Adenoma tóxico
b) Etionamida b) Bocio multinodular tóxico
c) Estreptomicina c) Tiroiditis de Hashimoto
d) Isoniazida d) Tiroiditis sugaguda de De Quervain
e) Etambutol e) Enfermedad de Graves Basedow
81.Con respecto al tiempo de sangría marque lo correcto: 88.¿Por dónde pasa el impulso al ventrículo?
a) Evalúa los factores de coagulación a) Has de Bachman
b) En pacientes hemofílicos se encuentra prolongado b) Has de Wenckebach
c) En trombocitopenias severas está prolongado c) Has de Thorel
d) En los defectos de agregación plaquetaria, se encuentra d) Has de Hiss
en valores normales e) Fibras de Purkinje
e) En pacientes anticoagulados el tiempo de sangría se
encuentra prolongado 89.Sangrado rojo vinoso en tercer trimestre, escaso, con
contractilidad uterina frecuente
82.Con respecto al estudio del L.C.R: ¿Cuál es el diagnóstico probable?
a) La celularidad normal es menor a 05 leucocitos por ml a)Rotura uterina
b) Normalmente encontramos predominancia b)Placenta previa
mononuclear c)Rotura de vasa previa
c) En las meningitis asépticas puede existir d)Desprendimiento prematuro de placenta
hipoglucorraquia e)Placenta de inserción baja
d) Un valor de más de 200 mg/dl de proteínas me sugieren
una meningitis séptica 90.En un hijo de madre con rubeola ¿Cuál es la cardiopatía
e) En enfermedad hemorrágica meníngea no se encuentra congénita más frecuente?
aumento de proteínas en el L.C.R a) CIV
b) CIA
83. Adolescente de 15 años con diagnóstico de depresión c) PCA
severa e intento de suicidio. d) Transposición de grandes vasos
¿Cuál es el tratamiento inicial? e) Tetralogía de Fallot
a) Lamotrigina
b) Haloperidol 91.Gestante multípara, con embarazo doble de 36
c) Clorpromazina semanas quien llega en periodo expulsivo. Luego de
d) Sertralina atender el parto en cefálica del primer producto se
e) Amitriptilina evidencia que el segundo se halla en situación
transversa con el dorso superior. ¿Cuál es la actitud
84.Hijo pretérmino de madre RPM con signos de más apropiada?
obstrucción y sangrado al defecar ¿Cuál es el a) Versión externa
diagnóstico más probable? b) Cesárea de emergencia
a) Neumoperitoneo c) Conducta expectante
b) Neumoretroperitoneo d) Tracción cefálica
c) Enterocolitis necrotizante e) Versión interna
d) Trombosis mesentérica
e) Aire en la vena porta 92.Una mujer de 63 años presenta angina de esfuerzo y
dos episodios de síncope en el pasado. En la
85. Niño de 3 años que vive en una institución para exploración se detecta un soplo sistólico con
menores abandonados y al que se le ha diagnosticado irradiación a las carótidas y un S2 suave, ¿Cuál de los
MEC por gérmenes Gram negativos. Usted le indicaría siguientes diagnósticos es el más probable?
tratamiento ATB durante: a) Estenosis mitral
a) Dos semanas como máximo b)Insuficiencia mitral
b) Una semana es lo ideal. c)Estenosis aórtica
c) 10 días d)Insuficiencia aórtica
d) Seis semanas e)Estenosis tricúspidea
e) Tres semanas mínimo 93.Una niña de 4 años es llevada al médico por un cuadro
de inquietud, intenso prurito anal nocturno y discreto
86. El tratamiento de mantenimiento de las convulsiones prurito vulvar. ¿Cuál es el diagnóstico?
febriles es: A. Teniasis.
a) Fenobarbital por 2 años. B. Oxiuriasis.
b) Ácido valproico por 6 meses C. Ascariasis.
c) No se debe dar tratamiento generalmente D. Trichiuriasis.
E. Eccema del pañal. 98.¿Cuál es una característica del Haemophilus
influenzae?
94.Una niña de 6 meses de edad es traída donde su A. La inmunización no ha reducido la incidencia de
pediatra por presentar incremento en la frecuencia infecciones por ese agente.
de las deposiciones durante las últimas dos semanas. B. La máxima incidencia se observa en niños mayores de
Sus heces son olorosas, “sueltas”, y flotan en el agua. 5 años.
Su madre refiere que después de darle de comer C. Los gérmenes más virulentos no pertenecen al
elimina flatos en exceso. Tiene como antecedente un serotipo B.
episodio de obstrucción meconial a las 24 horas de D. Forma parte de la flora respiratoria normal en el 60-
nacida. Su alimentación actual consiste en leche en 90% de niños sanos.
fórmula y ocasionalmente arroz. El examen físico E. Es un coco bacilo grampositivo.
revela un peso actual por debajo del percentil 5 para 99.Un hombre de 40 años, turista extranjero, es remitido
la edad. El examen de Thevennon en heces y cultivos a un centro de Urgencias por bajo nivel de conciencia,
resultan negativos. ¿Cuál de los siguientes exámenes tempe- ratura de 37,7 °C y disnea. No tiene
confirmaría el diagnóstico? traumatismos. Se realizan varias pruebas. La más
A. Biopsia guiada por endoscopía del intestino delgado inmediata es una gasometría arterial sin
B. Análisis cualitativo de grasas en heces oxigenoterapia que muestra pH 7,33, PCO2 50
C. Test de cloruro en sudor mmHg, PO2 65 mmHg, HCO3- 27mEq/l. Señale el
D. Ecografía de páncreas diag- nóstico más probable entre los siguientes:
E. Radiografía de abdomen y pelvis con contraste A. Tromboembolismo pulmonar.
95.En un niño de 8 años previamente sano, la causa más B. Neumonía lobar.
frecuente de meningitis bacteriana es: C. Edema agudo de pulmón.
A. H. influenzae. D. Hipoventilación alveolar.
B. S. pneumoniae. E. Bronconeumonía.
C. N. meningitidis.
D. S. aureus. 100¿Cuál es el antibiótico de elección para el tratamiento
E. Estreptococos del grupo B. de neumonía por Chlamydia pneumoniae en menores
de 4 años?
96.Un paciente de 32 años infectado por VIH consulta por A. Amoxicilina + ácido clavulánico.
astenia y fiebre ocasional, de unos 2 meses de B. Ceftriaxona.
evolución. En la exploración destaca: palidez cutánea, C. Eritromicina.
ausencia de adenopatías periféricas, esplenomegalia D. Rifampicina.
5 cm por debajo del re- borde costal. En la analítica E. Tetraciclina.
destaca: hemoglobina 6,5 g/dl, volumen corpuscular 101. Todos los siguientes fenómenos se asocian a
medio normal, leucocitos 1.300/ml, plaquetas taponamiento cardíaco, EXCEPTO:
40.000/ml, CD4 300/ml. En la TAC de abdomen A. Pulso paradójico.
presenta esplenomegalia homogénea, sin B. Presión venosa disminuida.
adenopatías. De los siguientes procesos, ¿cuál le C. Presión diferencial reducida.
parece más probable? D. Gasto cardíaco disminuido.
A. Infección por M. avium-intracellulare. E. Taquicardia sinusal.
B. Infección por Citomegalovirus. 102.Paciente de 48 años, con hipertensión pulmonar
C. Infección por Parvovirus B19. severa y cianosis, a quien en su infancia se le
D. Infección por Leishmania. diagnosticó comunica- ción interauricular. La
E. Toxoplasmosis diseminada. posibilidad diagnóstica es:
97.Un hombre de 30 años sano se realiza una radiografía A. Enfermedad de Lutembacher.
de tórax para formalizar un contrato laboral en una B. Hipertensión pulmonar primaria.
empresa. En la radiografía se observa un patrón C. Síndrome de Eisenmeger.
intersticial bilateral de tipo reticular, adenopatías D. Tetralogía de Fallot.
hiliares bilaterales y medias- tínicas. Se indica una E. Malformación de Ebstein.
broncofibroscopia con lavado broncoalveolar que en 103.Una paciente de 35 años refiere presentar disnea en
el recuento celular muestra los siguientes resultados: los últimos 10 días. A su llegada a urgencias se
linfocitos 50%, histiocitos 40%, eosinófilos 2%, observa crepitantes basales y ECG con taquicardia
polimorfonucleares 8% y el cociente CD4/CD8 5 ¿Cuál irregular de QRS estrecho, sin que se observen ondas
es el diagnóstico más probable? P. ¿Cuál de las siguientes es la actitud más correcta?
A. Alveolitis alérgica extrínseca. A. Cardioversión eléctrica sincronizada.
B. Neumoconiosis. B. Cardioversión eléctrica no sincronizada.
C. Sarcoidosis. C. Digoxina y diuréticos, anticoagulación durante 4
D. Linfangitis carcinomatosa. semanas y cardioversión posterior.
E. Hemosiderosis pulmonar idiopática. D. Digoxina y diuréticos, anticoagulación durante 4 días
y cardioversión posterior.
E. Digoxina y diuréticos, antiagregación durante 4 A. El alcohol puede desencadenar los ataques ya sea en
semanas y cardioversión posterior. el período de cefalea o intercrítica.
104.¿Cuál de los siguientes antiulcerosos produce B. Los ataques duran típicamente 8 – 10 horas y luego
quelación de otros medicamentos, impidiendo su remiten rápidamente.
absorción de forma significativa? C. Puede haber síndrome de Horner ipsilateral.
A. Cimetidina. D. Es más frecuente en el sexo femenino.
B. Ratinidina. E. Todo lo anterior.
C. Misoprostol.
D. Tetraciclina. 110.Un paciente diabético de 60 años consulta por
E. Sucralfato. primera vez respecto al tratamiento de su
enfermedad, ¿qué objetivo a
105.Un paciente de 78 años de edad, previamente sano, alcanzar de los siguientes le recomendaría en primer
que vive en una residencia de ancianos bastante lugar?
masificada y con insuficientes recursos higiénicos, A. Mantener la tensión arterial por debajo de 110/70
padece un cuadro diarreico desde hace 6 semanas. mmHg.
Refiere molestias abdominales tipo retortijón, B. Abandono del hábito tabáquico.
febrícula ocasional y 4-6 deposiciones diarias, alguna C. Mantener un índice de masa corporal (IMC) menor de
de ellas nocturna, con mucosidad y, en ocasiones, con 21.
hebras de sangre. Entre los diagnósticos que se D. Realizarse glucemia capilar basal a diario.
enumeran a continuación seleccione el que le parece E. Evitar las grasas animales en la dieta.
MENOS probable: 111.En una mujer de 55 años intervenida de cáncer de
A. Cáncer de colon. mama tres años antes, con buen estado general, se
B. Infección por Clostridium difficile. comprueba una hipercalcemia de 11,1 mg/dl. ¿Cuál
C. Enfermedad de Crohn. es la primera prueba a realizar?
D. Colitis isquémica. A. Determinación de PTH.
E. Salmonelosis. B. Determinación de 1,25(OH)2D.
C. Determinación de 25OHD.
106.¿Cuál de los siguientes procedimientos, es el menos D. Determinación de péptido relacionado con la PTH
indicado para el tratamiento de la úlcera péptica? (PRPTH).
A. Régimen lácteo. E. Gammagrafía ósea.
B. Anticolinérgicos. 112.Una niña de 11 años está hospitalizada debido al
C. Antiácidos. aumento de nerviosismo y palpitaciones; los
D. Bloqueadores H2. tranquilizantes no la han aliviado. El apetito ha sido
E. Tranquilizantes. bueno pero registró pérdida de peso. El
aprovechamiento escolar ha ido deteriorándose
107.La tomografía cerebral en los infartos cerebrales gradualmente. La niña fue enviada por el profesor a
isquémicos: un tutor guía que sugirió ayuda psiquiátrica. Al
A. Muestra típicamente área hipodensa rodeada de examen se en- contró un pulso amplio, la
edema. temperatura de la piel estaba aumentada,
B. Muestra área hiperdensa que capta contraste. transpiración excesiva y ROT rápidos. Los ojos
C. Puede ser normal en la fase inicial. presentaban una apariencia vidriosa. El diagnóstico
D. Distingue bien la zona de infarto de la de edema. más probable es:
E. Sólo distingue bien los infartos cerebelosos. A. Tirotoxicosis juvenil.
B. Psicosis juvenil.
108.Gestante en la décimo cuarta semana, desde hace C. Diabetes mellitus.
semanas presenta náuseas y vómitos persistentes sin D. Disautonomía familiar.
respuesta al dimenhidrinato, ha recibido en los E. Síndrome de Cushing.
últimos 15 días sueros glucosados por vía IV. 113.¿Cuál de los siguientes trastornos no produce
Actualmente presenta edema de miembros vasculitis de pequeños vasos?
inferiores, taquicardia, diplopía, dificultad para la A. Poliangeítis microscópica
marcha, nistagmo y alteraciones mentales. PA: B. Púrpura de Henoch-Schonlein
100/70; pulso: 115 lpm. Análisis: HB: 9 g%, Na: 130 C. Síndrome de Churg Strauss
mEq/L, bilirrubina total: 2 mg%, bilirrubina directa: D. Enfermedad de Kawasaki
1,4 mg%. ¿Cuál es la posibilidad diagnóstica? E. Crioglobulinemia mixta esencial
A. Encefalopatía hepática. 114.En el lupus cutáneo subagudo es característica de la
B. Pre – eclampsia severa. presencia de anticuerpos:
C. Síndrome de HELLP. A. Anti-Sm.
D. Hipertiroidismo de la gestante. B. Anticentrómero.
E. Encefalopatía de Wernicke. C. Anti-Ro.
D. Anti-Jo1.
109.La cefalea en racimos se caracteriza por lo siguiente: E. Antihistona.
115.La segunda generación de antidepresivos incluye a
todos los siguientes, EXCEPTO: 122.El linfoma de Burkitt está asociado a la infección de
A. Amoxapina. los linfocitos B con el:
B. Traxodona. A. Citomegalovirus.
C. Maprotilina. B. Virus delta.
D. Amitriptilina. C. Virus de Epstein Barr.
E. Fluoxetina. D. Papilomavirus.
E. Virus del polioma.
116.Señalar en cuál de los siguientes trastornos de la 123.En caso de muerte violenta. ¿Cuál es la razón
personalidad es más IMPROBABLE la aparición de fundamental para practicar la necropsia médico
sintomatología alucinatoria: legal?
A. Trastorno límite. A. Por imperativo de la ley.
B. Trastorno esquizotípico. B. Porque el juez instructor lo solicita
C. Trastorno obsesivo-compulsivo. C. Porque se necesita llenar el certificado
D. Trastorno esquizoide. D. Porque hay sospecha de homicidio
E. Trastorno paranoide. E. Para evitar problemas legales
117.¿Qué es cierto acerca de la oliguria? 124.Actualmente, el tratamiento de elección del
A. Se define como un volumen urinario inferior a 1.000 carcinoma epidermoide del canal anal es:
ml/día. A. Ablación local.
B. Es un hallazgo constante en la insuficiencia renal B. Radioterapia sola.
aguda. C. Radioterapia y quimioterapia combinadas.
C. Se asocia a dolor lumbar bilateral. D. Resección abdómino perineal.
D. Hace más difícil el manejo conservador del fracaso E. Resección anterior baja.
renal agudo. 125.En el tratamiento con inyección para esclerosar las
E. Mejora el pronóstico del fracaso renal agudo. hemorroides, es importante inyectar:
118.Mujer de 28 años de edad, consulta por hemoptisis A. En el lumen de la vena varicosa.
recurrente, disnea, anemia, presión arterial: 120/85 B. En la unión mucocutánea.
mmHg, res- piraciones 28 x’. Examen de orina: C. En la muscularis del intestino.
hematuria microscópica con cilindros hemáticos; D. En todas estas.
creatinina: 3,2 mg%, urea: 65 mg%; Hb: 6 g%; E. En ninguna de las anteriores.
anticuerpos antinucleares: negativo; anticuerpos
anticitoplasmáticos ANCA-C + 52 U. Radiografía de
pulmones: lesión homogénea 1/3 medio del 126.Un paciente con cirrosis hepática descompensada y
hemitórax izquierdo. ¿Cuál es el diagnóstico más ascitis, presenta dolor abdominal difuso y fiebre. Se
probable? sospecha pe- ritonitis bacteriana espontánea porque
A. Granulomatosis de Wegener. en el análisis del líquido ascítico se encuentra:
B. Lupus eritematoso sistémico. A. Hematíes mayores de 100/L.
C. Tuberculosis pulmonar y renal. B. Concentración baja de albúmina.
D. Síndrome de Goodpasture. C. Cuenta elevada de eosinófilos.
E. Aspergilosis pulmonar. D. Leucocitosis mayor de 500 células/L, a predominio de
119.La aparición de un pliegue extra de piel por debajo del polimorfonucleares.
párpado inferior es una característica de: E. Leucocitosis mayor de 10.000 células/ L.
A. Dermatitis seborreica 127.En pacientes con ataques repetidos de pancreatitis
B. Rosácea. biliar, es típica la existencia de:
C. Pitiriasis rosada. A. Fibrosis pancreática.
D. Dermatitis atópica. B. Colangitis.
E. Eccema de contacto. C. Diabetes.
F. Enfermedad injerto contra huésped. D. Esprue.
120.En Querion el tratamiento es: E. Narcomanía.
A. Antihistamínicos 128.El tratamiento paliativo más adecuado en caso de
B. Penicilina. cáncer de cabeza de páncreas es:
C. Clotrimazol A. Quimioterapia.
D. Griseofulvina. B. Operación de Whipple.
E. Anfotericina B. C. Derivación biliodigestiva sola.
121.Durante qué mes de gestación comienza la D. Derivación biliodigestiva más
hemopoyesis en la médula ósea: gastroyeyunoanastomosis.
A. Tercero. E. Radioterapia.
B. Quinto. 129.En las próximas 30 horas después de una laparotomía
C. Sexto. puede haber fiebre de causas infecciosas
D. Séptimo. importantes, EXCEP- TO:
E. Octavo. A. Lesión intestinal con filtración de líquidO intestinal.
B. Infección de tejidos blandos por Clostridium C. Adenocarcinoma de colon y colitis ulcerativa.
perfringes. D. Vólvulo del colon y amebiasis.
C. Infección de tejidos blandos por Gram negativos. E. Colitis granulomatosa y prolapso rectal.
D. Infección de tejidos blandos por Streptococo B- 138.Mejor operación (con menor índice de recurrencia)
hemolítico. en paciente joven con enfermedad acidopéptica:
E. Infección de tejidos blandos por Klebsiella. A. Gastrectomía total.
130. Una herida traumática ocurrida hace más de 4-6 B. Gastrectomía subtotal Billroth-II.
horas se considera que está: C. Vagotomía supraselectiva.
A. Contaminada. D. Vagotomía troncular y piloroplastia.
B. Limpia. E. Vagotomía troncular y antrectomía.
C. Sucia. 139.Mujer de 20 años de edad, G2 P2002. Último parto
D. Supurada. por cesárea, hace 3 meses lactando. ¿Cuál es el
E. Infectada. anticonceptivo
131.¿Cuál es el electrolito que más se altera en las hormonal MÁS adecuado?
primeras 48 horas post-quemadura? A. Estrógeno oral.
A. Sodio. B. Estrógeno más progesterona por vía oral.
B. Cloro. C. Progestágenos orales.
C. Zinc. D. Parche de etinilestradiol y progesterona.
D. Magnesio. E. Inyectable mensual de estrógeno y progesterona.
E. Todos. 140.Paciente con prurito vulvar, que al examen
132.La hernioplastía de tipo Liechtenstein se caracteriza ginecológico se encuentra secreción blanquecina
por el uso de: maloliente. Cuello uteri- no con punteado rojizo
A. Malla de polipropileno. como fresas. La infección vaginal sería causada por:
B. Incisiones relajantes.
C. Tapón de polipropileno en anillo inguinal. A. Cándida albicans.
D. Doble sutura a surget en la fascia transversalis. B. Trichomonas vaginalis.
E. Técnica laparoscópica. C. Gardnerella vaginalis.
133.En la Fase I del ATLS (Advanced Trauma Life Support), D. Flora mixta.
debe realizarse todo, excepto: E. Chlamydia trachomatis.
A. Control de la columna cervical. 141.En una gestante con condiloma genital de gran
B. Respiración y ventilación. volumen, el mejor tratamiento es:
C. Inserción de sonda nasogástrica. A. Podofilina local.
D. Exponer desnudo al paciente. B. Láser de dióxido de carbono.
E. Control de la hemorragia externa. C. Ácido tricloroacético local.
134.¿Cuál de las siguientes suturas es de material D. 6-fluorouracilo local.
absorbible?. E. Interferon sistémico.
A. Seda. 142.Paciente a quien se realizó un cono frío, cuya
B. Poliéster. anatomía patológica reporta carcinoma epidermoide
C. Nylon. invasor con menos de 5 mm de profundidad en el
D. Polipropileno. estroma. Según la clasificación de la FIGO,
E. Ácido poliglicolico corresponde al estudio:
135.En la fractura hepática con hemoperitoneo ¿qué A. I A.
estructura se tiene como referencia, para clampar y B. I B.
cohibir el san- grado? C. II A.
A. El ligamento de Treitz. D. II B.
B. La vía biliar principal. E. N.A.
C. La vena cava inferior. 143.Mujer de 60 años de edad. Presenta pérdida de orina
D. El ligamento colecistoduodenal. al esfuerzo físico sin poder controlarlo. Al examen se
E. Epiplón gastrohepático corrobora Con la maniobra de VaIsaIva. ¿Cuál es el
136.El dolor abdominal postprandial intenso a repetición diagnóstico MÁS probable?
en un paciente anciano con varios factores de riesgo A. Incontinencia urinaria de esfuerzo.
cardiovas- cular, nos debe hacer sospechar: B. Incontinencia urinaria de urgencia.
A. Embolia mesentérica. C. Desgarro perineal antiguo.
B. Isquemia mesentérica crónica. D. Prolapso uretral.
C. Colitis isquémica. E. Incontinencia urinaria por rebosamiento.
D. Rotura de aneurisma abdominal..
E. Aneurisma esplácnico. 144.El tratamiento MÁS apropiado en caso de carcinoma
137.¿Cuáles son las dos causas MÁS comunes de endometrial estadío I comprende:
hemorragia grave del colon? A. Histerectomía total abdominal.
A. Diverticulosis y angiodisplasia. B. Salpingooforectomía bilateral.
B. Enfermedad hemorroidaria y poliposis. C. Estadiaje.
D. Sólo A y B son correctas. C. No es necesario la colocación de sonda vesical, salvo
E. Todas las respuestas son correctas. cuando se maneja convulsiones.
145.¿Cuál de los siguientes signos NO suele aparecer en la D. Solo debe ser usado hasta 24 horas post parto con el
clínica del cáncer de mama? fin de prevenir las convulsiones.
A. Mastodinia. E. La arreflexia patelar y la depresión respiratoria son
B. Telorragia. signos de intoxicación por sulfato de magnesio.
C. Tumoración. 153.La complicación MÁS SEVERA de la hiperemesis
D. Retracción del pezón. gravídica es:
E. Adenopatía axilar. A. Deshidratación.
146.La modificación endocrina que aparece de modo MÁS B. Hipotensión.
PRECOZ en la menopausia es: C. Encefalopatía de Wemicke.
A. Aumento de FSH. D. Hiperkalemia.
B. Disminución de FSH. E. Hipematremia.
C. Aumento de LH.
D. Disminución de LH. 153.Gestante de 22 años, con 18 semanas por FUR, grupo
E. Aumento de estrógenos. sanguíneo A, factor Rh negativo. Antecedente de un
147.La cianosis de la mucosa vaginal y del cérvix es un parto eu- tócico, donde recibió inmunoglobulina Anti
signo presuntivo de embarazo y se le denomina signo D. el embarazo actual es de segundo compromiso.
de: ¿Cuál es la conducta adecuada?
A. Godell A. Esperar porque no hay problema.
B. Hegar B. Inmunoglobulina anti D a las 26 y 32 semanas.
C. Noble Budin C. Inmunoglobulina Anti D dosis única.
D. Test de Liley.
D. Chadwick E. Tipificación sanguínea de la pareja.
E. Mac Donald 155. En el perfil biofísico fetal se estudia lo siguiente,
148.¿Qué tiempo después de la enfermedad trofoblástica EXCEPTO:
molar (mola hidatiforme) generalmente hace su A. Situación y posición fetal.
aparición la enfermedad trofoblástica maligna? B. Movimientos fetales y respiratorios.
A. 4 a 6 años. C. Frecuencia y reactividad cardíaca fetal.
B. 5 a 10 años. D. Volumen del líquido amniótico.
C. 36 meses. E. Tono fetal.
D. 2 a 28 meses. 156. En relación a la valoración Apgar, señale lo
E. 6 a 24 meses. CORRECTO:
149.En la placenta acreta hay un defecto en la decidua A. Se toma desde el momento que sale completamente
que consiste en la AUSENCIA de: el RN del canal del parto
A. Las Estrías de Rohr. B. No necesariamente sirve como guía para la
B. La zona esponjosa. reanimación del RN.
C. La zona compacta. C. Un puntaje de 7 nos da un buen pronóstico
D. La zona basal. neurológico a largo plazo.
E. El estrato de Nitabuch. D. En un recién nacido grave debe evaluarse
150.El embarazo gemelar que se produce en el primer día básicamente el tono muscular.
de la división celular del huevo, tiene las siguientes E. Existe limitación de aplicación en el RN prematuro.
caracterís- ticas: 157.El hijo de madre diabética puede presentar los
A. Monoplacentario y biamniótico. siguientes problemas, EXCEPTO:
B. Monoplacentario y monocoriónico. A. Prematuridad.
C. Monocoriónico y monoamniótico. B. Macrosomía.
D. Monocoriónico y biamniótico. C. SDR I.
E. Bicoriónico y biamniótico D. Trombosis de la vena renal.
151.En el trabajo de parto normal, cuando la flexión se E. Mesomelia.
completa, el diámetro de la cabeza fetal que ingresa 158. La forma neurotóxica de la bilirrubina en el recién
en el estrecho superiorde la pelvis es: nacido es la:
A. Occipitofrontal. A. Bilirrubina no conjugada o indirecta.
B. Suboccipitobregmático. B. Bilirrubina conjugada o directa.
C. Occipitomentoniano. C. Urobilinógeno.
D. Biparietal. D. Biliverdina.
E. Bitemporal. E. Urobilina.
152.Con respecto al uso del sulfato de magnesio, señale 159.¿Cuál de los siguientes NO es parte del síndrome de
lo INCORRECTO: rubéola congénita?
A. La dosis de carga es de 4g /EV. A. Coriorretinitis.
B. La dosis de mantenimiento es de 1 - 2 g /EV por hora. B. Microftalmia.
C. Sordera.
D. Conducto arterioso persistente. 167.La alcaptonuria es una enfermedad producida por la
E. Hepatoesplenomegalia. carencia de la homogentísica oxidasa y afecta el
160.El feto postmaduro típicamente presenta todo lo metabolismo de la:
siguiente, EXCEPTO:
A. Piel suelta. A. Urea.
B. Fenilanina.
B. Uñas largas. C. Valina.
C. Caída del pelo. D. Prolina.
D. Disminución del vérmix. E. Creatinina.
E. Ninguna de las anteriores. 168.¿Cuál de las siguientes manifestaciones se presenta
161.Una madre se queja de que su niño de 3 meses vomita en la hipoglicemia neonatal?
frecuentemente, el control del peso del niño durante A. Hiperreflexia.
el período de 1 1/2 meses, revela que éste se B. Distensión abdominal.
mantiene dentro del percentil 70, ¿qué actitud C. Fontanela tensa.
tomaría usted? D. Hipertonía.
A. Solicita Rx del tracto digestivo alto. E. Temblores.
B. Solicita estudio de acidez gástrica. 169. El petit mal del niño se caracteriza por lo
C. Cambia la fórmula láctea. siguiente, EXCEPTO:
D. Agrega cereales a su alimentación. A. Puede observarse aura.
E. Ninguna de las anteriores. B. Por lo general las crisis duran menos de 15 segundos.
162.¿Cuál de los siguientes NO ha sido implicado como C. Las crisis no alteran la postura del niño.
agente causal en caso de una enfermedad parecida al D. Puede observarse contracciones de los miembros
coqueluche? superiores.
A. Bordetella pertusis. E. La conciencia se recupera rápidamente.
B. Echovirus tipo 21. 170.Niño de 2 meses de edad, es llevado a Emergencia por
C. Bordetella parapertussis. presentar convulsiones tónico-clónicas generalizadas
D. Bordetella bronchiseptica. desde hace 45 minutos, sin recuperar la conciencia.
E. Especies de Adenovirus. Antecedente de asfixia al nacer. Presenta fiebre de
163.En el primer trimestre de vida extrauterina, el 39ºC. ¿Cuál es el diagnós- tico más probable?
crecimiento en los niños se caracteriza por, EXCEPTO: A. Crisis convulsiva
A. Ganancia de peso diario de 30g. B. Convulsión febril compleja
B. Crecimiento en talla de 3.5 cm/mes. C. Estatus convulsivo
C. Crecimiento de perímetro cefálico: 2 cm/mes. D. Convulsión febril simple
D. Requerimiento calórico de 80 kcal/g/d. E. Epilepsia
E. B y D.
164.Los senos paranasales que se encuentran 171.Un niño de 10 años presenta anemia, hemorragias
desarrollados al nacer son: pulmonares y hepatoesplenomegalia, En la placa de
A. Maxilares y etmoidales. tórax se obser- va una masa mediastínica anterior,
B. esfenoidales y maxilares. ¿Qué proceso descartaría en primer lugar?
C. Frontales y etmoidales. A. Mononucleosis infecciosa
D. Esfenoidales y etmoidales. B. Leucemia linfoide crónica
E. Frontales y maxilares. C. Leucemia linfoblástica aguda tipo T
165.¿Cuál de las siguientes expresiones no es correcta D. Leucemia linfoblástica aguda tipo B
respecto a la laringotraqueobronquitis? E. Leucemia M2
A. Es la forma más común de obstrucción de las vías 172.Un niño de 6 años acude a consulta por un cuadro de
aéreas superiores. febrícula de 3 días de evolución, con dolor a la
B. El 75% de los casos es producida por los virus deglución. Los datos más relavantes de la exploración
parainfluenza. física son lesiones erosivas en el paladar y vesículas
C. La budesonida inhalada es útil en el tratamiento. intraepidérmicas no agru- padas en palmas y plantas.
D. El uso de antibióticos previene la infección Entre los siguientes diagnósticos, ¿cuál es el más
bacteriana. probable?
E. La dexametasona disminuye el edema inflamatorio. A. Eritema multiforme.
166.¿Cuál de las siguientes patologías está asociada a B. Rickettsiosis
displasia neuronal del intestino? C. Síndrome de Steven-Jonhson.
A. Íleo meconial. D. Enfermedad de pie, mano, boca
B. Megacolon secundario. E. Deshidrosis.
C. Enfermedad de Hirschsprung.
D. Colon irritable espástico. 173.El enteropatógeno parasitario que con más
E. Duplicación del colon. frecuencia causa diarrea con malabsorción intestinal
en niños menores de 5 años y en desnutridos es:
A. Isospora belli
B. Esfamoeba histolytica 180.Cual no es documento de gestión de un
C. Strongyloides stercolaris establecimiento de primer nivel:
D. Ascaris lumbricoides A. Manual de organización y funciones
E. Giardia lamblia B. Manual de procedimientos
C. Texto único de procedimientos administrativos
174.Si el valor de una canasta básica familiar en Tarma es D. Guía de procedimientos y trámites
de S/. 1500 y el valor de la canasta solo con alimentos E. Plan operativo anual
es de S/750, señale usted en que grupo se encontraría 181.Marque la respuesta CORRECTA, de acuerdo al tema
la familia “X” cuyo ingreso promedio mensual es de de SUNASA:
S/. 550 mensual: A. Es un órgano consultor adscrito al MINSA con
A. Quintil 4 autonomía funcional, administrativa y financiera.
B. Pobre no extremo B. Ejerce sus competencias y funciones desde Lima pero
C. Extrema pobreza a un ámbito nacional, regional y local.
D. Pobreza C. Ejerce función conciliatoria y arbitral para tramitar las
E. Clase media controversias entre el asegurado y el seguro.
D. Uno de sus objetivos es construir un sistema de
175.Son factores condicionantes de la salud pública: aseguramiento público sostenible.
A. Biología E. Es un órgano ejecutor del Seguro Integral de Salud
B. Medio ambiente 182.El coeficiente de correlación de Pearson (r):
C. Estilo de vida A. Varía de -1 a 0.
D. Sistema sanitario B. Tiene el mismo signo que la varianza de la cual
E. Todas procede.
176.¿Cuál de los siguientes documentos sobre promoción C. No sirve para estudiar la relación lineal entre dos
de la salud puso mayor énfasis en la perspectiva variables cuantitativas.
sociopolítica? D. Si se eleva al cuadrado, es el coeficiente de
A. Informe de Lalonde (Canadá, 1974) determinación.
B. Declaración de Santa Fe De Bogotá (Colombia, 1992) E. Todas son ciertas.
C. Declaración de Alma Ata (URSS, 1978) 183. La expectativa de vida al nacer en el Perú
D. Carta de Ottawa (Canadá, 1986) actualmente es de:
E. Declaración de Sundsvall (Suecia, 1991) A. 60,2 años
B. 74,1 años
177.¿Cuál es el programa social encargado de realizar C. 65,0 años
incentivos monetarios, que promuevan y apoyen el D. 63,4 años
acceso a los E. 68,8 años
1servicios en educación, salud y nutrición, para las
familias muy pobres? 184.Al tratar una artritis psoriásica con azatioprina,
A. Pronaa mejora el 60% de los pacientes; la 6-mercaptopurina
B. Cuna más mejora a un 55%. La diferencia entre ambos
C. Pensión 65 tratamientos es significativa (p<0,05), lo cual se debe
D. Foncodes interpretar como una de las siguientes opciones:
E. Juntos A. Con azatioprina mejorarán el 60% de tus pacientes.
B. La azatioprina es en un 95% mejor que la 6-
178.De acuerdo con las normas sectoriales, el mercaptopurina.
establecimiento de Salud que cuenta con servicio de C. El nivel de significación es del 1%.
emergencia, sala de operaciones, las cuatro D. Hay diferencias entre los dos tratamientos, con una
especialidades básicas y mediana capacidad probabilidad de que esta afirmación sea equivocada
resolutiva, es categorizado como: menor al 5%.
A. I-3 E. No se puede concluir nada.
B. I-4
C. II-1 185.Se llama probabilidad condicionada a:
D. II-2 A. La probabilidad de un suceso conocido previamente.
E. III-1 B. La condición que debe cumplir un suceso.
C. La probabilidad que ocurra un suceso habiendo
179. La existencia de un servicio de atención al cliente o al ocurrido otro.
usuario es un indicador de: D. La probabilidad de que suceda de que ocurra un
A. De eficacia suceso u otro.
B. De acceso E. El teorema de Bayes restringido.
C. De estructura
D. De proceso 186. Varón de 20 años de edad, que presenta
E. De resultado convulsiones en el servicio de Emergencia. ¿Cuál de
los siguientes fármacos
administraría? distribución normal, el test correcto para comparar la
A. Carbamazepina. respuesta es?:
B. Diazepam. A.La t de Student.
C. Metilfenitoína. B.El test de Wilcoxon.
D. Valproato. C.Análisis de la varianza.
E. Difenilhidantoína. D.El test de Krusal-Walls.
E.El test Chi-cuadrado.
187. Entre los efectos colaterales del litio NO se incluye:
A. Aumento de peso. 194.Si como resultado de un estudio prospectivo en el que
B. Gastritis. se han comparado dos grupos de individuos, unos
C. Polidipsia. expuestos y otros no expuestos a un factor de riesgo
D. Leucocitosis. causante de una enfermedad. Después de comprobar
E. Convulsiones. la comparabilidad de los dos grupos, obtenemos que
de los 500 individuos expuestos al factor de riesgo 50
188. ¿Cuál es el efecto adverso más frecuente en la padecen la enfermedad de estu- dio, y que de los 500
administración de Clindamicina? no expuestos sólo la padecen 10. ¿Cuál sería el riesgo
A. Colitis pseudomembranosa. de padecer la enfermedad que podríamos atribuir al
B. Nefrotoxicidad. factor de riesgo?:
C. Ototoxicidad. A. 0,08.
D. Anemia aplásica. B. 0,1.
E. Artropatia. C. 0,2.
D. 0,5.
189. Varón de 7 años, 15 días después de cumplir E. 10.
tratamiento por faringoamigdalitis estreptocócica
presenta náuseas, vó- mitos, cólicos abdominales e 195.En el estudio de Framingan (Framingan Heart Study)
ictericia. ¿Cuál de los siguientes fármacos se asocia a sobre cardiopatía coronaria, se encontró que en el
estos efectos adversos? examen ini- cial 17 personas por cada 1.000 tenían
A. Clindamicina fosfato síntomas evidentes de cardiopatía coronaria. Elija la
B. Cefradina tasa o razón que mejor describe este hecho:
C. Vancomicina A.Tasa de incidencia.
D. Ampicilina B.Tasa de prevalencia.
E. Eritromicina estolato C.Razón de morbilidad estandarizada.
D.Tasa específica por edad.
190. ¿Cuál es el hospedero definitivo de Hymenolepis E.Tasa de mortalidad ajustada.
diminuta?
A. Hombre. 196.En un estudio de cohortes diseñado para comprobar
B. Cerdo. la relación tabaco-cáncer de vejiga, se siguieron
C. Vaca. durante un año a 1.000 personas, 500 fuman y 500
D. Rata. no. De los 50 cánceres producidos en ese tiempo, 45
E. Venado. aparecieron en fumadores.
¿Cuál es la incidencia en expuestos?:
191. ¿Cuál es el tipo de estreptococo que produce
estreptolisina O? A. 50/1.000 = 5%.
A. Faecalis. B. 50/500 = 10%.
B. Bovis. C. 45/500 = 9%.
C. Pyogenes. D. 5/500 = 1%.
D. Agalactie. E. 45/1.000 = 4,5%.
E. Pneumoniae.
197. Niño de dos meses, sin antecedentes de importancia,
192.¿Cuál de las siguientes enfermedades es causada por presenta rinorrea hialina y fiebre de 38° C en los tres
un parásito intracelular? últimos días. Desde hace 24 horas, tiene tos y
A. Asepergilosis. dificultad respiratoria progresiva. En las últimas 12
B. Histoplasmosis. horas rechaza todas las tomas. Ha recibido la primera
C. Lepra. dosis de DTP acelular, Hemophilus influenzae tipo B,
D. Malaria. Meningococo C, Polio oral y dos dosis de Hepatitis B.
E. Tuberculosis. Examen físico: frec. cardíaca 135 l.p.m.,
frec.respiratoria 55r.p.m. y Sat O2: 90%. Tiraje
193.En un ensayo clínico se comparan 3 tratamientos (p.e. intercostal y subcostal,
placebo, tratamiento establecido y un tratamiento
nuevo). La variable respuesta es continua (p.e. nivel
de glucosa en sangre). ¿Si la variable no tiene una
1.C 21.C 41.A 61.C 81.C 101.B 121.B 141.D 161.C 181.D
subcrepitantes y sibilancias bilaterales. ¿Cuál es el 2.B 22.C 42.D 62.A 82.C 102.C 122.C 142.A 162.B 182.B
diagnóstico más probable? 3.B 23.A 43.A 63.B 83.B 103.C 123.A 143.A 163.D 183.D
A.Crisis asmática moderada - severa. 4.B 24.E 44.A 64.C 84.C 104.E 124.C 144.D 164.D 184.C
B.Bronquiolitis. 5.A 25.B 45.D 65.C 85.E 105.E 125.E 145.A 165.C 185.B
6.B 26.D 46.D 66.A 86.C 106.B 126.D 146.A 166.B 186.B
C.Neumonía bacteriana. 7.A 27.C 47.B 67.A 87.E 107.C 127.B 147.D 167.E 187.A
D.Tos ferina. 8.D 28.A 48.C 68.B 88.D 108.E 128.D 148.D 168.A 188.E
E.Neumonía por Clamidia trachomatis. 9.D 29.B 49.D 69.B 89.D 109.C 129.C 149.D 169.C 189.D
10.B 30.D 50.A 70.C 90.C 110.B 130.A 150.E 170.D 190.C
198.Paciente de 9 meses que presenta discreta fiebre, 11.E 31.C 51.D 71.D 91.B 111.A 131.A 151.B 171.D 191.D
12.A 32.B 52.A 72.C 92.C 112.A 132.A 152.C 172.E 192.C
adenopatías retroauriculares grandes y dolorosas y
13.C 33.C 53.C 73.C 93.B 113.D 133.B 153.C 173.C 193.D
además exante- ma maculopapular de color rosáceo 14.C 34.E 54.C 74.B 94.C 114.C 134.E 154.E 174.E 194.A
en cuerpo. ¿Cuál es el diagnóstico más probable? 15.E 35.C 55.C 75.D 95.C 115.D 135.B 155.A 175.E 195.A
a). Roséola infantil 16.E 36.D 56.C 76.B 96.D 116.C 136.B 156.E 176.E 196.C
b).Rubéola 17.D 37.B 57.B 77.E 97.C 117.D 137.A 157.E 177.C 197.B
18.B 38.A 58.D 78.C 98.D 118.A 138.E 158.A 178.D 198.B
c) Sarampión
19.E 39.B 59.A 79.E 99.D 119.D 139.C 159.A 179.C 199.C
d) Mononucleosis infecciosa 20.B 40.A 60.C 80A 100.C 120.D 140.B 160.C 180.C 200.D
e) Varicela

199.Niño de 13 años llevado a la consulta por presentar


estornudos, rinorrea acuosa y prurito nasal
persistentes. Al exa- men: intenso edema y palidez de
la mucosa nasal, con secreción clara. El diagnóstico
probable es:
a).Cuerpo extraño
b) Mastocitosis nasal
c) Rinitis alérgica
d) Rinitis neutrofílica
e) Rinitis vasomotora

200.A los 5 minutos de vida un neonato presenta estos


signos: 130 pulsaciones por minuto, manos y pies
cianóticos, buen tono muscular, llanto poderoso,
reactividad normal, frecuencia respiratoria 60 por
minuto. El índice de Apgar en este niño es:
a) 6
b) 7
c) 8
d) 9
e) 10
D. V
E. IV
6. Paciente de 26 años con 21 semanas de gestación,
4TO EXAMEN ENAM presenta dolor abdominal y sangrado vaginal escaso,
de 2 días de evolución. Al examen: altura uterina 20
1. Primigesta con 32 semanas de gestación por última cm, movimientos fetales presentes. Especuloscopía:
regla, acude a emergencia por dolor en epigastrio e se observa membranas ovulares prominentes e
hipocondrio derecho. PA: 180/110 mmHg, integras, que protruyen por el orificio externo
proteinuria ++, tacto vaginal: cérvix posterior, orificio abierto. ¿Cuál es el diagnóstico?
externo cerrado, pelvis ginecoide. El diagnóstico más A. Óbito fetal
probable es: B. Aborto incompleto
A. Preeclampsia severa y trabajo de parto C. Aborto inevitable
B. Preeclampsia superpuesta e hipertensión crónica D. Aborto inminente
C. Preeclampsia severa E. Aborto frustro
D. Preeclampsia severa y colecistitis 7. Primigesta en la 39 semana de gestación y con
E. Preeclampsia severa y DPP contracciones de parto. Todo ha transcurrido con
2. Puérpera de parto gemelar que presenta sangrado normalidad hasta que ha roto la bolsa
vaginal profuso luego del alumbramiento. El espontáneamente con 3 cm de dilatación. Nada más
diagnóstico más pro- bable es: romper la bolsa ha comenzado a sangrar (sangre roja
A. Laceración de cuello uterino en mediana cantidad) y han surgido signos de
B. Ruptura uterina sufrimiento fetal muy grave. La causa más probable
C. Coagulación es:
D. Atonía uterina A. Placenta previa.
E. Trombocitopenia B. Abruptio placentae.
3. Gestante a término de 35 años acude a emergencia C. Lesión de cérvix.
por contracciones uterinas cada 3 minutos y sangrado D. Síndrome de Hellp.
vaginal de 10cc. PA: 120/70, P: 84x’, FR: 16x’, E. Rotura de vasa previa
Ecografía: feto único en LCI, placenta previa marginal. 8. Gestante en la décimo cuarta semana, desde hace
LCF: 148x’. Se realiza TV en condiciones de semanas presenta náuseas y vómitos persistentes sin
operabilidad: incorporación: 90%, dilatación: 8cm, respuesta al dimenhidrinato, ha recibido en los
membranas íntegras, altura de presentación: 0, últimos 15 días sueros glucosados por vía IV.
variedad de posición: OIIA, pelvis ginecoide. ¿Cuál es Actualmente presenta edema de miembros
la conducta a seguir? inferiores, taquicardia, diplopía, dificultad para la
A. Acentuación del trabajo de parto marcha, nistagmo y alteraciones mentales. PA:
B. Esperar a que continúe el trabajo de parto 100/70; pulso: 115 lpm. Análisis: HB: 9 g%, Na: 130
espontáneo mEq/L, bilirrubina total: 2 mg%, bilirrubina directa:
C. Parto instrumentado 1,4 mg%. ¿Cuál es la posibilidad diagnóstica?:
D. Preparar para cesárea inmediata A. Encefalopatía hepática.
E. Transfusión de sangre B. Pre – eclampsia severa.
4. Paciente de 49 años, con mioma uterino de tamaño C. Síndrome de HELLP.
equivalente a una gestación de 12 semanas, que D. Hipertiroidismo de la gestante.
presenta hiperme- norreas y hemoglobinemia de 9 E. Encefalopatía de Wernicke.
g%. No existe patología asociada. La paciente está en 9. Después de un expulsivo normal y tras 60 minutos de
lista de espera para la práctica de una histerectomía periodo de alumbramiento no se aprecian signos de
programada para dentro de 3-4 meses. En esta despren- dimiento placentario a pesar de haberse
paciente está indicado el tratamiento preopera- torio aplicado masaje uterino y de haber incrementado
con: moderadamente la dosis de oxitocina. Se indica una
A. Estrógenos. extracción manual de placenta y la pared uterina
B. Ergóticos dopaminérgicos. ¿Cuál es el diagnóstico más probable?
C. Inhibidores de la fibrinolisis. A. Placenta incarcerada
D. Análogos de la GnRH. B. Engatillamiento placentario
E. Derivados del cornezuelo del centeno. C. Placenta adherente por acretismo placentario
5. Mujer con tumoración que protruye por genitales D. Placenta succenturiada con cotiledón aberrante
externos. Al examen tiene una longitud de vagina de E. Placenta circunvalata
11. El punto Aa 10. Paciente de 55 años, general en su activo, con
es +3. Según la clasificación de prolapso de órganos antecedentes de hipertensión, diabético y angina
pélvicos (POP-Q), ¿cuál es el grado de distopia inestable, traído a la urgencia tras caída montando en
genital? bicicleta. Radiográficamente presenta una fractura
A. III intracapsular desplazada del fémur proximal. ¿Cuál
B. I será el tratamiento más adecuado?
C. II A. Reducción y osteosíntesis con tornillo
B. Artroplastía parcial de cadera E. Queratitis medicamentosa
C. Reducción y osteosíntesis con tornillos canulados 16. Paciente de 37 años que desde hace 4 días presenta
D. Reducción incruenta dolor en región anal y desde hace 2 días fiebre. En
E. Artroplastía total de cadera urgencia se le observa una zona indurada tumefacta
11. Un hombre fumador de 50 años consulta por y enrojecida en la zona perianal derecha. A la presión
ronquera, afonía e hipo. En la exploración ORL se es muy dolorosa. Se le prescribe antibióticos y se le
evidencia parálisis de la cuerda vocal izquierda como recomienda consulta en Cirugía a las 48 horas, ¿Cuál
única anomalía. ¿Dónde localizaría la lesión? es el origen más probable de su patogenia actual?
A. Esófago distal A. Fisura anal
B. Mediastino posterior B. Prolapso rectal
C. Pleura C. Fístula anorrectal
D. Cavum D. Hemorroides internos
E. Língula E. Rectocele
12. Un paciente consulta por pérdida de peso, dolor 17. Una paciente diabética de 65 años acude a urgencias
abdominal y trombosis venosa. En la exploración por fiebre de 38.5°C y malestar general junto con
tiene esplenomega- síndrome mic- cional. PA: 90/60 mmHg, FC: 105x’ y
lia e ictericia. En las pruebas de imagen se confirma la FR: 22x’ con SO2: 89%. En la exploración física
sospecha clínica de tumor abdominal de: destacaba PPL (+). En el labo- ratorio sobresalía
A. Hígado leucocitosis de 15700/mm3 (87%: neutrófilos) y
B. Vesícula creatinina de 1.4 mg/dL. Plaquetas: 90000; señale
C. Páncreas aquella medida inicial cuyo retraso tenga una mayor
D. Estómago influencia negativa en la evolución del cuadro:
E. Colon A. Ingreso en UCI para ventilación mecánica asistida
13. Si un paciente queda disfónico después de haber sido B. Administración precozmente de bicarbonato
sometido a una hemitiroidectomía izquierda, es C. Administración de tratamiento inotrópico con
porque se ha lesionado: dobutamina
A. Nervio laríngeo superior izquierdo D. Administración de antibióticos y estabilización
B. Nervio frénico izquierdo hemodinámica
C. Cadena simpática izquierda E. Administración precozmente de corticoides y
D. Nervio laríngeo inferior izquierdo proteína C activada
E. Nervio frénico 18. Mujer de 51 años de edad que consulta por debilidad
14. Tras hacer la historia clínica y ordenar las pruebas generalizada y visión doble de 3 meses de evolución.
complementarias se llega al diagnóstico de que un Se le realiza un test de edrofonio y una
paciente de 45 años tiene una apendicitis aguda sin electromiografía que permiten un diagnóstico de
peritonitis que requiere una apendicectomía urgente, miastenia gravis. En Rx. y TAC torácico de localización
el paciente le pregunta sobre la laparoscopía. Su anteroposterior de 5x4x4cm, y densidad homogénea.
contestación es: ¿Qué patología mediastínica debemos sospechar
A. La laparoscopía solo sirve para el diagnóstico en casos como más probable?
de apendicitis aguda A. Bocio endotorácico
B. La cirugía laparoscópica solo está indicado para la B. Linfoma
colecistectomía C. Timoma
C. La apendicitis aguda es una contraindicación absoluta D. Teratoma
para el abordaje laparoscópico E. Tumor neurogénico
D. La apendicitis aguda sin peritonitis puede tratarse por 19. Un paciente de 65 años diagnosticado de EPOC, es
laparoscopía y puede ofrecer algunas ventajas sobre trasladado al hospital por disnea progresiva y
la laparotomía expectoración purulenta. Durante el traslado en la
E. La única indicación de cirugía laparoscópica es el ambulancia se le administra oxígeno y fluidoterapia.
plastrón inflamatorio palpable en fosa ilíaca derecha A su llegada el paciente está obnubilado y
15. Paciente de 52 años de edad que acude a consulta tembloroso. Se auscultan crepitantes en la base
manifestando que desde hace tres días nota visión derecha. La gasometría arterial muestra un pH: 7.08,
borrosa con mo- lestias en ojo derecho que se PO2: 106. El tratamiento inicial más adecuado sería:
acentúan cuando está en lugares muy iluminados o A. Reducir flujo de oxígeno a 1L/min. y administrar
en la calle si hace sol. Al ser explorado encontramos bicarbonato sódico
una discreta hiperemia conjuntival y tras la tinción B. Mantener flujo de oxígeno y administrar
con colirio de Fluoresceína sódica observamos sobre acetazolamida
la córnea central una figura lineal ramificada de unos C. Intubación y ventilación mecánica
5mm de longitud que nos sugiere que estamos ante: D. Retirar oxígeno y administrar bicarbonato sódico
A. Queratoconjuntivitis epidémica E. Administrar epinefrina i.v. y bicarbonato sódico
B. Queratitis bacteriana 20. Mujer de 35 años sin factores de riesgo
C. Queratitis vírica cardiovascular, con historia de una muerte fetal de 20
D. Queratitis por acanthamoeba semanas, que presenta un infarto de la cerebral
media derecha. En el hemograma presenta B. Parkinson arteriopático.
plaquetopenia y en el estudio de coagulación se C. Demencia arteriopática.
observa un alargamiento del TTPa que no corrige con D. Forma lacunar.
plasma fresco congelado. ¿Cuál es el diagnóstico más E. Neurasténica.
probable? 25. Mujer de 19 años que en los últimos 15 días presenta
A. Poliarteritis nodosa febrícula, sudoración nocturna, astenia y dolor
B. Micropoliangeítis torácico derecho. En la Rx de tórax se aprecia
C. Síndrome antifosfolípido derrame pleural derecho importante, sin otras
D. Hemofilia alteraciones. Se realiza toracocentesis diagnóstica,
E. Esclerosis sistémica obteniéndose un líquido serohemático con: pH 7,36,
21. Mujer de 62 años que acude a la consulta por proteínas 4,5 g/dl (sangre 7,5), LDH 850 U/l (sangre
presentar dolor en el borde interno del antebrazo y a 150), glucosa 40 mg/dl y amilasa en rango de
nivel del dedo anular y meñique de la mano derecha, normalidad. El Gram no muestra gérmenes. La
también explica hormigueos, frialdad y pérdida de citología muestra pre- dominio de linfocitos. Ante la
sensibilidad. ¿Qué tipo de neuropatía por compresión sospecha diagnóstica, ¿cuál sería el siguiente paso a
tiene el paciente? dar?
A. Síndrome canal carpiano A. Repetir de nuevo la toracocentesis, para descartar
B. Atrapamiento de nervio mediano y cubital a nivel de que sea un empiema.
la muñeca B. Realizar biopsia pleural.
C. Compresión proximal del nervio cubital C. Realizar toracoscopía.
D. Sección nerviosa cubital D. Iniciar tratamiento antibiótico y esperar respuesta.
E. Compresión del nervio radial E. Esperar el resultado del cultivo de Lowenstein.
22. Mujer de 72 años, con debilidad marcada que le 26. Paciente de 25 años de edad, hace dos semanas
impide caminar, diarreas de 3 meses de duración, presentó cuadro de influenza. Desde ayer parestesias
pérdida de peso 15 kg en un año. Antecedentes no en miembros infe- riores y debilidad progresiva que
contributorios. Al examen: palidez severa de piel y imposibilita deambular. Hoy pérdida de fuerza en
mucosas, glositis, edema de miem-bros inferiores. miembros superiores y dificultad respiratoria. Al
Hb: 4g%; VCM: 130, hcm: 37; leucocitos: 2,500; examen: cuadriparesia hipotónica e hiporreflexia. El
plaquetas: 25.000; creatininemía: 1mg%; glucosa: diagnóstico probable es:
160 mg%; lámina periférica: macrocitosis y A. Polimiositis aguda.
polisegmentación de neutrófilos. ¿Cuál es su B. Síndrome de Guillain- Barré.
posibilidad diagnóstica?: C. Polineuropatía carencial.
A. Leucemia aguda aleucémica. D. Mielitis transversa.
B. Anemia megaloblástica por déficit de folatos. E. Botulismo.
C. Aplasia medular. 27. Paciente de 40 años con disnea progresiva de
D. Anemia megaloblástica. esfuerzo hasta disnea de decúbito, pulso con arritmia
E. Púrpura trombótica trombocitopénica. completa, frecuen- cia cardiaca 156pm, soplo
23. Una niña de 12 años presenta como molestiaprincipal diastólico de tonalidad grave en area mitral PA
ptosis palpebral y visión doble. El examen revela 115/80 mmHg, crepitantes en ambas bases
debilidad de los músculos extraoculares, de los pulmonares ¿cuál es el diagnóstico más probable?
músculos del cuello y de los músculos faciales. Se A. Insuficiencia aortica
observa debilidad progresiva ante las contracciones B. Coartación de aorta
musculares sostenidas o repetidas. Cuando se solicita C. Estenosis mitral
a la paciente que mantenga la mirada dirigida hacia D. Insuficiencia mitral
arriba, se observa ptosis progresiva. La debilidad E. Estenosis aortica
muscular mejora con el reposo. Los estudios de 28. Un varón de 35 años lleva varios años trabajando de
tiroides son normales. El diagnóstico más probable pintor y ahora presenta cefalea de varios meses de
es: evolución, difi- cultad de concentracion y artralgias.
A. Parálisis de Bell. En la exploración física neuropatía periférica.
B. Miastenia gravis juvenil. Laboratorio: anemia normocitica
C. Ptosis congénita. ¿cuál de estas pruebas de laboratorio confirmaría el
D. Parálisis del nervio facial congénita. diagnostico?
E. Miositis osificante progresiva. A. Concentración de plomo en el suero
B. Concentración de plomo en la sangre
24. Un paciente con cerebro esclerosis que presenta C. Concentración de arsénico en la sangre
disturbios mnésicos, hipomimia, bradicinesia, D. Concentración de arsénico en el suero
hipertonía, Hiperre- E. Concentración de cadmio en el suero
flexia profunda, Babinski bilateral, temblor de actitud, voz 29. Paciente mujer de 45 años con enfermedad de Crohn
“monótona”, corresponde a la forma clínica y una pequeña fistula intestinal desarrolla tetania
siguiente: durante la 2da semana de nutrición parenteral. Los
A. Pseudobulbar. resultados de laboratorio incluyen del Ca 8.2 meq/L;
Na 135 meq/L ; K 3.2 meq/L; Cl 103 meq/L ; PO4 2.4 C. Klebsiella pneumoniae
meq/L ; albumina 2.4 ; pH 7.48 ; bicarbonato 25 D. Streptococcus pneumoniae
meq/L. La causa más probable de la tetania del E. Neisseria meningitidis
paciente es: 35. En relación a las etapas de desarrollo mamario, según
A. Hipomagnesemia Tanner, la presencia de montículos mamarios,
B. Hiperventilación incremento de la areola y desarrolla de pezón en
C. Hipocalcemia grado variable, corresponde a:
D. Deficiencia esencial del ácido graso A. Etapa 5
E. Convulsiones focales B. Etapa 3
30. Un varón de 30 años ha experimentado dificultades C. Etapa 2
para la deglución tanto de sólidos como de liquidos D. Etapa 0
en los últimos E. Etapa 1
6 meses. ¿Cuál es el diagnóstico más probable? 36. Un recién nacido presenta un área edematosa de
A. Carcinoma esofágico contenido seroso en la zona de presentación.
B. Anillos de Schatzki Presenta alguna pete- quia aislada, pero no existe
C. Acalasia colección de sangre. Se observa también
D. Estenosis esofágica benigna cabalgamiento de parietales. Este cuadro desapa-
E. Esófago de Barret rece en 24 horas. ¿Cómo se denomina dicho cuadro?
31. Un recién nacido postmaduro con el antecedente de A. Cefalohematoma
asfixia intraparto presenta en el momento del B. Caput venoso
nacimiento una atelectasia pulmonar asociada a C. Necrosis adiposa del tejido subcutáneo
neumotórax y neumomediastino, presentando en la D. Hemorragia subaponeurótica difusa
radiología condensaciones algo- donosas E. Caput succedaneum
diseminadas. ¿Cuál sería su diagnóstico? 37. Un bebé a término tiene una clínica de vómitos
A. Persistencia de circulación fetal tardíos, retraso en la eliminación de meconio,
B. TTRN episodios alternantes de diarrea-estreñimiento y
C. SALAM enterocolitis. Se le practica un tacto rectal
D. Neumonía estafilocócica observándose hipertonía con recto vacío. Usted
E. EMH decide hacer una biopsia de colon con el hallazgo de
32. Un recién nacido varón presenta una malformación ausencia de células ganglionares de los plexos
congénita que consiste en agenesia de timo y mioentéricos en un segmento colónico. ¿Cuál es el
paratiroides, arcoaórtico derecho, atresia esofágica y segmento que con más frecuencia se ve afectado en
anomalías faciales. ¿Qué síndrome polimalformativo esta enfermedad?
sospecharía? A. Colon transverso
A. Martin-Bell B. Recto-sigma
B. Potter C. Colon descendente
C. Pierre-Robin D. Recto
D. Prune-Belly E. Colon ascendente
E. Di George 38. Una madre acude con su hijo de 2 semanas al
33. Un varón de 10 meses presenta desde hace unos días pediatra ya que le encuentra irritable, rechaza el
signos de coriza y febrícula, hace unas horas alimento, tiene diarrea mucosanguinolenta y tinte
comienza con un episodio agudo de dificultad ictérico. A la exploración presenta distensión
respiratoria con tiraje intercostal y sibilancias en la abdominal con palpación dolorosa y aspecto séptico.
auscultación. Los padres nos comen- tan que nunca En una radiografía descubre una perforación
había sufrido un episodio similar. ¿Cuál es el agente intestinal. ¿Cuál será la causa más frecuente de este
etiológico que con más frecuencia relaciona este hallazgo teniendo en cuenta que el niño todavía
cuadro clínico? pertenece al período neonatal?
A. Virus sincitial respiratorio A. Invaginación intestinal
B. Parainfluenzae B. Estenosis pilórica
C. Adenovirus C. Enterocolitis necrotizante
D. Mycoplasma D. Celíaca precoz
E. Idiopático E. Apendicitis aguda
34. Varón de 3 años de edad que, encontrándose 39. Un niño de 7 meses deshidratado, con vómitos,
previamente bien, comienza de forma brusca con diarrea y fiebre; nos llega la siguiente analítica: Hb
fiebre, dolor de gargan- ta, estridor, disnea, acúmulo 11,6 g %; Hto: 39%; Leucocitos: 14.900 mm3
de secreciones orales y emisión de saliva por la boca. (Neutrófilos: 63%; Linfocitos: 30%; Monocitos: 7%);
Un hermano de 2 años lleva 72 horas ingresado por Plaquetas: 322.000 mm3; Os- molaridad: 295 mOs/l;
una meningitis bacteriana. El agente etiológico de Na: 137 mEq/l; K: 5.6 mEq/l; Ca iónico: 1,2 mmol/l;
ambos cuadros clínicos será con más probabilidad: pH: 7,20; pCO2: 25 mmHg; HCO3: 11 mEq/l; E.B.: -19
A. Providencia rettgeri mEq/l; Lactato: 5,3 mmol/l, Creatinina: 4,2 mg %.
B. Haemophilus influenzae
¿Cuál es la valoración metabólica de esta deshi- C. Anaplasia
dratación? D. Metaplasia
A. Deshidratación isotónica con acidosis mixta. E. Hiperplasia
B. Deshidratación hipotónica con acidosis metabólica. 46. Un paciente ha tenido un resultado positivo en un
C. Deshidratación isotónica con acidosis metabólica. Mantoux que se la ha realizado por sospecha de
D. Deshidratación hipertónica con acidosis metabólica. tuberculosis.
E. Deshidratación isotónica con acidosis respiratoria. ¿Cómo se denomina la probabilidad de que este individuo
40. Un niño de 2 años, previamente saludable, desarrolla, tenga realmente la enfermedad cuando el test
fiebre de 40,6º C y presenta convulsión generalizada realizado ha resultado positivo?
que dura 60 seg. El examen físico revela una infección A. Especificidad
respiratoria alta y los estudios de laboratorio son B. Riesgo atribuible
normales, mostrándose asintomático al día siguiente. C. Valor predictivo positivo
Ud. Explica a los padres que la convulsión es febril y D. Sensibilidad
que tiene buen pronóstico, no ne- cesitando E. Valor predictivo negativo
quimioprofilaxis con fenobarbital. Su opinión está 47. Entre los factores de riesgo modificables de la
basada en: aterosclerosis se encuentran el tabaquismo, la
1) Estuvo previamente sano hasta el inicio de la obesidad, la vida sedenta- ria, los trastornos de los
convulsión. lípidos, la hipertensión, la resistencia a la insulina,
2) La convulsión ocurrió luego del año de edad. entre otros; como factores no modificables están la
3) El episodio agudo duró menos de 15 minutos. edad, el sexo masculino y factores genéticos. Un
4) Al día siguiente no presentó manifestación varón de 35 años sufre un infarto agudo de
neurológica residual miocardio. Es fumador desde hace 20 años por lo que
5) La convulsión estuvo asociada a una infección se le aconseja dejar de fumar. Con esto usted realiza
respiratoria alta benigna. una:
A. La respuesta correcta es: B. 1,2 y 3. A. Promoción de la salud
C. 1,3 y 5. B. Atención curativa
D. 2,4 y 5. C. Prevención terciaria
E. 2,3 y 4. D. Prevención primaria
F. 1,4 y 5. E. Prevención secundaria
41. ¿A los cuántos meses de edad se recomienda colocar 48. Se determina la sensibilidad de la prueba a partir de
la vacuna contra la hepatitis A? la siguiente fórmula
A. 4 Resultado de la prueba Enfermos
B. 9 SI NO
C. 12 Positiva a b
D. 2 Negativa c d
E. 6 A. a/a+c
42. La asociación apropiada de antibióticos para tratar la B. c/a+c
sepsis en un recién nacido es: C. d/c+d
A. Vancomicina + Amoxicilina D. b/b+d
B. Amikacina + Penicilina E. a/a+b
C. Amikacina + Ampicilina 49. ¿Qué tipo de estudio epidemiológico emplearía en el
D. Gentamicina + Cefalotina estudio de una enfermedad rara, crónica, de larga
E. Cloranfenicol + Ampicilina evolución, y de larga inducción?
43. ¿Cuál de los siguientes cambios es un fenómeno de A. Transversal
adaptación? B. Cohortes
A. Autolisis C. Test de Welch
B. Metaplasia D. Casos y controles
C. Apoptosis E. Incidencia
D. Cariolisis 50. Un médico se encuentra realizando SERUMS en
E. Cariorrexis Cieneguilla y desea realizar una actividad de
44. ¿Cuál de las siguientes vitaminas se comporta como promoción en salud. Su equipo de salud le envía el
hormona esteroidea? siguiente listado de actividades. Considerando el
A. A concepto de promoción. ¿Cuál es la actividad que
B. B5 debe elegir?
C. E A. Consejería grupal de adolescentes sobre la
D. K transmisión del VIH.
E. D B. Charla a la comunidad en nutrición saludable.
45. ¿Cuál de las siguientes alteraciones caracteriza a las C. Campaña de vacunación en menores de 5 años.
células malignas? D. Búsqueda en la comunidad de sintomáticos
A. Atrofia respiratorios.
B. Hipertrofia
E. Control vectorial para disminuir casos de C. Malaria
enfermedades metaxénicas. D. Brucelosis
51. Si el valor de una canasta básica familiar en Tarma es E. Bartonelosis
de S/. 1500 y el valor de la canasta sólo con alimentos 57. Mujer de 32 años llega a consulta por cefalea intensa,
es de S/750, señale usted en que grupo se encontraría vómitos y rigidez de nuca. El LCR presenta pleocitosis
la familia “X” cuyo ingreso promedio mensual es de linfoci- taria. ¿Cuál es el diagnóstico más probable?
S/. 550 mensual: A. Meningitis por hongos
A. Quintil 4. B. Meningitis viral
B. Pobre no extremo. C. Meningitis bacteriana
C. Extrema pobreza. D. Hemorragia subaracnoidea
D. Pobreza. E. Meningitis por VIH
E. Clase media. 58. Paciente portador de prótesis cardiaca metálica,
52. En el análisis FODA de un hospital, “los permanentes alérgico a la penicilina, al que se le va a realizar
conflictos entre la dirección y el cuerpo médico” broncoscopia diag-nóstica. ¿Qué profilaxis antibiótica
corresponde a: utilizaría?
A. Debilidad. A. Cefalexina
B. Amenaza. B. Eritromicina
C. Fortaleza. C. Clindamicina
D. Oportunidad. D. Vancomicina
E. Recurrencia. E. Gentamicina
53. Varón de 18 años de edad, presenta un forúnculo en 59. Anciana de 70 años sin antecedente de DM es llevada
la cara y a los pocos días aparece edema con signos a emergencia por presentar estado mental alterado,
de inflama- ción en la rodilla izquierda, asociado a cuadro de deshidratación severa, el familiar solo
fiebre. A los 3 días aparece tos, disnea, dolor torácico, informa que días antes presentó polidipsia, poliuria.
fiebre elevada y signos de toxicidad sistémica. Una El diagnóstico planteado seria.
radiografía de tórax muestra múltiples infiltrados A. DM tipo 1
nodulares y neumatocele. El agente etiológico más B. DM tipo 2
probable es: C. Cetoacidosis diabética
A. Staphylococcus aureus D. Coma hiperosmolar no cetósico
B. Peptostreptococcus magnus E. Hipoglicemia
C. Streptococcus pyogenes 60. Un hombre asintomático presenta en dos análisis de
D. Pseudomonas aeruginosa sangre rutinarios glucemias basales de 132 y
E. Mycobacterium tuberculosis 130mg/dl. ¿Cuál será la actitud correcta?
54. Paciente varón de 18 años que presenta limitación A. Se le pide una curva de glucemia para diagnostico
funcional y dolor en región de columna dorsal, tiene B. Se inicia tratamiento con antidiabéticos orales
como ante- cedente haber sufrido de “pleuresia” y C. Se inicia tratamiento con dosis de insulina NPH
enfermedad venérea .Es consumidor de queso fresco. D. Se le pide nuevos análisis dentro de un año
Radiográficamente se evidencia lesión en columna E. Se le diagnostica diabetes mellitus tipo 2 y se le inicia
D12 con destrucción de las vértebras contiguas tratamiento con dieta
preservación de pedículos. El diagnóstico más 61. Paciente varón de 65 años con antecedente de
probable seria diabetes mellitus en control regular que acude a su
A. Brucelosis cita con los siguientes resultados: glicemia, creatinina
B. Neoplasia sérica y electrolitos en límites normales, en el examen
C. Espondilitis anquilosante de orina se aprecia una cruz de proteínas. En el mes
D. Mal de pott previo sus presiones arteriales han fluctuado entre
E. Sd de Reiter 150-160 / 70-80. ¿Cuál sería la mejor terapia
55. Paciente con antecedente de haber tenido impétigo farmacológica para su hipertensión arterial?
luego hipertensión, edema de párpados, oliguria, A. Calcio antagonistas
ictericia, hígado aumentado de tamaño ¿Cuál es el B. Beta bloqueadores
diagnóstico? C. Inhibidores de la enzima convertidora de
A. Enfermedad de Berger angiotensina
B. Glomerulonefritis post estreptocócica D. Diuréticos tiazídicos
C. GMN rápidamente progresiva E. Diuréticos ahorradores de potasio
D. Glomerulopatía membranosa 62. Paciente varón fumador de 68 años, que presenta tos
E. Cambios mínimos con expectoración matutina esporádica desde hace
56. Paciente de 54 años de edad, procedente de Pucallpa. 15 años y que desde los últimos 3 la tos y la
Acude a hospital de la capital porque tiene fiebre expectoración se han incrementado añadiéndose
intermitente y palidez. ¿Cuál es el diagnóstico más disnea al realizar grandes esfuerzos, las molestias
probable? ceden parcialmente con el uso de broncodilatadores.
A. Hepatitis C ¿Cuál es la prueba que Ud. le solicitaría primero para
B. Fiebre tifoidea completar el diagnóstico?
A. Tomografía axial computarizada de tórax A. Pancreatitis aguda
B. Espirometría B. Edema cerebral
C. Prueba de difusión de PCO2 C. Lesión de colon
D. Búsqueda de bacilos de Koch en el esputo D. Perforación ileal
E. Test de hiperreactividad bronquial con metacolina E. Hemorragia
63. Paciente mujer de 65 años que acude a la consulta 69. Paciente de 50 años de edad que consulta por dolor
con dolor en rodilla derecha de 15 días de evolución, en la fosa renal, poliaquiuria, disuria y hematuria. En
que se incre- menta al caminar y cede al reposo, sin el análisis de orina se observa piuria y pH ácido con
historia de trauma previo; tiene un índice de masa cultivos repetidamente negativos. ¿Cuál sería la
corporal en 32, aumento de volumen de la rodilla primera posibilidad diag- nóstica, de entre las
derecha pero sin enrojecimiento ni incremento de siguientes?
temperatura; a la movilización pasiva se percibe A. Pielonefritis aguda.
crujido. ¿Cuál es el diagnóstico más probable de la B. Síndrome nefrítico.
paciente? C. Tuberculosis genitourinaria.
A. Monoartritis infecciosa D. Prostatitis aguda.
B. Artropatía por cristales E. Carcinoma renal de células claras.
C. Osteoartritis de rodilla 70. A una mujer de 49 años de edad se le realiza un
D. Artritis reumatoide de rodilla Papanicolaou y el resultado se informa como
E. Meniscopatía de rodilla derecha neoplasia intracervical III o lesión intraepitelial
64.Ante un varón con antecedentes de tabaquismo y escamosa de alto grado, usted indicaría el siguiente
alcoholismo que consulta porque desde hace meses procedimiento :
nota una sen- sación de cuerpo extraño al tragar, A. Colposcopia y biopsia dirigida
parestesias faríngeas, pinchazos en los oídos y una B. Cono frio diagnostico
adenopatía cervical. ¿Qué debemos sospechar? C. Histerectomía total
A. Cáncer de las cuerdas vocales D. Inspección visual con ácido acético
B. Cáncer subglótico E. Aplicación del test de Schiller
C. Laringitis aguda 71. Ingresa una primigesta de 18 años de edad con 32
D. Cáncer de cavum semanas de gestación refiriendo cefalea intensa y
E. Cáncer de supraglotis epigastralgia tipo opresivo. Al examen tiene presión
65. Ante un niño de 5 años con un cuadro de hipertensión arterial 180/120 mm Hg. No hay dinámica uterina,
endocraneal, alteraciones visuales e hipotalámicas, presenta movimientos fetales y los latidos del feto
que pre- senta una radiografía lateral del cráneo con fluctúan entre 130 y 150, la altura uterina es de 25
calcificaciones en forma de paréntesis a nivel cms. con feto en LCI. Los reflejos patelares están en
supraselar. ¿Cuál será su diagnóstico presuntivo? 3+/4+. ¿Cuál es el manejo farmacológico inmediato?
A. Meduloblastoma A. Sulfato de magnesio 4 gms EV en 20’, nifedipino 10
B. Craneofaringioma mg VO
C. Adenoma hipofisario B. Diazepam 10 mg EV, labetalol 200 mg EV
D. Glioma del nervio óptico C. Sulfato de magnesio 10 gms IM, alfa metil dopa 1 gm
E. Pinealoma productor de hidrocefalia VO
66. Paciente de 65 años, con cuadro de ictericia D. Fenitoina 900 mg VO, hidralazina 10 mg EV
progresiva, coluria y dolor abdominal. En las últimas E. Diazepam 5 mg EV, nifedipino 10 mg Sublingual
58 horas se agregan fiebre y escalofríos. ¿Cuál es el 72. Gestante a término en fase activa de trabajo de parto,
diagnóstico más probable? presenta ruptura de membranas hace 30 minutos, y
A. Absceso hepático. usted de- tecta frecuencia cardiaca fetal basal de 132
B. Colangitis aguda. latidos por minuto. En 3 oportunidades ha
C. Hepatitis aguda. presentado latidos fetales en 110 latidos por minuto
D. Neoplasia de la vesícula biliar. durante 30 segundos fuera de las contracciones y
E. Ninguna de las anteriores. asociado a movimiento del bebe. ¿Cuál es la causa
67. Varón de 56 años de edad, desde hace 3 horas más probable de estas desaceleraciones en la
presenta dolor en la fosa ilíaca izquierda, fiebre, frecuencia cardiaca fetal?
vómitos. Niega moles- tias urinarias. ¿Cuál es la A. Compresión de cabeza fetal
primera impresión diagnóstica?: B. Acidemia fetal
A. Intoxicación aguda por plomo C. Compresión de cordón umbilical
B. Apendicitis aguda D. Hipoxia fetal
C. Cólico ureteral izquierdo E. Desprendimiento de placenta
D. Crisis hemolítica 73. Paciente en su tercer día de puerperio de cesárea por
E. Diverticulitis colónica aguda expulsivo prolongado, la intervención quirúrgica
68. Paciente de 24 años de edad, que sufre un discurrió sin complicaciones. El día de hoy presenta
politraumatismo por caída del segundo piso y a las 6 temperatura 38.50 C, pulso 120 por minuto,
horas fallece. ¿Cuál cree usted que es la causa más frecuencia respiratoria 20 por mi- nuto, presión
probable de muerte?: arterial 120/70 mmHg, llenado capilar < 2 segundos.
Luce en regular estado, al examen muestra mamas exclusivamente. ¿Qué anticonceptivo seria el
induradas y dolorosas, útero doloroso a la palpación apropiado en este caso?
con altura 3 cms. por encima del ombligo, la herida A. Anticonceptivo Oral Combinado
no muestra flogosis, loquios escasos sin mal olor, el B. Ligadura Tubárica Bilateral
resto del examen es normal. ¿Cuál es el diagnóstico C. Preservativos y espermicida
más probable? D. Progestágenos Orales
A. Atelectasia pulmonar E. Anticonceptivos Inyectables mensuales
B. Flebitis 79. Si un niño de 8 años hace shock anafiláctico por
C. Congestión mamaria penicilina, el medicamento de elección es:
D. Endometritis A. Clorfenamina
E. Infección urinaria B. Suero fisiológico
74. Primigesta de 34 semanas diagnosticada de infección C. Epinefrina
por VIH a las 18 semanas iniciándose tratamiento con D. Dexametasona
zidovudi- na y lamivudina. Acude a control prenatal. E. Dopamina
¿Cuál de las siguientes alternativas sería la más 80. La medicación que permite mejoría rápida de un
apropiada para el manejo del parto? lactante con crup infeccioso grave es:
A. Amniocentesis a las 38 semanas para madurez A. Adrenalina en nebulización
pulmonar B. Antibióticos
B. Cesárea segmentaria a las 40 semanas C. Dexametasona vía endovenosa
C. Inducción de parto a las 39 semanas D. Ambiente húmedo
D. Medir carga viral a las 36 semanas E. Salbutamol en inhalación
E. Medir leucocitos CD4 a las 36 semanas 81. El diagnóstico de desnutrición aguda se hace en base
75. Primigesta de 35 semanas acude a emergencia a la relación:
refiriendo hace 30 minutos dolor abdominal y A. Peso/talla2
sangrado vaginal oscuro de 300 cc. Al examen detecta B. Peso/edad
Presión arterial 90/60 mm de Hg, pulso 120 por C. Peso/talla
minuto, respiraciones 24 por minuto, afebril. D. Talla/edad
Abdomen con altura uterina de 40 cm, contracciones E. Talla/peso
uterinas cada 2 minutos, 50 segundos de duración, 82. La panencefalitis esclerosante subaguda de Dawson
intensidad 4 cruces, latidos fetales 100 a 120 por tiene como etiología al virus de:
minuto; en el examen con especulo sangrado a través A. Rubéola
del cervix, el cual se encuentra cerrado y largo. Se B. Sarampión
indica vía EV, hemograma grupo y Rh, prueba cruzada C. Influenza
y perfil de coagulación. D. Varicela
¿Cuál debe ser la siguiente conducta? E. Hepatitis B
A. Ecografia transvaginal 83. En las niñas, el agente causal más frecuente de
B. Perfil biofísico infección urinaria es:
C. Inducción de parto con oxitocina A. Escherichia coli.
D. Amniotomia y estimulación con oxitocina B. Ureoplasma.
E. Cesárea C. Parvovirus.
76. Paciente mujer de 27 años de edad, con IMC de 30, D. Campilobacter.
presenta oligomenorrea, acné, hirsutismo y desde E. Chlamydia trachomatis.
hace 3 años busca salir gestando. ¿Cuál sería el 84. Un niño de 4 años de edad con un cuadro de fiebre,
diagnóstico de la paciente? conjuntivitis, adenopatía cervical, lengua de fresa y
A. Hipotiroidismo edema con descamación membranosa de los dedos
B. Hiperplasia Suprarrenal de manos y pies, orientará nuestro diagnóstico hacia:
C. Síndrome de Ovario Poliquistico A. Enfermedad de Behcet
D. Deficiencia de la Fase Lútea B. PAN
E. Hiperprolactinemia C. Forma juvenil del lupus discoide
77. Mujer de 53 años de edad, quien cursa con sequedad D. Enfermedad de Still
vaginal, disminución del deseo sexual, disuria y E. Enfermedad de Kawasaki
poliaquiuria con urocultivos negativos. Estos 85. La saturación de oxígeno media en la arteria
síntomas están asociados a disminución de: pulmonar es aproximadamente de:
A. Progesterona A. 0.33
B. FSH B. 0.75
C. Estrógenos C. 0.5
D. LH D. En personas sanas es superior al 95%
E. Prolactina E. Depende de varios factores, pero se mueve en el
78. Paciente mujer de 21 años de edad, con pareja intervalo 80-95%
estable, quien tuvo su segundo parto vaginal hace 3 86. Su alteración es muy sensible en la fase inicial del
días con recién nacido sano. Planea amamantar trastorno tiroideo a veces el único cambio:
A. TSH técnica de enfermería que realice una visita
B. T3 libre domiciliaria, mientras que el médico manda a la
C. T3 misma técnica que lo ayude en consultorio externo.
D. T4 libre ¿Estos problemas a que componentes de la
E. T4 administración corresponden?:
87. ¿Cuál de las siguientes acciones corresponde a la A. Planificación.
vitamina E? B. Organización.
A. Cicatrización de las heridas C. Control.
B. Anitoxidante D. Dirección.
C. Síntesis de algunos factores de la coagulación E. Evaluación.
D. Síntesis de colágeno 95. Un centro de salud programó para el año 2009
E. Regularización de los niveles de calcio y fósforo en realizar 2 000 controles de crecimiento y desarrollo,
sangre para 1 500 niños menores de 5 años, utilizando 800
88. Al nacer, el volumen sanguíneo es: horas enfermera. AI finalizar el año atendió 200
A. 65 cc. x Kg. consultantes nuevos, 300 reingre- santes y 500
B. 80 cc. x Kg. continuadores, utilizando 600 horas enfermera. El
C. 110 cc. x Kg. porcentaje de avance de meta en atenciones fue:
D. 125 cc. x Kg. A. 50.
E. 150 cc. x Kg. B. 85.
89. El contenido de hierro corporal total en el adulto C. 25.
normal está entre: D. 75.
A. 5 y 10 mg. E. 100.
B. 50 y 100 mg. 96. Se quiere graficar un conjunto de datos cuantitativos
C. 25 y 50 mg. D. 0.5 y 1.5 mg. continuos, trabajados en una escala de razón. ¿Qué
E. 2 y 5 g. tipo de grá- fico sería más adecuado?:
90. El hierro molecular, Fe, es: A. Gráfico de barras.
A. Almacenado primariamente en el bazo. B. Gráfico circular.
B. Excretado en la orina como Fe++. C. Histograma.
C. Almacenado en el cuerpo en combinación con D. Pictograma.
ferritina. E. Gráfico semi-logarítmico.
D. Absorbido en el intestino por la transferrina. 97. En una muestra de 50 estudiantes la estatura
E. Absorbido en la forma férrica, Fe+3. promedio es de 1,65 m. Luego, considerando un
91. La vitamina B12, se absorbe en el: intervalo de confianza
A. Íleon distal. del 95%, el límite inferior es de 1,56 m. y el superior:
B. Yeyuno proximal. A. 91,74 m.
C. Duodeno. B. 1,68 m.
D. Colon. C. 1,70 m.
E. Yeyuno medio. D. 1,77 m.
92. El sistema nervioso simpático que sale de la médula E. No se puede calcular con la información disponible.
espinal: 98. Una encuesta determina que las mujeres consumen
A. Cesa de funcionar después de sección del bulbo. bebidas alcohólicas unas 0,65 veces menos que los
B. Contiene solamente fibras adrenérgicas. varones. En este caso, la medición usada es un(a):
C. Contiene solamente fibras colinérgicas. A. Proporción.
D. Incluye una sinapsis gangliónica. B. Fracción aritmética.
E. Ninguna de las anteriores. C. Porcentaje.
93. Se dice que el sistema extrapiramidal está D. Mediana.
conformado por estratos o niveles superpuestos. E. Razón.
Estos niveles son: 99. En un país “X”, durante los últimos cinco años, el
1. Cuerpo estriado. número total de casos de malaria mantiene una
2. Núcleo óptico. tendencia ascen- dente, superior a la del crecimiento
3. Hipotálamo. de la población. Este cambio anual en la frecuencia
4. Núcleos de la región subopto-estriada. total de la malaria, tiene su origen en la modificación
5. Sustancia reticular. Es cierto solamente: A. 1, 2. de su:
B. 2, 3. A. Incidencia.
C. 3, 4. B. Letalidad.
D. 1, 4. C. Mortalidad.
E. 1, 3. D. Riesgo atribuible.
94. En un centro de salud la enfermera da una charla E. Riesgo relativo.
sobre control prenatal a un grupo de madres, la 100.La tasa de incidencia de leucemia, en población
semana siguiente la obstetriz da la misma charla al expuesta a radiación ionizante es de 4 en mil y la
mismo grupo de madres. La enfermera ordena a una
población no expues- ta de 0,5 en mil. ¿Cuál es el 108.¿Cuál de los siguientes cambios es un fenómeno de
riesgo atribuible al factor de exposición?: adaptación?
A. 8,0. A. Autolisis
B. 4,5. B. Metaplasia
C. 3,5. C. Apoptosis
D. 4,0. D. Cariolisis
E. 0,5. E. Cariorrexis
101.Son las células encargadas de la contracción de la 109.¿Cuál de las siguientes alteraciones caracteriza a las
herida, durante el proceso de cicatrización: células malignas?
A. Atrofia
A. Neutrófilos. B. Hipertrofia
B. Polimorfonucleares. C. Anaplasia
C. Fibroblastos. D. Metaplasia
D. Macrófagos. E. Hiperplasia
E. Fibrocitos. 110.¿Cuál de las siguientes acciones corresponde a la
102.La cirugía moderna del cáncer gástrico, tiene vitamina E?
fundamento en la disección de las diferentes A. Cicatrización de las heridas
estaciones ganglionares. La extirpación del grupo N° B. Anitoxidante
9, corresponde a: C. Síntesis de algunos factores de la coagulación
A. Arteria coronaria estomáquica. D. Síntesis de colágeno
B. Arteria esplénica. E. Regularización de los niveles de calcio y fósforo en
C. Arteria hepática. sangre
D. Pedículo hepático. 111.Un centro de salud realizó la evaluación de sus
E. Tronco celíaco servicios correspondientes al año 2009. Población
103.La pepsina es inactivada por: total 12,000 habitan- tes. Atendidos 3000 personas,
A. La secreción acuosa del páncreas. consultas médicas realizadas 6000 ¿cuál es la
B. La concentración gástrica de hidrogeniones. intensidad de uso o concentración (%)?
C. Las prostaglandinas gástricas. A. 1.3
D. El ácido desoxicólico. B. 4
E. La concentración de gastrina. C. 25
104.La mayor absorción de sodio se realiza en: D. 2
A. La porción ascendente del asa de Henle E. 1
B. La porción descendente del asa de Henle 112.Se realizó un estudio para relacionar la ingesta
C. El túbulo proximal rica/pobre en grasas y la existencia / no existencia
D. El túbulo distal posterior de cáncer de próstata en adultos mayores
E. El túbulo colector de 40 años. La observación duró 10 años. ¿Cuál es el
105.¿Cuál es el antibiótico que inhibe la DNA girasa? tipo de estudio de la investiga- ción?
A. Ciprofloxacino A. Casos y controles
B. Penicilina B. Ecológica
C. Amikacina C. Cohortes
D. Clindamicina D. Experimental
E. Ceftriaxona E. Transversa
106.En relación a las fases del potencial de acción 113.En una localidad determinada, durante los recientes
cardíaco: 3 años, el número total de casos de fiebre amarilla
A. La fase 4 en las células marcapaso es estable. mantiene una tendencia ascendente, superior a la del
B. El nodo sinusal es el marcapaso cardíaco en crecimiento de la población. Este cambio anual en la
condiciones normales porque la pendiente de la fase frecuencia total de fiebre amarilla, tiene su origen en
4 es la menor. la modificación de su:
C. La pendiente de la fase 0 de una célula marcapaso es A. Incidencia
mayor que en una célula del sistema de conducción. B. Letalidad
D. En el nodo sinusal, el ion responsable de la fase 0 es C. Mortalidad
el calcio. D. Riesgo atribuible
E. El período refractario relativo engloba la fase 1 del E. Riesgo relativo
potencial de acción. 114.Un estudio observacional en el que se comparan 110
107. Normalmente la válvula ileocecal se cierra en individuos con cáncer de senos paranasales con 220
qué circunstancias: individuos sin la enfermedad, corresponde a un
A. Ocurre una onda peristáltica estudio:
B. Incrementa la presión ileal A. Longitudinal
C. Incrementa la presión colonica B. Caso control
D. Existe estimulación vagal C. De cohortes concurrente
E. Se contrae el ciego D. Transversal
E. Cohortes no concurrente D. Dirección
115.En un distrito rural del país se notificaron durante la E. Evaluación
primera semana de Octubre del 2005, dos casos 120.En un estudio para comprobar la relación entre el
locales o autóc- tonos de rabia humana, enfermedad consumo de tabaco y el cáncer de vejiga, se siguió
que no se observó́ en las últimas tres décadas. Esta durante un año a 1.000 personas, de las cuales 500
situación se define como una: fumaban, y otras 500 no; de las 50 personas que
A. Endemia padecieron cáncer de vejiga durante este tiempo, 45
B. Epidemia fumaban. ¿Cuál es el riesgo individual de enfermar
C. Epizootia entre los expuestos al tabaquismo?:
D. Hiperendemia A. 0,09.
E. Holoendemi B. 0,10.
116.Se desarrolla un estudio con la finalidad de buscar la C. 0,01.
asociación entre el polvo de una mina y la presencia D. 1,00.
de proble- mas respiratorios en la población. Se E. 1,11.
estudian dos poblaciones: Una expuesta al polvo de 121.Niño preescolar presenta bruscamente tos intensa. Al
la mina y la otra de agri- cultores; se hace seguimiento examen físico pulmonar: timpanismo y sibilantes
por un año y se registra la aparición de los casos en unilaterales. Debe sospecharse:
ambas poblaciones. ¿A qué tipo de diseño A. Epiglotitis
epidemiológico corresponde el estudio? B. Síndrome de escape de aire
A. Cohortes C. Cuerpo extraño
B. Casos y controles D. Neumonitis intersticial
C. Cuasi experimental E. Crisis asmática.
D. Longitudinal 122.Un niño de 3 años de edad presenta hace 3 días un
E. Transveral cuadro catarral leve, con fiebre leve, adenopatías
117.Durante su ejercicio profesional, en el centro de salud retroauriculares y exantema maculopaular rosado de
se presenta un elevado número de casos de diarrea inicio en cara. El diagnóstico más probable es:
acuosa, neumonía, desnutrición infantil y parasitosis. A. Eritema infeccioso.
De acuerdo a su observación, ¿qué acciones B. Escarlatina.
preventivas recomendaría? C. Exantema súbito.
A. Incrementar la compra de medicamentos para el D. Rubéola.
centro de salud E. Sarampión
B. Solicitar más personal profesional y técnicos de salud 123.Un lactante de seis meses, amamantado con leche
C. Mejorar los servicios de emergencia del materna, presenta desde hace 5 horas episodios
establecimiento repetitivos de llanto intenso y vómitos. Presenta
D. Ampliar el número de camas del establecimiento también desde hace 2 horas dos deposiciones
E. Acceso a agua potable, mejoramiento de la vivienda semilíquidas y sanguinolentas con moco. No se
y nutrición refiere fiebre. El diagnóstico más probable es:
118.En el análisis situacional de salud de un A. Diarrea por Shigella.
establecimiento se encuentra que su comunidad de B. Diarrea por Campylobacter.
referencia carece de agua potable, el año anterior se C. Divertículo de Meckel.
presentaron 40 casos de cólera con 8 fallecidos, y el D. lnvaginación intestinal.
establecimiento no tiene protocolos de atención. E. Alergia a la proteína de leche.
Según el análisis FODA institucional, la situación 124.Un recién nacido, de seis horas de vida, presenta
mencionada corresponde a: párpados edematizados y conjuntivas enrojecidas
A. Oportunidades y amenazas con secreción ocu- lar clara. El diagnóstico más
B. Oportunidades y fortalezas probable es:
C. Debilidades y fortalezas A. Conjuntivitis química
D. Amenazas y debilidades B. Dacriocistitis
E. Debilidades y oportunidades C. Oftalmia neumococica
119. En un centro de salud la enfermera da una D. Oftalmia gonocócica
charla sobre control prenatal a un grupo de madres, E. Conjuntivitis por clamydia
la semana siguiente la obstetriz da la misma charla al 125.Un lactante de 1700 gr sufrió asfixia al nacimiento y
mismo grupo de madres. La enfermera ordena a una después de maniobras de reanimación presentó
técnica de enfermería que realice una visita numerosos episo- dios apneicos. Al tercer día de vida
domiciliaria, mientras que el médico manda a la empezó a vomitar y presentó distensión abdominal
misma técnica que lo ayude en consultorio externo. con evacuaciones sanguinolen- tas. El diagnóstico
¿Estos problemas a que componentes de la más probable es:
administración corresponden? A. Intususcepción
A. Planificación B. Enterocolitis necrotizante
B. Organización C. Vólvulo
C. Control D. Aganglionosis
E. Gastroenteritis. D. Administrarle manitol.
126.RN pretérmino que, a las 6 h de vida, presenta quejido E. Administrarle dexametasona.
espiratorio a la auscultación, ausencia de retracción 132.La persistencia del sexto arco aórtico izquierdo y que
esternal, tiraje intercostal leve con movimientos se manifiesta en el periodo neonatal por dificultad
toraco-abdominales sincrónicos y sin aleteo nasal. Su respiratoria,
puntuación en el Score de Silverman será: precordio hiperdinámico y pulsos periféricos amplios es
A. 2 la:
B. 3 A. Atresia pulmonar con septum interventricular
C. 4 intacto.
D. 5 B. Persistencia del conducto arterioso.
E. 6 C. Comunicación interventricular.
127.Un recién nacido de parto vaginal y peso al nacer de D. Conexión anómala de las venas pulmonares.
4 kilos, presenta al examen: deformación de la cabeza E. Tetralogía de Fallot.
que sobre- pasa las suturas craneales y que deja fóvea 133.En un neonato que en las primeras horas de vida
a la presión. ¿Cuál es el diagnóstico más probable? presenta vómitos de contenido biliar, y cuya
A. Caput succedaneum radiografía de tórax y abdomen muestra el signo de
B. Céfalohematoma la “doble burbuja”, el diagnóstico más probable es:
C. Fractura de cráneo A. Estenosis congénita del píloro.
D. Rodete B. Atresia duodenal.
E. Hidrocefalia C. Invaginación intestinal.
128.Un lactante de seis meses y sin antecedentes de D. Hernia diafragmática.
importancia, presenta coriza y fiebre y tos desde hace E. Malrotación intestinal.
tres días. Desde hace 12 horas se agrega respiración 134.Un escolar de seis años que presenta temperatura de
rápida y al examen se auscultan roncantes y 39 grados C., movimientos tónicos del miembro
subcrepitantes. ¿Cuál es el diagnós- tico más superior dere- cho, midriasis y pérdida del estado de
probable? alerta, tiene una crisis convulsiva de tipo:
A. Neumonía A. Tónico.
B. Bronquiolitis B. Mioclónico.
C. Laringotraqueitís C. Parcial simple.
D. Rinofaringitis D. Parcial compleja.
E. Crisis asmática E. Febril compleja.
129.Un niño de 3 años traído a emergencia por haber 135.Una lactante de 6 meses que tiene potasio sérico de
presentado convulsiones tónico-clónicas 7.5 mEq/L y que además presenta arritmia cardiaca,
generalizadas que duraron 5’. Al examen: T: 40°C, se debe ser tratada con:
auscultan roncantes en ambos campos pulmonares y A. Bicarbonato de sodio, glucosa e insulina.
presenta déficit motor en miembro in- ferior derecho B. Gluconato de calcio, salbutamol inhalado y glucosa
y somnolencia post convulsión. No tiene con Insulina.
antecedentes relevantes. ¿Cuál es el diagnóstico más C. Glucosa y furosemida.
probable? D. Bicarbonato de sodio, sulfato de magnesio y
A. Convulsión febril compleja furosemida.
B. Episodio de escalofríos E. Resinas de intercambio iónico, glucosa y furosemida.
C. Meningoencefalitis bacteriana aguda 136.Un niño de 10 años ingresa al hospital por presentar
D. Convulsión febril simple un cuadro clínico de tres semanas de evolución
E. Convulsión asociada a fiebre caracterizado por edema generalizado. Las pruebas
130.Niño de 4 años que presenta tos y dificultad de laboratorio informan: hemoglobina 10.5 g/dl,
respiratoria todas las noches y durante la actividad leucocitos: 3 500/mm3, Urea 15 mg/dl, creatinina 2
física. ¿Cuál es el mg/dl, albúmina sérica 2.2 g/dl, globulinas séricas 4
tratamiento a seguir? g/dl, y colesterol total 280 mg/dl. El diagnóstico más
A. Corticoide inhalado a altas dosis y beta 2 de acción probable es:
prolongada A. Síndrome nefrótico.
B. Corticoide inhalado a baja dosis y adrenalina B. Glomerulonefritis aguda.
C. Corticoide inhalado a baja dosis y cromonas C. Lupus eritematoso.
D. Bromuro de ipratropio y fluticasona D. Hipotiroidismo.
E. Corticoide inhalado de acción corta y teofilina E. Insuficiencia renal.
131.Después de haber sufrido un traumatismo craneal, un 137.Un paciente de 7 años de edad presenta gingivorragia
niño de 10 años tiene 4 puntos en la escala de y lesiones petequiales en tronco y extremidades, sin
Glasgow y pre- senta respiración irregular, en este otras alte- raciones al examen físico. Hma: 8
momento está indicado: 940/mm3, Hb: 11.5 g/dl, Plaquetas: 60 000/mm3. El
A. Practicarle intubación endotraqueal. diagnóstico más probable es:
B. Administrarle naloxona. A. Púrpura Trombocitopénica idiopática
C. Administrarle Ringer lactato. B. Linfoma de Hodking
C. Leucemia linfoblástica aguda B. Esperar a que continúe el trabajo de parto
D. Síndrome de Wiskott Aldrich espontáneo
E. Púrpura de schonlein Henoch C. Parto instrumentado
138.¿Cuántos gramos de proteína le corresponden a un D. Preparar para cesárea inmediata
escolar de 30 Kg? E. Transfusión de sangre
A. 30 gr/día 144.Puérpera de parto gemelar que presenta sangrado
B. 15 gr/día vaginal profuso luego del alumbramiento. El
C. 20 gr/día diagnóstico más pro- bable es:
D. 1.5 gr/día A. Laceración de cuello uterino
E. 2 gr/día B. Ruptura uterina
139.En una paciente que se le realiza una amniografia y se C. Coagulación
aprecia imagen en panal de abeja. A que patología D. Atonía uterina
nos estamos refiriendo. E. Retención de restos placentarios
A. Aborto espontáneo 145.Primigesta con 32 semanas de gestación por última
B. Embarazo ectópico regla, acude a emergencia por dolor en epigastrio e
C. Pseudociesis hipocondrio derecho. PA: 180/110 mmHg,
D. Down proteinuria ++, tacto vaginal: cérvix posterior, orificio
E. Mola externo cerrado, pelvis ginecoide. El diagnóstico más
140.Femenina de 22 años de edad, casada hace 6 meses, probable es:
la cual refiere amenorrea de 16 semanas de A. Preeclampsia severa y trabajo de parto
evolución. A la explo- ración se observa, signo de B. Preeclampsia superpuesta e hipertensión crónica
Chadwick, Godell y útero entre la sinfisis del pubis y C. Preeclampsia severa
cicatriz umbilical. Con estos datos usted puede hacer D. Preeclampsia severa y colecistitis
el diagnóstico de E. Preeclampsia severa y DPP
A. Posible embarazo 146.Una gestante a término de 18 años, sin antecedentes
B. Embarazo confirmado de importancia, con 180/100 mm Hg de presión
C. Embarazo positivo arterial, en 2 tomas, y con una proteinuria de 350 mg
D. Posibilidad de embarazo en orina de 24 horas su diagnóstico más correcto
E. Probable embarazo sería:
141.Femenina de 23 años la cual cursa con su 38 semana A. Preeclampsia leve.
de gestación. Refiere que hace 2 horas arrojo una B. Hipertensión transitoria leve.
masa muco- sanguinolenta. A la exploración se C. Preeclampsia severa.
observa cuello con 3 cm de dilatación, presentación D. Hipertensión transitoria severa
cefálica en OIA, FCF 138. Con estos datos usted haría E. HTA crónica con PE severa
el diagnostico de: 147.En una paciente que cursa su 32 semana de gestación
A. Placenta previa usted le pide un ECO ya que la altura del fondo
B. DPPNI uterino es mayor a la esperada y el ECO le reporta que
C. Embarazo a término el líquido amniótico es de 2100 ml. Con esto usted
D. Trabajo de parto en fase pasiva puede decir que la paciente tiene
E. Trabajo de parto en fase activa A. Diabetes gestacional
142.En una paciente de 38 semanas de gestación la cual B. Producto con anencefalia
usted le pide un eco y este le reporta todo normal con C. Oligohidramnios
líquido am- niótico de 800 ml. Con estos datos usted D. Producto con atresia esofágica
hace el diagnostico de: E. Polihidramnios
A. Polihidramnios 148. Mujer obesa en edad reproductiva, con
B. ECO normal oligomenorrea, infertilidad e hirsutismo. El
C. Embarazo normal diagnóstico más probable es:
D. Insuficiencia útero placentaria A. Síndrome de ovario poliquístico
E. Oligohidramnios B. Tumor funcionante de ovario
143.Gestante a término de 35 años acude a emergencia C. Hiperprolactinoma
por contracciones uterinas cada 3 minutos y sangrado D. Hiperplasia suprarrenal
vaginal de 10cc. PA: 120/70, P: 84x’, FR: 16x’, E. Síndrome de Kallman
ecografía del día informa: feto único en LCI, placenta 149.Mujer de 35 años, acude a consulta por metrorragia
previa marginal. LCF: 148x’, se incida vía, análisis y se de dos meses, sin otro antecedente de importancia.
realiza TV en condiciones de operabilidad: Examen clínico: pálida, peso 61 kg, PA 110/76, pulso
incorporación: 90%, dilatación: 8cm, membranas 68 X’, útero se palpa dos cm por encima del pubis. Al
íntegras, altura de presentación: 0, variedad de examen pélvico: útero anterior con nódulo de 4 cm a
posición: OIIA, pelvis ginecoide. ¿Cuál es la conducta nivel fúndico, anexos normales. Hemoglobina 9 g/dL.
a seguir? ¿Cuál es el diagnóstico más probable?
A. Acentuación del trabajo de parto A. Endometrioma
B. Leiomioma uterino
C. Quiste ovárico última regla fue hace dos años. Al examen de acuerdo
D. Sarcoma uterino a la clasificación POPQ presenta las siguientes
E. Teratoma medidas: Aa +2; Ba +3; C
150.Una mujer de 53 años de edad que no ha menstruado -5; Hiato Genital (gH) 5; Cuerpo perineal (pB) 3; longitud
durante un año ha comenzado con tratamiento de total de vagina (Tvl) 10; Ap -1; Bp -2; D -6.
restitución hormonal cíclico. Tiene ligero sangrado +2 +3 -5
vaginal que dura dos días, mientras comienza su 5 3 10
segundo ciclo de restitución. Está sana, su índice de -1 2 -6
masa corporal (BMI) es de 21, su presión arterial es ¿Cuál es el grado de distopia de la paciente?
normal, y usó anticonceptivos orales hasta los 42 A. 0
años. Se rehúsa a una muestra endometrial. El B. 1
siguiente paso más apropiado para tratar su sangrado C. 2
es: D. 3
A. Comenzar un calendario menstrual. E. 4
B. Hacer un Papanicolaou, lo que incluye una muestra 155.Paciente mujer de 30 años de edad, que acude por
endocervical. dolor pélvico de una semana de evolución que se
C. Insistir en una muestra endometrial. inicia dos días después de haber sido sometida a una
D. Realizar una colposcopia. histerosalpingografía por infertilidad. Hace tres días
E. Realizar una ecografía transvaginal para medir el refiere fiebre de 38 0C. Al examen presenta dolor en
grosor endometrial. abdomen bajo y se insinúa rebote, el examen
151.Una mujer de 33 años no puede sentir el hilo de su bimanual del útero y anexos es difícil de evaluar por
dispositivo intrauterino. Su última menstruación fue el dolor, el resto del examen es normal. ¿Cuál es el
hace una semana. Una prueba sérica de embarazo es diagnóstico más probable en este caso?
negativa. La mejor acción a seguir es:
A. Insertar otro DIU para reemplazar el que se perdió A. Apendicitis aguda
B. Hacer una radiografía abdominal B. Endometriosis
C. Hacer una ecografía pélvica C. Enfermedad pélvica inflamatoria
D. Realizar una histerosalpingografía D. Ruptura folicular
E. Palpar el cuello uterino cuidadosamente para jalar el E. Infección urinaria
hilo 156.Paciente varón de 19 años de edad con fractura de
152.Una mujer de 29 años de edad G 6 P 1041 se presenta fosa orbitaria derecha, al examen presenta: Ptosis
con retraso menstrual de 2 semanas; su régimen palpebral y mirada hacia adentro y abajo, que nervios
catamenial es 3/28 regular, tiene un examen de podrían encontrarse afectados:
gonadotrofina en orina positivo. Ha tenido su primer A. IV nervio craneal y rama interna del oftálmico
embarazo a término normal, luego 2 abortos B. VI nervio craneal y rama interna del oftálmico
espontáneos, un aborto provocado y el último fue un C. III nervio craneal y rama interna del oftálmico
embarazo ectópico. No refiere enfermedades D. III y IV nervios craneales
previas, tuvo salpingectomía derecha. ¿Cuál de las E. III y VI nervios craneales
siguientes alternativas sería la primera medida a 157.Paciente que viene a consultar por sufrir de
adoptar? tumoración en la parte lateral superior e inferior
A. Dosaje sérico cuantitativo de HCG Beta. derecha del hígado, evidenciada por tomografía
B. Test triple (HCG, Alfa feto proteína, estriol) hepática, se pregunta: ¿Cuáles son los segmentos
C. Anticuerpos anticardiolipinas hepáticos afectados?
D. VDRL A. I Y II
E. Ecografía transvaginal B. II Y IV
153.Ingresa una primigesta de 18 años de edad con 32 C. V Y VIII
semanas de gestación refiriendo cefalea intensa y D. V Y VI
epigastralgia tipo opresivo. Al examen tiene presión E. VI Y VII
arterial 180/120 mm Hg. No hay dinámica uterina, 158.Paciente que sufre un accidente de tránsito por
presenta movimientos fetales y los latidos del feto impacto o choque de vehículo con otro, por detrás. El
fluctúan entre 130 y 150, la altura uterina es de 25 conductor del vehículo impactado resulto muerto
cms. con feto en LCI. Los reflejos patelares están en asido al timón, se pregunta: ¿Qué articulación(es)
3+/4+. ¿Cuál es el manejo farmacológico inmediato? vertebral resultaron compro- metidos?
A. Sulfato de magnesio 4 g EV en 20’, nifedipino 10 mg A. Las 3 primeras articulaciones interapofisarias
VO cervicales
B. Diazepam 10 mg EV, labetalol 200 mg EV B. Articulación atloidoaxoidea media
C. Sulfato de magnesio 10 g IM, alfa metil dopa 1 g VO C. Articulación interapofisaria de la prominente
D. Fenitoína 900 mg VO, hidralazina 10 mg EV D. Sexta articulación interapofisaria cervical
E. Diazepam 5 mg EV, nifedipino 10 mg Sublingual E. Articulación odontoatloidea
154.Una paciente de 47 años acude refiriendo sensación 159.Ante un niño de 5 años con un cuadro de hipertensión
de bulto en vagina, no hay incontinencia urinaria, su endocraneal, alteraciones visuales e hipotalámicas,
que pre- senta una radiografía lateral del cráneo con E. Desgarro perineal.
calcificaciones en forma de paréntesis a nivel 164. Para un paciente de 16 años, con diagnóstico
supraselar. ¿Cuál será su diagnóstico presuntivo? de torsión testicular de ocho horas de evolución, el
A. Meduloblastoma tratamiento más adecuado consiste en practicar la:
B. Craneofaringioma
C. Adenoma hipofisario A. Orquiectomía.
D. Glioma del nervio óptico B. Destorsión manual.
E. Pinealoma productor de hidrocefalia C. Orquidopexia.
160.Para un paciente con antecedentes de nefropatía D. Destorsión quirúrgica.
crónica y apendicitis aguda perforada con peritonitis E. Resección del área afectada.
localizada. 165.Una mujer de 20 años recibió un golpe contuso en la
¿Cuál de las siguientes alternativas de manejo sería la más región lumbar izquierda. Acude al hospital 8 días
efectiva y segura? después con ictericia generalizada, dolor en el
A. Apendicectomía, drenaje y una asociación de hombro izquierdo, temperatura de 38.5 grados C.,
Amikacina y Metronidazol por 48 horas. frecuencia cardiaca de 90 por minuto y presión
B. Apendicectomía, drenaje y una asociación de arterial de 90/60 mmHg. El procedimiento quirúrgico
Amikacina y Metronidazol por siete dÍas. más adecuado en este momento es:
C. Apendicectomía, lavado de cavidad y una asociación A. Esplenectomía.
de Cefuroxima y Clindamicina por siete dias. B. Nefrectomía.
D. Apendicectomía, drenaje y una asociación de C. Drenaje del hematoma.
Cefuroxima y Metronidazol por siete días. D. Laparotomía exploradora.
E. Apendicectomía, drenaje y antibiótico-profilaxis con E. Lavado peritoneal.
cualquiera de los esquemas mencionados. 166.Varón de 78 años, con antecedente de hiperplasia
161.Un paciente de 60 años acude a emergencia con prostática, que presenta elevación del PSA (100
quemaduras de 2do grado en la región anterior del ng/ml) y lumbago. En el tacto rectal se encuentra una
tronco, los genita-les y el dorso de la mano derecha. próstata grande, de consistencia dura en ambos
¿Qué porcentaje de superficie corporal quemada lóbulos, de superficie nodular y de bordes no bien
presenta? definidos. AL realizarle los procedimientos que
A. 20%. corresponden se realiza el diagnostico de adeno-
B. 18%. carcinoma de próstata pobremente diferenciado que
C. 11%. afecta ambos lóbulos y que infiltra las vesículas
D. 36%. seminales, con metástasis ósea. ¿Cuál es el
E. 38%. tratamiento más adecuado?
162.Chofer que sufre accidente de tránsito (choque) es A. Extirpación total de la próstata
llevado a emergencia porque presenta dolor B. Quimioterapia enérgica
abdominal. Al examen clínico presenta PA 80/40 C. Radioterapia pelviana externa
mmHg. Pulso 105 por minuto, despierto, dolor D. Braquiterapia prostática.
abdominal difuso a la palpación, reacción peritoneal E. Hormonoterapia
dudosa, ruidos hidroaereos escasos. Su conducta a 167.Una mujer de 24 años que acusa visión borrosa y
seguir será: enrojecimiento del ojo derecho de aproximadamente
A. Hospitalizar al paciente y programar a sala de 15 días de evo- lución. Hace 5 días se agrega dolor en
operaciones por tratarse de un traumatismo el mismo ojo acompañado de lagrimeo. El examen
abdominal cerrado. revela AV de 20/400 en ese ojo, inyección conjuntival
B. Enviar al paciente a su domicilio e indicarle que periquerática y una pupila irregular aparentemente
regrese si empeora. arreactiva. En un caso como éste, ¿cuál sería la causa
C. Mantener al paciente en observación con hidratación más probable del síndrome?
parenteral, sonda Foley, control de hematocrito A. Glaucoma agudo congestivo
seriado. B. Escleritis necrotizante
D. Colocar vía parenteral, aplicar antiespasmódico y C. Neuritis óptica
solicitar TAC abdominal. D. Iridociclitis aguda
E. Realizar paracentesis diagnóstica para determinar si E. Queratoconjuntivitis por cuerpo extraño
es traumatismo. 168.Paciente post operada de apendicitis + peritonitis.
163.Al ser atropellado, un hombre de 52 años recibe una Presenta cuadro respiratorio con tos y bulto que
contusión directa en la pelvis. En la exploración física protruye a través de herida operatoria con asas
se encuen- tra sangre en la uretra; al efectuar el tacto intestinales exteriorizadas. Se trata de:
rectal se percibe que la próstata está desplazada. El A. Eventración
diagnóstico clínico más probable es: B. Evisceración
A. Hematoma prostático C. Fuga intestinal
B. Lesión vesical. D. Estrangulación
C. Laceración de la uretra. E. Hernia inscional
D. Laceración del trígono.
169.Paciente mujer de 52 años que acude a la emergencia 174.Mujer de 38 años de edad, obesa, con trastorno del
por presentar dolor abdominal y tumoración en sensorio, sequedad de piel, caída del cabello,
región umbilical de 12 horas de evolución, se trata de bradipsiquia e hipo- tensión arterial. Dosaje de T4
reducir y con mucho esfuerzo se consigue pero disminuida y TSH elevada. ¿Cuál es el tratamiento de
protruye nuevamente de inmedia- to, se trata de una elección?:
hernia: A. Hormona tiroidea de inicio sin corticoides
A. Indirecta B. Hormona tiroidea a dosis altas y corticoides a dosis
B. Inguino-escrotal bajas
C. Incoercible C. Corticoides a dosis altas, luego hormona a tiroidea a
D. Encarcelada dosis bajas
E. Por deslizamiento D. Corticoides y luego adicionar hormona tiroidea
170.Paciente mujer de 64 años con cuatro días de E. Hormonas tiroideas, corticoides y diuréticas
enfermedad, caracterizado por dolor abdominal tipo 175. Varón de 56 años, presenta dolor
cólico, náuseas, vómitos fecaloideos, dificultad para retroesternal súbito e intenso de 6 horas de
defecar y no eliminación de flatos; al examen: evolución. El EKG revela onda Q y seg- mento ST
distensión abdominal, ausencia de ruidos elevado en DII, DIII, AVF ¿Cuál es el diagnóstico y que
hidroaéreos, poco depresible. Ecografía: Vesícula fármaco usaría?
pequeña, de paredes engrosadas, con cálculos, A. Tromboembolismo pulmonar / heparina.
colédoco 6 mm; Radiografía de abdomen: niveles B. Infarto agudo de miocardio / heparina.
hidroaéreos, edema de pared intestinal, neumobilia. C. Pleurodinea / antiinflamatorios no esteroideos.
¿Cuál es la causa más probable de obstrucción D. Trombosis pulmonar / trombolítico.
intestinal? E. Infarto agudo de miocardio / trombolítico.
A. Hernia crural complicada. 176.Paciente de 13 años, 15 días antes viajó a Piura.
B. Íleo biliar. Presenta fiebre desde hace 5 días, artralgia, cefalea,
C. Cáncer de ciego. dolor retro ocular y erupción cutánea. ¿Cómo se
D. Bridas y adherencias. clasifica el caso?
E. Hernia interna. A. Probable dengue con signos de alarma
1 71. Mujer de 62 años que acude a la consulta por B. Probable dengue grave
presentar dolor en el borde interno del antebrazo y a C. Probable dengue sin signos de alarma
nivel del dedo D. Dengue confirmado
anular y meñique de la mano derecha, también explica E. Dengue descartado
hormigueos, frialdad y pérdida de sensibilidad. ¿Qué 177.Mujer de 24 años con varios días de fiebre, de mayor
tipo de intensidad por las tardes. Recibe tratamiento
neuropatía por compresión tiene el paciente? sintomático sin mejoría; hace 3 días se agrega dolor a
A. Síndrome canal carpiano nivel sacro iliaco, persistiendo el cuadro febril. ¿Cuál
B. Atrapamiento de nervio mediano y cubital a nivel de es el diagnóstico más probable?
la muñeca A. Salmonelosis
C. Compresión proximal del nervio cubital B. Malaria
D. Sección nerviosa cubital C. Pielonefritis aguda
E. Compresión del nervio radial D. Leptospirosis
172.Un hombre asintomático presenta en dos análisis de E. Brucelosis
sangre rutinarios glucemias basales de 132 y 178.Una mujer de 64 años notó lesiones del tipo
130mg/dl ¿Cuál será la actitud correcta? ampollosas en sus muslos y axilas. Eran pruriginosas,
A. Se le pide una curva de glucemia para diagnóstico pero no dolían. No tenían otros síntomas. En la
B. Se inicia tratamiento con glibenclamida exploración las lesiones son grandes, a tensión, en
C. Se inicia tratamiento con dosis de insulina NPH forma de ampollas de contenido seroso. La biopsia
D. Se le pide nuevos análisis dentro de un año confirmo el diagnóstico de penfigoide ampolloso.
E. Se le diagnostica diabetes mellitus tipo 2 y se le inicia Cuál de las siguientes características histológicas es
tratamiento con dieta típica de esta condición:
173.Paciente adulto traído a la Emergencia por los A. Cambios inespecíficos
Bomberos, quienes lo encontraron inconsciente en la B. Depósitos de inmunoglobulina A (igA)
calle. Al examen: pálido sudoroso, sin signos de C. Lesiones dentro de la epidermis (acantolisis)
agresión. PA: 120/60 mmHg, FC: 100 por minuto, FR: D. Depósitos de inmunoglobulina M (igM)
12 por minuto, saturación de oxígeno 95%, pupilas E. Lesiones de la membrana basal
centrales de 3,5 mm, fotorreactivas lentas. Su 179.Es una causa de acidosis con hiato aniónico elevado:
diagnóstico más probable es coma…: A. Cetoacidosis diabética
A. Hiperglicémico B. Diarrea
B. Hipoglicémico C. Acidosis tubular renal
C. Barbitúrico D. Inhibidores de la enzima convertidora de
D. Hipoxémico angiotensina
E. Vigil E. Hipopotasemia
180.Una mujer de 85 años tiene lesiones ampollosas B. Perforación de colon / Colectomía de urgencia
extensas en el abdomen que aparecen y desaparecen C. Brote severo de colitis ulcerativa / Corticoides y
de manera espon- tánea, sin ningún tratamiento, El antibióticos
signo de Nikolsky es negativo. Cuál de los siguientes D. Sobreinfección / Moxifloxacino
diagnósticos es el más probable E. Megacolon tóxico / Colectomía si no hay mejoría en
A. Pénfigo vulgar 24 h
B. Dermatitis herpetiforme 185.Paciente varón de 59 años, fumador, con diagnóstico
C. Herpes gestacional de carcinoma broncogénico en pulmón izquierdo. Se
D. Eritema multiforme le realizan los estudios correspondientes
E. Penfigoide ampolloso encontrándose un nódulo ganglionar en la región
181. Masculino de 40 años con antecedente de paratraqueal baja derecha .Si aplica- mos la
ingesta crónica de esteroides y salicilatos, que ingresa clasificación TNM ¿Cuál sería el resultado?
a urgencias por dolor abdominal que inicio 6 horas A. T4.
antes, el cual se inició de manera súbita en epigastrio. B. N1.
Se difundió a todo el vientre y que describe como muy C. N3.
intenso. Está en posición de gatillo y no quiere D. N2.
movilizarse; aumenta su dolor a la palpación. El E. M1.
abdomen se palpa en madera. A la percusión hay
ausencia de la matidez hepática y se ausculta silencio 186.Ante la sospecha de quilotórax en un paciente que
abdominal. tiene un dosaje de triglicéridos en el líquido pleural
T.A. 140/90, pulso 100, respiración 20 y temperatura 37.5, de 100mg/dl
su diagnóstico más probable es: ¿Cuál es el estudio que se de realizar en el líquido pleural?
A. Oclusión intestinal A. LDL en el líquido pleural
B. Apendicitis aguda B. VLDL en el líquido pleural
C. Pancreatitis C. Quilomicrones en el líquido pleural
D. Cólico renal D. Gradiente de albúmina sérico .líquido pleural
E. Ulcera péptica perforada E. IDL en el líquido pleural
187.Paciente masculino de 19 años de edad, quien acude
a consultar porque presenta episodios intermitentes
182.Cuando al explorar la presión venosa yugular, se de disnea, la cual llega a ser de pequeños esfuerzos,
encuentra que el paciente tiene una onda “v” gigante, el cuadro se acompaña de tos irritativa y de ¨silbidos¨;
¿cuál es el diagnóstico más probable? es predominantemente nocturno, y este último
A. Estenosis aórtica episodio ha durado más de 24 horas. A la exploración
B. Insuficiencia mitral física se encuentra con frecuencia res- piratoria de 28
C. Insuficiencia tricuspídea por minuto, taquicárdico 120 por minuto, con aleteo
D. Estenosis mitral nasal y tiros intercostales, y en la auscultación de
E. Insuficiencia aórtica tórax se encuentran sibilancias difusas bilaterales.
183.Un hombre de 72 años, oxigenodependiente, con Con estos datos su sospecha clínica es:
antecedente de tabaquismo intenso desde su A. Neumotórax espontáneo primario
juventud, inicia su pade- cimiento actual hace 4 días B. Fibrosis pulmonar idiopática
con disnea, tos en accesos, expectoración purulenta, C. Enfisema pulmonar
fiebre y dolor torácico. La radiografía de tórax D. Asma bronquial
muestra infiltrado basal y broncograma aéreo. El E Neumonía
agente etiológico más probable en este caso es: 188.Mujer de 42 años acude por consulta por prurito
A. Klebsiella pneumoniae. generalizado. Ella niega otros síntomas. En los
B. Haemophilus influenzae. exámenes de labora- torio se encuentra una fosfatasa
C. Mycoplasma pneumoniae. alcalina aumentada en 10 VNS. Para confirmar una
D. Estreptococcus pneumoniae. etiología hepatobiliar se solicita
E. Moraxella catarrhalis. gammaglutamiltransferasa la cual esta aumentada 5
184.Varón de 58 años quien tiene diagnóstico desde hace VNS ¿cuál es el marcador serológico que se debería
6 años de colitis ulcerativa, presenta desde hace 5 solicitar?
días fiebre, dia- rrea inflamatoria, y dolor abdominal. A. Anticuerpos antinucleares
Al realizarle rectosigmoidoscopía el resultado fue B. Antimusculo liso
úlceras y exudado purulento y hemorrágico. .Luego C. Microsomico hepatorrenal
de 72 horas de tratamiento con antibióticos y D. Anticuerpos antimitocondriales
corticoides se asocia hipotensión, taquicardia y dolor E. Alfafetoproteina
abdominal intenso. En la radiografía de abdomen se 189.Hombre de 53 años acude a Emergencia por dolor
encuentra dilatación de 9 cm del colon ¿Cuál es el abdominal severo, náuseas y vómitos alimentarios.
diagnós- tico más probable y el tratamiento más Exámenes au- xiliares: Leucocitos: 18 000/ mm3 y
adecuado? amilasa sérica marcadamente elevada. ¿Qué otra
A. Colitis pseudomembranosa asociada / Imipenem
anormalidad en los exámenes de laboratorio podría exudado. Western blot para cisticercosis en sangre:
encontrarse? negativo. HIV: negativo. TAC cerebral: múltiples
A. Hipoglicemia imágenes heterogéneas hipo e hiperdensas
B. Hipercolesterolemia redondeadas en el parén- quima cerebral. El
C. Hiperglicemia diagnóstico más probable es:
D. Hipercalcemia A. Neurocisticercosis.
E. Hipercalemia B. Tuberculosis cerebral.
C. Toxoplasmosis cerebral.
D. Criptococosis cerebral.
190.Varón de 70 años presenta lesiones eritematosas y E. Melanoma metastásico.
ampollares pruriginosas que predominan en las 195.Una paciente de 25 años, sin antecedentes de interés
axilas, en la ingle y muslos. Hay algunas lesiones en acude a su consulta por presentar molestias
los antebrazos y en los miembros inferiores, además precordiales inter- mitentes, a veces prolongadas,
lesiones orales moderadamente dolorosas. El signo que no son claramente opresivas y que en ocasiones
de Nikolsky es negativo. No hay compromiso de los se modifican con la respiración y los movimientos. La
ojos. El diagnóstico más probable es: exploración física es anodina salvo que en la
A. Dermatitis herpetiforme auscultación cardíaca se pone de manifiesto un click
B. Pénfigo vulgar mesosistólico con un soplo suave telesistólico.
C. Penfigoide buloso Teniendo en cuenta la sospecha diagnóstica, cuál
D. Penfigoide cicatricial sería el tratamiento más indicado para esta paciente:
E. Epidermolisis bulosa A. AAS, nitratos y calcioantagonistas.
91. Paciente varón de 40 años presenta desde hace 3 B. AAS a dosis antinflamatorias y reposo en cama al
meses lesiones papulosas poligonales, localizadas en menos 2 semanas.
la cara anterior de las muñecas y tobillos, C. Corticoides a dosis altas y, si no hay respuesta, añadir
acompañadas de prurito .El estudio histopatológico inmunosupresores.
de una de las lesiones muestra una der- matitis de D. Hielo, AINEs y vendaje compresivo.
interfase con degeneración vacuolar de las células E. Propranolol.
basales. ¿Cuál es diagnóstico? 196.Mujer de 35 años, que desde hace 6 meses refiere
A. Pitiriasis rosada dolor y edema en ambas manos. Examen físico: Dolor
B. Papulosis linfomatoide con aumento de volumen en articulaciones
C. Psoriasis eruptiva metacarpofalángicas proximales y desviación cubital.
D. Liquen plano ¿Cuál es el diagnóstico más pro- bable?
E. Acantosis nigricans A. Artritis reumatoide
192.Paciente de 75 años que consulta por una mácula B. Fiebre reumática
pigmentada heterocroma , de contorno irregular, de C. Lupus eritematoso sistémico
2 x 3 cm de diámetro, localizada en la mejilla derecha, D. Artritis gotosa
que ha tenido un crecimiento muy lento en los E. Artritis infecciosa
últimos años ¿Cuál sería el diagnóstico más probable? 197.Paciente mujer de 20 años que presenta debilidad
A. Melanoma lentiginoso acral muscular en miembros inferiores; hiporeflexia,
B. Lentigo maligno disminución del
C. Carcinoma basocelular pigmentado tono muscular y se queja de parestesias, disestesias y
D. Nevo de Ota allodinea. Cuál es el diagnóstico más probable:
E. Eritema fijo pigmentario A. Polineuropatía.
193.Paciente mujer de 40 años de edad, que acude a B. Síndrome piramidal.
Emergencia por presentar hace dos semanas dolor C. Síndrome de motoneurona del asta anterior.
intenso en la cara anterior del cuello, que aumenta D. Síndrome de placa mioneural.
con la deglución, y malestar general. Al examen físico: E. Mielitis transversa.
dolor a la palpación en la re- gión anterior del cuello, 198.Mujer diabética de 42 años es llevada a emergencia
con leve crecimiento asimétrico del lado izquierdo de por presentar un cuadro de sopor alteración del
la tiroides. El diagnóstico más probable es tiroiditis: sensorio. Glucosa en sangre: 350mg/dl. ¿Cuál de los
A. Aguda siguientes hallazgos confirma el diagnóstico de
B. Subaguda cetoacidosis?
C. De Hashimoto A. Polidipsia, polifagia y poliuria.
D. Tuberculosa B. Sudoración, enfriamiento generalizado y trastorno de
E. De Riedel conciencia.
194.Mujer de 20 años, presenta confusión, somnolencia, C. Deshidratación severa hiperosmolaridad y
convulsiones tónico-clónicas generalizadas, al convulsiones.
examen rigidez de nuca, brudzinski positivo, D. Dolor abdominal, respiratorio de Kussmaul y
vibraciones vocales abolidas y egofonía en base de cetonuria.
hemitórax derecho. Punción lumbar: pleocitosis, E. Fiebre, decaimiento y malestar general.
glucosa: 15, proteínas 150, ADA: 7. Toracocentesis:
199.¿Qué medicamento elegiría en primer lugar para
prevenir nuevos episodios maníacos en un paciente
diagnosticado de trastorno maníaco-depresivo y sin
otros trastornos somáticos acompañantes?:
A. Carbamazepina.
B. Propanolol.
C. Carbonato de litio.
D. Diazepán.
E. Clonidina.
200.¿Cuál de las siguientes actuaciones es la más
importante en la prevención del fracaso renal
agudo?:
A. Administración de manitol
B. Administración de furosemida
C. Evitar los aminoglucósidos
D. Valorar y corregir la hipovolemia
E. Vigilar la diuresis
SIMULACRO 5A E. Neumonía aspirativa
7. Varón de 20 años con diagnóstico de asma bronquial desde
1. Mujer de 28 años, vida sexual activa, presenta hiporexia, fiebre, hace cinco años y en tratamiento con salmeterol de larga data.
escalofríos, disuria, polaquiuria y dolor lumbar. Examen: febril, ¿Cuál de los siguientes efectos adversos está relacionado a su
puño percusión lumbar bilateral positivo. ¿Cuál es el administración?
diagnóstico probable? A. Bradicardia
A. Pielonefritis B. Hiperkalemia
B. Cistitis C. Hipocalcemia
C. EPI D. Hiperglicemia
D. Anexitis E. Disminución de ácidos grasos libres
E. Endometritis 8. Varón de 18 años presenta tos seca, sensación de opresión en
2. Mujer de 25 años diabética mal controlada, acude a emergencia el tórax y disnea intermitente que empeora en el invierno desde
deshidratada y somnolienta. Examen: FC: 90 X´, FR: 28 X´, PA: hace 2 años. Examen: sibilantes en ambos campos pulmonares.
110/70 mmHg. Laboratorio: glicemia: 280 mg/dL, glucosuria y Se sospecha de asma bronquial. ¿Qué paráme- tro de la función
cetonuria. ¿En qué segmento de la nefrona se ha superado el pulmonar se encuentra más disminuido en el paciente?
mecanismo de transporte máximo de glucosa? A. Volumen espiratorio forzado al primer segundo
A. Túbulo distal B. Volumen de reserva inspiratoria
B. Asa de Henle C. Volumen de ventilación pulmonar
C. Túbulo proximal D. Capacidad inspiratoria
D. Túbulo colector E. Capacidad vital
E. Glomérulo 9. Mujer de 53 años, acude por cansancio, sequedad de piel, caída
3. Mujer de 65 años, consulta por pérdida de peso, astenia y de cabello y aumento de peso. Se sospecha hipotiroi- dismo
parestesias en manos y pies desde hace un año. En los últi- mos primario. ¿Cuál de los siguientes hallazgos confirma el
dos meses se añade palpitaciones y ageusia. Antecedente de diagnóstico?
gastritis crónica. Examen: palidez con ictericia leve, mucosa oral A. TSH baja y T4 baja
pálida, lengua roja y depapilada. Hemoglobina: 6.5 g/dL, B. TSH elevada y T4 normal
leucocitos: 4200 x mm3, plaquetas: 120000 x mm3; aumento C. TSH normal
de LDH y de bilirrubina indirecta. ¿Qué examen se solicita para D. TSH normal y T4 baja
determinar la etiología de la anemia? E. TSH elevada y T4 baja
A. Test de Coombs 10. Mujer de 60 años, consulta por debilidad muscular, poliuria,
B. Ferritina sérica polidipsia, dolores articulares y estreñimiento. Antece- dente:
C. Ácido fólico úlcera péptica. Examen: PA: 150/95 mmHg. EKG: QT corto y
D. Vitamina B 12 onda T ensanchada. Laboratorio: calcio sérico y paratohormona
E. Reticulocitos elevadas. ¿Cuál es el diagnóstico?
4. Mujer de 42 años, luego de viaje prolongado, refiere dolor e A. Hiperparatiroidismo secundario
inflamación de las pantorrillas y bruscamente dificultad B. Diabetes mellitus tipo II
respiratoria; por lo que acude a emergencia. Examen: PA: C. Hiperparatiroidismo primario
100/60 mmHg, FC: 118 X’, FR: 26 X’, peso: 88 kg. Pulmo- nes D. Osteodistrofia renal
normales. Extremidad inferior derecha con aumento de E. Síndrome pluriglandular
volumen y signo de Homans positivo. Gases arteriales: pH: 7.20; 11. Mujer de 35 años, quien desde hace aproximadamente 7 meses
pCO2: 19 mmHg; PaO2: 68 mmHg. SaO2: 80% con FiO2: 0.21. se encuentra constantemente preocupada por la enfer- medad
¿Cuál es el tratamiento farmacológico inicial? de su madre, se muestra intranquila, nerviosa y sufre de
A. Enoxaparina 1mg/Kg SC cada 12 horas insomnio, en la última semana se levanta varias veces durante
B. Fondaparinux 7.5 mg IM cada 24 horas la noche. Se siente triste y evita las reuniones sociales. ¿Cuál es
C. Tinzaparina 15 U/Kg cada 24 horas el diagnóstico más probable?
D. Warfarina 5 mg VO cada 24 horas A. Ansiedad generalizada
E. Daltaparina 50 Ul VO cada 24 horas B. Trastorno fóbico
5. Varón de 50 años, refiere tos con expectoración blanquecina, C. Estrés traumático
diaforesis vespertina y pérdida de peso desde hace un mes. D. Trastorno obsesivo
Examen: murmullo vesicular pasa bien en ambos campos E. Trastorno bipolar
pulmonares, no estertores patológicos. Radiografía de tórax: 12. Varón de 40 años con historia personal donde prima la
infiltrado apical derecho. ¿Cuál es el diagnóstico más probable? búsqueda de su propia satisfacción a costa del sufrimiento de
A. Aspergilosis pulmonar los demás, sin capacidad de cambio ni culpa a pesar de las
B. Tuberculosis pulmonar consecuencias. ¿Qué trastorno de personalidad presenta?
C. Neumonía atípica A. Limítrofe
D. EPOC B. Antisocial
E. Silicosis C. Obsesivo
6. Mujer de 45 años, obesa, dos horas después de haber sido D. Paranoide
operada de histerectomía total, súbitamente presenta dolor E. Esquizoide
intenso en el pecho, disnea, lipotimia y sensación de muerte 13. Varón de 70 años, presenta: halitosis, ronquera, disfagia a
inminente. Examen: no ingurgitación yugular, reforza- miento sólidos y líquidos, regurgitaciones de alimentos sin digerir y mal
del segundo ruido en foco pulmonar, cianosis y taquicardia. Rx olientes. Antecedente de faringitis crónica, niega otra
de pulmones: normal. ¿Cuál es el diagnóstico más probable? enfermedad. ¿Cuál es el diagnóstico más probable?
A. Tromboembolismo pulmonar A. Divertículo de Zenker
B. Coartación de aorta B. Acalasia
C. Infarto de miocardio C. Candidiasis esofágica
D. Aneurisma de aorta D. Cáncer de estómago
E. Esclerodermia
14. Adolescente de 15 años, es traído por presentar euforia y 21. Paciente de 26 años, con amenorrea primaria, desarrollo
ansiedad. Examen: FC: 120 X´, FR: 28 X´, PA: 160/110 mmHg, T: mamario normal, con escaso vello púbico y axilar. Tiene
38°C, ojos: midriasis bilateral. Antecedente: epistaxis y rinorrea informe de cariotipo XY. ¿Cuál es el diagnóstico?
crónica. ¿Cuál es la sustancia causante de este cuadro? A. Aplasia cervical
A. Heroína B. Septum vaginal transverso completo
B. Marihuana C. Insensibilidad a los andrógenos
C. Ácido lisérgico D. Síndrome de Turner
D. Cocaína E. Síndrome de Mayer Rokitansky
E. Benzodiazepinas 22. ¿Cuál de los siguientes, es un criterio que define el síndrome
15. Varón de 25 años con fiebre reciente de cinco días de evolución nefrótico?
y antecedente epidemiológico de exposición a dengue, refiere A. Disminución de las globulinas séricas
artralgias, mialgias, cefalea, dolor retro-ocular, dolor lumbar y B. Proteinuria mayor de 3.5 g / 24 horas
se evidencia erupción cutánea. ¿A qué tipo de caso de la C. Presencia de cilindros hemáticos en orina
clasificación de dengue corresponde? D. Disminución de los triglicéridos
A. Descartado E. Disminución de eritropoyetina
B. Sospechoso con signos de alarma 23. En pacientes con insuficiencia renal crónica. ¿Cuál de los
C. Confirmado por laboratorio siguientes hallazgos es indicación de diálisis de emergencia?
D. Probable sin signos de alarma
E. Grave A. Hiperkalemia severa
16. Mujer de 30 años, VIH positivo, hace 15 días presenta fiebre y B. Derrame pleural
cefalea. Examen: signos meníngeos presentes. Cultivo de C. Alcalosis metabólica
líquido cefalorraquídeo positivo a Cryptococcus neoformans. D. Hiponatremia severa
¿Cuál es el tratamiento de inducción para la menin- gitis? E. Hipoxemia severa
A. Anfotericin B + Flucitosina 24. Para el diagnóstico de anemia ferropénica. ¿Cuál es el marcador
B. Trimetroprim + Sulfametoxazol de laboratorio más sensible?
C. Ganciclovir + Corticoides A. Hemosiderina
D. Fluconazol + Corticoides B. Transferrina
E. Sulfadiazina + Pirimetamina C. Hierro sérico
17. Varón de 35 años, con cefalea diaria de varias semanas de D. Ferritina sérica
evolución que le impiden conciliar el sueño, se acompaña de E. Saturación de hierro
congestión nasal, rinorrea y epífora. Examen: ptosis palpebral, 25. ¿Cuál es el trastorno más frecuente asociado al síndrome de
miosis, exoftalmos y anhidrosis derecha. ¿Cuál es el diagnóstico dificultad respiratoria aguda (SDRA)?
más probable? A. Embolia grasa
A. Cefalea tensional B. Pancreatitis
B. Cefalea en racimos C. Sepsis
C. Migraña D. Tuberculosis miliar
D. Arteritis temporal E. Edema pulmonar neurogénico
E. Tumor cerebral 26. ¿Qué segmentos pulmonares compromete típicamente la
18. Varón de 48 años con antecedente de tabaquismo, refiere tuberculosis post primaria o del adulto?
cefaleas esporádicas desde hace 5 años, se automedica con A. Apicales y posteriores de lóbulos superiores
analgésicos. Desde hace 3 meses las cefaleas son constantes y B. Anteriores y laterales de lóbulos inferiores
se acompañan de mareos e insomnio. Examen: lúcido, C. Superiores y basales de lóbulos medios
colaborador, IMC: 32, PA: 150/100 mmHg, FC: 85 X’. D. Superiores y basales medios de lóbulos inferiores
Cardiovascular: RC: rítmicos regulares y no soplos. Resto de E. Anteriores y lingulares de lóbulos inferiores
examen físico sin alteraciones. ¿Cuál es la conducta terapéutica 27. En el tratamiento de la crisis asmática. ¿Qué grupo
inicial? farmacológico se indica inicialmente?
A. Cambios en los estilos de vida A. Beta 2 agonistas de acción larga
B. Solo monoterapia anti hipertensiva B. Beta 2 agonistas de acción corta
C. Iniciar terapia combinada anti hipertensiva C. Metilxantinas
D. Uso de antiagregantes plaquetarios D. Corticosteroides
E. Uso de hipolipemiantes E. Anticolinérgicos
19. Mujer de 65 años, consulta por dolor en columna dorso lumbar 28. ¿Cuál es la causa más frecuente de embolia de origen cardíaco?
de dos años de evolución. Se plantea el diagnóstico de A. Infarto agudo de miocardio
osteoporosis. ¿Qué examen solicita para confirmar el B. Fibrilación auricular
diagnóstico de osteoporosis? C. Miocardiopatía dilatada
A. Densitometría ósea D. Endocarditis infecciosa subaguda
B. Resonancia magnética con secuencia espectroscópica E. Insuficiencia aórtica
C. Resonancia magnética con secuencia de difusión 29. ¿En qué derivaciones del EKG se observan las alteraciones de
D. Resonancia magnética funcional un infarto lateral de miocardio?
E. Tomografía axial computarizada ósea A. DI, aVL
20. Varón de 57 años, obeso, consulta por polidipsia y poliuria. B. V1,V2
¿Cuál es un criterio que contribuye al diagnóstico de diabetes? C. V3,V4
A. Glicemia al azar de 160 mg/dL D. DI,V1
B. Glicemia postprandial menor a 160 mg/dL E. DII, aVF
C. Hemoglobina glucosilada menor a 6.5% 30. ¿Cuál es el método ideal para el diagnóstico de insuficiencia
D. Test de tolerancia a la glucosa menor de 140 mg/dL venosa periférica?
E. Glicemia en ayunas superior a 126 mg/dL A. Prueba de Perthes
B. Pletismografía
C. Flebografía C. Disminuida
D. Doppler manual D. Inestable
E. Prueba de Brodie-Trendelenburg E. Muy elevada
31. ¿Cuál de las siguientes hormonas interviene directamente en la 41. Mujer de 23 años, acude a la consulta por presentar congestión
excreción renal de fosfato? ocular bilateral, que se corrobora al examen, sin ante- cedente
A. Paratohormona de traumatismo ocular. ¿Cuál es la causa más frecuente de ojo
B. Tiroxina rojo que presenta la paciente?
C. Eritropoyetina A. Iritis aguda
D. Renina B. Uveítis anterior aguda
E. Grelina C. Glaucoma agudo
32. La mayor parte de hierro en el organismo se encuentra como… D. Conjuntivitis
A. Hemosiderina. E. Abrasión de la córnea
B. Mioglobina. 42. En el ojo que ha sufrido una lesión penetrante y después de
C. Ferritina. tomar las medidas locales oftalmológicas. Es aconsejable
D. Hemoglobina. prevenir un probable traumatismo posterior. ¿Qué fármaco se
E. Apoferritina. administra?
33. ¿Cuál de las siguientes sustancias aumenta la secreción de A. Antiemético
gastrina? B. Antihipertensivo
A. Glucagon C. Diurético de asa
B. Somatostatina D. Antiinflamatorio
C. Secretina E. Diurético osmótico
D. Adrenalina 43. Mujer de 80 años llega a la consulta con dolor post traumático
E. Péptido intestinal vasoactivo de hombro derecho y limitación de movimiento. Exa- men:
34. ¿En qué tipo de célula se produce principalmente la brazo con hematoma en cara interna del 1/3 superior con cierta
testosterona? angulación. ¿Cuál es el diagnóstico?
A. Germinativa A. Fractura del cuello del húmero
B. Sertoli B. Fractura de clavícula
C. Paneth C. Luxación acromio clavicular
D. Leydig D. Fractura de escápula
E. Intercalada E. Luxación escápulo humeral
35. ¿Cuál es el tipo de epitelio que constituyen los folículos 44. Varón de 48 años, consulta por presentar progresivamente
tiroideos? pérdida de audición en el oído derecho, acompañado de
A. Simple cúbico vértigos, acúfenos pulsátiles, dolor tenebrante profundo,
B. Estratificado plano parálisis facial y supuración. ¿Cuál es el diagnóstico probable?
C. Simple cilíndrico A. Tumor de oído medio o interno
D. Estratificado cúbico B. Síndrome vestibular
E. Seudoestratificado C. Otitis media supurada
36. ¿Dónde se sintetiza la hormona cardionatrina? D. Laberintitis
A. Miocardio E. Otitis externa supurada
B. Epicardio 45. Durante una colecistectomía laparoscópica a un hombre de 40
C. Endocardio años por colelitiasis, antes de concluir la cirugía se per- cata la
D. Subendotelio presencia de bilis en el campo operatorio. Al revisar se
E. Pericardio encuentra sección total del colédoco que mide 8 mm de
37. ¿Cuál de los siguientes músculos de la pared torácica diámetro, razón por la que se convierte la cirugía y se decide
contribuye a determinar el volumen de reserva espiratoria? reparar la vía biliar. ¿Cuál de las siguientes alternativas
A. Intercostal externo quirúrgicas es la más apropiada?
B. Intercostal interno A. Hepato-yeyunoanastomosis en Y de Roux término lateral
C. Subcostal B. Colédoco-duodenoanastomosis término lateral
D. Serrato anterior C. Colédoco-duodenoanastomosis latero lateral
E. Escaleno D. Reparación del colédoco + dren de Kerh
38. ¿Cuál de los siguientes enunciados caracteriza la ubicación de E. Colédoco-gastroanastomosis
la arteria pulmonar derecha? 46. Varón de 24 años, que hace 3 horas sufre accidente de tránsito.
A. Posterior a la vena pulmonar inferior Hemodinámicamente estable. Refiere dolor abdominal y
B. Anterior a la aorta descendente nauseas. Examen: dolor abdominal en epigastrio contractura y
C. Posterior a la aorta ascendente reacción peritoneal generalizada. Leucocitos: 18.000 x mm3.
D. Posterior al bronquio principal derecho ¿Cuál es el órgano lesionado?
E. Anterior a la aorta ascendente A. Hígado
39. ¿A partir de qué estructura se desarrollan las células B. Intestino delgado
yuxtaglomerulares? C. Bazo
A. Túbulo proximal D. Riñón
B. Mesangio E. Páncreas
C. Arteriola aferente 47. Varón de 70 años, es operado con el diagnóstico de hernia
D. Capilar peritubular inguinal izquierda, en el hallazgo operatorio se encuentra que
E. Asa de Henle la hernia es de tipo indirecto, con anillo interno dilatado, saco
40. ¿Cómo se encuentra la presión oncótica plasmática en los herniario que comprime medialmente la fascia transversal del
tejidos quemados? triángulo de Hesselbach, con contenido deslizado de vísceras en
A. Estable forma de pantalón. ¿Cuál es el tipo de hernia dentro de la
B. Elevada clasificación de Nyhus?
A. III-A D. V
B. III-B E. VI
C. I 55. La anestesia raquídea se logra mediante la inyección del
D. II anestésico a nivel:
E. IV A. Epidural
48. Varón de 20 años, llega a emergencia presentando herida por B. Subaracnoideo
arma blanca a nivel sub escapular izquierda y dificultad C. Local
respiratoria. Se diagnostica hemoneumotórax izquierdo. ¿Cuál D. Sitio operatorio
es el procedimiento a seguir? E. Plexo lumbar
A. Toracotomía mínima con drenaje cerrado 56. En el Centro de Salud San Camilo se está elaborando el Plan de
B. Toracotomía en sala de operaciones Salud Local requiriéndose la asistencia técnica (apoyo técnico,
C. Toracocentesis evacuatoria recursos y ejecución de proyectos). Según el MAIS Basado en
D. Videotoracoscopía Familia y Comunidad. ¿En cuál de los paquetes está
E. Observación + oxigenoterapia comprendida la asistencia técnica?
49. Mujer de 60 años con estreñimiento crónico, dolor y sangrado A. Promoción de comunidades saludables
rojo rutilante a la defecación, acompañado de protru- sión de B. Producción de servicios
un “bulto” por el ano que la reduce con la mano. ¿Cuál es el tipo C. Metas de desarrollo sanitario local
de hemorroides que presenta? D. Desarrollo institucional
A. Interna de primer grado E. Apoyo técnico de los servicios
B. Interna de cuarto grado 57. El médico jefe de un establecimiento de salud realiza un taller
C. Interna de tercer grado con su equipo de gestión. En base al ASIS identifican y priorizan
D. Externa trombosada problemas. Programan tareas y recursos que garanticen la
E. Interna de segundo grado ejecución de actividades para el logro de objeti- vos y metas del
50. Mujer de 60 años politraumatizada, con diagnóstico de fractura siguiente año fiscal. ¿Qué tipo de planificación han
de pelvis estable. Examen: PA: 90/60 mmHg, abdo- men desarrollado?
ligeramente doloroso sin reacción peritoneal. Ecografía FAST A. Táctica
negativa. ¿Cuál es el manejo inicial? B. De recursos
A. Laparoscopia diagnóstica C. Estratégica
B. Tomografía abdominal D. Operativa
C. Reposición de volumen E. Contingencial
D. Laparotomía exploradora 58. En un establecimiento I-4, la evaluación de indicadores del
E. Lavado peritoneal diagnóstico último quinquenio muestra cambio de las primeras causas de
51. Varón de 30 años, politraumatizado por accidente de tránsito, morbilidad y de las prioridades sanitarias de la demanda, sin
es evaluado en emergencia. Examen: confuso, pálido, pulso: incremento de la población total. ¿Qué decisión es- tratégica
>140 X’, hipotenso, FR: 40 X’ y diuresis: 5 ml/h. Abdomen debe tomar e implementar el equipo de gestión?
distendido y doloroso; se sospecha hemoperitoneo. A. Replantear la misión del establecimiento
¿Qué clase de shock hemorrágico presenta? B. Solicitar cambio del establecimiento a nivel II
A. III C. Implementar servicio de hospitalización
B. I D. Realizar mercadeo social para incremento de partos
C. II E. Mejorar las técnicas de diagnóstico de laboratorio
D. IV 59. En un centro de salud se utiliza un mapa epidemiológico
E. V consignando el tipo de aprovisionamiento de agua en los
52. Mujer de 45 años, acude por escape involuntario de orina hogares. Mensualmente se ubica en el mapa los casos de EDA,
desencadenada por actividad física y grandes esfuerzos desde observándose la relación de los hogares con suministro de agua
hace 2 años. G:5 P:5005. En la evaluación no es posible realizar inadecuado, lo cual facilita la planificación local para el control
pruebas urodinámicas. Examen: maniobra de Valsalva positiva. del EDA. Esta información está disponible en…
¿Qué tipo de incontinencia urinaria presenta? A. El monitoreo situacional
A. Rebosamiento B. Las estadísticas de gestión sanitaria.
B. Neurogénica C. El avance de metas sanitarias.
C. Urgencia D. La sala situacional.
D. Mixta E. El monitoreo epidemiológico integral
E. De esfuerzo 60. El alcalde de Oxapampa se reúne con todos los sectores y
53. Varón de 75 años que refiere estar en tratamiento por organizaciones representativas del distrito, para elaborar el
hiperplasia benigna de próstata; sin embargo nota hematuria e presupuesto participativo y el plan de desarrollo local. ¿Qué
infecciones urinarias repetidas. ¿Qué solicita inicialmente para dimensión de la promoción de la salud aplicó?
descartar cáncer de próstata? A. Cultural
A. Ecografía prostática B. Social
B. Citología de la orina C. Económica
C. Cistoscopía D. Psicosocial
D. Dosaje de antígeno específico de la próstata E. Política
E. Tomografía 61. En vigilancia de Aedes, al elaborar un mapa de riesgo
54. Mujer de 70 años con diagnóstico de colecistitis aguda. entomológico, localidad en escenario II, mediano riesgo, con
Antecedente: insuficiencia cardíaca congestiva y ortopnea. índice aédico: 1 - < 2%. ¿Qué color le corresponde?
¿Cuál es la escala ASA que le corresponde desde el punto de A. Blanco
vista anestesiológico? B. Amarillo
A. III C. Gris
B. II D. Verde
C. IV E. Rojo
62. Un Médico interviene un brote de enfermedad transmitida por A. Hiperplasia endometrial
alimentos (ETA) en trabajadores de una fábrica; identificando B. Síndrome de ovario poliquístico
como factor común el almuerzo consumido el día anterior en el C. Hipertiroidismo
comedor institucional. Para lo cual estudió a 20 trabajadores D. Endometriosis pélvica
afectados asignando un testigo para cada uno; encontrando E. Miomatosis uterina
que de 5 posibles causas, el alimento asociado fue la mayonesa 70. Mujer de 38 años acude por presentar leucorrea y sangrado
con un OR de 3.5 y un IC: 1.9 a 7.2. ¿Qué tipo de estudio realizó post coital hace 6 meses. FUR: hace 8 días. Primera re- lación
para encontrar el alimento asociado a la ETA? sexual: 16 años. N° de parejas sexuales: 4. Método
A. Casos y controles anticonceptivo: ritmo. PAP: LIE de alto grado. Biopsia por
B. Cohortes colposcopia: Ca in Situ. ¿Cuál es la conducta a seguir?
C. Transversal A. Histerectomía ampliada a vagina
D. Ecológico B. Histerectomía total
E. Correlacional C. Histerectomía radical
63. ¿Cuál es el propósito de la prevención primaria? D. Braquiterapia
A. Evitar estados graves de enfermedad E. Conización fría del cérvix
B. Evitar surgimiento de patrones de vida social 71. Gestante de 25 años, con embarazo a término, feto vivo,
C. Limitar incidencia mediante control de factores de riesgo ponderado fetal 3800 g, presenta contracciones uterinas
D. Aumentar curación de pacientes irregulares durante 3 días; los tactos vaginales repetidos en
E. Reducción del progreso de complicaciones periodos de 6 horas demuestran que la dilatación cervical no se
64. Al proceso de valorización razonada de factores: sociales, ha iniciado. ¿Qué tipo de dinámica uterina presenta?
económicos, demográficos, políticos, sanitarios y otros; con el A. Hipodinamia secundaria
fin de tomar decisiones sobre la cantidad, calidad y tipo de B. Hipodinamia primaria
recursos de salud para cubrir las demandas y problemas de C. Inercia verdadera
salud en un periodo determinado. ¿Cómo se le denomina? D. Inercia mixta
A. Organización de sistemas de salud E. Hipoinercia
B. Análisis situacional 72. Paciente de 23 años, G2 P1102, acude al consultorio
C. Planificación estratégica manifestando presentar secreción vaginal abundante verdosa e
D. Planificación en salud irritación vulvar desde hace 7 días. Examen: cérvix uterino en
E. Salud pública fresa. El diagnóstico es……………y el mejor manejo terapéutico
65. ¿Cuál de las siguientes alternativas caracteriza a la “Distribución es con……………
Estándar”? A. Tricomoniasis/ metronidazol.
A. Media 1 y desviación estándar 1 B. Vaginosis bacteriana / metronidazol.
B. Mediana 0 y desviación estándar 0 C. Candidiasis / fluconazol.
C. Media 0 y desviación estándar 1 D. Vulvovaginitis / clindamicina.
D. Media 1 y desviación estándar 0 E. Vaginitis atrófica / estrógenos.
E. Media 0 y moda 1 73. Gestante de 33 años, G4 P2012, de 34 semanas por FUR acude
66. Mujer de 30 años acude a emergencia por presentar sangrado a su primer control. Tiene grupo y factor O negativo y Coombs
vaginal abundante hace 3 días, tomó anticonceptivo oral de indirecto positivo, nunca se ha colocado la inmunoglobulina
emergencia hace 6 días. RC: 3/28. FUR: hace 15 días. Al examen: anti D. Para determinar la anemia fetal por ecografía el
útero de 7x4x3 cm. Resto del examen normal. párametro a evaluar, es la velocidad de…
¿Cuál es el examen a solicitar para definir la probable causa del A. Las arterias uterinas.
sangrado? B. La arteria umbilical.
A. Histerosalpingografía C. La arteria cerebral media.
B. Ecografía abdominal D. El ductus venoso.
C. Dosaje de progesterona E. Las arterias mesentéricas.
D. Ecografía transvaginal 74. Gestante de 38 semanas por FUR, G6 P5005, acude a
E. Histerosonografía emergencia por cefalea, trastornos visuales y oliguria. Examen:
67. Mujer de 29 años, con 10 semanas de embarazo por FUR, no trabajo de parto, FCF: 136 X’, PA: 152/112 mmHg en dos
G:2 P:0010. Citología de cuello uterino: células glandulares controles de 6 horas de intervalo. Trombocitopenia, pro-
atípicas de significado incierto. ¿Cuál es la conducta a seguir? teinuria 2+ y creatinina sérica elevada. ¿Cuál es el diagnóstico?
A. Conización A. Eclampsia
B. Control citológico posparto B. Hipertensión gestacional
C. Determinar ADN-VPH C. Hipertensión crónica
D. Biopsia de cérvix D. Preeclampsia con signos de severidad
E. Colposcopía E. Preeclampsia sobreimpuesta
68. Mujer de 42 años, casada sin hijos, acude por presentar 75. Gestante de 12 semanas por FUR, usuaria de DIU hace 6 meses.
irregularidad en su ciclo menstrual hace 5 meses, sofocos, Examen preferencial: se visualiza guía de DIU por el cérvix.
sudoración nocturna y cambios de carácter. FUR: hace 2 meses, ¿Cuál es la conducta a seguir?
no usa métodos anticonceptivos. Subunidad beta: negativo. A. Retiro de DIU
¿Cuál es el diagnóstico probable? B. Antibioticoterapia
A. Perimenopausia C. AMEU
B. Hipotiroidismo D. Uso de misoprostol
C. Menopausia E. Legrado uterino
D. Post menopausia 76. En un estudio de colposcopía. ¿Cuál es un hallazgo anormal?
E. Hipertiroidismo A. Epitelio cilíndrico
69. Mujer de 30 años acude por hipermenorrea de un año de B. Epitelio escamoso original
evolución, además de cansancio fácil, refiere que es usuaria de C. Epitelio captador de yodo
anticonceptivos orales combinados. ¿Cuál es el diagnóstico? D. Zona de transformación tipo I
E. Zona de transformación tipo II A. Síndrome de Lennox Gastaut
77. De los siguientes hallazgos. ¿Qué criterios mayores son B. Crisis febril simple
considerados para el diagnóstico de enfermedad inflamatoria C. Crisis febril compleja
pélvica? D. Epilepsia primaria
A. Dolor pélvico y dolor a la palpación anexial E. Convulsión asociada a fiebre
B. Fiebre (T>38°C) y dolor pélvico 86. Lactante de 3 meses presenta cuadro de 24 horas de evolución
C. Sensibilidad hipogástrica y hemograma con leucocitosis con 15 deposiciones líquidas sin moco ni sangre. Exa- men: FC:
D. PCR aumentada y dolor anexial a la movilización del cérvix 200 X´, FR: 62 X´, ojos hundidos, ausencia de lágrimas, mucosa
E. Líquido libre en fondo de saco por ecografía y dolor pélvico oral seca, cutis marmórea y llenado capilar 4 seg. ¿Cuál es el
78. ¿Cuál es la etiología más frecuente del aborto espontáneo? tratamiento inicial?
A. Endometriales A. Solución isotónica en bolo
B. Constitucionales B. Albúmina 5% en bolo
C. Infecciones C. Plasma en 2 horas
D. Ambientales D. Dextrosa 5% en bolo
E. Cromosómica E. Solución hipertónica 3% EV
79. ¿Cuál de las siguientes alternativas es indicación absoluta de 87. Lactante varón de 6 meses presenta desde hace 2 días: 4
cesárea? deposiciones líquidas al día, fiebre de 39°C, vómitos, hiporexia
A. Macrosomía fetal e irritabilidad. Antecedente: Infección urinaria a los 3 meses.
B. Cesárea anterior ¿Qué examen de imagen indica inicialmente?
C. Miomatosis múltiple A. Cistografía
D. Condilomatosis perineal B. Gammagrafía
E. Placenta previa total C. Radiografía
80. En una deflexión de III Grado. ¿Cuáles son las estructuras D. Tomografía
anatómicas palpables al tacto vaginal? E. Ecografía
A. Arcos supraorbitarios y mentón 88. Niño de 1año, es llevado a la emergencia por presentar hace 5
B. Sutura frontal y lambdoidea días rinorrea y tos seca. Hace 2 días fiebre de 39 °C y dificultad
C. Dorso de la nariz y sutura bregmática respiratoria. Examen: aleteo nasal, FR: 58X´, matidez en tercio
D. Fontanela anterior y borde orbitario superior derecho, disminución del murmu- llo vesicular,
E. Dorso de la nariz y fontanela lambdoidea vibraciones vocales aumentadas y subcrepitantes diseminados
81. Durante el trabajo de parto, la presentación más común es la en ambos campos pulmonares. ¿Cuál es el diagnóstico
variedad occipito iliaca… probable?
A. Izquierda posterior. A. Asma
B. Derecha posterior. B. Neumonía
C. Izquierda anterior. C. Derrame pleural
D. Derecha anterior. D. Atelectasia
E. Transversa derecha. E. Bronquitis
82. Mujer de 65 años que presenta una masa tumoral en la mama 89. Lactante de 9 meses, con deshidratación hiponatrémica, recibe
izquierda, fija a la pared torácica, y ganglios palpables en la tratamiento de reposición de Na >12 mEq/L en las primeras 24
región axilar ipsilateral. En la evaluación inicial de la paciente se horas. Dos días después presenta confusión, agitación y
debe incluir... tetraparesia flácida. ¿Cuál es el diagnóstico pro- bable?
A. Ultrasonido A. Hemorragia intracerebral
B. Biopsia incisional B. Trombosis venosa central
C. Biopsia excisional C. Infarto cerebral
D. Aspiración con aguja fina D. Mielinólisis pontínica central
E. Tomografia axial computarizada E. Disgenesia cerebral
83. En la Rx de tórax de un neonato prematuro con dificultad 90. Preescolar de 3 años presenta hace tres días fiebre de 39°C,
respiratoria se describe: parénquima con granulación reti- cular hace un día ojos inflamados con secreción amarillenta y eritema
fina y broncogramas aéreos. ¿En qué patología se sospecha? faríngeo. ¿Cuál es el agente causal probable?
A. Hernia de Bochdalek A. Adenovirus
B. Síndrome de aspiración meconial B. Streptococcus pneumoniae
C. Enfermedad de membrana hialina C. Virus sincitial respiratorio
D. Secuestro pulmonar D. Haemophilus influenzae
E. Taquipnea transitoria del RN E. Chlamydia trachomatis
84. Recién nacido a término, con diagnóstico de incompatibilidad 91. Niño en edad pre-escolar sin antecedentes de importancia,
Rh, a las primeras 12 horas de vida presenta ictericia hasta presenta tumoraciones cervicales bilaterales a lo largo del
plantas. Madre es A negativa y el niño es O positivo. El paciente borde del músculo esternocleidomastoideo. Examen: pequeñas
tiene indicación de exanguinotransfusión. ¿Qué grupo adenopatías palpables, móviles no dolorosas y de consistencia
sanguíneo y factor debe indicar para el procedimiento? blanda. ¿Cuál es la etiología probable?
A. O negativo A. Parvovirus
B. O positivo B. Virus de la gripe
C. A positivo C. Estreptococcus grupo A
D. B positivo D. Adenovirus
E. AB negativo E. Staphylococcus aureus
85. Lactante de 11 meses, presenta durante 20 minutos 92. Lactante de 2 meses, inicia a las cuatro semanas de vida cuadro
convulsiones en hemicuerpo derecho, que luego se generalizan catarral, afebril y dificultad respiratoria progresiva.
con estado post ictal de recuperación rápida. Sin antecedentes Antecedente: conjuntivitis aguda bilateral a los 10 días de vida.
de importancia. Al examen T: 39°C. FC: 110 X´. Resto del Examen: Sat O2: 91%, FR: 65 X’, leve tiraje subcos- tal,
examen normal. ¿Cuál es el diagnóstico probable? estertores inspiratorios y crepitantes. Leucocitos: 19.600/mm3,
IgG: 13.262/mm3. TAC tórax: patrón en mosaico con áreas en B. Cefalohematoma
vidrio deslustrado y zonas de hiperinflación. ¿Cuál es el C. Hemorragia subgaleal
probable germen etiológico? D. Hematoma temporal
A. Mycoplasma pneumoniae E. Hematoma frontal
B. Legionella pneumophila 100. Lactante de 10 meses con vacunas incompletas, desde hace 2
C. Bordetella pertussis días presenta vómitos en 8 oportunidades, 10 deposicio- nes
D. Chlamydia trachomatis líquidas sin moco ni sangre. ¿Cuál es el agente más frecuente?
E. Chlamydia pneumoniae A. Coronavirus
93. Para un lactante de 8 Kg con cuadro de deshidratación severa B. Adenovirus
por diarrea aguda infecciosa. ¿Qué volumen de suero fisiológico C. Calicivirus
en ml debe infundir? D. Rotavirus
A. 160 E. Norwalk
B. 80
C. 240
D. 40
E. 100
94. Recién nacido de 42 semanas, producto de parto vaginal,
distocia por sufrimiento fetal agudo, con líquido amniótico
meconial. APGAR: 6 al minuto 1 y 8 al minuto 5, recibió
ventilación por máscara. A las 2 horas de vida, reportan ta-
quipnea, Sat O2: 70%, tiraje subcostal y cianosis. Rx tórax:
infiltrado pulmonar en “parches”. ¿Cuál es el diagnóstico
probable?
A. Quiste pulmonar congénito
B. Asfixia neonatal
C. Neumonía neonatal
D. Síndrome de aspiración meconial
E. Taquipnea transitoria neonatal
95. En la atención inmediata de un recién nacido flácido,
impregnado en líquido amniótico meconial y no respira espon-
táneamente, con FC: 60 X’. ¿Cuál es la primera acción que se
debe realizar?
A. Intubación endotraqueal y aspiración
B. Ventilación a presión positiva
C. Ventilación y masaje cardíaco
D. Posicionar, estimular y secar
E. Dar oxígeno a flujo libre
96. Recién nacido de madre sin antecedentes de importancia,
presenta apnea y FC: <100 X’. Se realiza inmediatamente
intubación endotraqueal; a la reevaluación se encuentra FC:
<60 X’. ¿Cuál es el siguiente paso en la reanimación?
A. Naloxona EV
B. Adrenalina EV
C. Masaje cardíaco SIMULACRO 6A
D. Atropina EV
E. Adenosina EV 1. El diagnóstico de gastritis crónica se realiza fundamentalmente
97. Neonato de 18 horas de vida que presenta vómitos biliosos. El mediante:
estudio radiográfico muestra el signo de doble burbuja. A. Valoración de Helicobacter pylori.
¿Cuál es el diagnóstico? B. Ecoendoscopia.
A. Atresia esofágica C. Determinación de la vitamina B12.
B. Atresia duodenal D. Estudio histológico.
C. Páncreas anular E. Estudios radiológicos.
D. Malrotación intestinal 2. Una mujer de 46 años de edad, diabética en tratamiento con
E. Estenosis pilórica hipertrófica insulina desde hace unos 12 años, presenta sensación de
98. Recién nacido a término, peso 3500 g, APGAR 9 al minuto, en saciedad, náuseas, vómitos y pérdida de unos 3 kg de peso en
alojamiento conjunto, recibe lactancia materna, funcio- nes el último mes. Se sospecha una gastroparesia diabética, siendo
biológicas normales. Madre hace 15 horas ha presentado la mejor prueba para este diagnóstico una de las siguientes:
hemoptisis con resultados BK++. ¿Cuál es la conducta en el A. Estudio de vaciamiento gástrico de sólidos con isótopos
recién nacido? radiactivos.
A. Radiografía de tórax B. Manometría gástrica.
B. PPD C. Gastroscopia.
C. Quimioprofilaxis D. Electrogastrografía.
D. BCG E. Rx. con contraste baritado (tránsito gastrointestinal).
E. Solicitar BK 3. Hombre de 20 años, previamente sano y sin antecedentes
99. En un recién nacido se encuentra una colección de sangre por personales de interés, que es llevado al servido de urgencias
debajo de la aponeurosis que cubre el cuero cabelludo a lo largo por presentar agitación durante las últimas 24 horas. Durante
de todo el músculo occipitofrontal. ¿Cuál es el diagnóstico? la semana previa sus familiares le han notado colora- ción
A. Caput succedaneum
amarillenta de piel y de conjuntivas. Consumidor ocasional de D. La estrangulación es más frecuente en las hernias inguinales
cocaína y éxtasis y, los fines de semana, de alcohol (20 gramos). directas y la incarceración en las indirectas.
Mantiene relaciones sexuales de riesgo sin protección E. La hernia incarcerada se caracteriza por cursar con dolor
adecuada. Temperatura 37°C. Presión arterial 110/60 mmHg. intenso y permanente.
En la exploración física se observa agitación psicomotriz y 9. Un paciente con EPOC grave agudizado acude a Urgencias y
desorientación en tiempo y en espacio, as- terixis e ictericia en presenta una gasometría arterial extraída con Fi02 del 31% a
piel y mucosas, gingivorragias y epistaxis. El resto de la nivel del mar con una Pa02 de 86 mm Hg, PaC02 65 mm Hg, pH
exploración física sin datos patológicos. Las pruebas de 7.13 y Bicarbonato 27 mmol/litro. ¿Cuál de las siguientes
laboratorio demuestran hiperbilirrubinemia (25 mg/dl), ALT afirmaciones es FALSA?:
(GPT) y AST (GOT) mayores de 1000 UI/L; prolongación del A. El paciente está hiperventilando.
tiempo de protrombina (actividad < 40%, INR 15). Indique el B. El gradiente alveolo-arterial de oxígeno está elevado.
diagnóstico más probable del cuadro que presenta el paciente: C. El paciente está en acidosis respiratoria.
A. Síndrome hemolítico urémico. D. El nivel de bicarbonato es normal.
B. Hepatitis alcohólica (esteatohepatitis alcohólica grave). E. Se debería considerar iniciar ventilación mecánica.
C. Hepatitis aguda fulminante (insuficiencia hepática aguda 10. En relación al TORCH en RN, quien desarrolla la tétrada de
grave). SABIN:
D. Intoxicación por éxtasis. A. Toxoplasma
E. Síndrome de abstinencia de cocaína. B. Sífilis
4. Una mujer de 55 años ingresa en el hospital con el diagnóstico C. Rubeola
de pancreatitis aguda. ¿Cuál de las siguientes pruebas o D. Citomegalovirus
determinaciones analíticas NO es útil para predecir la gravedad E. Herpes
de esta enfermedad?: 11. En la auscultación respiratoria se producen una serie de sonidos
A. Tomografía axial computarizada (TAC) del abdomen. básicos que debemos reconocer, y por ello propongo una serie
B. Creatinina en sangre. de parejas (sonidos / posibles patologías) que se relacionan,
C. Niveles de amilasa y lipasa en sangre. salvo en un caso que no existe ninguna congruen- cia:
D. Hematocrito. A. Crepitantes finos - fibrosis intersticial.
E. Nitrógeno ureico en sangre (BUN). B. Estridor - obstrucción bronquiolar.
5. De acuerdo a las técnicas de planificación del sector público, el C. Crepitantes gruesos - bronquitis aguda.
enunciado: “ El Centro de Salud tiene que proteger la dignidad D. Una respiración superficial con difícil audición de los ruidos
personal, promoviendo la salud, previniendo las enfermedades respiratorios - enfermedad neuromuscular.
y garantizando la atención integral de salud de toda la E. Crepitantes que recuerdan al frote de dos trozos de cuero -
población residente en su ámbito de influencia”, corresponde a inflamación pleural.
una definición de la: 12. Teniendo en cuenta sus conocimientos de los tumores en
A. Imagen-horizonte pediatría. Marque la alternativa INCORRECTA:
B. Imagen-objetivo A. El tumor cerebral más frecuente es el astrocitoma
C. Meta-estrategia B. El tumor de Wilms se asocia a una delección del brazo corto
D. Misión del cromosoma 12
E. Visión C. El tumor abdominal primitivo más frecuente es el
6. Niño de año y medio de edad (peso al nacer 3200 gr) con neuroblastoma
lactancia materna exclusiva los primeros 6 meses de vida y D. El meduloblastoma es el segundo tumor más frecuente
ablactancia normal. Hace 6 meses presenta cuadros repetitivos infratentorial
de diarrea e infecciones respiratorias. En su valoracion E. El signo de la capas de cebolla es de Osteosarcoma
nutricional se encuentra P/E 62%, P/T 75%, T/E 95% ¿Cuál es el 13. En un paciente con parkinsonismo, ¿cuál de las siguientes
diagnostico nutricional? situaciones le parece MENOS probable que ocurra en la
A. Eutrófico enfermedad de Parkinson?:
B. Desnutrición aguda A. Ausencia de respuesta a la levodopa.
C. Desnutrición crónica B. Disquinesias coreicas bajo tratamiento con levodopa.
D. Obeso C. Distonía en el pie.
E. Desnutrición crónica reagudizada D. Alucinaciones visuales bajo tratamiento.
7. Respecto al insulinoma, señale la respuesta correcta: E. Antecedentes familiares de parkinsonismo.
A. Se asocia a MEN tipo IIa. 14. Ante un paciente que presenta problemas para la comprensión
B. Es un tumor endocrino pancreático cuya resección del lenguaje tanto hablado como escrito, incapacidad para
quirúrgica supone la curación en la mayoría de casos. denominar objetos y repetir palabras que se le dicen, habla
C. Suele ser un tumor multifocal y maligno en la mayoría de fluida incomprensible con parafasias semánticas y fonémicas,
los casos. se trata de una:
D. Suele ser un tumor de localización extrapancreática. A. Afasia global.
E. El tratamiento de elección es la radiofrecuencia B. Afasia de Wernicke.
percutánea. C. Afasia de Broca.
8. Las complicaciones preoperatorias de mayor interés de la D. Afasia transcortical sensitiva.
hernia inguinal son la estrangulación y la incarceración. E. Afasia transcortical motora.
¿Qué hecho la diferencia?: 15. El 90% de los aneurismas del ventrículo izquierdo, como
A. La hernia estrangulada se reduce con la manipulación y la complicación de un infarto de miocardio transmural, son
incarcerada no. secundarios a una oclusión aguda de la arteria coronaria:
B. La hernia incarcerada se caracteriza por cursar con A. Tronco común de la arteria coronaria izquierda.
interrupción del tránsito intestinal y la estrangulada no. B. Arteria descendente anterior.
C. La hernia estrangulada cursa con alteración de la circulación C. Arteria circunfleja.
sanguínea del contenido herniario. D. Arteria descendente posterior.
E. Arteria coronaria derecha.
16. Mujer de 45 años que presenta hipertensión arterial (190/120 Llama la atención que cualquier intento de manipulación le
mm Hg ) acompañada de K 2.5 mEq/l. Se realiza ecogra- fía produce intenso dolor. Debemos pensar que el paciente tiene:
abdominal que muestra estenosis de ambas arterias renales. A. Fractura subtrocantérea de fémur.
Indique qué tratamiento está contraindicado: B. Fractura subcapital de fémur.
A. Enalapril. C. Luxación coxofemoral anterior.
B. Propanolol. D. Luxación coxofemoral posterior.
C. Amiloride. E. Fractura pertrocantérea de fémur.
D. Prazosin. 23. Con relación a la técnica de amamantamiento , el signo que
E. Amlodipino. indica que el niño hace un buen agarre es:
17. Si en un paciente con insuficiencia cardiaca crónica detectamos A. La nariz del bebe está pegado a la areola
unas ondas v prominentes en el pulso venoso yugular y en la B. La boca del bebe esta semiabierta
auscultación cardiaca se ausculta un soplo holosistólico en el C. Se observa gran parte de la areola libre
área del apéndice xifoides que se acentúa con la inspiración D. El mentón del bebe está separado del pecho de su madre.
profunda, ¿cuál es la valvulopatía responsable de esta E. El labio inferior del bebe esta evertido.
exploración física?: 24. En el trabajo de parto normal, cuando la flexion se completa, el
A. Insuficiencia mitral. diametro de la cabeza fetal que ingresa en el estrecho
B. Insuficiencia pulmonar. superior de la pelvis es:
C. Insuficiencia tricúspide. A. Occipitofrontal
D. Insuficiencia aórtica. B. Suboccipitobregmatico
E. Estenosis aórtica. C. Occipitomentoniano
18. Niño de 18 meses de edad, con calendario vacunal completo D. Biparietal
hasta la fecha, que consulta en el Servicio de urgencias por E. Bitemporal
tumefacción de la rodilla derecha tras jugar en el parque, sin 25. Un paciente con infección VIH se presentó con 40 linfocitos
traumatismo evidente. En la anamnesis dirigida, la madre CD4/μL y una carga viral de 2 millones de copias/mL en el
refiere que un tío de ella tenía problemas similares. La momento del diagnóstico. Inició tratamiento antirretroviral con
exploración ecográfica es compatible con hemartros y en la efavirenz, tenofovir y emtricitabina, y 3 semanas más tarde
analítica que se realiza sólo destaca un alargamiento del TTPA desarrolló un cuadro de fiebre, malestar general y adenopatías
de 52” (normal 25-35”). ¿Cuál es la hipótesis diagnóstica más cervicales bilaterales. El Mantoux fue negativo, en la Rx de tórax
probable?: se observaban múltiples adenopatías mediastínicas y en los
A. Síndrome de Marfan. análisis más recientes presentaba 77 linfocitos CD4/μL y una
B. Enfermedad de Von Willebrand. carga viral de VIH-1 de 1000 copias/mL. ¿Cuál es el diagnóstico
C. Enfermedad de Ehlers-Danlos. más probable?:
D. Hemofilia A. A. Tuberculosis ganglionar como síndrome de reconstitución
E. Enfermedad de Bernard-Soulier. inmune.
19. Un hombre de 58 años, no fumador y sin antecedentes B. Linfoma de alto grado.
personales relevantes, es ingresado en la planta de neurología C. Reacción adversa al tratamiento antirretroviral.
por un accidente cerebrovascular agudo isquémico. Su D. Criptococosis sistémica.
hemograma muestra 18.5 g/dl de hemoglobina con un hemato- E. Infección por Pneumocystis jirovecii.
crito de 60%. Todos los siguientes datos concuerdan con el 26. Hombre de 35 años que presenta hematuria tras infecciones
diagnóstico de Policitemia vera, EXCEPTO uno. Señálelo: respiratorias desde hace varios años. En la analítica de sangre
A. Niveles de eritropoyetina séricos elevados. presenta creatinina 1 mg/ dl sin otras alteraciones y en la orina
B. Presencia de la mutación V617F del gen JAK-2. aparecen hematíes 50/campo, siendo el 80% dismórficos, con
C. Esplenomegalia moderada. proteinuria de 0.8 gramos en 24 horas. ¿Cuál es el diagnóstico
D. Presencia de prurito “acuágeno” y eritromelalgia. más probable?:
E. Presencia de leucocitosis neutrofílica y trombocitosis. A. Nefropatía de cambios mínimos.
20. Mujer de 32 años con clínica de una semana de evolución B. Glomerulonefritis membranosa.
consistente en febrícula, eritema nodo-so, inflamación pe- C. Nefropatía Ig A.
riarticular de tobillos y uveítis anterior. En la radiografía de D. Glomerulonefritis proliferativa difusa.
tórax presenta adenopatías hiliares bilaterales. ¿Cuál es el E. Glomeruloesclerosis focal y segmentaria primaria.
diagnóstico?: 27. Paciente de 48 años que consulta por un cuadro de fiebre,
A. Síndrome de Sjögren. cefalea frontal intensa y sensación nauseosa de 48 horas de
B. Tuberculosis ganglionar mediastínica. evolución. En la exploración física presenta un estado general
C. Sarcoidosis tipo síndrome de Löfgren. conservado. No tiene lesiones cutáneas. Se aprecia una discreta
D. Lupus eritematoso sistémico. rigidez de nuca con signo de Kernig positivo. Ante la sospecha
E. Poliarteritis nodosa. de meningitis se practica una punción lumbar que da salida a
21. Un paciente de 27 años es incapaz de impedir el un líquido de aspecto claro, con proteínas 170 mg/dl, glucosa
desplazamiento anterior del fémur sobre la tibia cuando la 54 mg/dl (glucosa plasmática 98 mg/dl) y células 280 con un
rodilla está 89% de linfocitos. ADA 4 Ul/L. ¿Cuál es la causa más probable
flexionada. ¿Cuál de los siguientes ligamentos está dañado?: de la meningitis de este paciente?:
A. Cruzado anterior. A. Neisseria meningitidis.
B. Colateral peroneo. B. Mycobacterium tuberculosis.
C. Rotuliano. C. Streptococcus pneumoniae.
D. Colateral tibial. D. Enterovirus.
E. Cruzado posterior. E. Virus herpes 6.
22. Paciente de 28 años, que tras sufrir un accidente de tráfico 28. De acuerdo a la convulsión febril en niños. Marque la
frontal de gran energía, presenta deformidad del miembro alternativa INCORRECTA:
inferior derecho en aducción y rotación interna de la cadera. A. Es más frecuente en el segundo año de vida
B. La compleja es cuando tiene paresias postictal
C. Aumentan el riesgo de epilepsia en un 10% E. Sólo debe administrarse si se objetiva taquicardia
D. Suelen aparecer el primer día de la fiebre ventricular polimorfa.
E. Las que duran más de 30 minutos son catalogadas como 35. Con respecto a las quemaduras, ¿qué manifestaciones clínicas
status convulsivo presentan las quemaduras de se-gundo grado?:
29. En cuanto a los índices urinarios en el diagnóstico del fracaso A. Eritema, dolor intenso y sequedad.
renal agudo prerrenal, indique la afirmación INCO- RRECTA: B. Superficie dura y dolor escaso o ausente.
A. La osmolalidad urinaria es superior a 400 mOsm/ Kgr. C. Dolor intenso, formación de ampollas y exudado.
B. El sodio urinario es inferior a 20 mEq/1. D. Dolor escaso o ausente, exudado y ampollas.
C. El índice de fallo renal (IFR) es superior a 1. E. Eritema, dolor y superficie costrosa y seca.
D. El cociente entre la urea urinaria y la urea plasmática es 36. Mujer de 56 años que presenta en los últimos años clínica de
superior a 10. vértigo rotatorio, recurrente, en forma de episodios que se
E. La excreción fraccional de Sodio (EFNa) es inferior al 1 %. inician con sensación de plenitud ótica derecha, que duran
30. Un paciente con insuficiencia renal crónica estadio IV (filtrado entre 2 y 3 horas y que posteriormente, durante unos días,
glomerular renal 25 ml/min) presenta una hemoglobi- na de 8.6 cursa con inestabilidad. Asimismo la paciente refiere acúfenos
g/dL. El estudio de anemia muestra un volumen corpuscular en oído derecho e hipoacusia fluctuante. A la pa- ciente se le ha
medio de 78 fl y los niveles de ferritina en sangre son de 48 ng/ practicado una RM craneal informada como normal y una
mL (valor normal 30-300 ng/mL). ¿Cuál de las siguientes audiometría que evidencia una hipoacusia neurosensorial
opciones es más adecuada?: moderada en oído derecho. ¿Cuál es el diagnóstico más
A. Iniciar tratamiento sustitutivo con hemodiálisis. probable?:
B. Administrar darbopoyetina alfa 0.70 microgamos vía A. Vértigo de origen central.
subcutánea cada 2 semanas. B. Enfermedad de Ménière.
C. Indicar realización de un aspirado medular para completar C. Vértigo posicional paroxístico benigno.
el estudio. D. Neuritis vestibular.
D. Reponer el déficit de hierro y, si persite la anemia, iniciar E. Schwannoma del nervio vestibular.
tratamiento con un agente eritropoyético. 37. En el proceso de coordinación para desarrollar entornos y
E. Realizar una endoscopia digestiva para descartar sangrado estilos de vida saludables en los municipios, instituciones
gastrointestinal. educativas y otras instituciones a nivel local, ¿Cuál de los
31. Un paciente de 80 años, con historia de hipertensión y en lineamientos de política de promoción de la salud debe
tratamiento con enalapril y espirono-lactona, acude al hos- aplicarse?:
pital por astenia y debilidad muscular severa. La presión arterial A. Desarrollar alianzas intra e intersectoriales para la
es de 110/70 mm Hg. En el ECG destacan ondas T picudas y promoción de la salud
elevadas, extrasístoles ventriculares y QT corto. ¿Cuál es el B. Empoderar a la ciudadanía, la participación comunal y la
diagnóstico más probable?: interculturalidad
A. Hipercalcemia. C. Promover la participación comunitaria conducente al
B. Hiperpotasemia. ejercicio de la ciudadanía.
C. Hipomagnesemia. D. Reorientar la inversión hacia la promoción de la salud y el
D. Hipocalcemia. desarrollo
E. Hipernatremia. E. Reorientar los servicios de salud, con enfoque de
32. Los fármacos finasteride y dutasteride, inhibidores de la 5-alfa- promoción de la salud.
reductasa, están indicados en el tratamiento único o 38. Mujer de 30 años y antecedentes patológicos de esclerosis
combinados con alfa-bloqueantes de los síntomas producidos múltiple que acude por miodesopsias en ambos ojos de una
por: semana de evolución. La exploración de fondo de ojo muestra
A. Adenocarcinoma de próstata. vitritis, conglomerados inflamatorios en forma de “bo- las de
B. Adenocarcinoma renal de células claras. nieve” en cavidad vítrea inferior y periflebitis periférica en
C. Hiperplasia prostática benigna. ambos ojos. ¿Cuál es su diagnóstico?:
D. Litiasis urinaria infectiva. A. Uveítis intermedia.
E. Infección urinaria recidivante. B. Síndrome de manchas blancas.
33. De los tumores renales, el de mayor agresividad es: C. Neuritis óptica.
A. Variedad papilar. D. Uveítis anterior.
B. Variedad cromófoba. E. Coroiditis punteada interna.
C. Variedad de células claras. 39. En relación al síndrome de ovario poliquístico, es cierto que:
D. Variedad sarcomatoide. A. Es una endocrinopatía muy poco frecuente en mujeres en
E. Variedad oncocítica. edad reproductiva.
34. Un hombre de 58 años ingresa inconsciente en Urgencias en B. Existe un mayor riesgo a largo plazo de desarrollar diabetes
situación de parada cardiorrespiratoria, procediéndose de mellitus y carcinoma endometrial.
forma inmediata a la aplicación de maniobras de RCP avanzada. C. Clínicamente, es típico en este síndrome la polimenorrea,
Se objetiva en el monitor Fibrilación ventricular (FV), por lo que obesidad e hirsutismo.
se realiza desfibrilación monofásica con carga de 300 Julios. La D. Habitualmente la concentración sérica de FSH es mayor que
FV persiste, motivo por el que se reinician nuevos bucles de la de LH.
masaje cardiaco-ventilación. ¿Cuándo considera que está E. No existe una imagen ecográfica ovárica característica.
indicada la administración de Amiodarona en este paciente?: 40. 40. Gestante de 36 semanas, primigesta, es trasladada al
A. En caso de continuar la FV después del 3º choque hospital para valoración tras accidente de coche en cadena en
desfibrilatorio. la autopista, presentando dolor cervicodorsal. Durante la
B. Debe aplicarse desde el inicio en RCP avanzada, cuando se exploración, la paciente inicia dolor abdominal intenso, leve
detecte FV. sangrado vaginal oscuro y aumento mantenido del tono
C. Tras el primer choque desfibrilatorio, en caso de uterino. ¿Qué diagnóstico le parece el más probable?:
persistencia de la FV. A. Rotura esplénica con hemoperitoneo.
D. No está indicada su administración en RCP avanzada. B. Desprendimiento prematuro de placenta normoinserta.
C. Amenaza de parto prematuro. B. La metilprednisolona por vía intramuscular es el
D. Rotura uterina. tratamiento de elección.
E. Rotura de vasa previa. C. Se trata de un cuadro de urticaria asociado a asma, y debe
41. 41. Mujer de 26 años, primigesta de 32 semanas, acude a ser tratado con antihistamínicos y broncodilatadores inhalados.
urgencias por dolor cólico y lumbalgia. No refiere pérdida de D. Se debe recomendar a los padres su traslado a un Servicio
líquido. En la exploración no se observan sangrado ni líquido de Urgencias hospitalario.
amniótico en vagina. Se confirma latido cardiaco fetal normal y E. Se debe administrar sin más dilación adrenalina por vía
la ecografía vaginal nos informa de un acortamiento cervical del intramuscular.
60%. En los primeros 10 minutos de vigi- lancia 47. ¿Cuándo se aconseja intervenir quirúrgicamente la hernia
cardiotocográfica presenta 3 contracciones. ¿Qué pensaría umbilical infantil?:
como primera opción en esta paciente?: A. Se debe intervenir cuanto antes para evitar el riesgo de
A. Antibióticos. estrangulación.
B. Sulfato de Magnesio. B. A partir de los 3 ó 4 años de vida, al ser frecuente su cierre
C. Hidratación intravenosa. espontáneo antes de esa edad.
D. Corticoesteroides y tratamiento tocolítico. C. Con el fin de utilizar prótesis en la cirugía, se recomienda
E. Ecografías seriadas. esperar a que el paciente sea mayor.
42. 42. ¿Cuál es el tratamiento antibiótico ambulatorio de elección D. Si el niño presentara vómitos durante los accesos febriles.
en la enfermedad inflamatoria pélvica leve/moderada?: E. Ante la sospecha de que se trate de un onfalocele.
A. Clindamicina y gentamicina. 48. En un contraste de hipótesis estadístico, ¿a qué definición
B. Metronidazol. corresponde con más exactitud el valor “p”?:
C. Azitromicina. A. La probabilidad de observar los resultados del estudio, u
D. Amoxicilina-clavulánico y doxicilina. otros más alejados de la hipótesis nula, si la hipótesis nula fuera
E. Ceftriaxona y doxiciclina. cierta.
43. En relación a la Neoplasia Trofoblástica Gestacional, NO es B. La probabilidad de que la hipótesis nula sea cierta.
cierto que: C. La probabilidad de observar los resultados del estudio si la
A. Engloba una serie de neoplasias que tienen en común una hipótesis nula fuera cierta.
producción aumentada de Beta-HCG. D. La probabilidad de que los resultados observados sean
B. El síntoma más frecuente de la mola total es la hemorragia debidos al azar.
genital. E. La probabilidad de observar los resultados del estudio, u
C. En la etiología de la mola completa, el origen es paterno. otros más alejados de la hipótesis nula, si la hipótesis
D. En la mola parcial, el cariotipo embrionario es diploide en el alternativa fuera cierta.
90% de los casos. 49. Un grupo de 1000 pacientes diagnosticados de Síndrome del
E. En la mola total es frecuente la aparición de hiperemesis Aceite Tóxico (SAT) fueron segui-dos desde 1981 hasta 1995
gravídica muy precoz y severa. junto con un número similar de vecinos sin dicho diagnóstico.
44. A un adolescente asintomático se le practica un ECG que Entre los pacientes con SAT se observó en 1995 un 20% con
muestra ritmo sinusal y bloqueo de rama derecha. En la signos de neuropatía periférica frente a un 2% en los vecinos.
exploración física se ausculta un segundo ruido desdoblado y Según el diseño descrito, ¿de qué tipo de estudio se trata?:
en la radiografía de tórax se objetiva un arco pulmonar A. Estudio transversal.
prominente. ¿Cuál sería su sospecha diagnóstica?: B. Estudio de cohortes.
A. Tetralogía de Fallot. C. Estudio de casos y controles.
B. Estenosis pulmonar. D. Estudio cuasiesperimental.
C. Comunicación interauricular. E. Ensayo clínico controlado.
D. Persistencia de ductus arterial. 50. Llega a emergencia un paciente que sufrió un accidente con
E. Comunicación interventricular. traumatismo directo de abdomen. El cirujano comenta que
45. 45. La leche humana es el alimento natural y apropiado durante probablemente tenga hematoma retroperitoneal por un
el primer año de vida. ¿Cuál de las siguientes asevera- ciones al órgano lesionado de la zona I. ¿Cuál de los siguientes
respecto NO es cierta?: pertenecen a dicha zona?
A. La lecha humana contiene anticuerpos bacterianos y A. Vejiga
víricos. B. Riñones
B. La lactancia natural se asocia con una menor incidencia de C. Huesos pélvicos
alergia o intolerancia a la leche de vaca. D. Duodeno
C. Están bien reconocidas las ventajas psicológicas de la E. Colon descendente
lactancia al pecho tanto para la madre como para el niño. 51. ¿Qué es un estudio de casos y controles anidado?:
D. No se ha documentado la transmisión de infección por VIH A. Es el tipo de estudio de casos y controles en el que la serie
por la leche materna. de controles está apareada con los casos en posibles factores
E. La leche humana contiene lactoferrina, que tiene un efecto de confusión.
inhibitorio en el crecimiento de E. coli. B. Es el tipo de estudio de casos y controles en el que la serie
46. Acude de urgencia al centro de salud un niño de 15 meses de de controles está muestreada aleatoriamente de la cohorte que
edad que, durante la cena, tras ingerir un bocado de tortilla, da origen a los casos.
presenta de forma súbita: enrojecimiento facial de predominio C. Es el tipo de estudio de casos y controles en el que tanto los
perioral, lesiones habonosas en tronco y extre- midades, y tos. casos como los controles se extraen del mismo hospital o
A su llegada al centro se encuentra consciente y se objetiva, centro de estudio.
además de lo descrito: tiraje supraesternal, rinorrea acuosa D. Es el tipo de estudio de casos y controles que se realiza para
abundante, hipoventilación bilateral sin sibilancias, y relleno estudiar los factores etiológicos de las malformaciones
capilar inferior a 2 segundos. De las siguientes afirmaciones, congénitas y que se llevan a cabo en las unidades de
señale la respuesta CORRECTA: neonatología.
A. Lo prioritario es canalizar una vía venosa.
E. Es el tipo de estudio de casos y controles que se realiza en con un peso fetal estimado de 1.150 g, que acude a Urgencias
poblaciones estáticas o cerradas en las que no se permita la con dinámica uterina regular y una dilatación cervical de 3 cm.
entrada o salida de la misma. La vía del parto más adecuada será:
52. De entre las modificaciones gravídicas, ¿cuál es INCORRECTA?: A. Cesárea por edad gestacional inferior a 32 semanas.
A. Además de los 2 litros de volemia extra, existe un aumento B. Cesárea por pesos fetales estimados inferiores a 1.500 g.
de 5 litros de volumen extravascular. C. Cesárea por gestación gemelar monocorial.
B. El aumento de la volemia no conlleva hipertensión arterial D. Parto vaginal de ambos gemelos.
al disminuir las resistencias vasculares periféricas. E. Parto vaginal con versión interna y gran extracción del
C. Existe un aumento del sistema renina-angiotensina- segundo gemelo.
aldosterona, aunque su efecto quede bloqueado. 59. ¿Qué tipo de linfocitos son los más abundantes en sangre
D. Los uréteres pueden dilatarse desde la semana 8 hasta tres periférica?:
meses después del parto, siendo más evidente en el uréter A. Linfocitos B CD5+ (B la).
izquierdo. B. Linfocitos T colaboradores CD4+.
E. A nivel bucal, puede aumentar las pérdidas dentarias C. Linfocitos T citotóxicos CD8+.
debido a un aumento de la salivación y a una disminución del D. Linfocitos T gamma/delta.
pH de dicha saliva. E. Linfocitos NK (Natural Killer o asesinos naturales).
53. ¿Cómo se denomina al ensayo clínico en el que los pacientes, 60. Paciente politraumatizado ingresado en la UCI que presenta
los investigadores y los profesionales sanitarios implica- dos en neumonía asociada a ventilación mecánica. El hemocul- tivo es
la atención de los pacientes desconocen el tratamiento positivo a Acineto-bacter baumannii resistente a carbapenem y
asignado?: ampicilina/ sulbactan. ¿Cuál de los siguientes antibióticos sería
A. Enmascarado. de elección?:
B. Triple ciego. A. Cefepima.
C. Abierto. B. Vancomicina.
D. Simple ciego. C. Linezolid.
E. Doble ciego. D. Amikacina.
54. Una prueba diagnóstica tiene una sensibilidad del 95%. ¿Qué E. Colistina.
nos indica este resultado?: 61. Mujer de 55 años, fumadora habitual, a la que se le descubre
A. La prueba dará, como máximo, un 5% de falsos negativos. en una revisión anual un nódulo de 1,5cm, duro, fijo e indoloro,
B. La prueba dará, como máximo, un 5% de falsos positivos. en el cuadrante superoexterno de la mama. En la exploración
C. La probabilidad de que un resultado positivo corresponda física no se palpan adenopatías axilares ni su- praclaviculares.
realmente a un enfermo será alta. La mamografía muestra imágenes sugestivas de malignidad. Se
D. La probabilidad de que un resultado negativo corresponda realiza biopsia intraoperatoria, cuyo resultado es carcinoma
realmente a un sano será alta. intraductal in situ. La actitud terapéutica más adecuada es:
E. La prueba será muy específica. A. Mastectomía simple.
55. La vena porta está formada por la confluencia de varias venas. B. Tumorectomía + radioterapia + biopsia del ganglio
De las respuestas que se ofrecen, ¿cuál es la verdade- ra?: centinela.
A. Vena mesentérica superior, vena gástrica izquierda y vena C. Quimioterapia + radioterapia local + hormonoterapia, si
gastro-omental izquierda. receptores estrogénicos positivos.
B. Vena mesentérica inferior, vena gástrica izquierda y vena D. Quimioterapia + hormonoterapia si receptores
renal. estrogénicos positivos.
C. Vena esplénica, vena mesentérica superior y vena E. Mastectomía radical (Halsted).
mesentérica inferior. 62. ¿Qué fármaco NO estaría indicado en el tratamiento preventivo
D. Vena esplénica, venas pancreato-duodenales y vena de la migraña?:
omental izquierda. A. Sumatriptan.
E. Venas pancreato-duodenales, vena mesentérica superior y B. Topiramato.
vena mesentérica inferior. C. Propranolol.
56. En una nefrona, el 60% del cloruro de sodio es reabsorbido en: D. Flunarizina.
A. Túbulo proximal. E. Ácido Valproico.
B. Rama descendente del asa de Henle. 63. ¿Qué hallazgo, de los siguientes, en la exploración que se realiza
C. Rama ascendente del asa de Henle. en las primeras 24 horas a un recién nacido obliga a realizar
D. Túbulo contorneado distal. exámenes complementarios para aproximar su diagnóstico?:
E. Conducto colector. A. Acabalgamiento de sutura parieto-occipital.
57. Carmen y Pedro tienen 3 hijos: Enrique, de 5 años, Isabel, de 4 B. Edema palpebral bilateral.
años, y Pablo, de 1 año. Recientemente han notado que Enrique C. Eritema tóxico en tronco y extremidades.
tiene ciertos problemas para subir las escaleras y se cansa D. Mancha mongólica en espalda y extremidades superiores.
mucho cuando corre. Después de unas pruebas mé- dicas, le E. Mechón de pelo en zona de columna lumbosacra.
han diagnosticado una enfermedad genética llamada distrofia 64. Un paciente presenta unas lesiones eritematoescamosas en la
muscular de Duchenne. Señale la alternativa correcta sobre el cara extensora de los brazos y manos, con áreas de piel
tipo probable de herencia de esta enfermedad: respetada dentro de algunas de ellas, y descamación fina en
A. La madre es la que le ha transmitido la enfermedad. áreas seborreicas de cara y cuello. El eritema es más acen-
B. El padre es el que ha transmitido la enfermedad. tuado en las palmas, donde además usted aprecia una llamativa
C. Ambos padres le han trasmitido la enfermedad. queratodermia. En las uñas, observa hiperqueratosis y
D. Si tienen una nueva hija, puede presentar la enfermedad. estriación longitudinal. ¿Qué diagnóstico haría?:
E. Si tienen un nuevo hijo, no puede heredar la enfermedad. A. Psoriasis.
B. Pitiriasis rubra pilaris.
58. Paciente primigesta, de 31 semanas, con un embarazo gemelar C. Pitiriasis rosada.
monocorial biamniótico con el primer gemelo en ce- fálica y un D. Pitiriasis liquenoide.
peso fetal estimado de 1.300 g y el segundo gemelo en cefálica, E. Dermatosis seborreica y eczema de contacto en las manos.
65. Un paciente es diagnosticado de tuberculosis pulmonar e inicia amniótico teñido de sangre, al mismo tiempo que se objetiva
tratamiento con isoniacida, rifampicina y pirazina- mida. A las un deterioro en el estado fetal, el diagnóstico más pro- bable
tres semanas de tratamiento se objetiva en los datos analíticos será:
hiperuricemia. ¿Cuál considera que es la causa más probable?: A. Una hipertonía uterina.
A. Toxicidad por isoniacida. B. Una placenta baja o previa.
B. Toxicidad por rifampicina. C. Una aplopejía uteroplacentaria o hemorragia
C. Toxicidad por pirazinamida. retroplacentaria.
D. Reactivación de su cuadro de tuberculosis. D. Una rotura de “vasa previa”.
E. Destrucción del microorganismo. E. Un desgarro de cuello uterino.
66. Varón de 35 años que acude por presentar enrojecimiento 73. ¿Cuál de los siguientes patógenos es el causante de la
ocular, fiebre y mialgias de 7 días de evolución. Sus familia- res enfermedad por arañazo de gato?:
han tenido síntomas similares, por lo que han sido A. Bartonella henselae.
diagnosticados de síndrome gripal. En los análisis practicados B. Mycobacterium tuberculosis.
destaca CPK 1.200 U/l, LDH 1.800 U/ml, 7.200 leucocitos con C. Mycobacterium avium complex.
30% de eosinófilos. En la biopsia muscular practicada como D. Toxoplasma gondii.
prueba diagnóstica, espera encontrar: E. Virus de Epstein-Barr.
A. Cisticercosis. 74. ¿Cuál de los siguientes emparejamientos entre fármaco
B. Taenia solium. anticoagulante y mecanismo de acción es INCORRECTO?:
C. Fasciola hepatica. A. Heparina - cofactor de la antitrombina III.
D. Triquinosis. B. Acenocumarol - inhibe la vitamina K epóxido reductasa.
E. Giardia lamblia. C. Dabigatrán - inhibe la trombina.
67. ¿Cuál es el tratamiento de elección de la brucelosis aguda?: D. Rivaroxabán - inhibe el factor Xa.
A. Doxiciclina 6 semanas. E. Warfarina - inhibe la absorción de la vitamina K.
B. Doxiciclina 6 semanas, más rifampicina las primeras dos 75. La placenta es un derivado de:
semanas. A. La zona pelúcida.
C. Doxiciclina 6 semanas, más estreptomicina las primeras dos B. El trofoblasto.
semanas. C. El ectodermo.
D. Cotrimoxazol 4 semanas. D. La línea primitiva.
E. Eritromicina. E. El mesodermo.
68. Todos los datos de laboratorio pertenecen a las anomalías 76. El signo de Rovsing, característico en las apendicitis agudas,
producidas por infección VIH en el sistema inmune, EX- CEPTO: consiste en:
A. Depleción de linfocitos T4. A. Dolor a la presión en epigastrio al aplicar una presión firme
B. Activación policlonal de células B. y persistente sobre el punto de McBurney.
C. Producción de inmunoglobulinas incrementada. B. Dolor agudo que aparece al comprimir el apéndice entre la
D. Aumento de la producción de interferón gamma en pared abdominal y la cresta ilíaca.
respuesta a los antígenos. C. Sensibilidad de rebote pasajera en la pared abdominal.
E. Aumento de beta-2-microglobulina. D. Pérdida de la sensibilidad abdominal al contraer los
69. Sobre la estructura del hueso, ¿cuál de las siguientes músculos de la pared abdominal.
afirmaciones NO es cierta?: E. Dolor en el punto de McBurney al comprimir el cuadrante
A. El hueso cortical tiene como unidad el sistema Haversiano, inferior izquierdo del abdomen.
compuesto por un canal rodeado de láminas óseas con osteo- 77. En una mujer de 29 años se hallan, en el curso de una analítica
citos. de rutina, los siguientes parámetros: Hb 11,5 g/dL, VCM 70 fl,
B. La función de los osteoclastos es la reabsorción del hueso. HCM 28 pg, ferritina 10 ng/mL, leucocitos 5.200/mm3,
C. La matriz orgánica está compuesta principalmente por plaquetas 335.000/ mm3. La exploración física es normal. La
colágeno tipo II. exploración más indicada en esta situación es:
D. El crecimiento óseo viene determinado por el platillo A. Exploración ginecológica.
epifisario (crecimiento longitudinal) y el periostio (creci- B. Estudio de sangre oculta en heces.
miento concéntrico). C. Estudio radiológico de aparato digestivo.
E. El esqueleto tiene un origen mesenquimal. D. Electroforesis de hemoglobina.
70. Señale la afirmación INCORRECTA en relación con la clínica del E. Test de Coombs.
asma: 78. Un hombre de 62 años, con una diabetes mellitus tipo 2 de 10
A. Radiológicamente, lo habitual es encontrar una Rx tórax años de evolución, realiza tratamiento con metformina y
normal, salvo en las crisis graves. sitagliptina. Hace ejercicio físico escaso y realiza una dieta
B. El neumotórax y el neumomediastino pueden ser adecuada. En los últimos 6 meses ha perdido peso y tiene más
complicaciones de las crisis asmáticas. astenia. Sus controles glucémicos se han deteriorado, pasando
C. El dato auscultatorio más típico son las sibilancias de glucemias basales de 110-140 mg/dl a glucemias de 170-200
inspiratorias. mg/dl, así como su hemoglobina glicosilada, que ha pasado de
D. La PaCO2 suele estar disminuida durante las crisis. 7,1 a 8,5%. La medida terapéutica más adecuada a realizar es:
E. En crisis asmáticas graves puede aparecer pulso paradójico. A. Aumentar la ingesta de proteínas e hidratos de carbono de
71. ¿Qué tipo de carcinoma mamario es especialmente difícil de cadena larga en la dieta para mejorar la astenia y la pérdida de
detectar por mamografía?: peso.
A. Tumor filodes. B. Asociar al tratamiento una dosis de insulina basal.
B. Tumor escirro. C. Asociar al tratamiento acarbosa.
C. Tumor medular. D. Sustituir la sitagliptina por pioglitazona.
D. Comedocarcinoma. E. Sustituir la metformina por glimepirida.
E. Carcinoma inflamatorio. 79. NO es cierto, respecto a la artritis reumatoide:
72. En una paciente de parto que se encuentra con 3 cm de A. Es fundamental el diagnóstico precoz, un tratamiento
dilatación y, coincidiendo con una contracción, expulsa líquido intensivo y un control estrecho de la enfermedad.
B. El metotrexato es el fármaco de primera elección para E. Infección fúngica por dermatofitos.
controlar la actividad de la enfermedad. 86. Un paciente con artritis simétrica que compromete más de 10
C. El factor reumatoide es típico de la enfermedad y es un articulaciones por más de 3 meses, ¿cuál marcador serológico
criterio obligado para el diagnóstico. positivo alto le daría el diagnóstico de artritis reumatoide?
D. La especificidad de los anticuerpos anti péptidos A. ANA
citrulinados es muy elevada y tiene valor pronóstico. B. Anti DNA
E. Los fármacos biológicos, como los anti TNF, han C. Factor reumatoideo
revolucionado el tratamiento. D. leucocitosis
80. Una mujer de 56 años presenta, tras caída casual al suelo, dolor, E. RNP
deformidad y aumento de volumen en el brazo iz- 87. ¿Cuál de las siguientes medidas es más adecuada para combatir
quierdo, con imposibilidad para la flexión dorsal de la mano. la hiperbilirrubinemia (11.2mg/dl) de un lactante de 3
¿Cuál es el diagnóstico más probable?: semanas, con un desarrollo y crecimiento normal por recibir
A. Fractura de troquíter en húmero izquierdo. adecuada técnica de lactancia?
B. Fractura diafisaria de húmero izquierdo con lesión del A. Fototerapia
nervio radial. B. Exanguinotransfusión
C. Fractura supraintercondílea de paleta humeral con lesión C. Fenobarbital
del nervio mediano. D. Observación
D. Fractura de cuello anatómico de húmero con lesión del E. Suspender lactancia materna por 2 días y dar leche
nervio radial. maternizada
E. Fractura de epitróclea desplazada con lesión del nervio 88. Paciente de 25 años, en la cual se colocó un dispositivo
cubital. intrauterino hace 20 días. Presenta dolor pélvico constante y
81. Paciente de 26 años con 21 semanas de gestación, presenta creciente. Al examen físico: cérvix doloroso a la movilización y
dolor abdominal y sangrado vaginal escaso, de 2 días de anexos dolorosos. La complicación más frecuente es:
evolución. Al examen: altura uterina 20 cm, movimientos A. Colpocervicitis
fetales presentes. Especuloscopia se observa membranas B. Infeccion urinaria
ovulares prominentes e integras, que protuyen por el orificio C. Perforacion uterina
externo abierto. ¿Cuál es el diagnostico? D. Expulsion del DIU
A. Óbito fetal E. Enfermedad inflamatoria pelvica
B. Aborto incompleto 89. Mujer de 40 años, con carcinoma de cervix, presenta sangrado
C. Aborto inevitable postcoital y dolor pélvico mal definido. Al examen: cérvix con
D. Aborto inminente eritroplasia perioficial de consistencia dura y sangrante. Tacto
E. Aborto frustro rectal: parametrio derecho se encuentra infiltrado en un tercio
82. ¿Cuál de las siguientes asociaciones (enfermedad - síntoma o interno ¿Cuál es el estado clínico?
signo clínico) es INCORRECTA?: A. II a
A. Sarampión - manchas de Koplik. B. I a
B. Exantema súbito - fiebre. C. II b
C. Eritema infeccioso - anemia por aplasia medular. D. I b
D. Varicela - adenopatías occipitales. E. III
E. Escarlatina - fiebre y disfagia. 90. Gestante de 36 ss por amenorrea. Ingresa por emergencia por
83. Ante la sospecha de estenosis hipertrófica de píloro, ¿cómo dolor abdominal, sangrado vaginal oscuro de aproxi-
iniciaría los exámenes complementarios?: madamente 100 ml, PA: 90/60 mmHg, pulso 120 x, FR 24 x,
A. Tránsito digestivo. afebril. Examen respiratorio y cardiovascular sin alte- raciones,
B. pHmetría gástrica. AU 37 cm, contracciones uterinas cada 2 min, de 50 segundos
C. Radiología simple. de duración, intensidad (+++), LCF: 100-120 x, examen vaginal
D. Ecografía. con especulo confirma sangrado oscuro a través del cérvix, el
E. Estudio isotópico. cual está cerrado y presenta una longitud de 3.5 cm ¿Cuál es el
84. Mujer de 16 años que consulta por amenorrea primaria. diagnóstico más probable?
Presenta un desarrollo femenino normal de los caracteres A. Placenta previa
sexuales secundarios. Los niveles de estradiol y testosterona B. DPPNI
son normales. En la exploración ginecológica se aprecia C. Vasa previa
ausencia de vagina. Se realiza ecografía y se aprecia ausencia de D. Rotura de seno marginal
útero. Los ovarios son normales ecográficamente. No se E. Cáncer de cérvix
observa riñón izquierdo. El diagnóstico más probable es: 91. Según Lalonde y los Campos de la Salud, ¿Cuál no es una de
A. Síndrome de Rokitanski. ellas?
B. Síndrome de Morris. A. Medio ambiente
C. Hiperplasia suprarrenal congénita. B. Estilos de vida
D. Síndrome de ovarios poliquísticos. C. Biología humana
E. Síndrome de Kallmann. D. Organización de los servicios de salud
85. Joven de 24 años que, a los 3 días de un contacto sexual de E. La familia
riesgo, presenta numerosas lesiones pustulosas, pequeñas, 92. Según la historia natural de la enfermedad y sus niveles de
muy pruriginosas, y que evolucionan a diminutas erosiones, prevención, ¿Cuál define mejor las acciones para evitar que
afectando todo el glande y cara interna del prepucio. Señale aparezcan los factores de riesgo?
cuál es, entre las siguientes, la orientación diagnóstica más A. Prevención primordial
probable: B. Prevención primaria
A. Candidiasis genital. C. Prevención esencial
B. Chancroide. D. Prevención inicial
C. Sífilis secundaria. E. Prevención incipiente
D. Balanitis por Trichomonas. 93. Con respecto a las hernias, marque la alternativa INCORRECTA:
A. La hernia femoral es muy frecuente en mujeres
B. La técnica de Bassini es un tipo de técnica con tensión
C. La hernia por deslizamiento es de órganos
retroperitoneales
D. Pueden existir hernias lumbares por ejemplo de Petit
E. La Técnica de Shouldice es un tipo de reparación tisular
94. Un paciente acude a consulta con una evidente paresia de la
musculatura facial derecha que le apareció tres días antes.
¿Cuál de los siguientes datos sugiere que la lesión causante no
es periférica y afecta al sistema nervioso central?
A. Oye los sonidos por el oído derecho con más intensidad
B. No nota el sabor de la comida por el lado derecho de la
lengua
C. Tiene nistagmus bilateral en la desviación de la mirada
hacia la derecha
D. Tiene una acusada debilidad del musculo orbilcular del ojo
derecho
E. Tiene erupción y dolor en el conducto auditivo externo
derecho
95. Un aloinjerto se define como:
A. El que se da entre individuos con antígenos idénticos
B. Trasplante a un sitio anormal
C. El que se da entre especies diferentes
D. El que ocurre entre miembros de la misma especie
genéticamente diferentes
E. Un injerto donde el donante es también receptor
96. En relación con la atención inmediata del RN, señale lo
correcto:
A. Secado inmediato para prevenir pérdidas por radiación
B. Aspiración primero de ambas fosas nasales
C. Debe tomarse la temperatura rectal
D. Administrar 1ml de Vitamina K para prevenir enfermedad
hemorrágica
E. El Capurro A se evalúa a las 12h de vida
97. Con respecto a la glicemia e hipoglicemia del recién nacido.
Marque lo INCORRECTO:
A. Las glucogenolisis dependen del aumento de adrenalina,
glucagón y disminución de la insulina
B. El menor valor de glicemia que un RN puede presentar se
da a las 2 horas
C. El VIG máximo por vía endovenosa es de 12.5
D. El minibolo consiste en 2g glucosa por cada kilo de peso
E. Luego del minibolo requiere darle dextrosa a un VIG de 6-8
98. En el manejo de RCP neonatal, si un recién nacido a los 30
segundos de manejo tiene una FC<100x´. ¿Qué medida
debemos instaurar?
A. Ventilación a presión positiva
B. Intubación
C. Uso de inotrópicos
D. Iniciar compresiones
E. Oxígeno a flujo libre Examen 7-A
99. Durante la evaluación de atención inmediata de un RN se
1. Con respecto a la terapia antitrombótica en el síndrome
detecta maniobra de Barlow y Ortolani positivas. Ante su
coronario, señale lo FALSO:
sospecha diagnostica que examen solicitaría y que tratamiento
instauraría. A. Se pueden emplear agentes fibrinolíticos
A. Ecografía y reducción abierta y osteosíntesis
B. Ecografía y reducción cerrada B. La Hirudina es un inhibidor directo de la Trombina
C. Ecografía y arnes de Pavlik C. El efecto de la Aspirina es consecuencia del bloqueo de formación
D. Radiografía y arnes de Pavlik de Tromboxano A2
E. Radiografía y reducción abierta y osteosíntesis
100. Una medida cuantitativa para determinar la extensión de un D. El Abciximab no es un inhibidor directo de la trombina
brote epidémico es la tasa de...
E. La ticlopidina y los inhibidores de receptores de glicoproteína IIb,
A. Ataque
IIIa son agentes antiplaquetarios
B. Incidencia
C. Prevalencia 2. Varón de 16 años, con meningoencefalitis de inicio brusco,
LCR muy turbio. Se inicia terapia con penicilina. ¿Con qué
antimicrobiano sustituiría a la penicilina?:
A. Dicloxacilina 8. El Cáncer de pulmón que suele producir síndrome de
Pancoast:
B. Clindamicina
A. Carcinoma microcítico de pulmón.
C. Ceftriaxona
B. Adenocarcinoma.
D. Eritromicina
C. Carcinoma epidermoide.
E. Metronidazol
D. Carcinoma de cèlulas grandes.
3. ¿Cuál es la etiología más frecuente de artritis séptica en la
población general?: E. Carcinoma bronquioloalveolar.

A. Staphylococcus aureus. 9. ¿Cuál de las siguientes suturas es de material absorbible?:

B. Pseudomonas aeruginosa. A. Seda

C. Chlarnydia trachomatis. B. Poliéster

D. Neisseria gonorrhoeae. C. Nylon

E. Haemophylus influenzae. D. Polipropileno

4. Con respecto a las células de Langerhans, marque lo E. Ácido poliglicolico


incorrecto:
10. En apendicitis aguda perforada, los gérmenes más
A. Proceden de la médula ósea frecuentemente aislados en el cultivo bacteriológico de líquido
peritoneal son:
B. Se ubican en el estrato granuloso
A. Pseudomonas y Bacteroides fragilis
C. Poseen enzimas hidrolíticas
B. Peptostreptococcus y Bacteroides esplanchnicus
D. Poseen gránulos de Birbeck
C. Bacteroides fragilis y Escherichia coli
E. Son captadoras de antígenos cutáneos
D. Lactobacillus y Escherichia coli
5. En una ITU en el niño, marque la incorrecta:
E. Pseudomonas y Escherichia coli
A. En el periodo neonatal las niñas la padecen con más frecuencia.
11. Varón de 17 años de edad, contacto de TBC. Acude por dolor
B. El E. coli es el más frecuente. progresivo en hemitórax derecho desde hace 10 días y sensación
de alza térmica. La radiografía de TÓRAX muestra derrame
C. Se recomienda una ecografía renal en el primer episodio de una pleural y el examen del líquido: proteínas 5g/ dL (sérico: 7 g/dL),
ITU DHL 300 mg/ dL. ¿Qué tipo de líquido es y cuál es la conducta
más adecuada?
D. Los antibióticos indicados son Cefalosporinas de tercera
A. Exudado / buscar etiologia.
E. La gammagrafía con Tc 99 es la mejor exploración para detectar
cicatrices renales. B. Trasudado / buscar etiologia.
6. Un niño de 10 años presenta, desde hace 4 horas, un cuadro de C. Exudado / drenaje toracico percutaneo.
fiebre elevada, tos, disnea y un estridor perfectamente audibles
en ambos tiempos respiratorios. El diagnóstico probable es: D. Trasudado / drenaje toracico percutaneo.

A. Absceso retrofaríngeo. E. Exudado / colocacion de tubo de drenaje.


B. Traqueítis bacteriana. 12. Una de las siguientes afirmaciones referidas a las
osteomielitis y artritis sépticas causadas por Staphylococus
C. Laringitis aguda. aureus NO es correcta:
D. Epiglotitis. A. La osteomielitis por S. aureus puede ser resultado de
diseminación hematógena, traumatismos o infección estafilocócica
E. Aspiración de cuerpo extraño.
sobreyacente.
7. Adolescente de 15 años de edad, sometido desde hace 4
meses a disminución de peso mediante dietas restrictivas por B. En los niños, la diseminación hematógena suele afectar las
metáfisis de los huesos largos, un área de crecimiento óseo muy
aludir sobrepeso. Expresa miedo intenso a volverse obesa,
vascularizada.
ausencia de 3 ciclos menstruales consecutivos. Al examen:
pálida, piel fría, IMC: 13, FC: 50 por minuto. ¿Cuál es el
C. En los adultos la osteomielitis hematógena suele afectar a las
diagnóstico más probable? vértebras, y rara vez aparece en los huesos largos.
A. Anorexia nerviosa D. La evidencia radiográfica de osteomielitis es previa al inicio de
los síntomas clínicos.
B. Hipertiroidismo
E. El absceso de Brodie es un foco aislado de osteomielitis
C. Síndrome de malabsorción
estafilocócica en el área metafisaria de los huesos largos.
D. Bulimia
13. La etiología más frecuente de la mediastinitis aguda es:
E. Síndrome depresivo
A. CA de pulmón.
B. Neumopatia necrotizante. B. Aumento de marcas vasculares con derrame

C. Perforación esofágica. C. Infiltrados dispersos con atrapamiento aéreo.

D. Traumatismo torácico abierto. D. Patrón en esponja

E. Traqueostomia. E. Normalidad radiológica

14. La diferenciación clínica fundamental y característica entre la 20. Recién nacido a término de 18 horas de vida, madre refiere
desnutrición de tipo marasmático y la de tipo Kwas- hiorkor está que su bebe no succiona bien, se encuentra hipoactivo, poco
dada por: reactivo al estímulo, con temperatura de 38° C. De los siguientes
enunciados ¿Cuál estaría en relación como fac- tor de riesgo para
A. La deshidratación. sepsis neonatal?

B. Los edemas. A. Toxemia materna


C. La coloración y consistencia del pelo. B. Ruptura de membranas menor a 8 horas
D. La anemia. C. Recién nacido grande para edad gestacional

E. Los signos tróficos de la piel. D. Infección de tracto urinario materno en el III Trimestre
15. Un niño que presenta signos de urticaria aguda en E. Rinofaringitis
consulta externa, no tiene antecedentes y tampoco presenta
ningún signo de gravedad. Lo más importante es: 21. Varón de 20 años de edad, que presenta convulsiones en el
servicio de Emergencia. ¿Cuál de los siguientes fármacos
A. Una cuidadosa historia clínica administraría?:
B. Una descripción adecuada de las lesiones A. Carbamazepina
C. Pedir exámenes de ayuda diagnostica, como: IgE y pruebas B. Diazepam
cutáneas de alergia.
C. Metilfenitoína
D. El estudio anatomopatológico por biopsia
D. Valproato
E. El manejo con antihistamínicos y corticoides.
E. Difenilhidantoína
16. En un niño de un año cuatro meses con el siguiente
hemograma: leucocitos: 2.400, abastonados: 0, segmentados: 20, 22. De los siguientes parásitos, ¿Cuál produce diarrea y síndrome
linfocitos: 74, monocitos: 6, Hb: 7,6 g/dL y plaquetas: 30.00 x de mala absorción con más frecuencia?:
mm3. El cuadro hematológico corresponde a:
A. Oxiuro
A. Eritroblastopenia transitoria
B. Áscaris
B. Leucopenia con neutropenia moderada
C. Tricocéfalos dispar
C. Pancitopenia con neutropenia severa
D. Giardia lamblia
D. Púrpura trombocitopénica
E. Hymenolepis nana
E. Síndrome anémico con neutropenia leve
23. Respecto a los tratamientos habituales o hábitos de un
17. Son vacunas de microorganismos vivos atenuados. Excepto: paciente programado para una intervención quirúrgica bajo
anestesia general, una de las respuestas siguientes es
A. Polio oral CORRECTA:
B. Hepatitis B A. Los bloqueantes beta adrenérgicos deben mantenerse durante
todo el periodo preoperatorio.
C. Sarampión
B. Los antiparkinsonianos deben suspenderse 48 hrs antes.
D. Rubéola
C. La heparina de bajo peso molecular a dosis profilácticas se
E. Parotiditis asocia a hemorragia intraoperatoria significativa.
18. Afectación muscular en la distrofia muscular de Duchenne: D. La ticlopidina no se asocia a riesgo de hemorragia intraoperatoria
significativa.
A. Comienza en los grupos musculares proximales de las
extremidades E. Evitar fumar cigarrillos durante las 24 hrs previas carece de
beneficio intraoperatorio.
B. Comienza en los grupos musculares distales de las extremidades
24. La localización ideal para hacer una traqueotomía es:
C. Afecta exclusivamente a la cintura pélvica
A. La membrana cricotiroidea
D. Afecta exclusivamente a la cintura escapular
B. La membrana miotiroidea.
E. Los músculos intercostales se afectan precozmente
C. Primer anillo traqueal.
19. Señale cuál sería el patrón radiológico que más
frecuentemente encontraríamos en un RN que ha sufrido un SAM:
D. Segundo o tercer anillo traqueal.
A. Patrón reticulogranular con broncograma E. Cuarto o quinto anillo traqueal.
25. Una lesión purpúrica que no palidece a la vitropresión, nos B. 10 a 15 mL
indica:
C. 25 a 30 mL
A. La presencia de edema dérmico.
D. 20 a 45 mL
B. La presencia de exocitosis
E. 50 a 90 mL
C. La presencia de extravasación hemática.
32. La arteria radial:
D. La presencia de una malformación venosa.
A. Es más grande que la arteria cubital
E. La presencia de una malformación capilar
B. Pasa a lo largo del lado cubital de antebrazo
26. ¿Cuál de las siguientes entidades está asociada con herpes
virus tipo 6? C. Pasa entre el tendón del musculo abductor largo del pulgar y el
ligamento radial colateral de la muñeca
A. Exantema súbito
D. Comienza a lo largo de la extremidad de la diáfisis del humero
B. Enfermedad febril de la infancia
E. Pasa sobre el musculo supinador largo
C. Síndrome de fatiga crónica
33. El musculo flexor común profundo esta inervado por los
D. Síndrome de Gianotti- Crosti nervios:

E. Todas las anteriores. A. Mediano e interóseo anterior

27. Micosis superficial que afecta el estrato córneo y se B. Mediano y ulnar


manifiesta con lesiones discrómicas con descamación fina:
C. Musculocutáneo y cubital
A. Tiña pedis.
D. Cubital y radial
B. Tiña corporis
E. Radial y mediano
C. Pitiriasis versicolor
34. En la pubertad, el cuerpo uterino crece bajo la influencia
D. Exofialosis directa de:

E. Tiña imbricada A. Hormonas hipofisiarias

28. ¿Cuál es el andrógeno fundamental en la mujer?: B. Hormonas ováricas

A. Androstenediona. C. Hormonas tiroideas

B. Dehidroepiandrosterona. D. Factores hipotalámicos

C. Androstenediol. E. Hormonas adrenales

D. Androsterona. 35. Un investigador pretende determinar si existe una asociación


entre las cifras de PA diastólica (medida en mmHg) y los niveles
E. Testosterona. de colesterol (medidos en mg/mL). Para ello, ha realizado estas
mediciones a 230 voluntarios. ¿Qué prueba estadística es la MÁS
29. En un ciclo menstrual de 30 días, con una gráfica de apropiada pare examinar esta asociación?:
temperatura corporal basal considerada ovulatoria; la duración
de la fase luteínica o secretoria será de: A. Regresión logística.
A. 21 ± 2 días B. Prueba de la t de Student.
B. 07 ± 3 días C. Prueba de Chi cuadrado.
C. 18 ± 1 días D. Correlación de Pearson.
D. 14 ± 2 días E. Prueba de Fisher.

E. 16 ± 2 días 36. Un estudio analiza la relación entre la PA sistólica (PAS) y la


edad en una muestra de mujeres adultas. Los autores presentan
30. Los siguientes factores pueden influir en la reserva de hierro los resultados como la siguiente ecuación de regresión lineal:
al nacer, EXCEPTO: PAS = 81,5 + 1,2 x edad. ¿Cuál de las siguientes afirmaciones
acerca de este análisis es FALSA
A. Transfusión feto-materna
A. La edad se ha utilizado como variable independiente
B. Transfusión de gemelo a gemelo
B. La pendiente de la recta es de 1,2 mmHg/año de edad
C. Rotura de vasos placentarios
C. El valor 81,5 corresponde a la media de PAS en la muestra de
D. Anemia de la madre
mujeres
E. Todas las anteriores
D. Por cada año más de edad de las mujeres de la muestra, su
31. En los neonatos normales a término, la capacidad gástrica es: PAS se incrementa en 1,2 mmHg de promedio

A. 5 a 9 mL E. Se ha asumido que la relación entre la edad y la PAS es lineal

37. El test de Capurro para la edad gestacional no incluye:


A. Forma de la oreja D. Virus vivo atenuado

B. Pliegues plantares E. Ninguna de las anteriores.

C. Textura de la piel 44. Niño de 6 años de edad con riesgo de tétanos. Si sufre una
herida contaminada la conducta correcta es, asumiendo que tiene
D. Nódulo mamario completas las inmunizaciones y refuerzo de vacuna antitetánica:

E. Ventana cuadrada A. Vacuna antitetánica


38. ¿En cuál de los siguientes casos suele indicarse la prueba de B. Antitoxina tetánica
estrógenos para evaluar el origen de una ameno- rrea?:
C. Inmunoglobulina
A. Cuando la prueba de la progesterona es positiva.
D. Antitoxina + vacuna antitetánica
B. Cuando la prueba de la progesterona es negativa.
E. No se aplica ni vacuna ni inmunoglobulina
C. Cuando se detectan niveles elevados de gonadotropinas.
45. La desviación de la atención hacia otros puntos del
D. Cuando se detectan niveles disminuidos de gonadotropinas. pensamiento, se denomina…. Y la afectividad inadecuada
respecto al contexto en que se produce, se denomina………:
E. En todos los casos de amenorrea primaria.
A. Hiperprosexia/Aprosodia
39. ¿Cuál de los siguientes tendones se inserta en el astrágalo?:
B. Hipoprosexia/Paratimia
A. Aquiles
C. Paraprosexia/Alexitimia
B. Peroneo lateral corto
D. Paratimia/Afasia
C. Peroneo lateral largo
E. Disprosexia/Distimia
D. Tibial anterior
46. Con respecto a la vacuna triple (DPT), señale lo falso:
E. El astrágalo carece de inserción tendinosa
A. Debe conservarse en refrigeración entre 4 y 8° C
40. Respecto al desarrollo psicomotor normal del niño, señale la
afirmación correcta: B. Se aplica por vía IM en una dosis de 0,5 mL
A. Presiona pinzando el pulgar con el índice a los 12 meses C. Se puede administrar desde los 3 meses hasta los 72 meses de
edad
B. Junta las manos en la línea media a los 6 meses
D. La duración de la inmunidad conferida es de 10 años
C. Vuelve las páginas de un libro a los 8 meses
E. Puede causar reacción febril en las primeras 24 – 48 horas
D. Usa 4-6 palabras a los 9 meses
47. Estructuras que forman el plexo de Kiesselbach, EXCEPTO:
E. Se sienta estable y sin apoyo a los 6 meses
A. Esfenopalatina.
41. La edad recomendada para la introducción de la ablactancia
es: B. Palatina mayor.
A. Entre los 15 días y los 2 meses C. Etmoidal anterior.
B. Entre los 2.3 meses D. Etmoidal posterior.
C. A los 4 meses E. Palatina menor.

D. A los 6 meses 48. Las pruebas no paramétricas:

E. A los 8 meses A. Únicamente se utilizan para comparar distribuciones de


variables continuas.
42. La leche materna, para evitar la sobrecarga osmótica renal en
el lactante, contiene menos: B. Requieren la comprobación del requisito de normalidad.
A. Sodio C. Originan unos valores de error alfa similares a los que se
calculan mediante pruebas paramétricas.
B. Hierro
D. Deben utilizarse siempre que manejemos muestras de gran
C. Potasio tamaño.
D. Cloro E. No realizan asunciones sobre el tipo de distribución de la
variable
E. Fósforo
49. Uno de los siguientes NO es un test no paramétrico:
43. De la inmunización activa del sarampión que se utiliza en la
actualidad, ¿Cuál es la más eficaz? A. U de Mann-Whitney
A. Gammaglobulina B. Test de Kruskal-Wallis
B. Virus muerto C. Test de Wilcoxon
C. Virus vivo con gammaglobulina D. Análisis de varianza.
E. Coeficiente de correlación de Spearman A. Antipsicóticos

50. Cuál de las siguientes afirmaciones es falsa: B. Benzodiacepinas

A. En una paciente perimenopáusica y deseos de descendencia C. Antidepresivos inhibidores de la recaptación de noradrenalina


cumplidos, que no responde al tratamiento con gestágenos es
candidata a una ablación endometrial o histerectomía D. Antidepresivos inhibidores de la recaptación de serotonina

B. Durante los 18 meses posteriores a la menarquia son frecuentas E. Estimulantes


los ciclos anovulatorios, a intérvalos irregulares.
56. Cuál es la finalidad de la administración entendida como una
C. Debido a lo comentado en el apartado anterior, son frecuentes ciencia:
el flujo menstrual irregular entre las adolescentes jóvenes
A. Explicar el comportamiento y la conducción de la organización
D. Cuando el sangrado menstrual excesivo, comienza con la
menarquia y como consecuencia del sangrado produce una B. Uso de técnicas para la conducción de la organización
anemia severa, hay que descartar un trastorno de la coagulación
C. Comprender el comportamiento del personal
(deficiencia del factor de Von Willebrand)
D. Explicar el porqué del éxito de la organización
E. En la valoración de la HUD es imprescindible la realización de
un estudio hormonal. E. Explicar el funcionamiento y evolución de la organización
51. El agente causal del impétigo ampollar en lactante y niños 57. La secuencia lógica del proceso administrativo es:
pequeños es:
A. Organización-dirección-control-planificación
A. Aeromona hydrophila
B. Dirección-planificación-organización-control
B. Estafilococo aureus coagulasa positivo
C. Planificación-organización-dirección –control
C. Estreptococo piógeno
D. Planificación-dirección-organización-control
D. Estreptococo pneumoniae
E. Organización-planificación-dirección-control
E. Legionella
58. Respecto a trastornos de la refracción, señale la respuesta
52. Varón de 32 años, tiene sensación de pesadez escrotal, con INCORRECTA:
tumoración difusa de consistencia lisa y firme distal a la cabeza
del epidídimo. No signos inflamatorios, marcadores tumorales A. La presbicia se corrige con lentes positivas.
negativos, transiluminación negativa. El diag- nóstico más
probable es: B. Las ametropías esféricas se corrigen con lentes cónicas.

A. Hidrocele. C. En las lentes negativas, el movimiento de la imagen acompaña al


movimiento de cristal.
B. Seminoma.
D. En la hipermetropía la imagen de la esquiascopÍa acompaña
C. Carcinoma embrionario. el movimiento del retinoscopio.

D. Hernia inguinal directa. E. El tratamiento del astigmatismo en la actualidad es la cirugía


láser (LASIK, etc.).
E. Hernia inguinal indirecta.
59. El reflejo de acomodación consta de los siguientes
53. Niño nacido de parto normal hace 2 días. Desde hoy presenta componentes:
edema palpebral y abundante secreción purulenta en ambos ojos.
El diagnóstico más probable es conjuntivitis neonatal por: A. Convergencia ocular, contracción pupilar y acomodación.

A. Gonococo B. Convergencia ocular, dilatación pupilar y aumento de la


concavidad de la superficie retiniana
B. Clamidia
C. Contracción pupilar, aumento del diámetro AP del cristalino y
C. Estreptococo aplanamiento retiniano.
D. Herpes simple D. Miosis, enoftalmos y reducción de la presión de la cámara
anterior.
E. Haemophilus
E. Contracción pupilar y contracción del musculo ciliar.
54. La hemorragia intracraneal neonatal es más frecuentemente
encontrada en los siguientes casos, EXCEPTO: 60. La vena porta generalmente se forma por la confluencia de las
venas:
A. Prematuros
A. Mesentérica superior y esplénica
B. Neonatos nacidos por cesárea
B. Mesentérica superior y mesentérica inferior
C. Neonatos con presentación podálica
C. Mesentérica superior y coronaria estomáquica
D. Neonatos nacidos con ayuda mecánica, excepto el fórceps bajo
D. Mesentérica inferior y esplénica
E. Neonatos que han sufrido asfixia
E. Esplénica y coronaria estomáquica
55. ¿Cuál es el principal tratamiento psicofarmacológico del
trastorno obsesivo compulsivo? 61. El ectropión puede ocasionar:
A. Blefaritis 67. Multípara de 40 años de edad, presenta vómitos y distensión
abdominal. Al examen físico de la región inguinocrural se
B. Triquiasis encuentra tumoración dolorosa no reductible por debajo del
ligamento iliopubiano. ¿Cuál es el diagnóstico más probable?:
C. Blefaroespasmo
A. Hernia inguinal indirecta
D. Distiquiasis
B. Hernia crural
E. Queratopía por exposición
C. Hernia inguinal directa
62. ¿Cuál de los siguientes pólipos tiene mayor potencial
maligno?: D. Várices del cayado de la safena
A. Pólipo juvenil E. Hernia obturatriz
B. Pólipo hamartamatoso. 68. El patógeno intestinal que puede asociarse al síndrome de
Guillan Barré es:
C. Pólipo hiperplásico.
A. Criptosporidium.
D. Pólipo adenomatoso.
B. Shigella.
E. Ninguna de las anteriores.
C. Salmonella.
63. ¿Cuál es el procedimiento quirúrgico curativo en el cáncer
localizado en el tercio distal del recto?: D. Yersinia.
A. Resección abdominoperineal E. Campylobacter.
B. Íleo transverso anastomosis 69. La alteración del equilibrio ácido-básico producida por los
vómitos repetidos por el síndrome pilórico es:
C. Extirpación mesorectal total
A. Acidosis hiperclorémica.
D. Colostomía sigmoidea en asa
B. Acidosis hipernatremica
E. Colectomía izquierda
C. Acidosis Hipoclorémica.
64. Paciente varón de 75 años que presenta prurito, ictericia y
dolor leve en cuadrante superior derecho del abdomen. Al D. Alcalosis hiperpotasémica.
examen hepatomegalia. En la ecografía se muestra dilatación de
vías biliares intra y extrahepaticas sin cálculos en vía biliar. La E. Alcalosis hipoclorémica.
TAC de abdomen no muestra masas en páncreas. Tiene BT de 10,
FA de 400 y amilasa normal. En el diagnóstico diferencial del 70. El seguimiento postoperatorio para detectar la recurrencia del
paciente debe incluirse lo siguiente: cáncer de colon se realiza con:

A. Cirrosis biliar primaria. A. Marcadores tumorales

B. Colestasis inducida por drogas. B. Resonancia magnética

C. Colangiocarcinoma intrahepático. C. Radiografía abdominal

D. Ampuloma. D. Estudio de la función hepática

E. Enfermedad de Caroli. E. Ecografía abdominal

65. Paciente de 65 años, con cuadro de ictericia progresiva, 71. El tratamiento de elección de las hemorroides externas
coluria y dolor abdominal. En las últimas 58 horas se agregan trombosadas es:
fiebre y escalofríos. ¿Cuál es el diagnóstico más probable?
A. Dieta rica en fibra
A. Absceso hepático.
B. Ablación quirúrgica
B. Colangitis aguda.
C. Baños de asiento
C. Hepatitis aguda.
D. Ligadura con banda de caucho
D. Neoplasia de la vesícula biliar.
E. Reducción manual
E. Ninguna de las anteriores.
72. Mujer de 72 años de edad, con 7 días de dolor en el cuadrante
66. ¿Cuál es el vaso sanguíneo que permite diferenciar una superior derecho, ictericia progresiva y temperatura de 39 ºC
hernia inguinal directa de la indirecta?: recibió tratamiento con cefalexina vía oral, sin mejoría. Ingresa a
Emergencia hipotensa, con compromiso del sensorio.
A. Femoral Leucocitos: 19.000 x mm2, bilirrubinas, transaminasas y
fosfatasa alcalina elevadas, concentración de amilasa sérica
B. Epigástrica inferior normal. ¿Cuál es su diagnóstico?
C. Folicular A. Hepatitis.

D. Deferente B. Colelitiasis.
E. Del epidídimo C. Enfermedad de Wilson

D. Colecistitis aguda alitiásica.


E. Colangitis supurativa aguda. E. Todas las anteriores son correctas

73. Señale lo CORRECTO en relación al carcinoma de la vesícula 79. En sala de recuperación Usted sospecha sobredosis de
biliar: anestesia por lo siguiente:

A. La frecuencia es menor en pacientes con “vesícula en porcelana” A. Paciente con hipotensión y taquicardia.

B. El tratamiento adyuvante con quimio o radioterapia es muy eficaz B. Paciente con hipertensión y bradicardia.

C. El adenocarcinoma no es el más frecuente C. Paciente con hipertensión y taquicardia.

D. Se asocia a cálculo de vesícula D. Paciente con hipotensión y bradicardia.

E. El coeficiente varón/mujer igual al 2/1 E. Paciente con excitación psicomotriz.

74. En el caso de un paciente con hiperplasia prostática benigna, 80. El tumor primario más frecuente en mediastino posterior es:
¿Cuál, entre las siguientes circunstancias, NO establece por sí
misma indicaciones de cirugía?: A. Linfoma.

A. Hematuria severa recurrente. B. Fibroma.

B. Retención urinaria que requiere sondaje vesical permanente. C. Quiste esofágico.

C. Infecciones de orina de repetición. D. Tumor neurogénico.

D. Nicturia de dos veces. E. Quiste broncogénico.

E. Hidronefrosis retrograda. 81. ¿Cuál es el test que tiene por objetivo comparar la audición
de un sonido transmitido por vía aérea, poniendo el dia- pasón
75. Varón de 30 años de edad con fractura cerrada transversal de delante del pabellón auricular, con la audición del mismo sonido
tercio medio de fémur, tras sufrir un accidente de trá- fico, sin transmitido por vía ósea, situado el diapasón sobre la mastoides?
otras lesiones asociadas. ¿Cuál es la conducta a seguir?:
A. Test de Weber.
A. Tracción transesquelética.
B. Test de Rinné.
B. Clavo intramedular.
C. Test de Schwabach.
C. Placa y tornillos.
D. Test de Fowler.
D. Fijador externo
E. Test de Sisi.
E. Yeso.
82. Hombre joven con dolor facial y congestión nasal, descarga
76. En la fractura de Monteggia hay: nasal amarilla después de infección de vías respiratoria alta hace
10 días. T° 38.2°C y sensibilidad del seno maxilar a la palpación y
A. Fractura del 1/3 distal del cúbito + luxación de cabeza de radio mucosas nasales pálidas con algo de dre- naje amarillento.
Obnubilación del seno maxilar a transiluminación. ¿Cuál es el
B. Fractura de 1/3 proximal del cúbito + luxación de la cabeza diagnóstico MÁS probable?:
radial
A. Sinusitis aguda.
C. Fractura de cúbito y radio + luxación de codo
B. Sinusitis crónica.
D. Fractura de 1/3 medio del radio + luxación del codo
C. Angina de Vincent.
E. Ninguno de ellos
D. Angina de Ludwig.
77. Paciente varón 24 años, cae mientras jugaba fútbol en la calle
golpeando directamente la rodilla derecha contra el pavimento, E. Celulitis orbitaria.
inmediatamente presenta intenso dolor, edema e incapacidad
para extender la pierna derecha. ¿Qué me- didas se deben tomar 83. El raquitismo se caracteriza por:
inmediatamente?
A. Craneotabes.
A. Frotación y que siga jugando.
B. Hendiduras de Harrison.
B. Hielo, ferulización y traslado a emergencia.
C. Retardo en el cierre de la fontanela.
C. Traccionar la rodilla hasta lograr extensión.
D. Tórax de paloma
D. Analgésicos y que descanse en su casa.
E. Todas las anteriores.
E. Traccionar la rodilla hasta lograr flexión.
84. Señale lo correcto con respecto al tratamiento de la
78. Los siguientes signos sugieren el diagnóstico de displasia de bronquiolitis en el niño:
cadera:
A. Administrar un antibiótico que tenga una menor resistencia.
A. Asimetría de pliegues glúteos
B. Incluir corticoides por ser beneficiosos.
B. Signo de Barlow positivo
C. La Ribavirina es útil en todos los casos
C. Limitación en la abducción
D. Los fármacos broncodilatadores se aplican empíricamente.
D. Signo de Ortolani positivo
E. En casos severos la traqueotomía no es beneficiosa. E. Síndrome nefrótico.

85. Lactante de 6 meses que hace tres días inicia con rinorrea y 90. Un lactante de 7 meses presenta, presenta fiebre 38ºC.
tos progresiva. Al examen: tiraje subcostal, taquipnea, espiración No existe ningún otro síntoma y la exploración física es nor- mal,
prolongada, sibilancias y cianosis perioral. La radiografía de salvo el hallazgo de un cuadro catarral leve, conjuntivitis y
pulmones revela hiperinflación. ¿Cuál es la terapia inmediata adenopatías cervicales posteriores. El niño parece encontrarse
inicial? bastante bien. ¿Cuál de las siguientes afirmaciones describe
mejor los factores diagnósticos que deben considerarse en este
A. Salbutamol caso?:

B. Amoxicilina A. Si apareciera una erupción cutánea 24 horas después de


comenzar la fiebre, sería probable el diagnóstico de roséola o
C. Ribavirina exantema súbito.
D. Oxígeno húmedo frío B. En la roséola infantil, el diagnóstico diferencial puede ser
difícil, puesto que en las primeras 36 horas cursa con leucocitosis con
E. Eritromicina
predominio de neutrófilos.
86. En un centro de salud la enfermera da una charla
sobre control prenatal a un grupo de madres, la semana siguiente C. La fiebre podría corresponder a los pródromos de una
Enfermedad de Kawasaki.
la obstetriz da la misma charla al mismo grupo de madres. La
enfermera ordena a una técnica de enfermería que realice una D. Si la fiebre dura 2 días, y luego aparece un exantema
visita domiciliaria, mientras que el médico manda a la misma confluyente en cara de 3 días después, es probable que sea una
técnica que lo ayude en consultorio externo. ¿Estos problemas a Rubeola
que componentes de la administración corresponden?:
E. Es probable una escarlatina, pues es una enfermedad
A. Planificación. propia de niños de 6 a 18 meses.
B. Organización. 91. Establezca la relación entre las siguientes columnas:
C. Control. A. Sarampión
D. Dirección. B. Rubéola
E. Evaluación. C. Varicela
87. El eczema atópico usualmente aparece a la edad de: D. Mononucleosis infecciosa
A. 2 semanas E. Herpes simple
B. 3 meses 1. Linfadenopatía
C. 2 años 2. Lesiones simultáneas en distintos estadíos generalizada +
esplenomegalia
D. 6 años
3. Vesículas pequeñas sobre base eritematosa
E. 12 años.

88. Varón de 5 años presenta de forma súbita dolor en 4. Manchas de Forchheimer


teste derecho, sin antecedente de traumatismo previo. 5. Manchas de Koplick
Exploración: afebril, teste tumefacto, doloroso a la exploración,
con ausencia del reflejo cremastérico. Señale la afirmación falsa: A. (a–5) (b–4) (c–2) (d–1) (e–3)
A. En el Eco-Doppler se observaría disminución del flujo B. (a–2) (b–5) (c–4) (d–3) (e–1)
sanguíneo en ese testículo.
C. (a–1) (b–4) (c–2) (d–3) (e–5)
B. El tratamiento consiste en antibióticos, antiinflamatorios, y
tras la fase aguda, cirugía. D. (a–3) (b–4) (c–2) (d–5) (e–1)

C. La mayor incidencia se produce en la niñez tardía y E. (a–4) (b–2) (c– 5) d–1) (e–3)
adolescencia temprana.
92. En Emergencia se recibe a una niña de 10 años de edad que
D. En mayores de 13 años, se debe hacer diagnóstico ha presentado durante 40 minutos 8 crisis convulsivas, entre las
diferencial con una epididimitis. cuales no ha recuperado la conciencia. El familiar refiere que las
convulsiones han tenido características tónico-clónicas
E. En el periodo neonatal, generalmente tiene mal generalizadas. Su diagnóstico es:
pronóstico.
A. Convulsiones tónico-clónicas simples
89. Niño pálido, con parpados hinchados y orinas
cargadas. Padres refieren antecedente de cuadro B. Epilepsia
faringoamigdaliano quince días antes, tratado medicamente.
¿Cuál es el diagnóstico más probable? C. Convulsión febril

A. Glomerulonefritis. D. Síndrome de West

B. Infección del tracto urinario. E. Estado epiléptico

C. Litiasis renal. 93. ¿Cuál es el tratamiento más efectivo en el espasmo infantil o


síndrome de West?
D. Nefritis intersticial.
A. Fenobarbital
B. Etosuximida C. La edad gestacional por examen físico es más exacta que la edad
gestacional por FUR
C. Difenilhidantoína
D. La edad gestacional por Capurro tiene una variación de +/– 6
D. Vigabatrina semanas y puede hacerse máximo hasta las 12 horas de vida.
E. Carbamazepina E. El método de Usher añade evaluaciones neurológicas
94. Según el calendario de inmunizaciones, ¿Qué vacuna debe 99. Niño de 4 semanas de edad, recibe lactancia materna
recibir el niño a los 12 meses de edad? exclusiva, gana peso 30 g x día, ictérico, hasta la raíz del muslo,
luce saludable, Grupo sanguíneo: O (+), Hto: 48%, Reticulocitos
A. AMA (AntiamarílicA) 1%, Bilirrubinas totales: 12 mg/dL, Bilirrubina di- recta: 0.5 mg/dL,
TSH normal. En relación a la lactancia materna exclusiva. ¿Cuál
B. Refuerzo DPT
es la conducta a seguir?
C. Polio oral
A. Suspender por una semana
D. SPR (Triple víricA)
B. Suspender por 48 horas
E. 3ra dosis Pentavalente
C. Continuar e iniciar fototerapia
95. Niña de 9 meses que inicia vómitos y dolor abdominal hace 12
horas, minutos antes de ir a emergencia, presento de- posiciones D. Suspender e iniciar fototerapia
color rojo oscuro. El abdomen se encuentra distendido y con E. Proseguir con la lactancia materna.
dolor discreto a la palpación. El diagnóstico más probable es:
100. La causa más frecuente de síndrome de dificultad
A. Estenosis pilórica respiratoria o enfermedad de membrana hialina en el recién
nacido prematuro es:
B. Apendicitis aguda
A. El defecto de surfactante pulmonar por inmadurez.
C. Infección urinaria
B. Un neumotórax a tensión.
D. Invaginación
C. Un síndrome de aspiración meconial.
E. Enfermedad ulcero péptica
D. Una infección respiratoria.
96. ¿Cuál de las siguientes afirmaciones es correcta respecto a la
talla baja asociada a déficit de hormona de crecimiento (GH)? E. La escasa fuerza muscular
A. La deficiencia de GH es la causa más frecuente de
hipocrecimiento armónico patológico.

B. En las formas congénitas de deficiencia de GH, el crecimiento


prenatal suele ser normal.

C. La maduración ósea en la deficiencia de GH, al contrario de lo que


ocurre en otras endocrinopatías, se encuentra muy elevada.

D. Los valores séricos de IGF-I se encuentran claramente elevados


en la deficiencia de GH.

E. El tratamiento con GH debe iniciarse precozmente, pero debe


suspenderse antes de la pubertad, por el riesgo de desarrollo de
leucemia en este periodo de la vida.

97. Por lo que hace referencia al tratamiento del asma bronquial,


una de las siguientes respuestas es FALSA:

A. El salbutamol se indica como medicación de rescate.

B. La prednisona oral a pequeñas dosis puede estar indicada en el


asma inestable moderada.

C. Los glucocorticosteroides inhalados se metabolizan en el hígado.

D. Los agonistas adrenérgicos-beta2 pueden emplearse en


combinación con los glucocorticosteroides inhalados en el asma
persistente leve-moderada.

E. El efecto del salmeterol dura 24 horas.

98. Señale lo correcto en relación a la edad gestacional por


examen físico del RN:

A. Test de Capurro evalúa el color del nódulo mamario y tiene mucha


correlación estadística con la edad gestacional Examen 8-A

B. El test de Ballard considera el examen físico y neurológico del RN 1. Paciente varón de 60 años con antecedente de HTA e
y puede ser aplicado a prematuros Insuficiencia renal crónica en terapia dialítica regular, ingresa a
emergencia por disnea, SatO2 79%, PA 210/110mmHg, crépitos
en ACP y alteración de conciencia. Marque la FALSA:
A. Se trata de una emergencia hipertensiva. A. Hematuria, proteinuria, hiperazoemia y retención renal de agua
y sal
B. Requiere descartar evento cerebrovascular.
B. Proteinuria de unos 4g/d con hematuria e HTA
C. Probablemente presente edema agudo de pulmón.
C. Disuria y leucocituria
D. Requiere control de PA inmediato con objetivo 120/80mmHg en 24
horas. D. Hematuria e HTA con función renal normal

E. Está contraindicado el uso de Betabloqueadores EV y VO. E. Microhematuria recidivante en niños

2. ¿Cuál es el agente etiológico de la endocarditis infecciosa en 8. Es un trastorno neuromuscular que se caracteriza por
un usuario de drogas endovenosas? debilidad muscular especialmente de los labios y extraoculares
(diplopía y ptosis) así como debilidad proximal y fatigabilidad de
A. Klebsiella pneumoniae. los músculos esqueléticos, con reflejos osteotendino- sos
profundos conservados:
B. Enterococos faecalis.
A. Miastenia Gravis.
C. Estreptococos viridans.
B. Miopatía por corticoides.
D. Neisseria gonorrhoeae.
C. Miopatía por hipertiroidismo.
E. Staphylococcus aureus.
D. Neurastenia.
3.¿Cuál de las siguientes es la manifestación más frecuente en la
depresión?: E. Botulismo.

A. Suicidio 9. Un chico de 17 años, deportista y sin antecedentes de interés,


acude al médico por presentar ictericia conjuntival oca-
B. Bajo rendimiento sionalmente. Niega ingesta de fármacos y dolor abdominal. En
los análisis realizados se observan cifras de ALT, AST, GGT y FA
C. Alcoholismo
normales, con bilirrubina total de 3,2 mg/dl y bilirrubina directa de
D. Pseudodemencia 0,4 mg/dl. No tiene anemia y el hígado es ecográficamente
normal. ¿Cuál es su diagnóstico?
E. Agresividad
A. Síndrome de Rotor.
4. En relación a la diabetes insípida es falso:
B. Coledocolitiasis.
A. La prueba de deshidratación se suspenderá cuando pierda un
10% del peso corporal inicial. C. Síndrome de Dubin-Johnson.

B. Ha sido descrito un origen autoinmune en algunos casos de D. Hepatitis aguda.


diabetes insípida central.
E. Síndrome de Gilbert.
C. El tratamiento de la diabetes insípida nefrogenica comporta la
10.El nervio laríngeo recurrente inerva todos los músculos
utilización de diuréticos.
intrínsecos de la laringe, EXCEPTO:
D. La lisina vasopresina no es la hormona antidiuretica natural
A. Cricotiroideo.
humana.
B. Cricoaritenoideo lateral.
E. La poliuria de la diabetes insípida no respeta el sueño.
C. Aritenoideo oblicuo.
5.El ERGE puede relacionarse con cualquier de las entidades
siguientes, EXCEPTO una: D. Cricoaritenoideo posterior.
A. Fibrosis pulmonar. E. Tiroaritenoideo.
B. Broncoespasmo. 11.Mujer de 13 años con dolor en mesogastrio que se irradia,
hacia fosa iliaca derecha, se acompaña de náuseas, y vómi- tos.
C. Neumonía recurrente.
Al examen: temperatura de 38,4 C, abdomen doloroso en
D. Sinusitis. hemiabdomen inferior derecho. Laboratorio: Sedimento urinario:
6-8 hematíes x campo. Hemograma: leucocitosis de 14 000 x
E. Laringotraqueitis. mm3, FUR hace 30 días. Tiene como presunción diagnóstica
apendicitis aguda. ¿Cuál de las siguientes patologías se
6. El esquema de elección para el tratamiento de la tuberculosis considera en el diagnóstico diferencial?
pulmonar en el adulto no tratado previamente es:
A. Poliposis intestinal
A. 2HRSE/4R2H3
B. Vólvulo de ciego
B. 2RHRZSE/4R2H2E2
C. TBC entero peritoneal
C. 2HRZ/4R2H2
D. Litiasis vesical
D. 2HRZE/4H3R3.
E. Diverticulitis de Meckel
E. HRZE/7R2H2
12.En la anestesia epidural, ¿Cuáles de los signos clínicos se
7.El síndrome nefrítico consiste en: presentan como complicación por punción e inyección inad-
vertida de la duramadre?
A. Hipertensión arterial y endocraneana B.85

B. Hipotensión grave, parálisis respiratoria C.25

C. Hipertensión arterial, fiebre D.75

D. Fiebre, hipotensión E. 100

E. Hipertensión, oliguria 18.La causa más frecuente de síndrome de Asherman es:

13.Frente a un paciente varón de 10 años de edad, con 72 horas A. Infección endouterina posparto.
de enfermedad, con fiebre, dolor ósea localizado en tibia, eritema
de partes blandas, con sospecha de osteomielitis, el método de B. Infección endouterina postaborto.
diagnóstico precoz más útil es:
C. Miomectomía.
A. Proteína C reactiva
D. Tuberculosis genital.
B. Hemocultivo
E. Legrado uterino diagnóstico.
C. Velocidad de sedimentación
19.El legrado como medio diagnostico puede ser omitido antes
D. Radiografía de tibia del tratamiento de una HUD en el siguiente caso:

E. Gammagrafía ósea A. Adolescencia.

14.Paciente varón de 70 años de edad, con diagnóstico clínico de B. Postmenopausia.


vólvulo de sigmoides, en mal estado general, tóxico,
taquicárdico, hipotenso y con reacción peritoneal. ¿Qué tipo de C. Premenopausia.
cirugía es la MÁS ADECUADA?:
D. Todas las anteriores.
A. Colostomía.
E. Ninguna de las anteriores.
B. Resección más colostomía.
20.¿Cuál de las siguientes alternativas NO corresponde a
endometriosis moderada a severa?
C. Cecostomia más pexia.

D. Resección más anastomosis. A. CA 125 disminuido

E. Ileostomía. B. Dismenorrea secundaria

15.Paciente de 72 años, que acude a Emergencia por C. Dispareunia


hematoquezia. Durante la evaluación presenta hipotensión y
D. Dolor pélvico crónico
shock hipovolemico que responde al tratamiento médico. Se
coloca sonda nasogástrica no evidenciándose sangrado. Por E. Subfertilidad
frecuencia, ¿cuál sería su primera posibilidad diagnóstica?:
21.¿Cuál de los siguientes criterios se usa para establecer el
A. Cáncer de colon derecho. diagnostico de Vaginosis bacteriana?
B. Colitis ulcerativa hemorrágica. A. Aumento de células indicio y aumento de leucocitos.
C. Hemorragia diverticular de colon. B. Secreción blanquecina adherente a la pared vaginal
D. Hemorroides. C. pH de secreción vaginal menor de 4,5
E. Malformación arteriovenosa. D. Test de aminas o KOH positivo
16.Con respecto al Sistema de Redes y Microrredes de Salud, E. Incremento de leucocitos en el frotis vaginal
marcar lo que no corresponda:
22.Mujer de 30 años de edad, presenta retraso menstrual de 7
A. Los establecimientos de salud del primer de salud se ordenan días. Para diagnosticar un posible embarazo, ¿Cuál es el examen
principalmente en base a Redes y Microrredes de Salud. confirmatorio?:
B. La unidad básica de gestión lo constituyen las microrredes de A. Estrógenos
salud.
B. Progesterona
C. Los puestos de salud por lo general son cabecera de redes de
salud C. HCG-beta

D. Los centros de salud pueden contar o no con internamiento. D. LH

E. La conformación y/u organización de las Redes y Microrredes E. FSH


está a cargo de los Gobiernos Regionales.
23.¿Cuáles son los signos y síntomas de la ruptura uterina
17.Un centro de salud programó para el año 2009 realizar 2000 consumada?
controles de crecimiento y desarrollo, para 1 500 niños menores
de 5 años, utilizando 800 horas enfermera. AI finalizar el año A. Acinturamiento del útero, dinámica uterina intensa, sufrimiento
atendió 200 consultantes nuevos, 300 reingre- santes y 500 fetal agudo
continuadores, utilizando 600 horas enfermera. El porcentaje de
avance de meta en atenciones fue: B. Cese de contracciones uterinas, palpación fetal fácil, sufrimiento
fetal agudo
A.50
C. Descenso del útero, sangrado silencioso, dinámica uterina B. Fluticasona
irregular
C. Hidrocortisona
D. Sangrado vaginal, crecimiento uterino, hipertonía
D. Metilprednisolona
E. Tono uterino aumentado, sangrado escaso, taquicardia,
hipotensión E. Prednisona

24.Una primigesta de 37 semanas de edad gestacional ingresa 29.Niño de 1 año y 9 meses con diarrea mucosanguinolenta y
por rotura prematura de membranas de 24 horas de evo- lución. fiebre alta. Hace 1 semana recibió de regalo un perro de pocos
No hay trabajo de parto ni signos de infección amniótica El test días de nacido. ¿Cuál es el germen causal más probable?
de Bishop (de maduración cervical) es de 6 puntos. El feto esta
en cefálico y no hay signos de sufrimiento fetal. ¿Cuál es la A. Campilobacter Yeyuni
conducta obstétrica indicada?:
B. Entamoeba Histolytica
A. Cesárea.
C. E. Coli
B. Inducción del parto con Oxitocina.
D. Salmonella No tiphica
C. Administrar Antibióticos y corticoides e inducir el parto 48 horas
E. Yersinia enterocolitica
después.
30.Niña de 2 años de edad, que desde los 9 meses presenta
D. Administrar Antibióticos y esperar el comienzo espontáneo del
episodios de lesiones eritematosas descamativas en mejillas,
parto.
abdomen y partes extensoras de miembros, con prurito intenso.
E. Esperar el comienzo espontáneo sin tratamiento y hacer cesárea Actualmente aparecen lesiones similares después de comer
si aparecen signos de infección amniótica. maní. Al examen se evidencia pliegues redundantes
infraorbitarios. ¿Qué tipo de dermatitis consideraría en su
25.En un examen, a un RN se le comprueba ausencia de presunción diagnóstica?:
murmullo vesicular en un lado del tórax. La causa más probable
es: A. Por ingesta de alimentos

A. B. Seborreica

B. Bronconeumonía. C. Alérgica de contacto

C. Neumotórax. D. Eccematoide infecciosa

D. Atelectasia E. Atópica

E. Síndrome de distrés respiratorio. 31.Las anastomosis entre el sistema portal y el de las venas
cavas se encuentran en los siguientes sitios, EXCEPTO:
F. Enfermedad de membrana hialina.
A. Recto.
26.RN de 38 semanas de edad gestacional presenta distensión
abdominal y vómitos biliosos durante el primer día de vida. En el B. Bazo.
examen físico se palpan cordones duros que siguen el marco
C. Retroperitoneo.
cólico. Una radiografía en bipedestación muestra una masa en
pompa de jabón en cuadrante inferior derecho. El tratamiento D. Cardias.
inmediato más apropiado es:
E. Ombligo.
A. Antibioterapia.
32.La sensibilidad de una prueba diagnóstica mide:
B. Supositorios de glicerina.
A. La proporción de casos de pacientes sin la enfermedad que
C. Enema de solución hiperosmolar. presentan un resultado negativo de la prueba diagnóstica.
D. Estimulación rectal. B. La proporción de casos de pacientes sin la enfermedad que
presentan un resultado positivo de la prueba diagnóstica.
E. Cirugía.
C. La proporción de pacientes que se someten a la prueba que
27.Un recién nacido, con peso al nacer de 4300 gramos, hijo de
tienen la enfermedad.
madre con diabetes gestacional, doble circular ajustada a cuello;
se muestra muy inquieto a los 30 minutos de vida; quince D. La proporción de casos de enfermos con resultado positivo de la
minutos más tarde, sufre una convulsión tónico clónica. El prueba diagnóstica.
diagnóstico más probable es :
E. La proporción de casos con resultado positivo de la prueba que
A. Hipoglucemia. son verdaderamente enfermos.
B. Hipocalcemia. 33.En una comunidad hay abundante rabia animal y se notifican
algunos casos humanos todos los años. Con el fin de fijar las
C. Hiponatremia. bases para iniciar un programa de control, entre las cuales se
D. Hiperviscosidad. señalan las siguientes medidas: ¿Cuál a su juicio, puede dar
resultados MÁS permanentes?:
E. Encefalopatía hipoxico isquémica
A. Campaña para exterminar perros vagabundos.
28.¿Cuál es el corticoide más utilizado en el tratamiento del
Crup?: B. Campaña masiva de educación sanitaria en la población.

A. Dexametasona
C. Campaña masiva para hacer tratamiento antirrábico a todas las B. Peritonitis biliar no traumática.
personas mordidas.
C. Paciente colecistectomizado hace dos meses.
D. Establecer un programa de vacunación canina (80% de los perros
estimados) en un año. D. Colecistitis crónica calculosa no complicada.

E. Sistema de inspección de carne de los mataderos. E. Paciente con cáncer vesicular no invasivo

34.Paciente de 38 años que, cuando se encuentra paseando por 40.En el caso de que el enfermo fuera intervenido
el campo e inmediatamente después de la picadura de una abeja, quirúrgicamente, los hallazgos quirúrgicos que encontraría más
presenta: urticaria, eritema, disnea y shock. ¿Cuál es el pro- bablemente serían:
tratamiento MÁS resolutivo para este paciente?:
A. Apendicitis perforada.
A. La administración de un corticoide inhalado.
B. Úlcera duodenal perforada encubierta con epiplón.
B. La administración endovenosa de teofilina.
C. Diverticulitis perforada.
C. La administración de un antihistamínico.
D. Isquemia mesentérica.
D. La administración de corticoides intramusculares.
E. Diverticulitis de Meckel.
E. La administración de adrenalina intramuscular
41. El procedimiento consistente en colocar un tubo en el
35.Los oligosacáridos unidos a proteínas o lípidos de la estómago que salga a la piel, con la intención de utilizarlo para
membrana plasmática se encuentran: alimentar a un paciente, se llama:

A. Atravesando la bicapa lipídica. A. Gastrotomía.

B. A ambos lados de la membrana. B. Gastrostomía.

C. En la cara citoplasmática de la membrana. C. Gastrectomía.

D. En la cara extracelular de la membrana. D. Tubulización gástrica.

E. No existen oligosácaridos unidos a la membrana. E. Gastroplastia.

36.Si la tasa de natalidad es de 10 por mil habitantes y la 42.En el hiperparatiroidismo se ve todo lo siguientes, EXCEPTO:
población de 100.000 habitantes, ¿Cuántos nacimientos se pro-
dujeron?: A. Ausencia de lámina dura.

A.10 B. Hiperreflexia.

B. 100 C. Incremento en la fosfatasa alcalina.

C. 1.000 D. disminución del fósforo sérico.

D. 5.000 E. Osteítis fibrosa quística.

E. 10.000 43.¿En cuál de las siguientes afecciones no está recomendada la


dieta hiposódica?:
37.Las complicaciones del infarto transmural incluyen a todo lo
siguiente, EXCEPTO: A. En la insuficiencia cardíaca.

A. Endocarditis marántica. B. En la nefritis intersticial.

B. Taponamiento cardiaco. C. En la insuficiencia renal crónica.

C. Arritmia. D. En el síndrome nefrótico.

D. Ruptura de músculo papilar. E. En la hipertensión.

44.Un paciente es portador de un nódulo tiroideo. El estudio


E. Ninguna de las anteriores.
citológico mediante punción con aguja fina revela la existen- cia
38.Un paciente acude al servicio de urgencia con una historia de de “cuerpos de Psamoma”. Este hallazgo es DEFINITORIO del
hematemesis. El pulso arterial es de 130x´. La PA es de diagnóstico de:
100/75mmHg. Frialdad y sudoración de manos y pies. ¿Qué tipo
de acción es prioritaria?: A. Linfoma tiroideo.

A. Realizar endoscopia. B. Carcinoma papilar tiroideo.

B. Colocar vías intravenosas y restablecer el volumen circulante. C. Tiroiditis de Riedel.

C. Determinar hemoglobina y valor hematocrito. D. Carcinoma medular tiroideo.

D. Practicar exploración radiológica con bario. E. Tiroiditis de Hashimoto.

E. Administrar somatostatina. 45.¿Cuál de los siguientes factores NO es causa de


macrocitosis?:
39.La aerobilia es un signo radiológico que puede observarse en
uno de los siguientes procesos patológicos: A. Hipotiroidismo.

A. Colecistitis aguda alitiásica. B. Alcoholismo.


C. Anemia perniciosa. 51.En la osteomielitis, el absceso subperióstico emigra a la
articulación vecina en los siguientes casos, EXCEPTO:
D. Hepatopatía crónica.
A. Cadera.
E. Intoxicación por aluminio.
B. Hombro.
46. Señale cuál de estas afirmaciones es la VERDADERA,
respecto al empleo de pruebas serológicas de la sífilis: C. Rodilla.

A. Para confirmar el diagnóstico de sífilis en un enfermo con prueba D. Codo.


de anticuerpo reagínico positiva o con sospecha clínica de sífilis, se
debe utilizar el VDRL. E. Tobillo.

B. Para medir cuantitativamente el título de anticuerpos reagínicos a 52.¿Cuál es el método diagnóstico de elección para una lesión de
fin de valorar la actividad clínica de la sífilis, se menisco?:

debe utilizar el RPR. A. Radiografía simple.

C. Para vigilar el título de reagina en respuesta al tratamiento de la B. Artrografía.


sífilis, se debe utilizar el FTA-Abs o el MHA-TP.
C. Ecografía.
D. Para analizar gran número de sueros con miras diagnósticas o de
detección selectiva, se debe utilizar el MHA-TP. D. TAC.

E. Si un enfermo tiene una prueba reagínica positiva falsa, no E. RNM.


podemos excluir la sífilis aunque obtengamos una prueba treponémica
53.El principal factor pronóstico de un melanoma maligno sin
negativa.
metástasis en tránsito, ganglionares ni hematógenas es:
47.Las cepas de Staphylococcus aureus resistentes a meticilina
A. La edad del paciente.
presentan las siguientes características EXCEPTO:
B. Su desarrollo sobre un nevus previo.
A. Son resistentes a todos los antibióticos betalactámicos.
C. Su localización en zonas acras.
B. La resistencia está mediada por las PBP (Proteínas Fijadoras de
Penicilina). D. El traumatismo previo de la lesión.
C. Las pruebas de sensibilidad convencionales pueden dar E. El espesor de la lesión medido en milímetros.
resultados equívocos para las cefalosporinas.
54.De acuerdo con las maniobras de Leopold, la relación que
D. Son sensibles a Vancomicina. existe entre el dorso fetal con los flancos maternos se deno-
mina:
E. El mecanismo de resistencia esta codificado por un plásmido.
A. Situación fetal.
48.La causa MÁS frecuente de síndrome nefrótico idiopático en
adultos es: B. Posición fetal.
A. Nefropatía membranosa. C. Actitud fetal.
B. Glomerulosclerosis focal. D. Presentación fetal.
C. Nefropatía mesangial IgA. E. Rotación interna.
D. Glomerulonefritis mesangiocapilar. 55.Señale la respuesta FALSA en relación con la endometriosis:
E. Síndrome nefrótico con cambios mínimos. A. Es una causa frecuente de esterilidad.
49.Los valores aumentados de la enzima Adenosindeaminasa B. Su origen está ligado al flujo menstrual retrógrado.
(ADA) en un líquido pleural sugieren diagnóstico de:
C. El órgano más frecuentemente afectado es la trompa.
A. Embolismo pulmonar.
D. En su patogenia cada día se da más importancia al factor
B. Empiema metaneumónico. inmunológico.
C. TBC pleural. E. La laparoscopia es la prueba más definitiva para el diagnóstico de
la endometriosis pélvica.
D. Hemotórax.
56.Una mujer postmenopáusica fue tratada de un carcinoma
E. Artritis reumatoide. mamario infiltrante de 1 cm mediante tumorectomía y lin-
50.Señale cuál es el tratamiento de elección en las distonías fadenectomía. Esta última fue negativa. ¿Qué tratamiento
focales cervicofaciales: añadiría:

A. Anticolinérgicos. A. Ovariectomía bilateral.

B. Benzodiacepinas. B. Monoquimioterapia con taxol.

C. Toxina botulínica. C. Poliquimioterapia.

D. Barbitúricos. D. Radioterapia.

E. Tenotomía. E. Radioterapia y tamoxifeno.


57.Una de las siguientes NO es complicación frecuente del E. Todas las anteriores
abruptio:
63.¿Cuál es la conducta más apropiada en una gestante de 38
A. CID. semanas con VIH confirmado recientemente, sin tratamien- to
antirretroviral y una carga viral de 2000 copias?
B. Insuficiencia renal.
A. Parto vaginal.
C. Shock.
B. Cesárea programada.
D. Síndrome de Sheehan.
C. Cesárea de urgencia.
E. Pielonefritis.
D. Parto instrumentado.
58.Un maestro de escuela de 50 años, fumador, le consulta por
disfonía y tos desde 3 semanas antes. No refiere síndrome E. Parto inducido.
constitucional. No presenta adenopatías. En la laringoscopia
indirecta se ve una lesión blanquecina e irregular en tercio 64.En el análisis situacional de salud de un establecimiento se
anterior de cuerda vocal izquierda. ¿En qué debe pensar encuentra que su comunidad de referencia carece de agua
preferentemente?: potable, el año anterior se presentaron 40 casos de cólera con 8
fallecidos, y el establecimiento no tiene protocolos de atención.
A. Laringitis aguda. Según el análisis FODA institucional, la situación mencionada
corresponde a:
B. Edema de Reinke.
A. Oportunidades y amenazas.
C. Carcinoma de laringe.
B. Oportunidades y fortalezas.
D. Divertículo hipofaríngeo.
C. Debilidades y fortalezas.
E. Nódulo glótico.
D. Amenazas y debilidades.
59.Todos los siguientes procesos se manifiestan frecuentemente
como colestasis neonatal intrahepática, EXCEPTO: E. Debilidades y oportunidades.

A. Hepatitis B. 65.Mujer de 50 años de edad, casada hace 5 meses presenta casi


todos los días: disminución del apetito y la concentración,
B. Sepsis. insomnio terminal, lentificación psíquica y motor, irritabilidad,
bajo estado de ánimo, ideas de muerte. El diagnóstico más
C. Hipotiroidismo. probable es:
D. Galactosemia. A. Melancolía.
E. Tirosinemia. B. Trastorno efectivo bipolar.
60.La cistoureterografía de micción está indicada, principalmente, C. Depresión mayor.
en el diagnóstico de:
D. Depresión con síntomas psicóticos.
A. Cáncer de vejiga urinaria.
E. Esquizofrenia.
B. Reflujo vésicoureteral.
66.Paciente de 45 años colecistectomizado hace un mes. Refiere
C. Obstrucción ureteral. continuar con la sintomatología previa a la operación. Dolor
cólico abdominal, vómitos e ictericia. ¿Cuál es el diagnóstico
D. Adenoma postático.
más probable?
E. Ureterocele.
A. Coledocolitiasis residual.
61.Indique la respuesta CORRECTA:
B. Hepatitis.
A. El 2º ruido cardíaco corresponde con el cierre de las válvulas
C. Estenosis biliar.
auriculoventriculares.
D. Ulcera péptica complicada.
B. Los soplos continuos pueden auscultarse tanto en estenosis como
insuficiencias valvulares severas. E. Pancreatitis aguda.
C. En el pulso venoso yugular la onda “a” se debe a la distensión 67.Varón de 52 años, obeso, diabético, con diagnóstico reciente
venosa producida por la contracción de la aurícula izquierda. de HTA. En relación a su terapéutica es cierto:
D. El pulso alternante es típico de la miocardiopatía hipertrófica. A. Debe iniciar manejo con Beta bloqueadores.
E. El signo de Kussmaul es un aumento de la presión venosa yugular B. Puede iniciar manejo con IECA por ser diabético.
durante la inspiración.
C. Como la gran mayoría de hipertensos podría requerir tres drogas.
62.Según el informe internacional Lalonde, los factores
determinantes de la salud son: D. Las terapia combinada IECA + tiazida ha demostrado ser superior
a IECA + Bloqueador de canal de calcio.
A. El control del medio ambiente.
E. Se sugiere iniciar metildopa por su mayor potencia.
B. Sistemas de asistencia sanitaria.
68.¿Cuál de las siguientes afirmaciones acerca de los criterios de
C. La biología humana. Ranson NO es correcta?:0
D. Solo A y C
A. Predice la severidad de la pancreatitis aguda. 74.Un paciente de 60 años acude por historia de 2 días de
melenas al hospital. Tiene un hematocrito en 25. Esta hemodi-
B. Pacientes con 2 signos positivos tienen una mortalidad elevada. namicamente estable. Se le realiza una endoscopia la cual es
normal. ¿Cuál es el siguiente paso?
C. Se basa en 11 criterios.
A. Una nueva endoscopía alta.
D. Los parámetros son evaluados al ingreso del paciente.
B. Una colonoscopía con ileoscopía.
E. Los parámetros son evaluados a las 48 horas del ingreso del
paciente. C. Gammagrafía con glóbulos rojos marcados.
69.Un niño capaz de correr, subir escaleras solo, sin alternar los D. Una arteriografía.
pies, hacer una torre de seis cubos y elaborar frases de tres
palabras, presenta un desarrollo psicomotor propio de la E. Cápsula endoscópica.
siguiente edad:
75.Los Documentos de Gestión Institucional requeridas a las
A. 15 meses. instituciones de salud son, excepto:

B. 18 meses. A. Reglamento de Organización y Funciones.

C. 24 meses. B. Guías de prácticas clínicas.

D. 36 meses. C. Manual de Organización y Funciones.

E. 48 meses. D. Manuales de Procedimientos.

70.Mujer de 45 años presenta dolor abdominal episódico y refiere, E. Texto Único de Procedimientos Administrativos.
que el dolor aumenta posteriormente a la ingesta de comidas
ricas en grasas ¿La acción de cuál de las siguientes hormonas es 76.¿Qué aspectos clínicos se evalúan principalmente para
responsable de la intensificación postpran- dial de sus síntomas? realizar el plan preoperatorio y el desarrollo de la aneste- sia?:

A. Gastrina. A. Edad del paciente, perfil hepático, sistema hematopoyético

B. Secretina. B. Sistema digestivo, edad del paciente, sistema nervioso central

C. Colecistocinina. C. Aparato urinario, estado dermatológico y respiratorio

D. Pepsina. D. Edad del paciente, estado cardiovascular y estado respiratorio

E. Somatostatina. E. Masa corporal, estudio inmunológico y cardiovascular

71.Paciente de 50 años de edad que consulta por dolor en la fosa 77.Respecto a los tratamientos habituales o hábitos de un
renal, poliaquiuria, disuria y hematuria. En el análisis de orina se paciente programado para una intervención quirúrgica bajo
observa piuria y pH ácido con cultivos repetidamente negativos anestesia general, una de las respuestas siguientes es
¿Cuál sería la primera posibilidad diag- nóstica, de entre las CORRECTA:
siguientes?
A. Los bloqueantes beta adrenérgicos deben mantenerse durante
A. Pielonefritis aguda. todo el periodo preoperatorio.

B. Síndrome nefrítico. B. Los antiparkinsonianos deben suspenderse 48 hrs antes.

C. Tuberculosis genitourinaria. C. La heparina de bajo peso molecular a dosis profilácticas se


asocia a hemorragia intraoperatoria significativa.
D. Prostatitis aguda.
D. La ticlopidina no se asocia a riesgo de hemorragia intraoperatoria
E. Carcinoma renal de células claras. significativa.

72.Los estudios transversales son llamados: E. La evitación de fumar cigarrillos durante las 24 hrs previas carece
de beneficio intraoperatorio.
A. Ecológicos.
78.La localización ideal para hacer una traqueotomía es:
B. Cohortes.
A. La membrana cricotiroidea
C. Ensayo de campo.
B. La membrana miotiroidea.
D. Estudios analíticos.
C. Primer anillo traqueal.
E. De prevalencia.
D. Segundo o tercer anillo traqueal.
73.Son signos de corioamninitis, EXCEPTO:
E. Cuarto o quinto anillo traqueal.
A. Taquicardia materna.
79.Una lesión purpúrica que no palidece a la vitropresión, nos
B. Útero doloroso. indica:
C. Líquido amniótico meconial. A. La presencia de edema dérmico.

D. Taquicardia fetal persistente. B. La presencia de exocitosis


E. Leucocitosis materna. C. La presencia de extravasación hemática.

D. La presencia de una malformación venosa.


E. La presencia de una malformación capilar 86.En los neonatos normales a término, la capacidad gástrica es:

80.¿Cuál de las siguientes afirmaciones referentes a la A. 5 a 9 mL


leucoplasia vellosa oral se asocia a la infección por VIH es INCO-
RRECTA? B. 10 a 15 mL

A. Suele ser asintomática C. 25 a 30 mL

B. Existen células balonizadas en la histopatología D. 20 a 45 mL

C. Existe un marcador clínico de progresión de la infección por el E. 50 a 90 mL


VIH
87.La arteria radial:
D. Puede encontrarse en otras situaciones de inmunodeficiencia
A. Es más grande que la arteria cubital
E. El primer implicado en su patogenia parece ser Candida
albicans B. Pasa a lo largo del lado cubital de antebrazo

81.¿Cuál de las siguientes entidades está asociada con herpes C. Pasa entre el tendón del musculo abductor largo del pulgar y el
virus tipo 6? ligamento radial colateral de la muñeca

A. Exantema súbito D. Comienza a lo largo de la extremidad de la diáfisis del humero

B. Enfermedad febril de la infancia E. Pasa sobre el musculo supinador largo

C. Síndrome de fatiga crónica 88.El musculo flexor común profundo esta inervado por los
nervio:
D. Síndrome de Gianotti- Crosti
A. Mediano e interóseo anterior
E. Todas las anteriores.
B. Mediano y ulnar
82.Micosis superficial que afecta el estrato córneo y se
manifiesta con lesiones discrómicas con descamación fina: C. Musculocutáneo y cubital

A. Tiña pedis. D. Cubital y radial

B. Tiña corporis E. Radial y mediano

C. Pitiriasis versicolor 89.En la pubertad, el cuerpo uterino crece bajo la influencia


directa de:
D. Exofialosis
A. Hormonas hipofisiarias
E. Tiña imbricada
B. Hormonas ováricas
83.¿Cuál es el andrógeno fundamental en la mujer?:
C. Hormonas tiroideas
A. Androstenediona.
D. Factores hipotalámicos
B. Dehidroepiandrosterona.
E. Hormonas adrenales
C. Androstenediol.
90.Un investigador pretende determinar si existe una asociación
D. Androsterona. entre las cifras de PA diastólica (medida en mmHg) y los niveles
de colesterol (medidos en mg/mL). Para ello, ha realizado estas
E. Testosterona. mediciones a 230 voluntarios. ¿Qué prueba estadística es la MÁS
apropiada pare examinar esta asociación?:
84.En un ciclo menstrual de 30 días, con una gráfica de
temperatura corporal basal considerada ovulatoria; la duración A. Regresión logística.
de la fase luteínica o secretoria será de:
B. Prueba de la t de Student.
A. 21 ± 2 días
C. Prueba de Chi cuadrado.
B. 07 ± 3 días
D. Correlación de Pearson.
C. 18 ± 1 días
E. Prueba de Fisher.
D. 14 ± 2 días
91.Un estudio analiza la relación entre la PA sistólica (PAS) y la
E. 16 ± 2 días edad en una muestra de mujeres adultas. Los autores presentan
los resultados como la siguiente ecuación de regresión lineal:
85.Los siguientes factores pueden influir en la reserva de hierro PAS = 81,5 + 1,2 x edad. ¿Cuál de las siguientes afirmaciones
al nacer, EXCEPTO: acerca de este análisis es FALSA?
A. Transfusión feto-materna A. La edad se ha utilizado como variable independiente
B. Transfusión de gemelo a gemelo B. La pendiente de la recta es de 1,2 mmHg/año de edad

C. Rotura de vasos placentarios C. El valor 81,5 corresponde a la media de PAS en la muestra de


mujeres
D. Anemia de la madre
D. Por cada año más de edad de las mujeres de la muestra, su PAS
E. Todas las anteriores se incrementa en 1,2 mmHg de promedio
E. Se ha asumido que la relación entre la edad y la PAS es A. Sodio
lineal
B. Hierro
92.El test de Capurro para la edad gestacional no incluye:
C. Potasio
A. Forma de la oreja
D. Cloro
B. Pliegues plantares
E. Fósforo
C. Textura de la piel
99.De la inmunización activa del sarampión que se utiliza en la
D. Nódulo mamario actualidad, ¿Cuál es la más eficaz?

E. Ventana cuadrada A. Gammaglobulina


B. Virus muerto
93.Los hongos dermatofitos que causan cuadros clínicos con C. Virus vivo con gammaglobulina
mayor componente inflamatorio son: D. Virus vivo atenuado
E. Ninguna de las anteriores.
A. Antropofílicos
100.Niño de 6 años de edad con riesgo de tétanos. Si sufre una
B. Geofílicos
herida contaminada la conducta correcta es, asumiendo que tiene
C. Candidiasis completas las inmunizaciones y refuerzo de vacuna antitetánica:

D. Pytrosporum A. Vacuna antitetánica


B. Antitoxina tetánica
E. Zoofílicos C. Antibiótico tipo penicilina
D. Antitoxina + vacuna antitetánica
94.¿En cuál de los siguientes casos suele indicarse la prueba de E. No se aplica ni vacuna ni inmunoglobulina
estrógenos para evaluar el origen de una ameno- rrea?:

A. Cuando la prueba de la progesterona es positiva.

B. Cuando la prueba de la progesterona es negativa.

C. Cuando se detectan niveles elevados de gonadotropinas.

D. Cuando se detectan niveles disminuidos de gonadotropinas.

E. En todos los casos de amenorrea primaria.

95.¿Cuál de los siguientes tendones se inserta en el astrágalo?:

A. Aquiles

B. peroneo lateral corto

C. peroneo lateral largo

D. tibial anterior

E. El astrágalo carece de inserción tendinosa

96.Respecto al desarrollo psicomotor normal del niño, señale la


afirmación correcta:

A. Presiona pinzando el pulgar con el índice a los 12 meses

B. Junta las manos en la línea media a los 6 meses

C. Vuelve las páginas de un libro a los 8 meses

D. Usa 4-6 palabras a los 9 meses

E. Se sienta estable y sin apoyo a los 6 meses

97.La edad recomendada para la introducción de la ablactancia


es:

A. Entre los 15 días y los 2 meses

B. Entre los 2.3 meses

C. A los 4 meses

D. A los 6 meses

E. A los 8 meses

98.La leche materna contiene menos..... lo cual es para evitar la


sobrecarga osmótica renal en el lactante.
EXAMEN N°08 D. Paciente con HTA de raza afroamericana indicamos
beta block

E. En gestantes con HTA severa con labetalol


SIMULACRO 11A
6. El agente Gram negativo más frecuente de la neumonía
1. Cuadro clínico con foco neumónico en radiografía se ve asociada a Ventilación es:
lesión en base de pulmón derecho que borra aurícula derecha
¿qué lóbulo está afectado? A. Escherichia coli

A. Inferior B. Enterococco faecalis

B. Apical C. Pseudomona aeruginosa

C. Medio D. Staphylococcus aureus

D. Lingula E. Haemophilus influenzae

E. Superior 7.¿Cuál es la tasa de filtración glomerular compatible con


paciente diabético en estadio temprano?
2. Gestante de 40 semanas que acude en periodo expulsivo
del trabajo de parto. ¿Cuál de los movimientos principales del A. 90 ml/min
trabajo es el más precoz?
B. 60 ml/min
A. Rotación interna
C. 150 ml/min
B. Extensión
D. 30 ml/min
C. Flexion
E. 15 ml/min
D. Rotación externa
8. Mujer de 20 años de edad, peladora de espárragos desde
E. Expulsión. hace 4 meses, presenta lesiones eritematosas pruriginosas
micropapulares localizada en dorso de antebrazos, cara y
3. ¿Cuál es el volumen de rehidratación que debería recibir un cuello. Las lesiones también se han identificado en algunas
lactante en quien se clasifica como deshidratación severa sin compañeras de trabajo. No presenta antecedentes de atopía.
shock? Su primera impresión diagnóstica es y el tratamiento es:
A. 200 ml/Kg/día A. Acarosis / permetrina
B. 100 ml/Kg/día B. Dermatitis por contacto / corticoides
C. 100 ml/SC/día C. Acarosis / griseofulvina
D. 3500 ml/Kg/día D. Dermatofitosis / clotrimazol
E. 150 ml/Kg/día E. Dermatofitosis / itraconazol
4. Respecto a la enfermedad diverticular, señale lo correcto: 9. ¿ Cuál es el antibiótico de elección para la diarrea
producida por campylobacter jejuni
A. La diverticulosis siempre requiere tratamiento
A. Azitromicina
B. La hemorragia diverticular suele ceder
espontáneamente B. Dicloxacilina
C. Los divertículos son más frecuentes en el color C. Clindamicina
ascendente
D. Cotrimoxazol
D. En el manejo de la diverticulitis siempre se opta por la
colostomía E. Ciprofloxacino

E. Es más frecuente entre los 30 y 50 años 10. Son criterios mínimos para el diagnóstico de la
enfermedad inflamatoria pélvica:
5. Sobre tratamiento de HTA señale lo falso
A. Dolor abdominal inferior, VSG elevada examen del
A. Paciente con HTA con microalbuminuria es de flujo vaginal positivo para N. Gonorrhoeae
elección captopril
B. Dolor a la movilización cervical, absceso tuboovarico
B. Paciente con HTA e IRC terminal con K < 5.5 ́ , examen del flujo vaginal positivo para C.
en ecografia
indicamos calcioantagonistas
Trachomatis
C. Paciente con HTA y antecedente de osteoporosis
indicamos tiazidas C. Dolor abdominal inferior, dolor a la movilización
cervical, dolor anexial bilateral.
D. Dolor anexial bilateral, temperatura oral > 38 0C, flujo C. La tipo I se ubica en curvatura mayor
vaginal o cervical anormal.
D. La tipo IV cursa con hipersecreción de acido
E. Dolor hipogástrico persistente, protein
́ a C reactiva
elevada, flujo vaginal anormal. E. La tipo V se asocia a lesión por AINES

11. Primigesta en trabajo de parto, 6 cm de dilatación con 16. En relación al diagnóstico de cardiopatía isquémica señale
membranas íntegras prominentes, C–4. Se tacta cordón; con- lo falso
jugado diagonal 10,5 cm; ponderado fetal 3.500 g. ¿Cuál es su
A. En caso de duda diagnostica en la angina estable
diagnóstico?:
solicitamos ergometria

B. El estudio de mayor dato pronóstico es la angiografía


A. Desproporción céfalo-pélvica por macrosomía fetal. coronario

B. Desproporción céfalo-pélvica y prolapso de cordón. C. La forma aguda produce un trombo que causa
oclusión coronaria
C. Estrechez pelviana y deflexión II.
D. El cuadro crónico se produce una estenosis fija que
D. Estrechez pelviana y procúbito de cordón. va progresando en años

E. Estrechez pelviana y prolapso de cordon E. la oclusión total se manifiesta eléctricamente sin


elevación del segmento ST
12. Acude a consulta un paciente de 3 años de edad. Mama
refiere que presento un cuadro de rinorrea y estornudos leves. 17. Respecto al cáncer de colon, es incorrecto:
Luego empezó a presentar lesiones tipo maculopapulares que
se extendieron en todo su cuerpo al primer dia de exan- tema A. La presentación clínica del cáncer de hemicolon
y que luego desaparecieron con descamación furfurácea al derecho es fundamentalmente obstructiva
tercer día de enfermedad. ¿Cuál es el diagnostico?
B. Los adenomas colorrectales se consideran lesiones
A. Exantema súbito premalignas

B. Roseola C. El tipo más frecuente es el adenocarcinoma

C. Sarampion D. Es más frecuente en mayores de 50 años

D. Rubeola E. El consumo de grasas saturadas aumenta el riesgo

E. Escarlatina. 18. Paciente adulto con diagnóstico de neumonía por


estafilococo. Recibe tratamiento durante varios días con
13. Paciente joven leptosomico con colelitiasis, además se Vancomici- na, pero no mejora. ¿Qué antibiótico recomendaría
encuentra el examen físico esplenomegalia, además presente en este paciente?
astenia y en su perfil hepático destaca la elevación de
bilirrubina a predominio indirecto., ante la sospecha A. Ceftriaxona
diagnostica solicitamos
B. Piperacilina tazobactam
A. Test de HAM
C. Linezolid
B. Electroforesis de hemoglobina
D. Meropenem
C. PCR enzimático
E. Clindamicina
D. Test de fragilidad osmótica
19. ¿Cuál de las siguientes es un tipo de exantema conocido
E. Gota gruesa como la quinta enfermedad?

14. En el fenómeno de Raynaud, el paciente puede recibir A. Exantema súbito


como terapia farmacológica:
B. Eritema infeccioso
A. Prednisona
C. Varicela
B. Isorbide
D. Sarampion
C. Minoxidil
E. Rubeola.
D. Propanolol
20. Mujer de 25 años, puérpera de 1 horas de parto vaginal,
E. Amlodipino RN de 4 500 gr, presenta abundante sangrado rojo rutilan- te,
se encuentra pálida, taquicárdica, sudorosa, útero a 3 cm
15. Según la clasificación de Johnson de ulcera gástrica debajo de la cicatriz umbilical. ¿Cuál es el diagnóstico
señale lo verdadero probable?

A. La tipo II es la más frecuente A. Atonía uterina

B. La tipo III cursa con secreción de ácido disminuido B. Retención de restos placentarios
C. Hipotonía uterina A. Atraviesa la duramadre

D. Embolia de líquido amniótico B. Siempre ocasiona cefalea post punción

E. Laceración del tracto genital C. Es necesario siempre atravesar el ligamento amarillo

21. Es incorrecto respecto a las quemaduras de tercer grado: D. Es un tipo de anestesia general

A. La superficie es indolora E. Puede causar hipertensión arterial

B. Destruye toda la dermis 27. Paciente varón joven con datos de trombosis venosa
profunda, además se encuentra ictericia a predominio indirecto
C. Tiene un color oscuro con coluria, la sospecha diagnostica es:
D. La superficie es generalmente seca A. Hemoglobinuria paroxística nocturna
E. Presenta ampollas en la superficie B. Esferocitosis
22.¿Cuál de los siguientes enteroparásitos NO transmite por C. Talasemias
fecalismo?:
D. Hemoglobinopatías
A. Lumbricoides
E. Eliptocitosis
B. Cryptosporidium parvum
28.Mujer de 32 años de edad, con diagnóstico de artritis
C. Entamoeba histolytica reumatoide. ¿Cuál de los siguientes hallazgos indica un buen
pronóstico?:
D. Strongyloides stercolaris
A. Elevación de la velocidad de sedimentación
E. Trichuris trichiuria
B. Títulos bajos de factor reumatoide
23. Mujer de 64 años, G:1, P:1001, menarquia: 10 años. FUR:
55 años, obesa, diabética e hipertensa desde hace 10 años. C. Títulos elevados de haptoglobina
Acude por ginecorragia. Ecografía: endometrio 20 mm y
volumen uterino 110 cc. PAP: AGUS ¿cuál es el diagnóstico D. Presencia de nódulos subcutáneos
más probable?
E. Presencia erosiones en radiografía de manos
A. Adenocarcinoma endometroide
29.Paciente de 80años de edad, que presenta en ambas
B. Carcinoma de células claras extremidades ampollas grandes, tensas, algunas con
contenido sanguíneo, sobre una base urticariana y con
C. Carcinoma indiferenciado moderado prurito. No hay compromiso del estado general y la
histopato- logía muestra lesiones subdermicas con acumulo
D. Carcinoma mucinoso
de eosinofilos en la dermis papilar. La inmunofluorescencia
E. Carcinoma seroso papilar muestra depósito lineal de IgG y de C3. ¿Cuál es la
dermatopatia más probable?
24. No es un criterio de causalidad de Hill:
A. Dermatitis herpetiforme
A. Fuerza de asociación
B. Penfigo cicatricial
B. Número necesario a tratar
C. Penfigo seborreico
C. Experimentación
D. Penfigo vulgar
D. Consistencia
E. Penfigoide ampollar
E. Gradiente biológica
30.Paciente varón de 3 años de edad, quien desde hace
25. ¿Cuál de las siguientes es INCORRECTA respecto a la aproximadamente 4 días presenta deposiciones liquidas en
leche y lactancia materna? fre- cuencia de 5 por día sin sangre sin moco. Al examen físico
presenta deshidratación de grado moderado. ¿Cuál es el
A. La galactosemia es una contraindicación
rango de volumen que debería recibir dicho paciente?
B. El calostro tiene más contenido de grasas en
A. 20-80 ml/Kg/día
comparación a la leche madura
B. 30-80 ml/Kg/día
C. La leche madura se obtiene a partir del dia 14
C. 30-80 ml/Kg/4horas
D. El contenido de Calcio es menor en la leche humana
en comparación a la leche de vaca D. 80-100 ml/Kg/3horas
E. La hierro de la leche materna es mejor absorbido que E. 30-50 ml/Kg/12horas.
el de la leche de vaca.
31.Mencione el dato de mal pronóstico en pancreatitis aguda
26. Es característica de la anestesia epidural:
A. El índice de severidad tomografía más de 5 puntos B. Dolor pélvico

B. Secuestro de líquidos al ingreso C. Secreción vaginal maloliente

C. Hipocalcemia a las 48 horas D. Sangrado vaginal

D. Apache más de 7 puntos E. Fiebre a partir del segundo día puerperio

E. Disminución de PCR 37. Paciente de 40 años de edad, que acude a la consulta por
sangrado postcoital de 2 semanas de evolución al examen
32. El precursor metabólico inmediato de la noradrenalina es: pélvico:cérvix ulcerado hipertróficos, sangrante, poco móvil.
Cuerpo uterino 6cm, anexos no palpables. Al tacto rec- tal:
A. Isoproterenol.
útero duro con parametrios tomados en sus 2/3 internos. El
B. Ibopamina. diagnóstico clínico es:

C. Dobutamina. A. Cáncer de cérvix IB

D. Dopamina. B. Cáncer de cérvix IIIA

E. Efedrina. C. Cáncer in situ

33. Paciente de 68 años con disnea progresiva hasta en D. Cáncer de cérvix IIB
reposo. Examen físico: tórax amplio vibraciones vocales dismi-
E. Ectropión sangrante
nuidas, hipersonoridad, murmullo vesicular disminuido,
ingurgitación yugular y disnea severa. Cuál es la actitud 38. Varón de 17 años, con procesos respiratorio viral,
inmediata? posteriormente presenta hematuria con hematíes dismorficos
ade- más presenta disminución de complemento (C3 Y c4)
¿Cuál es su posibilidad diagnostica?
A. Tomar radiografía de tórax
A. Glomerulonefritis por vasculitis
B. Evaluación por cirujano de tórax
B. Glomerulonefritis postinfecciosa
C. Drenaje torácico inmediato
C. Glomerulonefritis membranoproliferativa
D. Tomografía de pulmones
D. Glomerulonefritis rapidament progresiva
E. Antibióticos y corticoides
E. ́ por IgA .
Nefropatia
34.Marque lo falso respecto a los niveles de prevención:
39.Señora que sufre caída y acude a consulta con miembro
A. La prevención primordial evita la causación primaria inferior en extensión y aducción, a la radiografía no se eviden-
de la enfermedad. cia fractura ¿Cuál es el diagnóstico más probable?

B. La prevención primaria busca reducir la incidencia de A. Luxación pubiana de la cadera


enfermedad a través de pruebas de screening.
B. Luxación posterior de la cadera
C. La prevención secundaria se sustenta en la utilidad
C. Luxación congénita de cadera
de las pruebas de screening.
D. Luxación obturatriz de cadera
D. La prevención terciaria busca recuperar parte de la
funcionabilidad previa. E. Luxofractura de cadera
E. La prevención cuaternaria evita el sobrediagnostico y 40.En este estudio de evalúan los efectos adversos a largo
sobretratamiento. plazo y las interacciones medicamentosas en un gran número
de pacientes / es un estudio previa a la comercialización del
35.Con respecto a la convulsión febril, marque la alternativa
fármaco pero que ya recibió la aprobación de la FDA para
incorrecta:
realizar ensayos clínicos controlados:
A. La convulsión febril más frecuente es la simple
A. Fase III / II
B. Es el tipo de convulsión febril más frecuente en niños
B. Fase V / II
con una frecuencia de 3-5% en menores de 5 años
C. Fase IV / III
C. El que los padres hayan tenido en la niñez no apoya
el riesgo de que el niño lo tenga D. Fase IV / II
D. Generalmente se resuelven solas E. Fase I / IV
E. Existe manejo preventivo de convulsión febril 41.Llega a consulta una madre con su hijo de 18 meses de
edad quien refiere que solo recibió la primera dosis de la Pen-
36.¿Cuál es un principal criterio para endometritis?
tavalente. Con respecto a esta vacuna, ¿Qué le diría usted?
A. Fiebre en el primer día pureperio
A. Ya no puede vacunarse por el riesgo de ESAVI
B. Debería completar con solo una segunda dosis C. Limpia

C. Completar con dos dosis más separadas por 1 mes D. Contaminada

D. Debería reiniciar las tres dosis y darlos cada 2 meses E. Infectada

E. Completar con dos dosis más separadas por 2 47.Mujer de 28 años, hace 6 meses presenta malestar
meses. general, Poliartritis de mano y disnea. Al examen físico: PA:
125/80 mmHg, FC: 100 x minuto, FR: 28 x minuto. Alopecia,
42. En un paciente con glucosa en ayunas de 130 mg/dL y dos palidez y adenopatías cervicales múltiples. Hemoglobina 7G/
exámenes adicionales dudosos, con un test de tolerancia a la dl, urea 65 mg/dl, creatinina 2.2 mg/dl. Examen de orina:
glucosa en limite, debemos solicitar: hematuria, cilindros hemáticos y proteínas (+), ANA y anti-
Smith (+). Rx de pulmones: lesión homogénea en el tercio
A. Curva de insulina
inferior del hemitórax izquierda: ¿Con respecto al diagnós- tico
B. Glucosa post prandial señale lo correcto?:

C. Repetir el test de tolerancia a la glucosa 75 A. Mediado por una reacción de hipersensibilidad tipo II

D. Hemoglobina A1c B. El rash malar es una manifestación aguda especif́ ica

E. Electroforesis de hemoglobina. C. Asocia frecuentemente con síndrome de Sjogren

43. Respecto a los aneurismas cerebrales, es INCORRECTO: D. Mejora con la gestación

A. La mayoría corresponden al tipo sacular E. Cuando presenta anticuerpo anticardiolipina es buen


pronóstico
B. Es la principal causa de hemorragia subaracnoidea
atraumática 48. Mujer de 25 años de edad, con gestación de 37
semanas. Acude a Emergencia por presentar dolor abdominal
C. La principal arteria donde se les puede encontrar es soste- nido, de inico brusco y de intensidad fuerte, con leve
la arteria meníngea anterior sangrado vaginal. Al examen físico: PA 130/90 mmHg, LCF
160 por minuto. ¿Cuál es el diagnóstico más probable?
D. La mayoría son de tipo congénito
A. Placenta marginal
E. La diplopía y la cefalea son síntomas característicos
B. Ruptura uterina espontánea

C. Ruptura del seno marginal


44. Una madre con tipo de sangre AB tiene un hijo con
tipo AB. A ella le gustaría establecer la paternidad a través del D. Desprendimiento prematuro de placenta
tipo sanguíneo. ¿Cuál tipo de sangre excluye a un varón de
ser el padre biológico? E. Parto pretérmino

A. AA. 49.El tipo de sangrado más característico que producen los


leimiomas uterinos:
B. BB.
A. Hipermenorrea
C. BO.
B. Metrorragia
D. AO.
C. Menorragia
E. OO.
D. Menometrorragia
45.El tipo de medicamento indicado para el tratamiento de la
Angina de Prizmetal es: E. Amenorrea

A. Calcioantagonistas 50. ¿Cuál de las siguientes vacunas es de tipo inmunización


activa tipo toxoide?
B. IECA
A. Disteferia y Tetanos
C. B- Bloqueantes
B. Sarampion, Paperas y Rubeola
D. ARA II
C. Pentavalente
E. Digoxina
D. Antineumococica
46. Paciente mujer quien se realiza una colecistectomía
laparoscópica electiva por antecedente de litiasis vesicular, E. BCG.
acci- dentalmente se lacera el duodeno, sin perforarlo. El tipo
51.Respecto a las escalas de medicina marque lo falso:
de herida es:
A. Las variables en la escala nominal son excluyentes.
A. Sucia
B. La escala ordinal presenta una diferencia de grado
B. Limpia contaminada
entre variables.
C. La escala de intervalo muestra diferencias 56. Gestante de 38 años, con diagnóstico de placenta
cuantificables de los valores pero no establece ausencia de previa. ¿Cuál es el factor de riesgo más asociado y a partir de
fenómeno. qué edad gestacional se diagnostica dicha patología?

D. La escala de razón muestra diferencias cuantificables A. Cesárea – 28 semanas


de los valores y establece ausencia de fenómeno.
B. Parto distocico – 26 semanas
E. La escala de intervalo muestra diferencias
cuantificables de los valores y establece ausencia de C. Cesárea – 30 semanas
fenómeno.
D. Parto distocico – 28 semanas
52. En un puesto de salud se han producido 2.800
E. Cesárea – 22 semanas
consultas médicas al año y se dispuso para el mismo 400
horas-médico. ¿Cuál es el rendimiento? 57. Un neonato presenta ictericia, dificultad respiratoria,
Tº inestable, distensión abdominal y hepatomegalia. Diagnósti-
A. 9
co de presunción:
B. 5
A. Kernicterus.
C. 6
B. Meningitis neonatal.
D. 8
C. Enfermedad granulomatosa crónica.
E. 7
D. Lupus neonatorum.
53.¿Cuál es la característica que corresponde al sarcoma de
E. Sepsis neonatal.
Kaposi?
58. ¿Cómo se define talla baja en pediatría?
A. Es una neoplasia maligna de endotelio causada por
HVS8 A. P/T<3p
B. Las lesiones son dolorosas y pruriginosas causadas B. T/E<5p
por HVS8
C. P/T<5p
C. Es un cáncer de endotelio por HVS1
D. T/E<3p
D. Es típico de VIH causado por HVS5
E. P/E<10p
E. La etología es el VIH que causa cáncer de endotelio
59. Respecto a los tumores de esófago, marque lo
INCORRECTO

A. Los tumores benignos más frecuentes son los


leiomiomas
54. ¿Cuál es la alternativa que NO corresponde a la
anemia ferropenia?. B. La acalasia crónica aumenta el riesgo
A. La respuesta al tratamiento se mide con reticulocitos C. El consumo de alcohol y tabaco aumenta el riesgo
al 10 día
D. El tumor maligno más frecuente es el
B. La causa más frecuente es carencial en jóvenes adenocarcinoma
C. Para diferenciarla de la anemia por enfermedad E. El cáncer más frecuente es el carcinoma
crónica se mide transferrina epidermoide.
D. ́ ico mas especifico es la ferritina
El test bioquim 60. Paciente con ECV con hemiparesia derecha a
predominio braquial y afasia de expresión ¿qué arteria está
E. El hierro se absorbe solo el 10% de lo ingerido a nivel
afectada?
intestinal.
A. Cerebral media rama superior izquierda
55. En el tratamiento de la hiperkalemia ¿Qué
medicamento actúa produciendo disminución del potasio B. Cerebral media rama superior derecha
corporal total?
C. Cerebral anterior rama superior izquierda
A. Gluconato de calcio
D. Cerebral anterior rama superior derecha
B. bicarbonato
E. Cerebral media rama inferior izquierda
C. Dextrosa hipertónica
61. Mide la desviación estándar de una media de todas
D. Beta bloqueadores las muestras posibles de una población infinita y se usa para
la inferencia por:
E. Resina Kayexalate
A. Error estándar de la media – intervalos de confianza
B. Desviación medial de la media – intervalos de C. Doxepina / bupropion
confianza
D. Fluoxetina / paroxetina
C. Error estándar de la media – prueba de hipotesis
E. Trazodona / sertralina
D. Amplitud intercalar - prueba de hipotesis
67. ¿Cuál es la frecuencia cardiaca fetal normal?
E. Correlacion lineal – intervalos de confianza
A. 60 B. 120
62. El impétigo ampolloso es causado por y debe ser
tratado con: C. 120 a 160

A. Staphylococcus aureus coagulasa positivo / D. 80 a 120


mupirocina
E. >180
B. Streptococcus pneumoniae / dicloxacilina VO
68. Varón de 40 años que presenta en el brazo derecho
C. St. Pyogenes / clindamicina VO lesión máculo-papulosa de 3 cm, pruriginosa, indolora que
progre- sa a vesícula y luego a costra negruzca rodeada de
D. Staphylococcus aureus coagulasa positivo / edema. Ocupación crianza de ganado lanar y vacuno. Niega
dicloxacilina VO fiebre. Examen: Pulso: 115 x minuto, FR: 24 x minuto, PA:
100/60 mmHg. ¿Cuál es la presunción diagnóstica y el manejo
E. Pseudomonas aeruginosa / aztreonam VO adecuado?
63. Paciente de 30 años de edad, con hemorragia uterina A. Carbunco / doxiciclina
normal y biopsia endometrial informada como hiperplasia sim-
ple típica. ¿Cuál es la conducta más apropiada a seguir?: B. Leishmaniasis / nifurtimox

A. Tratamiento cíclico con progesterona C. Lactrodactismo / suero especifico

B. Histeroscopia D. Leishmaniasis / antimonio pentavalente

C. Histerectomía abdominal total E. Carbunco / ciprofloxacino

D. Esperar un nuevo control 69. La insuficiencia adrenal primaria se denomina y es


causada:
E. Ablación endometrial
A. Síndrome de Cushing / falla de regulación central
64. En un escolar de 7 años con claudicación de la
marcha y dolor en la cadera, usted sospecharía en la B. Enfermedad de Addison / falla de la glándula
enfermedad de:
C. Síndrome de Addison / falla de glándula
A. Sever
D. Enfermedad de Addison / falla de regulación central
B. Osgood-Schlatter
E. Síndrome de Addison / falla de regulación central.
C. Khole
70. El test de Apgar valora el estado vital del neonato
D. Legg-Calve-Perther mediante la comprobación de los siguientes signos,
EXCEPTO:
E. Panner
A. Frecuencia cardiaca.
65. La elaboración del Plan Operativo de un
establecimiento de Salud tiene como una de sus B. Movimientos respiratorios.
características que:
C. Respuesta a estímulos.
A. La misión expresa lo que seremos en el futuro
D. Reactividad osteotendinosa.
B. Promueve la participación del equipo de gestión
solamente E. Coloración de la piel.

C. Se programan actividades a lo largo de un año 71. Respecto a las hernias inguinales marque lo
incorrecto
D. Las fortalezas parten del entorno
A. Las hernias inguinales directas salen mediales a los
E. Busca plantear problemas de la oferta para vasos epigástricos inferiores
resolverlos operativamente
B. Para el tratamiento de las hernias inguinales directas
66. El antidepresivo con menor efecto sedante y el que se prefieren las técnicas sin tensión
tiene mayor efecto sedante son respectivamente:
C. Las hernias indirectas ocurren por persistencia del
A. Amitriptilina / sertralina conducto peritoneo vaginal

B. Clomipramina / imipramina D. Las hernias directas son generalmente congénitas


E. Las hernias indirectas recorren el canal inguinal B. Ecografía previa en el primer trimestre: DBP

72. Qué diámetro es el de mayor medida en el feto: C. Fecha de inicio de náuseas y vómitos

A. Suboccipitobregmatico D. Ecografía previa en el primer trimestre: LCC

B. Suboccipitomentoniano E. FUR confiable

C. Occipitofrontal 78. Paciente con pancreatitis aguda severa se colicita


TAC con contraste donde se informa edema de páncreas con
D. Bitrocantereo una colección de líquido por lo cual según el índice de
Baltazar corresponde a:
E. Bi acromial
A. A
73. Agente etiológico del Chancroide y de la cervicitis
respectivamente: B. B
A. Haemophilus vaginalis / clamidia trachomatis C. C
B. Calymmabacterium Granulomatosis / gonococo D. D
C. Treponema pallidum / H. ducreyi E. E
D. Chlamydia trachomatis / H. ducreyi 79. ¿Clínica más frecuente de esclerodermia?
E. Hemophilus ducrey / C. trachomatis A. Fenómeno de Raynaud
74. El síndrome de Cushing se refiere a: B. Atrofia de piel
A. Excesiva producción adrenal de glucocorticoides C. Cianosis
B. Al exceso de glucocorticoides producidos D. Diarrea
C. Excesiva exposición multiorganica a glucocorticoides E. Microstomia
D. Excesiva estimulación de las adrenales por la ACTH 80. Paciente varón de 75 años, con dolor abdominal
intenso desde hace 8 horas, distensión abdominal marcada y
E. Excesiva actividad glucocorticoide
vómitos biliosos. En la radiografía se evidencia distensión de
75. El motivo por el cual la sulfonilureas deben ser asa intestinal. El diagnóstico más probable es:
usadas con cuidado en ancianos y enfermos renales es:
A. Hernia inguinal incarcerada

B. Vólvulo de ciego
A. Pueden causar acidosis láctica
C. Diverticulitis aguda
B. Hipoglicemia
D. Vólvulo de sigmoides
C. Hiperkalemia
E. Malrotación intestina
D. Agitación
81. El modelo de atención de salud tiene como principio
E. Daño hepático fundamental y la estrategia principal es la :

76. Se recibe en Emergencia a un niño de 5 años de A. Calidad / APS


edad, febril, con 15 minutos continuos de crisis convulsiva, no
B. Integridad / APS
ha recu- perado la conciencia aun y continua convulsionando,
la madre del niño describe cuadro compatible con C. Eficacia / MAIS
convulsiones tónico-clónicas generalizadas. ¿Cuál es el
diagnóstico más probable y el manejo inicial más adecuado? D. Universalidad / MAIS

A. Estado convulsivo / Diazepam o lorazepam E. Solidaridad / APS

B. Epilepsia / Diazepam o lorazepam 82. De las siguientes alternativas, la conducta de


tratamiento más adecuado para el trauma de uretra posterior
C. Convulsión febril / fenitoina es:
D. Estado convulsivo / fenitoina A. Colocación de sonda Foley uretral
E. Convulsión tónico –clónica / fenobarbital B. Talla vesical
77. ¿Cuál es el parámetro más exacto para determinar la C. Reparación primaria de la uretra
edad gestacional?
D. Ureterostomía
A. Ecografía previa en el primer trimestre : LF
E. Nefrostomía
83. Se desea estudiar un factor de riesgo asociado a la C. Ibuprofeno
prostatitis, se nota que la población presenta varios factores
con- fusores pero no pueden ser considerados dentro de los D. Observación
criterios de exclusión y deben muestrear a todos. Para obtener
E. Azitromicina.
una muestra representativa se debe optar por el muestreo:
88. Gestante de 38 semanas con PA: 160/110 y
A. Estratificado
Albuminuria ( ++++) ¿Cuál es la alternativa terapéutica más
B. Aleatorio simple con restitución apropiada?

C. Aleatorio simple sin restitución A. Sulfato de Magnesio 6gr Ev en bolo y 2gr cada hora

D. Sistemático B. Betametasona 12 mg EV c/12 horas

E. Por conglomerado C. Misoprostol 200 ug via vaginal cada 6 horas

84. Paciente usuaria de DIU, asintomática, con ecografía D. Sulfato de Magnesio 2gr IM en cada glúteo
que reporta gestación intrautero de 10 semanas, Cola de DIU
E. Sulfato de Magnesio 1gr Ev en bolo y 1gr cada hora
visible al examen con especulo. La conducta a seguir es:
89. Anciana llega a la consulta con dolor postraumático
A. Control del embarazo más antibiótico
de hombro derecho, con limitación de movimiento. Examen
B. Control del embarazo más progesterona clíni- co: hematoma en cara interna del tercio superior del
brazo. ¿Cuál es el diagnóstico más probable?
C. Retiro de DIU más antibióticos
A. Luxación escapulohumeral
D. Retiro de DIU más progesterona
B. Fractura de clavícula
E. Retiro de DIU y observación
C. Luxación acromioclavicular
85. ¿A qué edad aproximadamente empieza a salir la
primera dentición? D. Fractura de escápula

A. 4 meses E. Fractura del cuello del húmero

B. 5 meses 90. ¿Qué se desarrolla en la administración de un


establecimiento nivel I, al relacionar las actividades de sus
C. 6 meses diversas unidades para alcanzar las metas propuestas?

D. 7 meses A. Negociación

E. 8 meses B. Organización

86. Un prematuro de 32 semanas de edad; a la primera C. Coordinación


hora de vida presenta una disnea progresiva con cianosis y
tiraje. En la radiografía hay un patrón de vidrio esmerilado y D. Motivación
broncograma aéreo. A pesar de la ventilación, el oxígeno y los
E. Planificación
antibióticos, continúa mal. ¿Qué terapéutica añadiría en primer
término? 91. Infante con hipoacusia cursa con prueba de Rinne
negativo, además en la otoscopia se menciona membrana
A. Surfactante endotraqueal.
timpánica con niveles hidroaéreos.
B. Indometacina oral.
A. Otitis media aguda
C. Prednisona intravenosa.
B. Otitis media crónica
D. Bicarbonato intravenoso.
C. Otitis externa difusa
E. Glucosa intravenosa.
D. Otitis secretora
87. Durante la noche en una guardia de pediatría llega a
E. Otitis externa maligna.
consulta un paciente varón de 20 meses quien presenta
aguda- mente un cuadro de dolor de oído leve a moderado, 92. Las macula densa se encuentran en:
mama refiere que días antes presento un cuadro de resfrió y
que ahora presenta esta molestia. Al examen presenta una T A. La arteria aferente
38.5°C, orofaringe congestiva sin exudado, oído derecho con
B. La arteria eferente
tímpano eritematomoso, congestivo, abombado y nivel
hidroaereo, pulmones sin alteraciones. Según su diagnóstico, C. Túbulo contorneado distal
¿Qué le indicaría?
D. Asa de Henle
A. Amoxicilina
E. Capsula de Bowman
B. Cefuroxima
B. El cardias.

93. ¿Cuál es la contraindicación absoluta para el C. El fondo.


tratamiento hormonal en post menopausia?
D. El cuerpo.
A. Ginecorragia de origen desconocido.
E. El antro.
B. Varices superficiales
99. Una niña de 9 años de edad tiene una historia de
C. Bochornos sibilancias intermitentes de varios años; no ha recibido
medicamen- tos hace algún tiempo. Se halla febril, agitada y
D. Dislipidemia controlada con cianosis peribucal. Tirajes intercostales y supraesternales;
los ruidos respiratorios están disminuidos y hay sibilancias
E. HTA controlada.
bilaterales. La intervención inicial más apropiada es:
94. Cuál de las siguientes es un material de sutura
A. Administración de aminofilina intravenosa.
absorbible:
B. Solicitar radiografía torácica.
A. Nylon
C. Prescribir la nebulización de cromoglicato sódico.
B. Polidioxanona
D. Pedir biometría hemática completa y hemocultivo.
C. Polipropileno
E. Prescribir salbutamol nebulizado.
D. Seda
100. La trompa de Eustaquio y la mucosa del oído medio
E. Acero quirúrgico
se originan en:
95. ¿Cuál es la profundidad a la cual se debe comprimir
A. La cuarta bolsa faríngea
el torax en un lactante en el RCP?
B. El neuroectodermo notocordal
A. 3cm
C. El primer arco branquial
B. 4cm
D. La cuerda del tímpano
C. 5cm
E. La primera bolsa faríngea
D. 6cm

E. 7cm.

96. Bebé de 30 horas de edad, nació con 4300 g, hijo de


madre con diabetes gestacional, ha tomado bien su alimento,
pero se ve muy inquieto a los 30 minutos de vida. 15 minutos
más tarde sufre una convulsión tónico-clónica. Diagnóstico
más probable:

A. Hipoglucemia

B. Hipocalcemia

C. Hipomagnesemia

D. Hiponatremia

E. Hiperviscosidad

97. El grado de afectación fetal en la isoinmunización Rh


se aprecia con mayor exactitud por medio de:

A. Test de Coombs indirecto.

B. Test de Coombs directo.

C. Niveles de b-HCG en líquido amniótico.

D. Medición de los niveles de bilirrubina en el líquido


amniótico-curvo de Liley.

E. Test de Coombs y b-HCG. SIMULACRO 12A

98. La porción mayor del estómago está constituida por: 1. Son agentes patógenos de neumonía adquirida en la
comunidad los siguientes, excepto:
A. La unión gastroesofágica.
A. Streptococcus pneumoniae B. Linfoma y carcinoma gástrico / glioblastoma
multiforme
B. Acinetobacter baumanii
C. Cáncer de pulmón y de mama // astrocitoma grado IV
C. Mycoplasma pneumoniae
D. Cáncer gástrico y de páncreas // meduloblastoma
D. Hemophilus influenzae
E. Cáncer de pulmón y de mama // metastasis
E. Legionella pneumophila
7. Primigesta añosa de 34 semanas de gestación no
2.¿Cuál de las siguientes no es característica de convulsion controlada, acude por presentar cefalea, tinnitus, dolor
febril simple? epigástrico. Examen fiś ico: PA: 150/90 mm Hg, FC: 90 x ́, FR:
20 x ́, T: 36.5°C, edema en miembros inferiores. Altura uterina:
A. Duración menor de 15 minutos
30 cm, Feto LCI, FCF:140 x ́. Exámenes de Laboratorio: Hb:10
B. 2 o más en 24 horas g/dL, plaquetas 110,000/mm3, Creatinina: 1 mg/dL, TGO: 70
UI/L, LDH:1200 UI/L, Bilirrubina:1.4 mg/dL a predominio
C. No dejar déficit motor indirecto. ¿Cuál es el diagnóstico más probable?
D. Convulsion Tonicoclonico generalizado A. Síndrome HELLP
E. Convulsion asociado a fiebre B. Preeclampsia severa
3. Mujer de 30 años, acude por presentar Poliartrálgias y C. Hipertensión gestacional
pápulas eritematosas aplanadas sobre los nudillos de ambas
manos (signo de Gottron) y telangiectasias periungüeales. D. Hipertensión crónica
¿Cuál es lo correcto con respecto al diagnóstico?:
E. Eclampsia
A. Existe dolor muscular
8. ¿Cuáles son los agentes terapéuticos de uso
B. Asocia con esclerodermia frecuente en la epilepsia y convulsiones paciales y en las
generalizadas respectivamente?
C. Para el diagnostico se usa biopsia muscular
A. Lamotrigina, fenobarbital // carbamazepina
D. Presencia del anti jo 1 indica buen pronostico
B. Etosuximida, ácido valproico, lamotrigina //
E. Asocia con panarteritis nodosa lamotrigina
4. Es la relación que existe entre los resultados con C. Carbamazepina, fenilhidantoína, gabapentina // acido
respecto a los recursos valproico, lamotrigina
A. Eficiencia D. Primidona, topiromato // Carbamazepina, Lamotrigina
B. Efectividad E. Zonisanida, ácido valproico // Carbamazepina,
Lamotrigina
C. Eficacia
9. RN con hepatoesplenomegalia, rinitis purulenta,
D. Costo – Beneficio
anemia, rash maculopapular y descamación de Palmas y
E. Aceptación. plantas. Cuál es el diagnóstico más probable.

5. Mujer de 20 años, con cesárea por inducción fallida. A. CMV


Al tercer diá de puerperio presenta malestar general, fiebre y B. Sífilis congénita
loquios fétidos. Al Examen clínico: T: 39°C, útero
subinvolucionado y doloroso a la palpación, secreción fétida. C. Toxoplasmosis
¿Cuál es el diagnóstico más probable?
D. Rubeola
A. Dehiscencia de la histerorrafia
E. Herpes
B. Endometritis puerperal
10. Con respecto a la HTA señale lo correcto
C. Absceso de pared abdominal
A. Principal factor de riesgo para hemorragia
D. Retención de restos placentarios subaracnoidea
E. Necrosis isquémica de mioma uterino B. La hipertensión maligna genera necrosis fibrinoide
6. ¿Cuáles son los tumores primarios que más C. La hipertensión produce lesión de arterias de gran
frecuentemente producen metástasis cerebrales y cuáles son calibre
los tumores primarios más frecuentes en adultos?
D. La estenosis arteriolar corresponde a la retinopatía
A. Cáncer de cuello uterino y de próstata // astrocitoma grado II
grado II
E. Produce dilatación ventricular izquierda
11. Gestante de 38 semanas, acude a Emergencia por B. Hipertension transitoria
presentar dolor abdominal intenso hace 4 horas acompañado
de sangrado vaginal rojo vinoso. Examen físico: PA: 140/90 C. Pre-eclampsia severa
mmHg, LCF: 128 x’, AU: 37 cm, hipertonía uterina y se con-
D. Pre-eclampsia superpuesta
firma sangrado de cavidad uterina ¿Cuál es el diagnóstico
más probable? E. Sindrome de HELLP
A. Desprendimiento prematuro de placenta 16. Llega a emergencia un lactante de 3 meses quien 4
días antes de su ingreso desarrolla rinorrea y estornudos,
B. Placenta previa
ahora tiene dificultad para respirar, taquipena y a la
C. Ruptura uterina auscultación subcrepitos. Según su sospecha diagnostica,
¿Cuál es el manejo más indicado?
D. Vasa previa
A. Iniciar amoxicilina
E. Inversión uterina
B. Brindar terapia de rescate con beta-agonistas
12. Si el cuadro clínico de asma bronquial cursa con
sibilancias, tos, disnea menos de 2 veces por semana, C. Dar broncodilatadores más corticoide
exacerbaciones breves (desde horas a dias), síntomas
D. Iniciar macrolidos
nocturnos mayores de una vez por semana, asintomática
entre exacerbaciones. E. Nebulización con suero hipertónico.
¿Qué clasificación de gravedad le corresponde? 17. Varón de 56 años, que presenta pérdida de peso de 6
kg en el último año, cambios de la voz, incremento del
A. Intermitente
volumen de la glándula tiroides a predominio derecho. No
B. Persistente leve adenomegalias y pruebas de función tiroidea normales. En el
preope- ratorio, ¿qué examen complementario de tiroides
C. Persistente moderado solicitaría?

D. Persistente grave A. Aspiración con trocar

E. Intermitente moderado. B. ́
Gammagrafia

13. Acude a consulta un paciente de 6 meses de edad C. Ultrasonografía con biopsia


que en sus 3 primeros días de enfermedad presenta cuadro
catarral, luego aumenta su frecuencia respiratoria por lo que D. Resonancia magnética
acude a emergencia donde se evidencia taquipnea, tirajes,
E. Tomografía
sibi- lantes. ¿Cuál es el agente etiológico más probable?
18. Paciente proviene de Piura, con fiebre de menos de 5
A. Adenovirus
́
dia s de duració n, sin afección de las via
́ s aéreas superiores
B. Micoplasma con cefalea y mioartalgias. ¿Cuál es el diagnóstico más
probable?
C. Penumococo
A. A.Dengue sospechoso.
D. Virus sincitial respiratorio
B. B.Probable dengue grave
E. Rinovirus
C. C.Probable dengue hemorrágico
14. ¿Cuál es la alternativa que NO corresponde a la
anemia ferropenia? D. D.Dengue confirmado

A. La respuesta al tratamiento se mide con reticulocitos E. E.Dengue descartado


al 10 dia
19. Inmunoglobulina aumentada en purpura de Henoch
B. La causa más frecuente es carencial en jovenes Schonlein

C. Para diferenciarla de la anemia por enfermedad A. Ig G


crónica se mide transferrina
B. Ig E
D. El test bioquimico mas especifico es la ferritina
C. Ig M
E. El hierro se absorbe solo el 10% de lo ingerido a nivel
D. Ig D
intestinal.
E. Ig A
15. Adolescente de 15 años, con 32 semanas de
gestación , sin control prenatal, acude a emergencia por dolor 20. Paciente mujer de 20 años con palpitaciones de inicio
epigástrico y cefalea. Al examen impresiona como embarazo y fin súbito, con PA normal, la primera medida terapuetica es:
de 6 meses por altura uterina. PA: 160/110mmHg, Proteinuria
(+++). El diagnostico más probable es : A. Digoxina

A. Eclampsia B. Atenolol
C. Verapamilo 26. Señale lo correcto respecto a la menopausia:

D. Adenosina A. Los niveles de FSH se elevan antes que los


trastornos del ciclo menstrual.
E. Maniobras vagales
B. El estradiol es la hormona predominante.
21. Gestante de 8 semanas acude por sangrado vaginal
profuso desde hace 3 horas, acompañado de dolor abdominal C. La progesterona se eleva tras la menopausia.
intenso. Examen físico: PA: 80/60 mmHg, FC: 98 x’, FR: 18 x’.
Especuloscopía: cérvix con sangrado y coágulos abun- D. El climeterio es más intenso en los primero 10 años
dantes. Tacto vaginal: orificio externo e interno abiertos. ¿Cuál anteriores al diagnóstico.
es el diagnóstico más probable?
E. La terapia de reemplazo hormonal es lo más
A. Aborto inevitable importante en todas las menopausicas.

B. Aborto frustro 27. Lactante de 8 días de nacido con distensión


abdominal y vómitos biliosos abundantes, signo de la doble
C. Aborto inminente burbuja.

D. Amenaza de aborto A. Páncreas tubular

E. Aborto diferido B. Estenosis hipertrófica de piloro

22. Mencione lo que no corresponde al score de forrest C. Vólvulo de ciego


para hemorragia digestiva alta
D. Atresia duodenal
A. El tipo Ia tiene alta tasa de mortalidad
E. Bridas
B. El tipo IIb es coagulo adherido
28. ¿Con que prueba se realiza el diagnostico de asma
C. El tipo III es sangrado inactivo bronquial en un niño mayor de 5 años?

D. El tipo Ib requiere tratamiento endoscópico más A. Espirometria simple


antiacidos
B. Flujometro
E. El tipo IIc es ulcera con fibrina
C. Espirometria forzada
23. ¿En qué mecanismo del trabajo de parto se produce
cambio de la variedad de posicion? D. Pletismografia

A. Expulsión E. Radiografia de torax

B. Rotación interna 29. Paciente con IMA que presenta crepitantes en ambos
campos pulmonares signos de ICC severa se clasifica como
C. Descenso
A. Killip I
D. Extensión
B. Killip II
E. Flexion
C. Killip III
24. Paciente programado para cirugía de apendicitis
congestiva, en sala se produce mínima lesión del colon. Como D. Killip IV
́ .
lo clasificaria
E. Forrester III
A. Limpia
30. Paciente con disfagia motora más pérdida de peso,
B. Sucia ante la sospecha diagnostica quien confirma la etiologia

C. Limpia contaminada A. Ph metria de 14 hrs

D. Sucia contaminada B. Endoscopia alta

E. Contaminada C. Impedanciometria

25. El diagnóstico bioquímico del empiema pleural se D. Manometría esofagica


realiza sobre la base de los siguientes criterios, excepto:
E. Test de secretina
A. Triglicéridos mayor de 100

B. Glicemia menor de 40 mg%

C. pH menor de 7.2
31. En relación con la etiología del RCIU II, señale factor
D. proteínas mayor de 3g% más frecuente

E. Gran celularidad a predominio de polimorfonucleares A. Enfermedad hipertensiva


B. Rubéola E. Eutrófico

C. Alcoholismo 37. El Caso índice o caso cero Corresponde a:

D. Drogadicción A. Primer caso ocurrido en una epidemia.

E. Tabaquismo B. Primer caso reportado al sistema de salud.

32. ¿Cuál es el vector del ciclo de transmisión selvático C. Primer caso muerto.
de la fiebre amarilla en la América del sur?
D. Primer caso tratado.
A. Mosquitos de genero Haemagogus
E. Primer caso publicado.
B. Mosquitos de genero Aedes
38. Paciente de 15 años, con náuseas, vómitos, boca
C. Mosquitos de genero Crotalus seca, y somnolencia, PA 60/40 IMC 17KG/M2 , glicemia
450mg/dl; pH
D. Mosquitos de genero Phlebotomus
7,15; HCO3 12mEq/kg , la terapia indicada comprende fluidos
E. Mosquitos de genero Vivax más:
33. ¿Cuál de las siguientes entidades causa derrame A. Insulina cristalina en bolos
pleural tipo trasudado?
B. Insulina cristalina por vía SC
A. ́ bacteriana
Neumonia
C. Insulina cristalina por infusión
B. TBC pleural
D. ́ IM
Insulina cristalina por via
C. Cirrosis hepática
E. ́ SC
Insulina intermedia por via
D. Metástasis pleural
39. Respecto a los ACO señale lo falso:
E. Mesotelioma .
A. Si se dan gestàgenos solos el efecto anticonceptivo
34. En el Perú actual, el cuidado de la salud está en los disminuye.
primeros lugares de la agenda política del Acuerdo Nacional,
al mismo tiempo el Ministerio de Salud sólo ha podido B. Si se dan progestágenos solos puede ocurrir atrofia
desarrollar precariamente su rol rector y conductor sensorial. endometrial.
La combinación de estos dos factores en una matriz FODA
permite identificar la siguiente estrategia: C. El principal efecto anticonceptivo se obtiene a nivel
central hipotalámico.
A. ́
Desafio
D. Los progestágenos de tercera generación tienen
B. Limitación buen perfil lipídico.

C. Potencialidad E. Los gestagenos en depósito son de elección en


adolescentes y nulíparas.
D. Restricción
40. Con respecto a la angina inestable señale lo
E. Riesgo. verdadero
35. Paciente varon joven con datos de trombosis venosa A. La etiología más frecuente es la placa de ateroma
profunda, además se encuentra ictericia a predominio indirecto con capa fibrosa gruesa
con coluria, la sospecha diagnostica es:
B. El factor de riesgo más importante es la hipertensión
A. Hemoglobinuria paroxiś tica nocturna arterial
B. Esferocitosis C. Las mujeres tienen más riesgo en comparación con
varones
C. Talasemias
D. Una variante clínica es la angina por vasoespasmo
D. ́ s
Hemoglobinopatia
coronario
E. Eliptocitosis
E. Presenta elevación de CPK MB
36. ¿Cuál es el diagnostico nutricional de un lactante de
41. Niño con alopecia, lesión en piel (acrodermatitis),
11 meses que tiene P/T 94%, T/E 94%?
diarrea, deficiencia de que vitamina:
A. Desnutrición aguda
A. Zinc
B. Desnutrición crónica
B. Niacina
C. Desnutrición crónica reagudizada
C. Tiamina
D. Desnutrición global
D. Magnesio
E. Hierro C. Parkinsonismo / levodopa y memantina

42. Paciente varón de 25 años, politraumatizado en D. Enfermedad de Parkinson / levodopa y carbidopa


accidente de tránsito. Al examen neurológico: apertura ocular
al do- lor, emite sonidos incomprensibles y extiende el E. Wilson / EDTA.
miembro superior al dolor. De acuerdo a la escala de
47. Acude a emergencia un paciente con estridor que
Glasgow, cuál sería la conducta inmediata a seguir:
días antes presento clínica de rinorrea, estornudos y
A. Observación sensación de alza térmica, ahora presenta dicho estridor y tos
perruna. Al evaluarlo se le clasifica como leve. ¿Cuál es su
B. Intubación orotraqueal manejo?

C. Craneotomía A. Antipiréticos

D. Traqueotomía B. Nebulización con adrenalina

E. TAC C. Nebulización con suero hipertónico

43. Mujer de 40 años de edad, presenta D. Dexametasona endovenoso


predominantemente en la región nasogeniana legiones
papulosas eritomatosas y descamaticas, las cuales se E. Dexametasona vía oral
exacerban con alguna comida y tensión emocional. el
48. Fractura conminuta no desplazada del primer MTC:
diagnóstico más probable es y el manejo adecuado debería
ser: A. Benett
A. Rosácea / minociclina B. Rolando
B. Acné / retinoides C. Jones
C. Rosácea / metronidazol topico D. Murphy
D. Acné / metronidazol oral E. Mallampati
E. Psoriasis / retinoides o metronidazol 49. En la pubarquia, de acuerdo con la clasificación de
Tanner, si se encuentra vello pubiano que extiende solo en el
44. La nueva estructura de etapas del ciclo de vida según
trián- gulo pubiano, corresponde al estadio:
el modelo de atención integral de salud incluye a la etapa
joven. ¿Cuál es el rango de edad que le corresponde? A. VP1
A. 18a29años B. VP2
B. 19a29años C. VP3
C. 20 a 29 años D. VP4
D. 19a30años E. VP5
E. 20a30años 50. Paciente varón de 18 años, quien ingresa a
emergencia con dolor abdominal desde hace 7 horas,
45. Adulto joven en cuyo perfil hepático solo se encuetra
localizado en fosa iliaca derecha y náuseas, afebril. Dolor en
alterado la bilirrubina que esta aumentado ligeramente a
punto de Mc Burney positivo. Leucocitosis y PCR levemente
predo- minio indirecto, la sospecha diagnostica es
aumentado. De acuerdo al diagnóstico más probable, señale
A. Síndrome de rotor lo correcto:

B. Síndrome de dubin jonson A. La etiología es básicamente la perforación de la


víscera
C. ́ drome de Gilbert
Sin
B. El germen que se puede identificar con más
D. Sindrome de clijer najar tipo I probabilidad es el Bacteroides fragilis

E. Sindrome de clijer najar tipo II C. El tratamiento antibiótico requiere Amikacina +


Metronidazol por una semana
46. Varón de 45años de edad, que presenta desde hace
12 meses de manera progresiva dificultad para la marcha. Al D. La causa más frecuente es la hipertrofia de las placas
exa- men se evidencia hipomimia facial, lentitud para iniciar la de Peyer.
marcha, dificultad para atar los cordones de sus zapatos,
temblor fino en miembro superior derecho y resistencia a la E. El germen más frecuente es E. coli
flexo-extensión pasiva de dicho miembro. La sospecha
51. El establecimiento de sector salud de categoría I-2
diagnóstica es ------ y el manejo adecuado es --------:
corresponde a:
A. Alzheimer / memantina
A. Puesto de salud con médico
B. Huntington / levodopa y carbidopa
B. Puesto de salud sin médico
C. Puesto de salud con 10 médicos 57. En el embarazo normal, la altura del fondo uterino se
vuelve extrapelvico durante la semana y se considera un
D. Puesto de salud con hospitalizacion síntoma… de diagnóstico de embarazo
E. Puesto de salud con quirófano. A. 12 - probable
52. ¿A qué edad aproximadamente un niño duplica su B. 16 - presuntivo
talla de nacimiento?
C. 18 - certeza
A. 1 año
D. 10 - presuntivo
B. 2 años
E. 20 – certeza
C. 3 años
58. ¿Cuál es la neoplasia más frecuente en pediatría?
D. 4 años
A. Leucemias
E. 5 años
B. Linfomas
53. Paciente con epistaxis posterior mencione lo
verdadero C. Tumor de willms

A. Lesión del plexo de kiesellbach D. Astrocitoma

B. Fácil de controlar E. Meduloblastoma.

C. Resuelve con taponamiento anterior 59. En la prueba de hipótesis, el investigador comete un


error tipo II o beta cuando:
D. La arteria esfenopalatina es rama de la maxilar
interna A. No establece el nivel de significancia

E. La causa más frecuente es infecciones. B. No rechaza la hipótesis nula siendo falsa

54. Paciente de 40 años de edad que presenta necesidad C. Rechaza la hipótesis alterna, siendo falsa
imperiosa de miccionar y se le escapa la orina antes de llegar
al baño, ya que no puede retenerla. Al examen se constata D. Rechaza la hipótesis alterna, siendo verdadera
uretrocistocele de primer grado. El diagnóstico es:
E. Rechaza la hipótesis nula siendo verdadera.
A. Incontinencia urinaria de esfuerzo
60. ¿Cuál es la vacuna que no debe darse en alérgicos a
B. Incontinencia urinaria de urgencia la proteína del huevo?

C. Infección de vías urinarias A. BCG

D. Cistitis intersticial B. Antineumococica

E. Cistitis aguda. C. Antiamarilica

55. La denominación sinclitismo y asinclitismo se refiere D. VPH


a la relación entre:
E. Rotavirus
A. La sutura interparietal y las tuberosidades izquierdas.
61. Varón de 50 años de edad, que ingresa al Hospital
B. La sutura metódica y el promontorio. por presentar un cuadro de disnea, palidez de piel y mucosas.
Al examen clínico: PA 150/90 mmHg. Se detecta anasarca y
C. La sutura occipital y el promontorio. signos de derrame pleural derecho. Exámenes de laboratorio:
hemoglobina 9g/dL, riñones con pérdida de la diferenciación
D. La sutura sagital con el promontorio y el pubis. corticomedular ¿Cuál es el diagnóstico más probable?:
E. La sutura sagital y las espinas ciáticas. A. Síndrome nefrótico
56. Multigesta de 10 sem con sangrado vaginal y dolor B. ́ drome nefrítico
Sin
abdominal, altura uterina 18 cm orificio cervical
entreabierto,Además presenta hemoptisis ¿Cuál es el C. Glomerulonefritis aguda
diagnóstico?
D. Nefritis aguda
A. Mola parcial
E. Insuficiencia renal crónica
B. Mola completa
62. ¿Cuál de los siguientes es el mejor parámetro para
C. Aborto molar evaluar el crecimiento de un niño?

D. Degeneración hidrópica A. IMC en valores absolutos

E. Coriocarcinoma B. Una sola visita en su vida


C. El seguimiento con las curvas de crecimiento D. Uso de diuréticos de asa

D. Con los datos del nacimiento E. Uso de resina de intercambio iónico

E. Evaluación psicológica

63. El índice ecográfico que mejor identifica el retardo del


crecimiento fetal es… , y para evaluar sus pronostico se
68. Señale que cambios ocurren en el embarazo normal
utiliza la ecografía doppler de la arteria
A. Disminución del tamaño renal
A. La longitud del fémur - uterina
B. Dilatación uretereal bilateral
B. La circunferencia abdominal - umbilical
C. Disminución del índice de filtración glomerular
C. La longitud del fémur – cerebral media
D. Aumento del índice de flujo plasmático renal
D. La circunferencia abdominal – ductus venoso
E. Pronunciada proteinuria
E. La longitud del fémur – cerebral media
69. ¿Cuál es la secuencia lógica en el proceso
64. Varón de 65 años, con T: 38°C, náuseas, vómitos y administrativo?
deposiciones líquidas con moco, sangre y tenesmo. Estando
hospi- talizado presenta una convulsión por primera vez. A. Planificación, organización, dirección y control.
¿Cuál es el agente causal?
B. Planificación dirección, organización, y control
A. Entamoeba histolit́ ica
C. organización, dirección control Planificación
B. Salmonella
D. Planificación, control organización, dirección
C. Escherichia colie
E. dirección Planificación, organización, y control.
D. Clostridiun perfingens
70. RN con antecedente de DIPII y líquido amniótico
E. Shiguella biflexa. meconial, nace hipotónico, apnea y bradicardico. Qué medida
to- maría.
65. Es un complicación tardía de la traqueotomía
A. Aspiración de secreciones
A. ́ gea
Estenosis larin
B. Estimulación táctil y aspiración de secreciones
B. Hemorragia
C. VPP
C. Neumotórax
D. VPP con mascarilla
D. Atelectasia pulmonar
E. Ventilación mecánica
E. Enfisema subcutáneo
71. En varones adultos con anemia ferropénica ¿con que
66. Niño de 2 años de edad, que desde los 9 meses prueba monitorizamos la respuesta al tratamiento?
presenta episodios de lesiones eritematosas descamativas en
mejillas, abdomen y parte extensoras de miembros, con prurito A. Hemoglobina a los 20 dias
intenso. Actualmente aparecen lesiones similares después de
B. HTO a los 15 dias
comer maní. Al examen se evidencia pliegues redundantes
infraorbitarios. ¿Qué tipo de dermatitis consideraría en su C. Reticulocitos a los 10 dias
presunción diagnóstica?:
D. Reticulocitos a los 5 dias
A. Por ingesta de alimentos
E. Ferritina a los 30 dias
B. Seborreica
72. ¿Cuál es la caracteriś tica del flujo vaginal en la
C. Alérgica de contacto Vaginosis bacteriana?
D. Eccematoide infecciosa A. Seco y espumoso
E. Atópica B. Grumoso y blanquecino
67. En el tratamiento de la hiponatremia en un paciente C. Fétido y grisáceo
con insuficiencia renal aguda oligúrica, la medida terapéutica
más apropiada es: D. Espeso y blanquecino

A. Restricción de agua libre E. Espumosos y sanguinolento.

B. Restricción de sodio 73. ¿Cuál es el principal mecanismo de la ovulación a


nivel hormonal
C. Administración de sodio
A. El pico de LH aislado.
B. El pico de FSH aislado. E. Quetiapina

C. El pico de Estradiol y FSH. 79. Que serotipo VPH está asociado a las verrugas
genitales
D. El pico de Estradiol y LH.
A. 6 y 11
E. El pico de Progesterona aislado.
B. 16 y 18
74. Recién nacido con fibrosis quiś tica, cual es la
manifestación más relacionada: C. 24 y 28

A. Ileo meconial D. 15 y 11

B. Policitemia E. 6 y 18

C. Hipoglicemia 80.¿Cuál de las siguientes medidas de tendencia central es la


más usada para el análisis estadístico si los datos son asi-
D. Dermatitis crónica metricos?
E. ́
Neumonia A. La mediana
75. ¿Qué porcentaje del área bajo la curva normal B. La media
estándar se localiza entre +2 desviaciones estándares de la
media? C. La moda

A. 58,6 D. La desviación estándar

B. 95,4 E. La varianza

C. 68,3 81.Según la clasificación de Bormann para cáncer gástrico, las


lesiones ulceradas infiltrantes corresponden al tipo:
D. 90,4
A. 5
E. 99,9
B. 4
76. Las fracturas que se producen por
microtraumatismos, ya sea por intensidad o por repetición que C. 3
́ icas del hueso, se
alteran las propie-dades fisicoquim
denominan y la más frecuente es la fractura : D. 2

A. Por fatiga / del recluta E. 1

B. Patológicas / colles 82.Varón de 23 años de edad: refiere fácil sangrado de las


enciá s al uso del cepillo dental. Al examen; palidez y
C. Por fatiga / cargador mucosas, ausencia de gingivitis; punta de bazo palpable.
Hematocrito 31%, leucocitos 2.300/dL, plaquetas 100.000/dL.
D. Osteoporoticas / colles Frotis de sangre periférica: cuerpos de Auer. El diagnóstico
presuntivo es:
E. Patologicas / metatarsiano 2
A. Leucemia linfoide aguda
77. Marque lo falso respecto a las siguientes patologías
neurológicas: B. Leucemia mieloide aguda
A. La demencia más frecuente es la demencia de C. Leucemia mieloide crónica
Alzheimer.
D. Mielofibrosis
B. La demencia de Alzheimer es predominantemente
subcortical. E. Mononucleosis infecciosa

C. El nivel de lesión mesencefalico se evalúa con la 83. Paciente de 78 años con te: 2 días, presenta
rigidez de decorticacion o descerebración. deposiciones líquidas 3 v/día, volumen de ½taza por vez, s/s,
con pa: 100/60 mmHg, p: 100x’, fr: 22x’; presión ocular
D. El tumor más frecuente en el SNC es metastasico de disminuida. Además tiene Na: 120 mmol/l, k: 3,5 mmol/l, cl:
pulmón. 100 mmol/l. respecto al estado hidroelectrolítico de la
paciente, podemos afirmar que presenta:
E. El tumor pilocitico infantil es frecuente en cerebelo.
A. Hiponatremia Hiperosmolar.
78. Tratamiento para crisis de agitación psicomotriz:
B. SIADH
A. Haloperidol
C. Hiponatremia con hipovolemia
B. Benzodiazepinas
D. Hiponatremia con normokalemia
C. Fenitoina
E. Hiponatremia con hipovolemia
D. Carbamazepina
84. Paciente diabético cursa con pérdida progresiva de C. Antibióticos orales y AINES
agudeza visual, la etiología más probable seria:
D. Antibióticos intra-articulares
A. Desprendiemiento de retina
E. Irrigación intra-articular continua
B. Edema macular
89. Mujer de 20 años de edad, peladora de espárragos
C. Hemovitreo desde hace 4 meses, presenta lesiones eritematosas
pruriginosas micropapulares localizada en dorso de
D. Glaucoma de Angulo estrecho antebrazos, cara y cuello. Las lesiones también se han
identificado en algunas compañeras de trabajo. No presenta
E. Miopia maligna
antecedentes de atopía. Su primera impresión diagnóstica es
85. Mujer de 22 años de edad, sexualmente activa, con y el tratamiento es:
dolor pélvico intenso y fiebre. Al examen: anexos dolorosos y
A. Acarosis / permetrina
leucorrea maloliente. Hemograma con 15.000 leucocitos x
mm3. ¿Cuál es el diagnóstico y el manejo más apropiado? B. Dermatitis por contacto / corticoides
A. Enfermedad pélvica inflamatoria + hospitalización + C. Acarosis / griseofulvina
Ceftriaxona + metronidazol + doxiciclina
D. Dermatofitosis / clotrimazol
B. Enfermedad pélvica inflamatoria + hospitalización +
cefoxitin + metronidazol + azitromicina E. Dermatofitosis / itraconazol

C. Enfermedad pélvica inflamatoria + tratamiento 90. El antidepresivo con menor efecto sedante y el que
ambulatorio con ciprofloxacino, doxiciclina y metronidazol. tiene mayor efecto sedante son respectivamente:

D. Quise ovárico complicado + tratamiento quirúrgico A. Amitriptilina / sertralina

E. Enfermedad pélvica inflamatoria + tratamiento B. Clomipramina / imipramina


quirúrgico.
C. Doxepina / bupropion
86. Mujer de 26 años, con fiebre de 39°C, de 4 semanas
de evolución que calma con antipiréticos; fatiga intensa y D. Fluoxetina / paroxetina
malestar general. Al Examen físico: Linfoadenomopatías
E. Trazodona / sertralina
cervicales, submaxilares y supraclaviculares y
hepatoesplenomega- lia. Hemograma con 15% de linfocitos 91. Mujer con infección por chlamydia, que esquema de
́ icos y leve trombocitopenia. Anticuerpos heterofilos
atip tratamiento indicaría.
positivos. ¿Cuál es el diagnóstico más probable?
A. Metronidazol 2gr dosis única
A. Dengue hemorragico
B. Fluconazol 150mg
B. Mononucleosis infecciosa por CMV
C. Doxiciclina 100 mg
C. Fiebre Chikungunya
D. Ketoconazol
D. Mononucleosis infecciosa por VHB
E. Ceftriaxona 250 mg
E. Tuberculosis pulmonar
92. Ubicación más frecuente del tumor de klastkin
87. Paciente varón de 45a con DM tipo I. Acude por
cefalea, somnolencia, náuseas y vómitos persistentes. Al A. En conducto cístico
examen físi- co, despierto, pálido y mucosa oral seca; PA
B. Colédoco proximal
140/70, FC: 100; FR: 28; extremidades inferiores con fóvea++
Hb 10 pH: 7.24; Hto 44 K 2.9 Na 139; HCO3 15. Glucosa 280. C. Confluencia de hepáticos
¿Cuál es el Diagnostico?
D. Colédoco distal
A. Estado Hiperosmolar
E. Hepático derecho
B. Cetoacidosis Diabética
93. Varón de 19 años que refiere aumento de volumen en
C. Pielonefritis crónica el hemiescroto derecho transiluminacion positiva, deberíamos
descartar
D. Cetoacidosis ayuno
A. Hernia inguinoescrotal.
E. Insuficiencia Renal Aguda
B. Hidrocele
88. En la artritis bacteriana aguda de rodilla, la conducta
terapéutica inmediata es: C. Quiste del epidídimo.
A. ́ uido sinovial y AINES
Drenaje de liq D. tumor testicular tipo seminoma
B. Antibióticos parenteral y drenaje E. Tumor testicular tipo no seminoma
94. Beta bloqueador utilizado en glaucoma crónico D. Clopidogrel.

A. Metoprolol E. Abciximab.

B. Bisoprolol 100.El tipo de sangrado más característico que producen los


leimiomas uterinos:
C. Nadolol
A. Hipermenorrea por intramurales
D. Carvedilol
B. Metrorragia por submucosos
E. Timolol
C. Menorragia por submucosos
95. Paciente varón de 50 años, fumador, con
Hipertensión arterial controlada, programado para cura D. Menometrorragia por intramurales
quirúrgica de her- nia inguinal. Su riesgo ASA es:
E. Hipermenorrea por submucosos F.
A. V

B. II

C. I

D. IV

E. III

96. Se cuenta con 13 valores de nivel de bilirrubinas en


pacientes con ictericia y se desconoce la varianza poblacional
para esta variable. En la construcción del intervalo de
confianza al 95% para el valor promedio de bilirrubina en este
grupo de pacientes, la prueba estadística requerida es:

A. ANOVA

B. Binomial

C. Chi-cuadrado

D. Exacta de Fisher

E. T de Student.

97. Paciente con WPW, cual es la alternativa correcta:

A. Se debe a fenómenos de reentrada

B. Su tratamiento definitivo es con beta bloqueadores

C. Su tratamiento de urgencia es ablación con


radiofrecuencia

D. Cursa con onda delta en el EKG

E. Más frecuente en personas adultas mayores

98.¿Cómo se define un RN con peso adecuado para la edad


gestacional?

A. P10-90

B. P5-95

C. P15-85

D. P3-97

E. P3-90

99.La inhibición de la agregación plaquetaria por ADP


(adenosina difosfato) es característica de:

A. Ácido acetilsaliciĺ ico.

B. Tirofibán.

C. Sulfinpirazona.
SIMULACRO 10A C. Pseudomona aeruginosa
D. Staphylococcus aureus
E. Haemophilus influenzae
1. Cuadro clínico con foco neumónico en radiografía se
ve lesión en base de pulmón derecho que borra
7. ¿Cuál es la tasa de filtración glomerular compatible
aurícula derecha
con paciente diabético en estadio temprano?
¿qué lóbulo está afectado?
A. 90 ml/min
A. Inferior
B. 60 ml/min
B. Apical
C. 150 ml/min
C. Medio
D. 30 ml/min
D. Lingula
E. 15 ml/min
E. Superior
8. Mujer de 20 años de edad, peladora de espárragos
2. Gestante de 40 semanas que acude en periodo
desde hace 4 meses, presenta lesiones
expulsivo del trabajo de parto. ¿Cuál de los
eritematosas pruriginosas micropapulares
movimientos principales del trabajo es el más
localizada en dorso de antebrazos, cara y cuello.
precoz?
Las lesiones también se han identificado en
A. Rotación interna
algunas compañeras de trabajo. No presenta
B. Extensión
antecedentes de atopía. Su primera impresión
C. Flexion
diagnóstica es y el tratamiento es:
D. Rotación externa
A. Acarosis / permetrina
E. Expulsión.
B. Dermatitis por contacto / corticoides
C. Acarosis / griseofulvina
3. ¿Cuál es el volumen de rehidratación que debería
recibir un lactante en quien se clasifica como D. Dermatofitosis / clotrimazol
deshidratación severa E. Dermatofitosis / itraconazol
sin shock?
A. 200 ml/Kg/día 9. ¿ Cuál es el antibiótico de elección para la diarrea
B. 100 ml/Kg/día producida por campylobacter jejuni
C. 100 ml/SC/día A. Azitromicina
D. 3500 ml/Kg/día B. Dicloxacilina
E. 150 ml/Kg/día C. Clindamicina
D. Cotrimoxazol
4. Respecto a la enfermedad diverticular, señale lo E. Ciprofloxacino
correcto:
A. La diverticulosis siempre requiere tratamiento 10. Son criterios mínimos para el diagnóstico de la
B. La hemorragia diverticular suele ceder enfermedad inflamatoria pélvica:
espontáneamente A. Dolor abdominal inferior, VSG elevada examen del
C. Los divertículos son más frecuentes en el color flujo vaginal positivo para N. Gonorrhoeae
ascendente B. Dolor a la movilización cervical, absceso tuboovarico
D. En el manejo de la diverticulitis siempre se opta por en ecografi ́a, examen del flujo vaginal positivo para
la colostomía C.
E. Es más frecuente entre los 30 y 50 años Trachomatis

C. Dolor abdominal inferior, dolor a la movilización


5. Sobre tratamiento de HTA señale lo falso cervical, dolor anexial bilateral.
A. Paciente con HTA con microalbuminuria es de D. Dolor anexial bilateral, temperatura oral > 38 0C,
elección captopril flujo vaginal o cervical anormal.
B. Paciente con HTA e IRC terminal con K < 5.5 E. Dolor hipogástrico persistente, protei ́na C reactiva
indicamos calcioantagonistas elevada, flujo vaginal anormal.
C. Paciente con HTA y antecedente de osteoporosis
indicamos tiazidas 11. Primigesta en trabajo de parto, 6 cm de dilatación
D. Paciente con HTA de raza afroamericana indicamos con membranas íntegras prominentes, C–4. Se
beta block tacta cordón; con- jugado diagonal 10,5 cm;
E. En gestantes con HTA severa con labetalol ponderado fetal 3.500 g. ¿Cuál es su diagnóstico?:
A. Desproporción céfalo-pélvica por macrosomía fetal.
B. Desproporción céfalo-pélvica y prolapso de cordón.
6. El agente Gram negativo más frecuente de la
C. Estrechez pelviana y deflexión II.
neumonía asociada a Ventilación es:
D. Estrechez pelviana y procúbito de cordón.
A. Escherichia coli
E. Estrechez pelviana y prolapso de cordon
B. Enterococco faecalis
B. Los adenomas colorrectales se consideran lesiones
12. Acude a consulta un paciente de 3 años de edad. premalignas
Mama refiere que presento un cuadro de rinorrea y C. El tipo más frecuente es el adenocarcinoma
estornudos leves. Luego empezó a presentar D. Es más frecuente en mayores de 50 años
lesiones tipo maculopapulares que se extendieron E. El consumo de grasas saturadas aumenta el riesgo
en todo su cuerpo al primer dia de exan- tema y que
luego desaparecieron con descamación furfurácea 18. Paciente adulto con diagnóstico de neumonía por
al tercer día de enfermedad. ¿Cuál es el estafilococo. Recibe tratamiento durante varios
diagnostico? días con Vancomici- na, pero no mejora. ¿Qué
A. Exantema súbito antibiótico recomendaría en este paciente?
B. Roseola A. Ceftriaxona
C. Sarampion B. Piperacilina tazobactam
D. Rubeola C. Linezolid
E. Escarlatina. D. Meropenem
E. Clindamicina
13. Paciente joven leptosomico con colelitiasis,
además se encuentra el examen físico 19. ¿Cuál de las siguientes es un tipo de exantema
esplenomegalia, además presente astenia y en su conocido como la quinta enfermedad?
perfil hepático destaca la elevación de bilirrubina a A. Exantema súbito
predominio indirecto., ante la sospecha diagnostica B. Eritema infeccioso
solicitamos C. Varicela
A. Test de HAM D. Sarampion
B. Electroforesis de hemoglobina E. Rubeola.
C. PCR enzimático
D. Test de fragilidad osmótica 20. Mujer de 25 años, puérpera de 1 horas de parto
E. Gota gruesa vaginal, RN de 4 500 gr, presenta abundante
sangrado rojo rutilan- te, se encuentra pálida,
14. En el fenómeno de Raynaud, el paciente puede taquicárdica, sudorosa, útero a 3 cm debajo de la
recibir como terapia farmacológica: cicatriz umbilical. ¿Cuál es el diagnóstico probable?
A. Prednisona A. Atonía uterina
B. Isorbide B. Retención de restos placentarios
C. Minoxidil C. Hipotonía uterina
D. Propanolol D. Embolia de líquido amniótico
E. Amlodipino E. Laceración del tracto genital

15. Según la clasificación de Johnson de ulcera gástrica 21. Es incorrecto respecto a las quemaduras de tercer
señale lo verdadero grado:
A. La tipo II es la más frecuente A. La superficie es indolora
B. La tipo III cursa con secreción de ácido disminuido B. Destruye toda la dermis
C. La tipo I se ubica en curvatura mayor C. Tiene un color oscuro
D. La tipo IV cursa con hipersecreción de acido D. La superficie es generalmente seca
E. La tipo V se asocia a lesión por AINES E. Presenta ampollas en la superficie

16. En relación al diagnóstico de cardiopatía isquémica 22. ¿Cuál de los siguientes enteroparásitos NO
señale lo falso transmite por fecalismo?:
A. En caso de duda diagnostica en la angina estable A. Lumbricoides
solicitamos ergometria B. Cryptosporidium parvum
B. El estudio de mayor dato pronóstico es la angiografía C. Entamoeba histolytica
coronario D. Strongyloides stercolaris
C. La forma aguda produce un trombo que causa E. Trichuris trichiuria
oclusión coronaria
D. El cuadro crónico se produce una estenosis fija que 23. Mujer de 64 años, G:1, P:1001, menarquia: 10
va progresando en años años. FUR: 55 años, obesa, diabética e hipertensa
E. la oclusión total se manifiesta eléctricamente sin desde hace 10 años. Acude por ginecorragia.
elevación del segmento ST Ecografía: endometrio 20 mm y volumen uterino
110 cc. PAP: AGUS ¿cuál es el diagnóstico más
17. Respecto al cáncer de colon, es incorrecto: probable?
A. La presentación clínica del cáncer de hemicolon A. Adenocarcinoma endometroide
derecho es fundamentalmente obstructiva
B. Carcinoma de células claras
C. Carcinoma indiferenciado A. Dermatitis herpetiforme
D. Carcinoma mucinoso B. Penfigo cicatricial
E. Carcinoma seroso papilar C. Penfigo seborreico
D. Penfigo vulgar
E. Penfigoide ampollar
24. No es un criterio de causalidad de Hill:
A. Fuerza de asociación 30. Paciente varón de 3 años de edad, quien desde
B. Número necesario a tratar hace aproximadamente 4 días presenta
C. Experimentación deposiciones liquidas en fre- cuencia de 5 por día
D. Consistencia sin sangre sin moco. Al examen físico presenta
E. Gradiente biológica deshidratación de grado moderado. ¿Cuál es el
rango de volumen que debería recibir dicho
25. ¿Cuál de las siguientes es INCORRECTA respecto a la paciente?
leche y lactancia materna? A. 20-80 ml/Kg/día
A. La galactosemia es una contraindicación B. 30-80 ml/Kg/día
B. El calostro tiene más contenido de grasas en C. 30-80 ml/Kg/4horas
comparación a la leche madura D. 80-100 ml/Kg/3horas
C. La leche madura se obtiene a partir del dia 14 E. 30-50 ml/Kg/12horas.
D. El contenido de Calcio es menor en la leche humana
en comparación a la leche de vaca 31. Mencione el dato de mal pronóstico en pancreatitis
E. La hierro de la leche materna es mejor absorbido aguda
que el de la leche de vaca. A. El índice de severidad tomografía más de 5 puntos
B. Secuestro de líquidos al ingreso
26. Es característica de la anestesia epidural: C. Hipocalcemia a las 48 horas
A. Atraviesa la duramadre D. Apache más de 7 puntos
B. Siempre ocasiona cefalea post punción E. Disminución de PCR
C. Es necesario siempre atravesar el ligamento amarillo
D. Es un tipo de anestesia general 32. El precursor metabólico inmediato de la
E. Puede causar hipertensión arterial noradrenalina es:
A. Isoproterenol.
27. Paciente varón joven con datos de trombosis B. Ibopamina.
venosa profunda, además se encuentra ictericia a C. Dobutamina.
predominio indirecto con coluria, la sospecha D. Dopamina.
diagnostica es: E. Efedrina.
A. Hemoglobinuria paroxística nocturna
B. Esferocitosis 33. Paciente de 68 años con disnea progresiva hasta en
C. Talasemias reposo. Examen físico: tórax amplio vibraciones
D. Hemoglobinopatías vocales dismi- nuidas, hipersonoridad, murmullo
E. Eliptocitosis vesicular disminuido, ingurgitación yugular y
disnea severa. Cuál es la actitud inmediata?
28. Mujer de 32 años de edad, con diagnóstico de A. Tomar radiografía de tórax
artritis reumatoide. ¿Cuál de los siguientes B. Evaluación por cirujano de tórax
hallazgos indica un buen pronóstico?: C. Drenaje torácico inmediato
A. Elevación de la velocidad de sedimentación D. Tomografía de pulmones
B. Títulos bajos de factor reumatoide E. Antibióticos y corticoides
C. Títulos elevados de haptoglobina
D. Presencia de nódulos subcutáneos 34. Marque lo falso respecto a los niveles de
E. Presencia erosiones en radiografía de manos prevención:
A. La prevención primordial evita la causación primaria
29. Paciente de 80años de edad, que presenta en de la enfermedad.
ambas extremidades ampollas grandes, tensas, B. La prevención primaria busca reducir la incidencia de
algunas con contenido sanguíneo, sobre una base enfermedad a través de pruebas de screening.
urticariana y con moderado prurito. No hay C. La prevención secundaria se sustenta en la utilidad
compromiso del estado general y la histopato- de las pruebas de screening.
logía muestra lesiones subdermicas con acumulo D. La prevención terciaria busca recuperar parte de la
de eosinofilos en la dermis papilar. La funcionabilidad previa.
inmunofluorescencia muestra depósito lineal de E. La prevención cuaternaria evita el sobrediagnostico
IgG y de C3. ¿Cuál es la dermatopatia más y sobretratamiento.
probable?
35. Con respecto a la convulsión febril, marque la A. Fase III / II
alternativa incorrecta: B. Fase V / II
A. La convulsión febril más frecuente es la simple C. Fase IV / III
B. Es el tipo de convulsión febril más frecuente en D. Fase IV / II
niños con una frecuencia de 3-5% en menores de 5 E. Fase I / IV
años
C. El que los padres hayan tenido en la niñez no apoya 41. Llega a consulta una madre con su hijo de 18 meses
el riesgo de que el niño lo tenga de edad quien refiere que solo recibió la primera
D. Generalmente se resuelven solas dosis de la Pen- tavalente. Con respecto a esta
E. Existe manejo preventivo de convulsión febril vacuna, ¿Qué le diría usted?
A. Ya no puede vacunarse por el riesgo de ESAVI
36. ¿Cuál es un principal criterio para endometritis? B. Debería completar con solo una segunda dosis
A. Fiebre en el primer día pureperio C. Completar con dos dosis más separadas por 1 mes
B. Dolor pélvico D. Debería reiniciar las tres dosis y darlos cada 2 meses
C. Secreción vaginal maloliente E. Completar con dos dosis más separadas por 2 meses.
D. Sangrado vaginal
E. Fiebre a partir del segundo día puerperio 42. En un paciente con glucosa en ayunas de 130
mg/dL y dos exámenes adicionales dudosos, con
un test de tolerancia a la glucosa en limite,
37. Paciente de 40 años de edad, que acude a la debemos solicitar:
consulta por sangrado postcoital de 2 semanas de A. Curva de insulina
evolución al examen pélvico:cérvix ulcerado B. Glucosa post prandial
hipertróficos, sangrante, poco móvil. Cuerpo C. Repetir el test de tolerancia a la glucosa 75
uterino 6cm, anexos no palpables. Al tacto rec- tal: D. Hemoglobina A1c
útero duro con parametrios tomados en sus 2/3 E. Electroforesis de hemoglobina.
internos. El diagnóstico clínico es:
A. Cáncer de cérvix IB 43. Respecto a los aneurismas cerebrales, es
B. Cáncer de cérvix IIIA INCORRECTO:
C. Cáncer in situ A. La mayoría corresponden al tipo sacular
D. Cáncer de cérvix IIB B. Es la principal causa de hemorragia subaracnoidea
E. Ectropión sangrante atraumática
C. La principal arteria donde se les puede encontrar es
38. Varón de 17 años, con procesos respiratorio viral, la arteria meníngea anterior
posteriormente presenta hematuria con hematíes D. La mayoría son de tipo congénito
dismorficos ade- más presenta disminución de E. La diplopía y la cefalea son síntomas característicos
complemento (C3 Y c4) ¿Cuál es su posibilidad
diagnostica? 44. Una madre con tipo de sangre AB tiene un hijo con
tipo AB. A ella le gustaría establecer la paternidad a
A. Glomerulonefritis por vasculitis
través del tipo sanguíneo. ¿Cuál tipo de sangre
B. Glomerulonefritis postinfecciosa
excluye a un varón de ser el padre biológico?
C. Glomerulonefritis membranoproliferativa
D. Glomerulonefritis rapidament progresiva
A. AA.
E. Nefropati ́a por IgA .
B. BB.
39. Señora que sufre caída y acude a consulta con C. BO.
miembro inferior en extensión y aducción, a la D. AO.
radiografía no se eviden- cia fractura ¿Cuál es el E. OO.
diagnóstico más probable?
A. Luxación pubiana de la cadera 45. El tipo de medicamento indicado para el
B. Luxación posterior de la cadera tratamiento de la Angina de Prizmetal es:
C. Luxación congénita de cadera A. Calcioantagonistas
D. Luxación obturatriz de cadera B. IECA
E. Luxofractura de cadera C. B- Bloqueantes
D. ARA II
40. En este estudio de evalúan los efectos adversos a E. Digoxina
largo plazo y las interacciones medicamentosas en
un gran número de pacientes / es un estudio previa 46. Paciente mujer quien se realiza una
a la comercialización del fármaco pero que ya colecistectomía laparoscópica electiva por
recibió la aprobación de la FDA para realizar antecedente de litiasis vesicular, acci- dentalmente
ensayos clínicos controlados: se lacera el duodeno, sin perforarlo. El tipo de
herida es: cuantificables de los valores pero no establece
A. Sucia ausencia de fenómeno.
B. Limpia contaminada D. La escala de razón muestra diferencias cuantificables
C. Limpia de los valores y establece ausencia de fenómeno.
D. Contaminada E. La escala de intervalo muestra diferencias
E. Infectada cuantificables de los valores y establece ausencia de
fenómeno.
47. Mujer de 28 años, hace 6 meses presenta malestar
general, Poliartritis de mano y disnea. Al examen 52. En un puesto de salud se han producido 2.800
físico: PA: 125/80 mmHg, FC: 100 x minuto, FR: 28 consultas médicas al año y se dispuso para el mismo
x minuto. Alopecia, palidez y adenopatías 400 horas-médico.
cervicales múltiples. Hemoglobina 7G/ dl, urea 65 ¿Cuál es el rendimiento?
mg/dl, creatinina 2.2 mg/dl. Examen de orina: A. 9
hematuria, cilindros hemáticos y proteínas (+), B. 5
ANA y anti- Smith (+). Rx de pulmones: lesión C. 6
homogénea en el tercio inferior del hemitórax D. 8
izquierda: ¿Con respecto al diagnós- tico señale lo E. 7
correcto?:
A. Mediado por una reacción de hipersensibilidad tipo 53. ¿Cuál es la característica que corresponde al
II sarcoma de Kaposi?
A. Es una neoplasia maligna de endotelio causada por
B. El rash malar es una manifestación aguda especi ́fica
HVS8
C. Asocia frecuentemente con síndrome de Sjogren
B. Las lesiones son dolorosas y pruriginosas causadas
D. Mejora con la gestación
por HVS8
E. Cuando presenta anticuerpo anticardiolipina es buen
C. Es un cáncer de endotelio por HVS1
pronóstico
D. Es típico de VIH causado por HVS5
48. Mujer de 25 años de edad, con gestación de 37 E. La etología es el VIH que causa cáncer de endotelio
semanas. Acude a Emergencia por presentar dolor
54. ¿Cuál es la alternativa que NO corresponde a la
abdominal soste- nido, de inico brusco y de
anemia ferropenia?
intensidad fuerte, con leve sangrado vaginal. Al
A. La respuesta al tratamiento se mide con reticulocitos
examen físico: PA 130/90 mmHg, LCF 160 por
al 10 día
minuto. ¿Cuál es el diagnóstico más probable?
B. La causa más frecuente es carencial en jóvenes
A. Placenta marginal
C. Para diferenciarla de la anemia por enfermedad
B. Ruptura uterina espontánea
crónica se mide transferrina
C. Ruptura del seno marginal
D. El test bioqui ́mico mas especifico es la ferritina
D. Desprendimiento prematuro de placenta
E. El hierro se absorbe solo el 10% de lo ingerido a nivel
E. Parto pretérmino
intestinal.
49. El tipo de sangrado más característico que
producen los leimiomas uterinos:
55. En el tratamiento de la hiperkalemia ¿Qué
A. Hipermenorrea
medicamento actúa produciendo disminución del
B. Metrorragia potasio corporal total?
C. Menorragia
D. Menometrorragia A. Gluconato de calcio
E. Amenorrea B. bicarbonato
C. Dextrosa hipertónica
50. ¿Cuál de las siguientes vacunas es de tipo D. Beta bloqueadores
inmunización activa tipo toxoide?
E. Resina Kayexalate
A. Disteferia y Tetanos
B. Sarampion, Paperas y Rubeola 56. Gestante de 38 años, con diagnóstico de placenta
C. Pentavalente previa. ¿Cuál es el factor de riesgo más asociado y
D. Antineumococica a partir de qué edad gestacional se diagnostica
E. BCG. dicha patología?
A. Cesárea – 28 semanas
51. Respecto a las escalas de medicina marque lo falso: B. Parto distocico – 26 semanas
A. Las variables en la escala nominal son excluyentes. C. Cesárea – 30 semanas
B. La escala ordinal presenta una diferencia de grado D. Parto distocico – 28 semanas
entre variables. E. Cesárea – 22 semanas
C. La escala de intervalo muestra diferencias
57. Un neonato presenta ictericia, dificultad conducta más apropiada a seguir?:
respiratoria, Tº inestable, distensión abdominal y
hepatomegalia. Diagnósti- co de presunción: A. Tratamiento cíclico con progesterona
A. Kernicterus. B. Histeroscopia
B. Meningitis neonatal. C. Histerectomía abdominal total
C. Enfermedad granulomatosa crónica. D. Esperar un nuevo control
D. Lupus neonatorum. E. Ablación endometrial
E. Sepsis neonatal.
64. En un escolar de 7 años con claudicación de la
58. ¿Cómo se define talla baja en pediatría?
marcha y dolor en la cadera, usted sospecharía en
A. P/T<3p la enfermedad de:
B. T/E<5p A. Sever
C. P/T<5p B. Osgood-Schlatter
D. T/E<3p C. Khole
E. P/E<10p D. Legg-Calve-Perther
E. Panner
59. Respecto a los tumores de esófago, marque lo
INCORRECTO
65. La elaboración del Plan Operativo de un
A. Los tumores benignos más frecuentes son los establecimiento de Salud tiene como una de sus
leiomiomas características que:
B. La acalasia crónica aumenta el riesgo A. La misión expresa lo que seremos en el futuro
C. El consumo de alcohol y tabaco aumenta el riesgo B. Promueve la participación del equipo de gestión
D. El tumor maligno más frecuente es el solamente
adenocarcinoma C. Se programan actividades a lo largo de un año
E. El cáncer más frecuente es el carcinoma D. Las fortalezas parten del entorno
epidermoide. E. Busca plantear problemas de la oferta para
resolverlos operativamente.
60. Paciente con ECV con hemiparesia derecha a
predominio braquial y afasia de expresión ¿qué
66. El antidepresivo con menor efecto sedante y el que
arteria está afectada? tiene mayor efecto sedante son respectivamente:
A. Cerebral media rama superior izquierda A. Amitriptilina / sertralina
B. Cerebral media rama superior derecha
B. Clomipramina / imipramina
C. Cerebral anterior rama superior izquierda
C. Doxepina / bupropion
D. Cerebral anterior rama superior derecha
D. Fluoxetina / paroxetina
E. Cerebral media rama inferior izquierda E. Trazodona / sertralina
61. Mide la desviación estándar de una media de
67. ¿Cuál es la frecuencia cardiaca fetal normal?
todas las muestras posibles de una población
infinita y se usa para la
A.60
inferencia por:
B.120
A. Error estándar de la media – intervalos de confianza
B. Desviación medial de la media – intervalos de C. 120 a 160
confianza D. 80 a 120
C. Error estándar de la media – prueba de hipotesis E. >180
D. Amplitud intercalar - prueba de hipotesis
E. Correlacion lineal – intervalos de confianza 68. Varón de 40 años que presenta en el brazo derecho
lesión máculo-papulosa de 3 cm, pruriginosa,
62. El impétigo ampolloso es causado por y debe ser indolora que progre- sa a vesícula y luego a costra
tratado con: negruzca rodeada de edema. Ocupación crianza de
A. Staphylococcus aureus coagulasa positivo / ganado lanar y vacuno. Niega fiebre. Examen:
mupirocina Pulso: 115 x minuto, FR: 24 x minuto, PA: 100/60
B. Streptococcus pneumoniae / dicloxacilina VO mmHg. ¿Cuál es la presunción diagnóstica y el
C. St. Pyogenes / clindamicina VO manejo adecuado?
D. Staphylococcus aureus coagulasa positivo / A. Carbunco / doxiciclina
dicloxacilina VO B. Leishmaniasis / nifurtimox
E. Pseudomonas aeruginosa / aztreonam VO C. Lactrodactismo / suero especifico
D. Leishmaniasis / antimonio pentavalente
63. Paciente de 30 años de edad, con hemorragia E. Carbunco / ciprofloxacino
uterina normal y biopsia endometrial informada
como hiperplasia sim- ple típica. ¿Cuál es la 69. La insuficiencia adrenal primaria se denomina y es
causada: convulsiva, no ha recu- perado la conciencia aun y
A. Síndrome de Cushing / falla de regulación central continua convulsionando, la madre del niño
B. Enfermedad de Addison / falla de la glándula describe cuadro compatible con convulsiones
C. Síndrome de Addison / falla de glándula tónico-clónicas generalizadas. ¿Cuál es el
D. Enfermedad de Addison / falla de regulación central diagnóstico más probable y el manejo inicial más
E. Síndrome de Addison / falla de regulación central. adecuado?
A. Estado convulsivo / Diazepam o lorazepam
70. El test de Apgar valora el estado vital del neonato B. Epilepsia / Diazepam o lorazepam
mediante la comprobación de los siguientes signos, C. Convulsión febril / fenitoina
EXCEPTO:
D. Estado convulsivo / fenitoina
A. Frecuencia cardiaca.
E. Convulsión tónico –clónica / fenobarbital
B. Movimientos respiratorios.
C. Respuesta a estímulos.
77. ¿Cuál es el parámetro más exacto para determinar
D. Reactividad osteotendinosa.
la edad gestacional?
E. Coloración de la piel.
A. Ecografía previa en el primer trimestre : LF
B. Ecografía previa en el primer trimestre: DBP
71. Respecto a las hernias inguinales marque lo
C. Fecha de inicio de náuseas y vómitos
incorrecto
A. Las hernias inguinales directas salen mediales a los D. Ecografía previa en el primer trimestre: LCC
vasos epigástricos inferiores E. FUR confiable
B. Para el tratamiento de las hernias inguinales directas
78. Paciente con pancreatitis aguda severa se colicita
se prefieren las técnicas sin tensión
TAC con contraste donde se informa edema de
C. Las hernias indirectas ocurren por persistencia del
conducto peritoneo vaginal páncreas con una colección de líquido por lo cual
D. Las hernias directas son generalmente congénitas según el índice de Baltazar corresponde a:
E. Las hernias indirectas recorren el canal inguinal A. A
B. B
72. Qué diámetro es el de mayor medida en el feto: C. C
A. Suboccipitobregmatico D. D
B. Suboccipitomentoniano E. E
C. Occipitofrontal
D. Bitrocantereo 79. ¿Clínica más frecuente de esclerodermia?
E. Bi acromial A. Fenómeno de Raynaud
B. Atrofia de piel
73. Agente etiológico del Chancroide y de la cervicitis C. Cianosis
respectivamente: D. Diarrea
A. Haemophilus vaginalis / clamidia trachomatis E. Microstomia
B. Calymmabacterium Granulomatosis / gonococo
C. Treponema pallidum / H. ducreyi
D. Chlamydia trachomatis / H. ducreyi 80. Paciente varón de 75 años, con dolor abdominal
E. Hemophilus ducrey / C. trachomatis intenso desde hace 8 horas, distensión abdominal
marcada y vómitos biliosos. En la radiografía se
74. El síndrome de Cushing se refiere a: evidencia distensión de asa intestinal. El
A. Excesiva producción adrenal de glucocorticoides diagnóstico más probable es:
B. Al exceso de glucocorticoides producidos A. Hernia inguinal incarcerada
C. Excesiva exposición multiorganica a glucocorticoides B. Vólvulo de ciego
D. Excesiva estimulación de las adrenales por la ACTH C. Diverticulitis aguda
E. Excesiva actividad glucocorticoide D. Vólvulo de sigmoides
E. Malrotación intestina
75. El motivo por el cual la sulfonilureas deben ser
usadas con cuidado en ancianos y enfermos renales 81. El modelo de atención de salud tiene como
es: principio fundamental y la estrategia principal es la
A. Pueden causar acidosis láctica :
B. Hipoglicemia A. Calidad / APS
C. Hiperkalemia B. Integridad / APS
D. Agitación C. Eficacia / MAIS
E. Daño hepático D. Universalidad / MAIS
E. Solidaridad / APS
76. Se recibe en Emergencia a un niño de 5 años de
edad, febril, con 15 minutos continuos de crisis 82. De las siguientes alternativas, la conducta de
tratamiento más adecuado para el trauma de presento un cuadro de resfrió y que ahora presenta
uretra posterior es: esta molestia. Al examen presenta una T 38.5°C,
A. Colocación de sonda Foley uretral orofaringe congestiva sin exudado, oído derecho
B. Talla vesical con tímpano eritematomoso, congestivo,
C. Reparación primaria de la uretra abombado y nivel hidroaereo, pulmones sin
D. Ureterostomía alteraciones. Según su diagnóstico,
E. Nefrostomi ́a ¿Qué le indicaría?
83. Se desea estudiar un factor de riesgo asociado a la A. Amoxicilina
prostatitis, se nota que la población presenta B. Cefuroxima
varios factores con- fusores pero no pueden ser C. Ibuprofeno
considerados dentro de los criterios de exclusión y D. Observación
deben muestrear a todos. Para obtener una E. Azitromicina.
muestra representativa se debe optar por el
muestreo: 88. Gestante de 38 semanas con PA: 160/110 y
A. Estratificado Albuminuria ( ++++) ¿Cuál es la alternativa
B. Aleatorio simple con restitución terapéutica más apropiada?
C. Aleatorio simple sin restitución A. Sulfato de Magnesio 6gr Ev en bolo y 2gr cada hora
D. Sistemático B. Betametasona 12 mg EV c/12 horas
E. Por conglomerado C. Misoprostol 200 ug via vaginal cada 6 horas
D. Sulfato de Magnesio 2gr IM en cada glúteo
84. Paciente usuaria de DIU, asintomática, con E. Sulfato de Magnesio 1gr Ev en bolo y 1gr cada hora
ecografía que reporta gestación intrautero de 10 89. Anciana llega a la consulta con dolor
semanas, Cola de DIU visible al examen con postraumático de hombro derecho, con limitación
especulo. La conducta a seguir es: de movimiento. Examen clíni- co: hematoma en
A. Control del embarazo más antibiótico cara interna del tercio superior del brazo. ¿Cuál es
B. Control del embarazo más progesterona el diagnóstico más probable?
C. Retiro de DIU más antibióticos A. Luxación escapulohumeral
D. Retiro de DIU más progesterona B. Fractura de clavícula
E. Retiro de DIU y observación C. Luxación acromioclavicular
D. Fractura de escápula
85. ¿A qué edad aproximadamente empieza a salir la
E. Fractura del cuello del húmero
primera dentición?
90. ¿Qué se desarrolla en la administración de un
A. 4 meses
establecimiento nivel I, al relacionar las
B. 5 meses
actividades de sus diversas unidades para
C. 6 meses
alcanzar las metas propuestas?
D. 7 meses
A. Negociación
E. 8 meses
B. Organización
C. Coordinación
D. Motivación
86. Un prematuro de 32 semanas de edad; a la primera
E. Planificación
hora de vida presenta una disnea progresiva con
cianosis y tiraje. En la radiografía hay un patrón de
91. Infante con hipoacusia cursa con prueba de Rinne
vidrio esmerilado y broncograma aéreo. A pesar de
negativo, además en la otoscopia se menciona
la ventilación, el oxígeno y los antibióticos,
membrana timpánica con niveles hidroaéreos.
continúa mal. ¿Qué terapéutica añadiría en primer
A. Otitis media aguda
término?
B. Otitis media crónica
C. Otitis externa difusa
A. Surfactante endotraqueal.
D. Otitis secretora
B. Indometacina oral.
E. Otitis externa maligna.
C. Prednisona intravenosa.
D. Bicarbonato intravenoso.
92. Las mácula densa se encuentran en:
E. Glucosa intravenosa.
A. La arteria aferente
B. La arteria eferente
C. Túbulo contorneado distal
87. Durante la noche en una guardia de pediatría llega
D. Asa de Henle
a consulta un paciente varón de 20 meses quien
E. Capsula de Bowman
presenta aguda- mente un cuadro de dolor de oído
leve a moderado, mama refiere que días antes
93. ¿Cuál es la contraindicación absoluta para el
tratamiento hormonal en post menopausia? intercostales y supraesternales; los ruidos
A. Ginecorragia de origen desconocido. respiratorios están disminuidos y hay sibilancias
B. Varices superficiales bilaterales. La intervención inicial más apropiada
C. Bochornos es:
D. Dislipidemia controlada A. Administración de aminofilina intravenosa.
E. HTA controlada. B. Solicitar radiografía torácica.
C. Prescribir la nebulización de cromoglicato sódico.
94. Cuál de las siguientes es un material de sutura D. Pedir biometría hemática completa y hemocultivo.
absorbible: E. Prescribir salbutamol nebulizado.
A. Nylon
B. Polidioxanona
C. Polipropileno 100. La trompa de Eustaquio y la mucosa del oído medio
D. Seda se originan en:
E. Acero quirúrgico A. La cuarta bolsa faríngea
B. El neuroectodermo notocordal
95. ¿Cuál es la profundidad a la cual se debe comprimir C. El primer arco branquial
el torax en un lactante en el RCP? D. La cuerda del tímpano
A. 3cm E. La primera bolsa faríngea
B. 4cm 101. Son agentes patógenos de neumonía adquirida en
la comunidad los siguientes, excepto:
A. Streptococcus pneumoniae
C. 5cm
B. Acinetobacter baumanii
D. 6cm
C. Mycoplasma pneumoniae
E. 7cm.
D. Hemophilus influenzae
E. Legionella pneumophila
96. Bebé de 30 horas de edad, nació con 4300 g, hijo de
102. ¿Cuál de las siguientes no es característica de
madre con diabetes gestacional, ha tomado bien su
convulsion febril simple?
alimento, pero se ve muy inquieto a los 30 minutos
A. Duración menor de 15 minutos
de vida. 15 minutos más tarde sufre una
B. 2 o más en 24 horas
convulsión tónico-clónica. Diagnóstico más
C. No dejar déficit motor
probable:
D. Convulsion Tonicoclonico generalizado
A. Hipoglucemia
E. Convulsion asociado a fiebre
B. Hipocalcemia
C. Hipomagnesemia
D. Hiponatremia 103. Mujer de 30 años, acude por presentar
E. Hiperviscosidad Poliartrálgias y pápulas eritematosas aplanadas
sobre los nudillos de ambas manos (signo de
Gottron) y telangiectasias periungüeales. ¿Cuál es
97. El grado de afectación fetal en la isoinmunización
lo correcto con respecto al diagnóstico?:
Rh se aprecia con mayor exactitud por medio de:
A. Existe dolor muscular
A. Test de Coombs indirecto.
B. Asocia con esclerodermia
B. Test de Coombs directo.
C. Para el diagnostico se usa biopsia muscular
C. Niveles de b-HCG en li ́quido amniótico.
D. Presencia del anti jo 1 indica buen pronostico
D. Medición de los niveles de bilirrubina en el líquido
E. Asocia con panarteritis nodosa
amniótico-curvo de Liley.
E. Test de Coombs y b-HCG.
104. Es la relación que existe entre los resultados con
respecto a los recursos
98. La porción mayor del estómago está constituida
A. Eficiencia
por:
B. Efectividad
A. La unión gastroesofágica.
C. Eficacia
B. El cardias.
D. Costo – Beneficio
C. El fondo.
E. Aceptación.
D. El cuerpo.
E. El antro.
105. Mujer de 20 añ os, con cesárea por inducció n
fallida. Al tercer diá de puerperio presenta
99. Una niña de 9 años de edad tiene una historia de
malestar general, fiebre y loquios fétidos. Al
sibilancias intermitentes de varios años; no ha
Examen clínico: T: 39°C, ú tero subinvolucionado y
recibido medicamen- tos hace algún tiempo. Se
doloroso a la palpación, secreción fétida. ¿Cuál es
halla febril, agitada y con cianosis peribucal. Tirajes
el diagnóstico más probable? 110. Con respecto a la HTA señale lo correcto
A. Dehiscencia de la histerorrafia A. Principal factor de riesgo para hemorragia
B. Endometritis puerperal subaracnoidea
C. Absceso de pared abdominal B. La hipertensión maligna genera necrosis fibrinoide
D. Retención de restos placentarios C. La hipertensión produce lesión de arterias de gran
E. Necrosis isquémica de mioma uterino calibre
D. La estenosis arteriolar corresponde a la retinopati ́a
106. ¿Cuáles son los tumores primarios que más grado II
frecuentemente producen metástasis cerebrales y E. Produce dilatación ventricular izquierda
cuáles son los tumores primarios más frecuentes
en adultos? 111. Gestante de 38 semanas, acude a Emergencia por
A. Cáncer de cuello uterino y de próstata // presentar dolor abdominal intenso hace 4 horas
astrocitoma grado II acompañado de sangrado vaginal rojo vinoso.
B. Linfoma y carcinoma gástrico / glioblastoma Examen físico: PA: 140/90 mmHg, LCF: 128 x’, AU:
multiforme 37 cm, hipertonía uterina y se con- firma sangrado
C. Cáncer de pulmón y de mama // astrocitoma grado de cavidad uterina ¿Cuál es el diagnó stico más
IV probable?
A. Desprendimiento prematuro de placenta
D. Cáncer gástrico y de páncreas // meduloblastoma
B. Placenta previa
E. Cáncer de pulmón y de mama // metastasis
C. Ruptura uterina
107. Primigesta añosa de 34 semanas de gestación no D. Vasa previa
controlada, acude por presentar cefalea, tinnitus, E. Inversión uterina
́
dolor epigástrico. Examen fisico: PA: 150/90 mm
112. Si el cuadro clínico de asma bronquial cursa con
Hg, FC: 90 x ́, FR: 20 x ́, T: 36.5°C, edema en
sibilancias, tos, disnea menos de 2 veces por
miembros inferiores. Altura uterina: 30 cm, Feto
semana, exacerbaciones breves (desde horas a
LCI, FCF:140 x ́. Exámenes de Laboratorio: Hb:10
dias), síntomas nocturnos mayores de una vez por
g/dL, plaquetas 110,000/mm3, Creatinina: 1
semana, asintomática entre exacerbaciones.
mg/dL, TGO: 70 UI/L, LDH:1200 UI/L, Bilirrubina:1.4
¿Qué clasificación de gravedad le corresponde?
mg/dL a predominio indirecto. ¿Cuál es el
A. Intermitente
diagnóstico más probable?
A. Si ́ndrome HELLP B. Persistente leve
B. Preeclampsia severa C. Persistente moderado
C. Hipertensión gestacional D. Persistente grave
D. Hipertensión crónica E. Intermitente moderado.
E. Eclampsia
113. Acude a consulta un paciente de 6 meses de edad
108. ¿Cuáles son los agentes terapéuticos de uso que en sus 3 primeros días de enfermedad presenta
frecuente en la epilepsia y convulsiones parciales y cuadro catarral, luego aumenta su frecuencia
en las generalizadas respectivamente? respiratoria por lo que acude a emergencia donde
A. Lamotrigina, fenobarbital // carbamazepina se evidencia taquipnea, tirajes, sibi- lantes. ¿Cuál
B. Etosuximida, ácido valproico, lamotrigina // es el agente etiológico más probable?
lamotrigina A. Adenovirus
C. Carbamazepina, fenilhidantoína, gabapentina // B. Micoplasma
Ácido valproico, lamotrigina C. Penumococo
D. Primidona, topiromato // Carbamazepina, D. Virus sincitial respiratorio
Lamotrigina E. Rinovirus
E. Zonisanida, ácido valproico // Carbamazepina,
Lamotrigina 114. ¿Cuál es la alternativa que NO corresponde a la
anemia ferropenia?
109. RN con hepatoesplenomegalia, rinitis purulenta, A. La respuesta al tratamiento se mide con reticulocitos
anemia, rash maculopapular y descamación de al 10 dia
Palmas y plantas. Cuál es el diagnóstico más B. La causa más frecuente es carencial en jovenes
probable. C. Para diferenciarla de la anemia por enfermedad
A. CMV crónica se mide transferrina
B. Sífilis congénita D. El test bioquimico mas especifico es la ferritina
C. Toxoplasmosis E. El hierro se absorbe solo el 10% de lo ingerido a nivel
D. Rubeola intestinal.
E. Herpes
115. Adolescente de 15 años, con 32 semanas de
gestación , sin control prenatal, acude a A. Digoxina
emergencia por dolor epigástrico y cefalea. Al B. Atenolol
examen impresiona como embarazo de 6 meses C. Verapamilo
por altura uterina. PA: 160/110mmHg, Proteinuria D. Adenosina
(+++). El diagnostico más probable es : E. Maniobras vagales

A. Eclampsia 121. Gestante de 8 semanas acude por sangrado vaginal


B. Hipertension transitoria profuso desde hace 3 horas, acompañado de dolor
C. Pre-eclampsia severa abdominal intenso. Examen físico: PA: 80/60
D. Pre-eclampsia superpuesta mmHg, FC: 98 x’, FR: 18 x’. Especuloscopía: cérvix
E. Sindrome de HELLP con sangrado y coágulos abun- dantes. Tacto
vaginal: orificio externo e interno abiertos. ¿Cuál
116. Llega a emergencia un lactante de 3 meses quien 4 es el diagnó stico más probable?
días antes de su ingreso desarrolla rinorrea y A. Aborto inevitable
estornudos, ahora tiene dificultad para respirar, B. Aborto frustro
taquipena y a la auscultació n subcrepitos. Según su C. Aborto inminente
sospecha diagnostica, ¿Cuál es el manejo más D. Amenaza de aborto
indicado? E. Aborto diferido
A. Iniciar amoxicilina
B. Brindar terapia de rescate con beta-agonistas 122. Mencione lo que no corresponde al score de forrest
C. Dar broncodilatadores más corticoide para hemorragia digestiva alta
D. Iniciar macrolidos A. El tipo Ia tiene alta tasa de mortalidad
E. Nebulización con suero hipertónico. B. El tipo IIb es coagulo adherido
C. El tipo III es sangrado inactivo
117. Varón de 56 años, que presenta pérdida de peso de D. El tipo Ib requiere tratamiento endoscópico más
6 kg en el ú ltimo año, cambios de la voz, antiacidos
incremento del volumen de la glándula tiroides a E. El tipo IIc es ulcera con fibrina
predominio derecho. No adenomegalias y pruebas
de función tiroidea normales. En el preope- ratorio, 123. ¿En qué mecanismo del trabajo de parto se
¿qué examen complementario de tiroides produce cambio de la variedad de posicion?
solicitaría? A. Expulsión
A. Aspiración con trocar B. Rotación interna
B. Gammagrafi ́a C. Descenso
C. Ultrasonografi ́a con biopsia D. Extensión
D. Resonancia magnética E. Flexion
E. Tomografi ́a
124. Paciente programado para cirugía de apendicitis
118. Paciente proviene de Piura, con fiebre de menos de congestiva, en sala se produce mínima lesión del
́ de duració n, sin afecció n de las vias
5 dias ́ aéreas colon. Como lo
superiores con clasificaría.
cefalea y mioartalgias. ¿Cuál es el diagnóstico más A. Limpia
probable? B. Sucia
C. Limpia contaminada
A. A.Dengue sospechoso.
D. Sucia contaminada
B. B.Probable dengue grave
C. C.Probable dengue hemorrágico E. Contaminada
D. D.Dengue confirmado
E. E.Dengue descartado 125. El diagnóstico bioquímico del empiema pleural se
realiza sobre la base de los siguientes criterios,
119. Inmunoglobulina aumentada en purpura de excepto:
Henoch Schonlein A. Triglicéridos mayor de 100
A. Ig G B. Glicemia menor de 40 mg%
B. Ig E C. pH menor de 7.2
C. Ig M D. protei ́nas mayor de 3g%
D. Ig D E. Gran celularidad a predominio de
E. Ig A polimorfonucleares

120. Paciente mujer de 20 añ os con palpitaciones de 126. Señale lo correcto respecto a la menopausia:
inicio y fin súbito, con PA normal, la primera medida A. Los niveles de FSH se elevan antes que los trastornos
terapuetica es: del ciclo menstrual.
B. El estradiol es la hormona predominante. pleural tipo trasudado?
C. La progesterona se eleva tras la menopausia. A. Neumoni ́a bacteriana
D. El climeterio es más intenso en los primero 10 años B. TBC pleural
anteriores al diagnóstico. C. Cirrosis hepática
E. La terapia de reemplazo hormonal es lo más D. Metástasis pleural
importante en todas las menopausicas. E. Mesotelioma .

127. Lactante de 8 días de nacido con distensión 134. En el Perú actual, el cuidado de la salud está en los
abdominal y vómitos biliosos abundantes, signo de primeros lugares de la agenda política del Acuerdo
la doble burbuja. Nacional, al mismo tiempo el Ministerio de Salud
A. Páncreas tubular sólo ha podido desarrollar precariamente su rol
B. Estenosis hipertrófica de piloro rector y conductor sensorial. La combinació n de
C. Vólvulo de ciego estos dos factores en una matriz FODA permite
D. Atresia duodenal identificar la siguiente estrategia:
E. Bridas A. Desafi ́o
B. Limitación
128. ¿Con que prueba se realiza el diagnostico de asma C. Potencialidad
bronquial en un niño mayor de 5 años? D. Restricción
A. Espirometria simple E. Riesgo.
B. Flujometro
C. Espirometria forzada 135. Paciente varon joven con datos de trombosis
D. Pletismografia venosa profunda, además se encuentra ictericia a
E. Radiografia de torax predominio indirecto con coluria, la sospecha
diagnostica es:
129. Paciente con IMA que presenta crepitantes en A. Hemoglobinuria paroxi ́stica nocturna
ambos campos pulmonares signos de ICC severa se B. Esferocitosis
clasifica como C. Talasemias
A. Killip I D. Hemoglobinopati ́as
B. Killip II E. Eliptocitosis
C. Killip III
D. Killip IV 136. ¿Cuál es el diagnostico nutricional de un lactante
E. Forrester III de 11 meses que tiene P/T 94%, T/E 94%?
A. Desnutrición aguda
130. Paciente con disfagia motora más pérdida de peso, B. Desnutrición crónica
ante la sospecha diagnostica quien confirma la C. Desnutrición crónica reagudizada
etiología
D. Desnutrición global
A. Ph metria de 14 hrs
E. Eutrófico
B. Endoscopia alta
C. Impedanciometria 137. El Caso índice o caso cero Corresponde a:
D. Manometri ́a esofagica A. Primer caso ocurrido en una epidemia.
E. Test de secretina B. Primer caso reportado al sistema de salud.
C. Primer caso muerto.
131. En relación con la etiología del RCIU II, señale factor
D. Primer caso tratado.
más frecuente
E. Primer caso publicado.
A. Enfermedad hipertensiva
B. Rubéola 138. Paciente de 15 años, con náuseas, vómitos, boca
seca, y somnolencia, PA 60/40 IMC 17KG/M2 ,
C. Alcoholismo
glicemia 450mg/dl; pH
D. Drogadicción 7,15; HCO3 12mEq/kg , la terapia indicada
E. Tabaquismo comprende fluidos más:
A. Insulina cristalina en bolos
132. ¿Cuál es el vector del ciclo de transmisió n selvático B. Insulina cristalina por vi ́a SC
de la fiebre amarilla en la América del sur? C. Insulina cristalina por infusión
A. Mosquitos de genero Haemagogus D. Insulina cristalina por vi ́a IM
B. Mosquitos de genero Aedes E. Insulina intermedia por vi ́a SC
C. Mosquitos de genero Crotalus
D. Mosquitos de genero Phlebotomus 139. Respecto a los ACO señale lo falso:
E. Mosquitos de genero Vivax A. Si se dan gestàgenos solos el efecto anticonceptivo
disminuye.
133. ¿Cuál de las siguientes entidades causa derrame B. Si se dan progestágenos solos puede ocurrir atrofia
endometrial. B. 19a29años
C. El principal efecto anticonceptivo se obtiene a nivel C. 20 a 29 años
central hipotalámico. D. 19a30años
D. Los progestágenos de tercera generación tienen E. 20a30años
buen perfil lipídico.
E. Los gestagenos en depósito son de elección en 145. Adulto joven en cuyo perfil hepático solo se
adolescentes y nuli ́paras. encuetra alterado la bilirrubina que esta
aumentado ligeramente a predo- minio indirecto,
la sospecha diagnostica es
140. Con respecto a la angina inestable señale lo A. Si ́ndrome de rotor
verdadero B. Si ́ndrome de dubin jonson
C. Si ́ndrome de Gilbert
A. La etiología más frecuente es la placa de ateroma D. Sindrome de clijer najar tipo I
con capa fibrosa gruesa E. Sindrome de clijer najar tipo II
B. El factor de riesgo más importante es la hipertensión
arterial 146. Varó n de 45añ os de edad, que presenta desde hace
C. Las mujeres tienen más riesgo en comparación con 12 meses de manera progresiva dificultad para la
varones marcha. Al exa- men se evidencia hipomimia facial,
D. Una variante cli ́nica es la angina por vasoespasmo lentitud para iniciar la marcha, dificultad para atar
coronario los cordones de sus zapatos, temblor fino en
E. Presenta elevación de CPK MB miembro superior derecho y resistencia a la flexo-
extensió n pasiva de dicho miembro. La sospecha
141. Niñ o con alopecia, lesió n en piel (acrodermatitis), diagnóstica es ------ y el manejo adecuado es -------
diarrea, deficiencia de que vitamina: -:
A. Zinc A. Alzheimer / memantina
B. Niacina B. Huntington / levodopa y carbidopa
C. Tiamina C. Parkinsonismo / levodopa y memantina
D. Magnesio D. Enfermedad de Parkinson / levodopa y carbidopa
E. Hierro E. Wilson / EDTA.
142. Paciente varón de 25 años, politraumatizado en 147. Acude a emergencia un paciente con estridor que
accidente de tránsito. Al examen neurológico: días antes presento clínica de rinorrea, estornudos
apertura ocular al do- lor, emite sonidos y sensación de
incomprensibles y extiende el miembro superior al alza térmica, ahora presenta dicho estridor y tos
dolor. De acuerdo a la escala de Glasgow, cuál sería perruna. Al evaluarlo se le clasifica como leve.
la conducta inmediata a seguir: ¿Cuál es su manejo?
A. Observación
B. Intubación orotraqueal A. Antipiréticos
C. Craneotomi ́a B. Nebulización con adrenalina
D. Traqueotomi ́a C. Nebulización con suero hipertónico
E. TAC D. Dexametasona endovenoso
E. Dexametasona vi ́a oral
143. Mujer de 40 años de edad, presenta
predominantemente en la región nasogeniana 148. Fractura conminuta no desplazada del primer MTC:
legiones papulosas eritomatosas y descamaticas, A. Benett
las cuales se exacerban con alguna comida y B. Rolando
tensión emocional. el diagnóstico más probable es C. Jones
y el manejo adecuado debería ser: D. Murphy
A. Rosácea / minociclina E. Mallampati
B. Acné / retinoides
C. Rosácea / metronidazol topico 149. En la pubarquia, de acuerdo con la clasificació n de
D. Acné / metronidazol oral Tanner, si se encuentra vello pubiano que extiende
E. Psoriasis / retinoides o metronidazol solo en el trián- gulo pubiano, corresponde al
estadio:
144. La nueva estructura de etapas del ciclo de vida A. VP1
segú n el modelo de atención integral de salud B. VP2
incluye a la etapa joven. C. VP3
¿Cuál es el rango de edad que le corresponde? D. VP4
A. 18a29años E. VP5
150. Paciente varón de 18 años, quien ingresa a C. La sutura occipital y el promontorio.
emergencia con dolor abdominal desde hace 7 D. La sutura sagital con el promontorio y el pubis.
horas, localizado en fosa iliaca derecha y náuseas, E. La sutura sagital y las espinas ciáticas.
afebril. Dolor en punto de Mc Burney positivo.
Leucocitosis y PCR levemente aumentado. De 156. Multigesta de 10 sem con sangrado vaginal y dolor
acuerdo al diagnóstico más probable, señale lo abdominal, altura uterina 18 cm orificio cervical
correcto: entreabierto,.
Además presenta hemoptisis ¿Cuál es el
A. La etiologi ́a es básicamente la perforación de la diagnóstico?
vi ́scera A. Mola parcial
B. El germen que se puede identificar con más B. Mola completa
probabilidad es el Bacteroides fragilis C. Aborto molar
C. El tratamiento antibiótico requiere Amikacina + D. Degeneración hidrópica
Metronidazol por una semana E. Coriocarcinoma
D. La causa más frecuente es la hipertrofia de las placas
de Peyer.
157. En el embarazo normal, la altura del fondo
E. El germen más frecuente es E. coli
uterino se vuelve extrapelvico durante la semana
…………………y se considera un síntoma…………..de
151. El establecimiento de sector salud de categoría I-2
diagnóstico de embarazo
corresponde a:
A. 12 - probable
A. Puesto de salud con médico
B. 16 - presuntivo
B. Puesto de salud sin médico
C. 18 - certeza
C. Puesto de salud con 10 médicos
D. 10 - presuntivo
D. Puesto de salud con hospitalizacion
E. 20 – certeza
E. Puesto de salud con quirófano.
158. ¿Cuál es la neoplasia más frecuente en pediatría?
152. ¿A qué edad aproximadamente un niño duplica su
A. Leucemias
talla de nacimiento?
B. Linfomas
A. 1 año
C. Tumor de willms
B. 2 años
D. Astrocitoma
C. 3 años
E. Meduloblastoma.
D. 4 años
E. 5 años
159. En la prueba de hipótesis, el investigador comete
un error tipo II o beta cuando:
153. Paciente con epistaxis posterior mencione lo
A. No establece el nivel de significancia
verdadero
B. No rechaza la hipótesis nula siendo falsa
A. Lesión del plexo de kiesellbach
C. Rechaza la hipótesis alterna, siendo falsa
B. Fácil de controlar
D. Rechaza la hipótesis alterna, siendo verdadera
C. Resuelve con taponamiento anterior
E. Rechaza la hipótesis nula siendo verdadera.
D. La arteria esfenopalatina es rama de la maxilar
interna
160. ¿Cuál es la vacuna que no debe darse en alérgicos a
E. La causa más frecuente es infecciones.
la proteína del huevo?
A. BCG
154. Paciente de 40 años de edad que presenta
B. Antineumococica
necesidad imperiosa de miccionar y se le escapa
C. Antiamarilica
la orina antes de llegar al baño, ya que no puede
D. VPH
retenerla. Al examen se constata uretrocistocele
E. Rotavirus
de primer grado. El diagnóstico es:
A. Incontinencia urinaria de esfuerzo
161. Varón de 50 años de edad, que ingresa al Hospital
B. Incontinencia urinaria de urgencia
por presentar un cuadro de disnea, palidez de piel
C. Infección de vi ́as urinarias
y mucosas. Al examen clínico: PA 150/90 mmHg. Se
D. Cistitis intersticial
detecta anasarca y signos de derrame pleural
E. Cistitis aguda.
derecho. Exámenes de laboratorio: hemoglobina
9g/dL, riñones con pérdida de la diferenciación
155. La denominació n sinclitismo y asinclitismo se
corticomedular ¿Cuál es el diagnóstico más
refiere a la relació n entre:
probable?:
A. La sutura interparietal y las tuberosidades A. Si ́ndrome nefrótico
izquierdas. B. Si ́ndrome nefri ́tico
B. La sutura metódica y el promontorio. C. Glomerulonefritis aguda
D. Nefritis aguda A. Restricción de agua libre
E. Insuficiencia renal crónica B. Restricción de sodio
C. Administración de sodio
162. ¿Cuál de los siguientes es el mejor parámetro para D. Uso de diuréticos de asa
evaluar el crecimiento de un niño? E. Uso de resina de intercambio iónico
A. IMC en valores absolutos
B. Una sola visita en su vida 168. Señale que cambios ocurren en el embarazo normal
C. El seguimiento con las curvas de crecimiento A. Disminución del tamaño renal
D. Con los datos del nacimiento B. Dilatación uretereal bilateral
E. Evaluación psicológica C. Disminución del índice de filtración glomerular
D. Aumento del índice de flujo plasmático renal
163. ́
El indice ecográfico que mejor identifica el retardo E. Pronunciada proteinuria
del crecimiento fetal es…………., y para evaluar sus
pronostico se 169. ¿Cuál es la secuencia lógica en el proceso
utiliza la ecografía doppler de la arteria administrativo?
A. La longitud del fémur - uterina A. Planificación, organización, dirección y control.
B.La circunferencia abdominal - umbilical B. Planificación dirección, organización, y control
C.La longitud del fémur – cerebral media C. organización, dirección control Planificación
D.La circunferencia abdominal – ductus venoso D. Planificación, control organización, dirección
E.La longitud del fémur – cerebral media E. dirección Planificación, organización, y control.

164. Varón de 65 años, con T: 38°C, náuseas, vómitos y 170. RN con antecedente de DIPII y líquido amniótico
deposiciones líquidas con moco, sangre y meconial, nace hipotónico, apnea y bradicardico.
tenesmo. Estando hospi- talizado presenta una Qué medida to- maría.
convulsión por primera vez. ¿Cuál es el agente A. Aspiración de secreciones
causal? B. Estimulación táctil y aspiración de secreciones
A. Entamoeba histoli ́tica C. VPP
B. Salmonella D. VPP con mascarilla
C. Escherichia colie E. Ventilación mecánica
D. Clostridiun perfingens
E. Shiguella biflexa. 171. En varones adultos con anemia ferropénica ¿con
que prueba monitorizamos la respuesta al
165. Es un complicación tardía de la traqueotomía tratamiento?
A. Estenosis lari ́ngea
B. Hemorragia A. Hemoglobina a los 20 dias
C. Neumotórax B. HTO a los 15 dias
D. Atelectasia pulmonar C. Reticulocitos a los 10 dias
E. Enfisema subcutáneo D. Reticulocitos a los 5 dias
E. Ferritina a los 30 dias
166. Niño de 2 años de edad, que desde los 9 meses
presenta episodios de lesiones eritematosas 172. ́
¿Cuál es la caracteristica del flujo vaginal en la
descamativas en mejillas, abdomen y parte Vaginosis bacteriana?
extensoras de miembros, con prurito intenso. A. Seco y espumoso
Actualmente aparecen lesiones similares después B. Grumoso y blanquecino
de comer maní. Al examen se evidencia pliegues C. Fétido y grisáceo
redundantes infraorbitarios. ¿Qué tipo de D. Espeso y blanquecino
dermatitis consideraría en su presunción E. Espumosos y sanguinolento.
diagnóstica?:
A. Por ingesta de alimentos 173. ¿Cuál es el principal mecanismo de la ovulación a
B. Seborreica nivel hormonal
C. Alérgica de contacto A. El pico de LH aislado.
D. Eccematoide infecciosa B. El pico de FSH aislado.
E. Atópica C. El pico de Estradiol y FSH.
D. El pico de Estradiol y LH.
E. El pico de Progesterona aislado.
167. En el tratamiento de la hiponatremia en un
paciente con insuficiencia renal aguda oligúrica, la 174. ́
Recién nacido con fibrosis quistica, cual es la
medida terapéutica manifestació n más relacionada:
más apropiada es: A. Ileo meconial
B. Policitemia
C. Hipoglicemia 180. ¿Cuál de las siguientes medidas de tendencia
D. Dermatitis crónica central es la más usada para el análisis estadístico
E. Neumoni ́a si los datos son asi- metricos?
A. La mediana
175. ¿Qué porcentaje del área bajo la curva normal B. La media
estándar se localiza entre +2 desviaciones C. La moda
estándares de la media? D. La desviación estándar
A. 58,6 E. La varianza

B. 95,4 181. Según la clasificació n de Bormann para cáncer


gástrico, las lesiones ulceradas infiltrantes
C. 68,3
corresponden al tipo:
D. 90,4 A. 5
B. 4
E. 99,9 C. 3
D. 2
E. 1
176. Las fracturas que se producen por
microtraumatismos, ya sea por intensidad o por
182. Varó n de 23 añ os de edad: refiere fácil sangrado de
repetición que alteran las propie-
las enciaś al uso del cepillo dental. Al examen;
dades fisicoquímicas del hueso, se denominan y la
más frecuente es la fractura : palidez y mucosas, ausencia de gingivitis; punta de
bazo palpable. Hematocrito 31%, leucocitos
A. Por fatiga / del recluta 2.300/dL, plaquetas 100.000/dL. Frotis de sangre
B. Patológicas / colles periférica: cuerpos de Auer. El diagnóstico
C. Por fatiga / cargador presuntivo es:
D. Osteoporoticas / colles A. Leucemia linfoide aguda
E. Patologicas / metatarsiano 2 B. Leucemia mieloide aguda
C. Leucemia mieloide crónica
177. Marque lo falso respecto a las siguientes patologías
D. Mielofibrosis
neurológicas:
A. La demencia más frecuente es la demencia de E. Mononucleosis infecciosa
Alzheimer.
B. La demencia de Alzheimer es 183. Paciente de 78 años con te: 2 días, presenta
predominantemente subcortical. deposiciones líquidas 3 v/día, volumen de ½taza
C. El nivel de lesión mesencefalico se evalúa con la por vez, s/s, con pa: 100/60 mmHg, p: 100x’, fr:
rigidez de decorticacion o descerebración. 22x’; presión ocular disminuida. Además tiene Na:
D. El tumor más frecuente en el SNC es metastasico 120 mmol/l, k: 3,5 mmol/l, cl: 100 mmol/l.
de pulmón. respecto al estado hidroelectroliticó de la
E. El tumor pilocitico infantil es frecuente en paciente, podemos afirmar que presenta:
cerebelo. A. Hiponatremia Hiperosmolar.
B. SIADH
178. Tratamiento para crisis de agitación psicomotriz: C. Hiponatremia con hipovolemia
A. Haloperidol D. Hiponatremia con normokalemia
B. Benzodiazepinas E. Hiponatremia con hipovolemia
C. Fenitoina
D. Carbamazepina 184. Paciente diabético cursa con pérdida progresiva de
E. Quetiapina agudeza visual, la etiología más probable seria:
A. Desprendiemiento de retina
179. Que serotipo VPH está asociado a las verrugas B. Edema macular
genitales C. Hemovitreo
D. Glaucoma de Angulo estrecho
A. 6 Y 11 E. Miopia maligna
B. 16 Y 18
C. 24 Y 28 185. Mujer de 22 añ os de edad, sexualmente activa,
D.15 Y 11 con dolor pélvico intenso y fiebre. Al examen:
anexos dolorosos y
E. 6 y 18
leucorrea maloliente. Hemograma con 15.000
leucocitos x mm3. ¿Cuál es el diagnóstico y el
manejo más apropiado?
A. Enfermedad pélvica inflamatoria + B. Dermatitis por contacto / corticoides
hospitalización + Ceftriaxona + metronidazol + C. Acarosis / griseofulvina
doxiciclina D. Dermatofitosis / clotrimazol
B. Enfermedad pélvica inflamatoria + E. Dermatofitosis / itraconazol
hospitalización + cefoxitin + metronidazol +
azitromicina 190. El antidepresivo con menor efecto sedante y el que
C. Enfermedad pélvica inflamatoria + tratamiento tiene mayor efecto sedante son respectivamente:
ambulatorio con ciprofloxacino, doxiciclina y A. Amitriptilina / sertralina
metronidazol. B. Clomipramina / imipramina
D. Quise ovárico complicado + tratamiento C. Doxepina / bupropion
quirúrgico D. Fluoxetina / paroxetina
E. Enfermedad pélvica inflamatoria + tratamiento E. Trazodona / sertralina
quirúrgico.
191. Mujer con infección por chlamydia, que esquema
186. Mujer de 26 añ os, con fiebre de 39°C, de 4 semanas de tratamiento indicaría.
de evolució n que calma con antipiréticos; fatiga A. Metronidazol 2gr dosis única
intensa y malestar general. Al Examen físico: B. Fluconazol 150mg
Linfoadenomopatías cervicales, submaxilares y C. Doxiciclina 100 mg
supraclaviculares y hepatoesplenomega- lia. D. Ketoconazol
Hemograma con 15% de linfocitos atipicos ́ y leve E. Ceftriaxona 250 mg
trombocitopenia. Anticuerpos heterofilos
positivos. ¿Cuál es el diagnóstico más probable? 192. Ubicación más frecuente del tumor de klastkin
A. Dengue hemorragico A. En conducto ci ́stico
B. Mononucleosis infecciosa por CMV B. Colédoco proximal
C. Fiebre Chikungunya C. Confluencia de hepáticos
D. Mononucleosis infecciosa por VHB D. Colédoco distal
E. Tuberculosis pulmonar E. Hepático derecho

193. Varó n de 19 añ os que refiere aumento de volumen


187. Paciente varón de 45a con DM tipo I. Acude por en el hemiescroto derecho transiluminacion
cefalea, somnolencia, náuseas y vómitos ́
positiva, deberiamos
persistentes. Al examen físi- co, despierto, pálido y descartar
mucosa oral seca; PA 140/70, FC: 100; FR: 28; A. Hernia inguinoescrotal.
extremidades inferiores con fóvea++ Hb 10 pH: B. Hidrocele
7.24; Hto 44 K 2.9 Na 139; HCO3 15. Glucosa 280. C. Quiste del epidi ́dimo.
¿Cuál es el Diagnostico? D. tumor testicular tipo seminoma
A. Estado Hiperosmolar E. Tumor testicular tipo no seminoma
B. Cetoacidosis Diabética
C. Pielonefritis crónica 194. Beta bloqueador utilizado en glaucoma crónico
D. Cetoacidosis ayuno
E. Insuficiencia Renal Aguda A. Metoprolol
B. Bisoprolol
188. En la artritis bacteriana aguda de rodilla, la C. Nadolol
conducta terapéutica inmediata es: D. Carvedilol
A. Drenaje de li ́quido sinovial y AINES E. Timolol
B. Antibióticos parenteral y drenaje
C. Antibióticos orales y AINES
D. Antibióticos intra-articulares
E. Irrigación intra-articular continua

189. Mujer de 20 años de edad, peladora de espárragos


desde hace 4 meses, presenta lesiones
eritematosas pruriginosas micropapulares
localizada en dorso de antebrazos, cara y cuello.
Las lesiones también se han identificado en
algunas compañeras de trabajo. No presenta
antecedentes de atopía. Su primera impresión
diagnóstica es y el tratamiento es:
A. Acarosis / permetrina
195. Paciente varón de 50 años, fumador, con
Hipertensión arterial controlada, programado
para cura quirú rgica de her- nia inguinal. Su
riesgo ASA es:
A. V
B. II
C. I
D. IV
E. III

196. Se cuenta con 13 valores de nivel de bilirrubinas


en pacientes con ictericia y se desconoce la
varianza poblacional para esta variable. En la
construcció n del intervalo de confianza al 95%
para el valor promedio de bilirrubina en este
grupo de pacientes, la prueba estadística
requerida es:
A. ANOVA
B. Binomial
C. Chi-cuadrado
D. Exacta de Fisher
E. T de Student.

197. Paciente con WPW, cual es la alternativa correcta:


A. Se debe a fenómenos de reentrada
B. Su tratamiento definitivo es con beta
bloqueadores
C. Su tratamiento de urgencia es ablación con
radiofrecuencia
D. Cursa con onda delta en el EKG
E. Más frecuente en personas adultas mayores

198. ¿Có mo se define un RN con peso adecuado para


la edad gestacional?
A. P10-90
B. P5-95
C. P15 -85
D. P3-97
E.P3-90

199. La inhibición de la agregación plaquetaria por


ADP (adenosina difosfato) es característica de:
A. Ácido acetilsalici ́lico.
B. Tirofibán.
C. Sulfinpirazona.
D. Clopidogrel.
E. Abciximab.

200. El tipo de sangrado más característico que


producen los leimiomas uterinos:
A. Hipermenorrea por intramurales
B. Metrorragia por submucosos
C. Menorragia por submucosos
D. Menometrorragia por intramurales
E. Hipermenorrea por submucosos
D. Recto superior izquierdo y oblicuo inferior izquierdo.
EXAMEN 12° FECHA: 16 /08/2019
E. Recto superior derecho y oblicuo derecho.
SIMULACRO 3B
6.¿De qué músculo forma parte el ligamento inguinal?
1.El tipo de muestreo que analiza las unidades de una
población que pueden encontrarse agrupadas en personas, A. Oblicuo externo del abdomen.
fami- lias, viviendas o manzanas es: B. Oblicuo interno del abdomen.
A. Muestreo aleatorio estratificado. C. Transverso del abdomen.
B. Muestreo sistemático. D. Psoas.
C. Muestreo aleatorio por recomendación. E. Pectíneo.
D. Muestreo aleatorio de conglomerados. 7.La leptina se sintetiza en:
E. Muestreo aleatorio simple. A. Hígado
2.En el proceso de planificación estratégica, la identificación B. Hipotálamo
de las fortalezas y debilidades así como las oportunidades y
amenazas es importante. Califique cada situación: I: C. Tejido adiposo
Producto readicionado en el mercado, II: Presencia de nuevo
proveedor de mi producto, III: Personal desmotivado y no D. Intestino delgado
capacitado y IV: Suscripción de un tratado de libre co-
E. Páncreas
mercio que incluye mis productos; según se trate de una
fortaleza (F), debilidad (D), oportunidad (D) o amenaza (A): 8.De las siguientes arterias uterinas, ¿cuáles son las que
irrigan la capa funcional del endometrio?:
A. (I-F) (II-D) (III-A) (IV-O)
A. Arterias arcuatas.
B. (I-D) (II-A) (III-F) (IV-O)
B. Arterias radiadas.
C. (I-F) (II-O) (III-D) (IV-A)
C. Arterias helicinas menores.
D. (I-D) (II-O) (III-F) (IV-A)
D. Arterias helicinas mayores.
E. (I-F) (II-A) (III-D) (IV-O)
E. Arterias espirales.
3.La potencia o poder estadístico de un test es lo mismo
que: 9.La nefrotoxicidad de los aminoglicósidos se produce por
daño en:
A. Complementario del error alfa (I - alfa)
A. Células del túbulo proximal.
B. Complementario del error beta (I – beta)
B. Membrana basal glomerular.
C. Error alfa
C. Células del túbulo distal.
D. Grado de significación estadística
D. Células del túbulo colector.
E. Error beta
E. Células de la papila.
4.Con respecto al grado de significación estadística (p),
señale lo que es cierto:

A. Cuanto menor sea “p”, mayor evidencia a favor de


la hipótesis nula
10.Un niño normal duplica su talla de nacimiento a la edad
B. Nos da la probabilidad de que los resultados de:
obtenidos sean fruto de la casualidad o azar
A. 3 años.
C. Es una medida de la fuerza de asociación
B. 2 años.
D. Si “p” > 0.05 se acepta la hipótesis alternativa
C. 5 años.
E. Ninguna de las anteriores
D. 4 años.
5.En la mirada abajo a la izquierda, exploramos los
siguientes músculos extraoculares: E. 6 años.

A. Recto superior derecho y oblicuo superior izquierdo. 11.¿En qué momento se recomienda administrar la vacuna
de Hepatitis B para evitar la hepatitis de transmisión vertical?
B. Recto inferior derecho y oblicuo superior izquierdo.
A. En las primeras 12 horas de vida
C. Recto inferior izquierdo y oblicuo superior derecho.
B. A las 24 horas de vida B. Enfermedad de Paget

C. A la semana C. Osteoporosis

D. A los 14 días de vida D. Metástasis ósea de primario a definir

E. Al mes de edad E. Osteodistrofia por insuficiencia renal crónica

12.Un niño de 3 años es llevado a la emergencia con inicio 16.Mujer de 60 años, con historia de infiltración en rodilla
súbito de alteración de la conciencia. Al examen se derecha 10 días antes de la consulta actual, acude refiriendo
encuentra miosis, bradicardia y fasciculaciones musculares. tres días de dolor, eritema, incremento de volumen,
Estos hallazgos son sugerentes de intoxicación por: incapacidad funcional a nivel de rodilla derecha asociado a
fiebre, cefalea y malestar. El diagnóstico más probable es:
A. Antidepresivos tricíclicos
A. Artritis por depósito de cristales en rodilla derecha
B. Salicilatos
B. Artritis por estafilococo aureus en rodilla derecha
C. Cocaína
C. Artritis por neisseria gonorreae en rodilla derecha
D. Organofosforados
D. Artritis por mycobacterium tuberculoso en rodilla
E. Narcóticos derecha
13.Una mama lleva a su hija de 2 años y medio a la E. Artritis por ruptura de meniscos en rodilla derecha
emergencia por haber presentado una convulsión tónico-
clónica ge- neralizada de 2 minutos de duración. Ella refiere 17.Mujer de 69 años con una densitometría ósea que reveló
haber sentido “caliente” a su hija más o menos una hora un valor T=-2.8 en región lumbar y valor T=-1.9 en cadera.
antes de la convulsión. No hay historia familiar de Usted le indica estilos de vida saludables y le menciona:
convulsiones. La niña llega a la emergencia despierta pero
un poco irritable, con temperatura de 39° y examen A. Tiene osteopenia y prescribe calcio más vitamina D.
neurológico normal. Su manejo, además de disminuir la
B. Tiene osteoporosis vertebral y coxofemoral y
temperatura es
prescribe calcio más vitamina D y un bifosfonato.
A. Interconsultar al neuropediatra
C. Tiene osteoporosis vertebral y prescribe calcio y
B. Iniciar fenobarbital vitamina D y un bifosfonato.

C. Iniciar fenitoína D. Tiene osteoporosis coxofemoral y prescribe calcio y


vitamina D y un bifosfonato.
D. Observar en emergencia por 6 horas
E. Tiene ostepenia y prescribe un bifosfonato.
E. Realizar un EEG
18.Mujer de 20 años presenta un mes de edemas de
14.Paciente varón de 21días de vida que presenta vómitos miembros inferiores que progresa a anasarca, se asocia
inmediatamente después de lactar. Al examen se le fiebre y po- liartritis simétrica. Al examen PA 110/70, edema
encuentra irritable, con pobre ganancia de peso. Tiene blando en extremidades inferiores, derrame pleural bilateral,
además una alcalosis metabólica hiperclorémica. Que derra- me pericárdico y ascitis. El estudio del líquido pleural
examen sería el más recomendable para llegar al reveló un contenido de proteínas de 4.5g / 100 ml y la
diagnóstico: albumina del líquido ascítico de 3 g / 100 ml. Lo más
probable es que se trate de:
A. Radiografía simple de abdomen
A. Tuberculosis multisistémica
B. Ecografía de abdomen
B. Lupus Eritematoso Sistémico
C. Gammagrafía
C. Artritis reumatoide con poliserositis
D. Enema con bario
D. Poliangeítis microscópica
E. T4 y TSH.
E. Síndrome antifosfolipídico
15.Paciente mujer de 78 años que luego de realizar sus
labores cotidianas de la casa presenta dolor intenso en la 19.Mujer de 55 años, con historia de AR de 10 años de
región lumbar que limita la deambulación, pero sin evolución, ha recibido eventualmente prednisona; se
compromiso motor ni sensitivo. En la radiografía lateral de la presenta a consulta con un tiempo de enfermedad de 6
columna lumbar se aprecia colapso en cuña de la cuarta meses caracterizado por disnea progresiva y tos episódica,
vértebra lumbar, se tiene además un calcio sérico: 7.5mg/dl, al examen crépitos bibasales y S2 incrementado de
fosfatasa alcalina: 120mg/dl, creatinina: 1.2 mg/dl, proteínas intensidad. Por frecuencia cuál de las siguientes entidades
totales: 5 mg/dl, albúmina: 3 mg/dl, hemoglobina: 12 mg/dl. es más probable:
¿Cuál es la enfermedad de fondo más probable de la
paciente? A. Fibrosis pulmonar.

A. Mieloma múltiple B. Bronquiolitis obliterante con neumonía asociada.


C. Nódulos reumatoides. E. Lamivudina

D. Obstrucción de las vías aéreas pequeñas. 24.Paciente mujer de 62 años con insuficiencia cardiaca
descompensada desarrolla neumonía y en la radiografía de
E. Engrosamiento pleural. tórax se aprecia un gran derrame pleural. ¿Qué hallazgo
indicaría que la efusión pleural se debe a la insuficiencia
20.Mujer de 18 años con historia de gingivorragia, equimosis
cardiaca?
espontánea e hipermenorrea. Al examen: palidez, ausencia
de esplenomegalia, petequias en hemiabdomen inferior y A. Contenido de proteína de 6 g/dl
muslos. Plaquetas: 30,000, hematocrito: 30%, leucocitos:
6,000. ¿Cuál de las siguientes opciones es la causa más B. pH de 7.13
probable de la trombocitopenia?
C. Glucosa de 20 mg/dl
A. Lupus Eritematoso Sistémica
D. Relación LDH fluido/sérica de 0,3.
B. Purpura de Henoch-Schönlein
E. Relación de proteína fluido/sérica de 0.7
C. Purpura Trombocitopénica autoinmune
25.Un joven de 16 años se presenta a emergencia por inicio
D. Trombocitopenia por parvovirus B19 agudo de un rash morbiliforme, fiebre, debilidad y oliguria.
Estos síntomas aparecieron una semana después de haber
E. Síndrome de Evans iniciado tratamiento con ampicilina por una faringoamig-
dalitis estreptocócica. Su temperatura es 38,8 ºC, PA 100/70,
21.Una mujer de 43 años sufre la mordedura de una araña.
FC 92x’ y FR 16x’. El examen de orina muestra micro-
Presenta en el brazo derecho una lesión violácea de 3 cm d
hematuria, leucocituria y numerosos eosinófilos con escasos
diámetro de bordes eritematosos dolorosa no pruriginosa. En
cilindros leucocitarios. Los exámenes de sangre muestran
el examen de orina muestra hematuria. La lesión des- crita
títulos elevados de ASO y eosinofilia moderada. La urea esta
sugiere la mordedura por:
en 92mg/dl y la Creatinina en 2,5mg/dl. ¿Cuál de los
A. Lactrodectus mactans siguientes es el diagnóstico más probable?

B. Loxosceles laeta A. Nefritis intersticial aguda

C. Phoneutria nigriventer B. Purpura de Henoch Schonlein

D. Atrax robustus C. Necrosis tubular aguda

E. Lycosa sp. D. Sindrome urémico hemolitico

22.Un hombre de 25 años procedente de Iquitos presenta E. Glomérulonefritis post estreptocócica


fiebre e ictericia de 1 semana de evolución. Al examen físico
26.Un adulto mayor malnutrido de 58 años es llevado a la
se evidencia dolor muscular intenso y rash eritematoso
emergencia. Se encuentra afebril; PA 110/80 mmHg, FC
homogéneo a nivel de tórax y extremidades. El paciente
108/ min, FR 20/min. El examen neurológico muestra
camina descalzo en la selva. Las pruebas de laboratorio
desorientación con tendencia a la somnolencia. Lab. inicial:
muestran CPK: 900 mg/dl y Leucocitos en 18 000/mm3. El
Na 105 meq/l, K 3,9 meq/l, Cl 101 meq/l, osmolaridad
diagnóstico más probable sería:
plasmática 256 mOsm/kg y excreción fraccionada de sodio
A. Dengue menor de 0,5%. Fue tratado con Tiamina EV 300mg, infusión
de dextrosa 5% e infusión de ClNa 3%. 12 horas después se
B. Fiebre amarilla torna cuadripléjico, aunque puede comunicarse a través de
movimientos de los parpados. Una muestra de sangre en
C. Leptospirosis
ese momento mostró: Na 126 meq/l, K 4 meq/l, Cl 105meq/l.
D. Hepatitis Viral B Luego de 3 días el paciente fallece. ¿De las siguientes
posibi- lidades cual es la causa más probable de la muerte?
E. Malaria
A. Encefalopatía de Wernicke.
23.Mujer de 30 años infectada por el virus de
inmunodeficiencia humana se encuentra en TARGA desde B. Mielinolisis central pontina
hace 3 semanas y presenta debilidad generalizada, sueño
C. Hemorragia en área motora del hemisferio
incrementado y palidez marcada. El hematocrito es de 28 %,
dominante
se sabe que inició la terapia con 40 %. ¿Cuál de los
siguientes fármacos es el más probable de producir este D. Degeneracion aguda de la medula espinal
efecto adverso?
E. Infarto en capsula interna del hemisferio dominante
A. Efavirenz
27.Paciente mujer de 20 años que presenta debilidad
B. Estavudina muscular en miembros inferiores; hiporeflexia, disminución
del tono muscular y se queja de parestesias, disestesias y
C. Nevirapina
lodinia. Cuál es el diagnóstico más probable:
D. Zidovudina
A. Polineuropatía.
B. Síndrome piramidal. pulsaciones/min. Despierto, con dolor abdominal difuso a la
palpación, reacción peritoneal dudosa, ruidos hidroaéreos
C. Síndrome de motoneurona del asta anterior. escasos. SU CONDUCTA A SEGUIR SERA:
D. Síndrome de placa mioneural. A. Hospitalizar al paciente y programar a sala de
operaciones por tratarse de un traumatismo abdominal
E. Mielitis transversa.
cerrado.
28.Paciente de 38 años acude a Urgencias tras sufrir herida
B. Enviar al paciente a su domicilio e indicarle que
por arma blanca a nivel de hemitórax derecho, octavo
regrese si empeora
espacio intercostal. PA 120/60 mmHg; FC 92x´ y FR 22x´, la
auscultación del hemitórax derecho demuestra disminución C. Mantener al paciente en observación con
de los ruidos respiratorios y la percusión del hemitórax hidratación parenteral, sonda Foley, control de hematocrito
demuestra timpanismo. ¿Cuál es la MEJOR conducta a seriado.
seguir?:
D. Colocar vía parenteral, aplicar antiespasmódico y
A. TAC de tórax. solicitar TAC abdominal
B. Toracotomía urgente. E. Realizar paracentesis diagnóstica para determinar
si es traumatismo.
C. Observación clínica.
33.Cuando hay ruptura de víscera hueca se pierde la
D. Radiología simple de tórax y según hallazgos
matidez hepática y es producto de la interposición de aire en
colocación de drenaje pleural.
el espacio hepatofrénico convirtiéndose en sonoridad a la
E. Ingreso en UCI. percusión. Este signo es patognomónico de ruptura de
vísceras huecas se denomina signo de:
29.Paciente en el 5° día post operatorio presenta fiebre alta,
taquicardia y signos inflamatorios de la herida A. Murphy
peratoria.¿Cuál es la conducta prioritaria a seguir?
B. Jobert
A. Apertura amplia de la herida
C. Mc burney
B. Cambiar los antibióticos
D. Winston
C. Aplicación de hielo local
E. Blumberg
D. Colocar un dren laminar
34.Paciente de 37 años que desde hace 4 dias presenta
E. Observación y antipiréticos dolor en región anal y desde hace dos días fiebre. En
urgencias se le observa una zona indurada, tumefacta y
30.Mujer de 41 años de edad, que acude a Emergencia por enrojecida en la zona perianal derecha. A la presión es muy
dolor abdominal en epigastrio de tipo cólico desde hace tres dolorosa. Se le prescriben antibióticos y se le recomienda
días, desencadenado por la ingesta de alimentos grasos, consulta en cirugía a las 48 horas. ¿Cuál es el origen más
escalofríos y fiebre. El diagnóstico más probable es: probable de su patología actual?
A. Íleo biliar A. Fisura anal
B. Colecistitis aguda B. Hemorroides internas
C. Ulcera péptica C. Abceso perianal
D. Gastritis aguda D. Prolapso rectal
E. Obstrucción intestinal E. Fístula anorrectal
31.Paciente de 20 años de edad, que presenta traumatismo 35.Llega a emergencia un paciente politraumatizado,
cerrado de abdomen. En la radiografía simple de abdomen inconsciente por un accidente de tránsito. Presenta la cara
de pie se aprecia aire en el espacio hepatofrénico derecho, aplasta- da y la boca y nariz llenas de sangre. Presenta
esto se observa en: obvia deformidad y crepitación de dos extremidades, y
sangra pulsátil y abundantemente por la cara medial del
A. Perforación intestinal
brazo. El orden de las primeras actuaciones debe ser:
B. Traumatismo de bazo
A. a. Inmovilización de la fractura de las extremidades,
C. Perforación de vesícula biliar seguido de TAC craneal y facial.

D. Traumatismo hepático B. b. Diagnóstico de lesión intracraneal, seguido de


osteosíntesis de fractura de extremidades.
E. Traumatismo de páncreas
C. c. Control del sangrado arterial, seguido de
32.Chofer que sufre accidente de tránsito (choque) es diagnóstico de lesiones intracraneales.
llevado a la emergencia porque presenta dolor abdominal. Al
examen clínico presenta PA: 80/60 mmHg. Pulso: 105
D. d. Control del sangrado arterial, seguido de C. Citomegalovirus
inmovilización de las extremidades.
D. Virus papiloma humano
E. e. Asegurar vía aérea permeable, seguido de
control del sangrado arterial. E. Virus de la hepatitis B

36.Paciente de 35 años de edad se encuentra en el tercer 41.Si un tumor testicular, es secretor de alfafetoproteínas,
día postoperatorio de colecistectomía sin colangiografía probablemente se tratará de:
intrao- peratoria, cursa con ictericia progresiva. El estudio
A. Sarcoma
ecográfico muestra dilatación de la vía biliar principal con
cálculo de 1.5cm de diámetro. El procedimiento más B. Tumor de células de Leydig
recomendable es:
C. Seminoma espermatocito
A.Exploración quirúrgica del colédoco y extracción del
cálculo D. Carcinoma embrionario

B. Esfinteroplastía E. Seminoma puro

C. Sólo drenaje con sonda de Kher 42.¿Qué opción debemos considerar de elección en el
tratamiento de un pseudoquiste inferior de 6cm de diámetro
D. Pancreatocolangiografía retrógrada endoscópica y asin- tomático que se ha desarrollado como complicación
en una pancreatitis aguda?
E. Derivación biliodigestiva
A. Drenaje quirúrgico interno
37.¿Cuál es la causa más frecuente de obstrucción de
colon? B. Somatostatina (perfusión EV) u octreótide SC
A. Diverticulitis C. Drenaje endoscópico percutáneo guiado por
ECO/TAC abdominal
B. Carcinoma
D. Esperar evolución clínica de 4–6 semanas
C. Invaginaciones
E. Ninguna de las anteriores.
D. Vólvulos
43.Una paciente de 25 años G0 P0 acude a su control
E. Hernias
ginecológico de rutina sin manifestar molestias. Su FUR fue
38.¿Cuál de estas afirmaciones es cierta sobre la fractura de hace 10 días, normal; usa anticonceptivos orales hace 8
la extremidad distal del radio? meses. El examen clínico es normal salvo la presencia de
ectropión. Se toma un examen de secreción cervical para
A. La mayoría se producen como consecuencia de una Papanicolau y luego de una semana es informado como LIE
caída con la muñeca en flexión de alto grado.¿Cuál debe ser la siguiente indicación?

B. El tratamiento quirúrgico está contraindicado A. Electrofulguración

C. Más frecuente en hombres B. Criocauterización con nitrógeno líquido

D. La rotura del extensor largo del pulgar no es una C. Colposcopía y biopsia dirigida
complicación de estas fracturas
D. Conización con asa electroquirúrgica
E .La mayoría responden favorablemente al tratamiento
conservador E. Conización con LASER

39.En una fractura de tercio medio de diáfisis de húmero, el 44.Paciente mujer de 28 años, quien presenta
nervio más vulnerable de lesionarse cuando existe desplaza- oligomenorrea, acné e hirsutismo. Hace 18 meses intenta
miento de los fragmentos es: salir gestando sin lograrlo. ¿Qué examen de laboratorio
solicitaría?
A. Axial
A. Cortisol.
B. Safeno
B. Androstenediona
C. Cubital
C. Estradiol día 3 del ciclo
D. Mediano
D. FSH/LH día 3 del ciclo
E. Radial
E. TSH ultrasensible
40.¿Cuál es el microorganismo que se asocia con el linfoma
gástrico? 45.Paciente de 25 años, de raza negra, cursa con
menometrorragia de 6 meses. Su Hematocrito es de 31%, la
A. Virus de la hepatitis C ultraso- nografia transvaginal detecta imagen sólida de 1.5
cm. en cavidad endometrial confirmada por hidrosonografia.
B. Helicobacter pylori El procedimiento quirúrgico indicado es:
A. Biopsia de endometrio en consultorio 50.Mujer de 20 años de edad, G2 P2002. Último parto por
cesárea, hace 3 meses lactando. ¿Cuál es el anticonceptivo
B. Dilatación y curetaje hormonal más adecuado?:
C. Histeroscopia quirúrgica A. Estrógeno oral
D. Laparoscopia quirúrgica B. Estrógeno más progesterona por vía oral
E. Histerectomía Abdominal C. Progestágenos orales
46.Paciente mujer de 30 años de edad, que acude por dolor D. Parche de etinilestradiol y progesterona
pélvico de una semana de evolución que se inicia dos días
después de haber sido sometida a una histerosalpingografía E. Inyectable mensual de estrógeno y progesterona
por infertilidad. Hace tres días refiere fiebre de 38°C. Al
examen presenta dolor en abdomen bajo y se insinúa 51.La localización más frecuente del embarazo ectópico
rebote, el examen bimanual del útero y anexos es difícil de tubárico es:
eva- luar por el dolor, el resto del examen es normal. ¿Cuál
A. Intersticial
es el diagnóstico más probable en este caso?
B. Columnar
A. Apendicitis aguda
C. Fímbrico
B. Endometriosis
D. Ampollar
C. Enfermedad pélvica inflamatoria
D. Ruptura folicular E. Ístmico
E. Infección urinaria 52.¿Qué maniobra obstétrica permite comprobar el
encajamiento de la presentación fetal?
47.Paciente de 25 años de edad, que presenta flujo vaginal
blanco homogéneo, no hay presencia de células clave a la A. Tercera maniobra de Leopold
mi- croscopia, el PH es mayor de 4.5 y se percibe olor a
aminas al mezclar con KOH al 10%. ¿Cuál es el agente B. Cuarta maniobra de Leopold
etiológico en este caso?:
C. Peloteo
A. Trichomonas vaginalis
D. Primera maniobra de Leopold
B. Gardnerella vaginalis
E. Segunda maniobra de Leopold
C. Candida Albicans
53.Retención de restos placentarios Los desgarros
D. Chlamydia trachomatis perineales de tercer grado comprometen:

E. Neisseria gonorrheae A. Mucosa rectal exponiendo la luz rectal

48.Usted realiza una amniocentesis en una gestante de 31 B.Piel, mucosa vaginal, aponeurosis y músculos del cuerpo
semanas en la cual sospecha corioamnionitis. ¿Qué perineal
exámenes de laboratorio del líquido amniótico debe solicitar
para tomar una decisión? C. Piel perineal y mucosa vaginal

A. Proteinas-neutrófilos-gram D. Piel, mucosa vaginal, cuerpo perineal y esfínter anal

B. Cultivo-proteinas-glucosa E. Horquilla, piel perineal y mucosa vaginal

C. Glucosa-neutrófilos-gram 54.Señale que cambios ocurren en el embarazo normal:

D. Neutrófilos-PCR-VSG 1) Aumento de tamaño renal

E. Gram-cultivo-neutrófilos 2) Dilatación ureteral

49.Mujer de 25 años con tumoración amarilla bilobulada de 3) Aumento del índice del flujo plasmático renal
2.5cm de diámetro, móvil, lisa con consistencia de caucho 4) Pronunciada proteinuria
en mama derecha. El diagnóstico más probable es:
5) Aumento del valor de la creatinina sérica
A. Mastitis
6) Aumento de la excreción renal de aminoácidos y
B. Carcinoma vitaminas hidrosolubles
C. Enfermedad fibroquística A. 1, 2, 3 y 6
D. Fibroadenoma B. 1, 2, 3, 4 y 5
E. Cistosarcoma filoide C. 3, 4, 5 y 6

D. 2, 3, 4 y 5
E. 1, 2, 3 y 5 D. Pre-eclampsia

55.¿Cuál de los tipos de virus de papiloma humano (VPH) E. Neumonitis intersticial


está asociado al carcinoma del cuello uterino?
59.Paciente varón de 55 años de edad que ingresa a la
A. 44 emergencia con 5 días de ictericia sin coluria, a los 3 días se
agrega trastorno de la conciencia; en el examen se
B. 6 comprueba la ictericia y la palidez, MV se ausculta bien, no
hay soplos, es- plenomegalia ni signos de focalización. En
C. 11
los exámenes iniciales se encuentra anemia severa, con
D. 42 hiperbilirrubinemia indirecta y en el frotis de sangre periférica
se ve esquistocitos. El diagnóstico más probable sería:
E. 16
A. Anemia hemolítica por anticuerpos fríos
56.Varón 70 años, HTA crónica mal controlada; acude por 6
horas de trastorno del sensorio y hemiparesia derecha. PA: B. Purpura trombocitopénica trombótica
200/100, resto de FV normales. Soporoso y con hemiparesia
C. Esferocitosis hereditaria
demostrada. TAC cerebral no muestra alteraciones. Mar- car
el enunciado verdadero: (PAM = presión arterial media). D. Síndrome urémico hemolítico
A. El manejo incluye uso de antihipertensivos E. Síndrome de Evans
sublinguales inmediatamente hasta normalizar la PAM.
60.Paciente de 63 años que está en tratamiento con
B. El manejo incluye monitorización y no inciar heparina i.v. y desarrolla un cuadro compatible con
antihipertensivos si la PAM se mantiene entre 130 – 150 apendicitis que requiere intervención en las próximas 24
mmHg durante las primeras 24 – 48 horas. horas. ¿Qué actitud plantearía?
C. Monitorizar la primera hora y de no bajar se debe A. Administrar sulfato de protamina
llevar la PAM entre 70-80 mmHg, para proteger de mayor is-
quemia la zona de penumbra. B. Administrar complejo protrombínico

D. La nitroglicerina es el antihipeetensivo de elecciòn C.Suspender la heparina y operar pasadas 6 o más horas


cuando el órgano blanco es el cerebro.
D. Pasar a heparina de bajo Pm
E. Llevar la PAM entre 80-100 mmHg las primeras 24
horas para disminuir la probabilidad de un nuevo evento. E. Administrar plasma fresco congelado

57.Paciente varón de 44 años con dolor epigástrico urente 61.Una mujer de 30 años refiere un cuadro de mialgias,
desde hace un mes, al realizársele la endoscopía se fiebre, palpitaciones, nerviosismo y dolor en el cuello. La
encuentra una úlcera duodenal, se biopsia en busca de H veloci- dad de sedimentación globular está elevada, los
pylori, pero el resultado va a demorar una semana, no se niveles séricos de triiodotironina (T3) y tiroxina (T4) están
dispone de test de la ureasa, usted decide: elevados y los de tirotropina (TSH) están bajos. ¿Cuál sería
el tratamiento más adecuado para esta paciente?:
A. Iniciar tratamiento para H pylori
A. Corticosteroides y antitiroideos
B. Iniciar tratamiento con inhibidores de la bomba de
protones B. Antiinflamatorios no esteroideos y betabloqueantes

C. No iniciar algún tratamiento hasta tener el resultado C. Antiinflamatorios no esteroidesos y antitiroideos


de la biopsia.
D. Sólo antitiroideos
D. Iniciar protectores de mucosa gástrica como
E. Solo betabloqueantes
Magaldrato
62.En relación al tratamiento del dolor agudo posoperatorio,
E. Iniciar antagonistas de los receptores tipo 2 de
señalar la correcta:
histamina
A. Se puede prescindir de la participación de la
58.Paciente segundigesta de 32 semanas de gestación que
enfermería en un equipo interdisciplinar.
ingresa a la emergencia por disnea de inicio agudo y palpita-
ciones. Niega antecedentes de importancia. PA: 135/75 B. La escala analógica visual no permite valorar la
mmHg, FR: 22x´, FC: 115x´, T oral: 36 grados, peso: 100 Kg, eficacia del tratamiento.
obesa, no edema, MV se ausculta bien, RC rítmicos,
taquicárdicos, no soplos. Radiografía tórax normal, EKG: C. La mejor técnica es aquella que produce menores
taqui- cardia sinusal. AGA: hipoxemia. El diagnóstico más efectos secundarios.
probable sería:
D. El fármaco de referencia es el paracetamol.
A. Endocarditis bacteriana
E. La técnica de elección es la analgesia controlada
B. Tromboembolismo venoso por el interno de medicina.

C. Crisis de ansiedad
63.Relacione los signos y síntomas con la entidad A. Membrana sinovial
correspondiente:
B. Cápsula articular
a) Infección de una cripta anal.
C. Músculo
b) Trayecto infectado.
D. Hueso subcondral
c) Rectorragia y dolor.
E. Cartílago articular
d) Irritación crónica y leucoplasia.
68.Las alteraciones del electrocardiograma típicamente
e) Dolor, ulceración y espasmo del esfínter. diagnósticas de la pericarditis aguda obligan a plantearse el
diagnóstico diferencial con una de las siguientes entidades
1) Hemorroides. clínicas, ¿Cuál es?
2) Tumor del canal anal. A. Insuficiencia aórtica aguda
3) Fístula ano rectal. B. Estenosis valvular aórtica
4) Fisura anal. C. Transposición corregida de los grandes vasos
5) Absceso anorectal. D. Enfermedad de Ebstein
A. (a,4) (b,5) (c,1) (d,2) (e,3). E. Infarto de miocardio
B. (a,5) (b,3) (c,1) (d,2) (e,4). 69.¿Cuál de las siguientes es característica de la
enfermedad celíaca?
C. (a,3) (b,4) (c,5) (d,1) (e,2).
A. Riesgo incrementado de linfoma en 0.1%
D. (a,1) (b,2) (c,3) (d,4) (e,5).
B. Niveles altos de anticuerpo antigliadina
E. (a,2) (b,1) (c,2) (d,3) (e,4).
C. Mejora de los síntomas con la ingestión de fibra
64.¿Cuál es la manifestación extraarticular más frecuente de
la espondilitis anquilosante? D. Compromiso intestinal distal
A. La Amiloidosis secundaria E. Vellosidades predominantes en la biopsia del
intestino delgado
B.La afectación pulmonar intersticial de campos superiores
70.La ausencia de pulsos femorales es un signo
C. La uveítis anterior aguda
característico de:
D. La uveítis posterior con coriorretinitis
A. Tetralogía de Fallot
E. La nefropatía IgA
B. Desembocadura anómala de las venas pulmonares
65.La presencia en la vulva de vesículas agrupadas y
C. Estenosis aórtica
úlceras pequeñas orienta al diagnóstico de:
D. Atresia tricuspídea
A. Chancroide
E. Coartación aórtica
B. Molusco contagiosos
71.Con respecto al tétanos. ¿Cuál de las siguientes frases
C. Herpes genital
es la correcta?
D. Condiloma acuminado
A. La mortalidad del tétanos es todavía en la
E. Sífilis actualidad superior al 50%

66.La hepatitis B y C comparten una de las siguientes B. El tétanos aparece tras un período mínimo de
afirmaciones: incubación de 14 días

A. Desarrollan hepatitis crónica en más de 60% C. En el diagnóstico del tétanos es fundamental haber
efectuado cultivos previos al inicio del tratamiento
B. La infección activa está asociada con una alta
mortalidad D. El tétanos está caracterizado por una rigidez
generalizada junto a crisis de espasmos musculares
C. Raros en consumidores habituales de drogas
E. En el tratamiento del tétanos es fundamental
D. La vacunación provee adecuada protección contra mantener el estado de alerta del enfermo, con el fin de evitar
la enfermedad aspira- ciones
E. Colocan al paciente en riesgo incrementado de 72.La diarrea sin sangre está asociada con uno de los
hepatocarcinoma siguientes:
67.En la osteoartrosis, la lesión principal radica en: A. Infección por Giardia lamblia
B. Isquemia colónica C. CD20

C. Shigelosis D. EGF1 (factor de crecimiento epidérmico tipo 1)

D. Colitis ulcerativa E. GM-CSF (factor estimulante de colonias)

E. E. coli serotipo 0157-H7 79.El agente etiológico del cancroide (chancro blando) es:

73.Son enfermedades clasificadoras de estadío “C” de la A. Chlamydia trachomatis


infección VIH, según CDC:
B. Haemophilus ducreyi
A. Sepsis por Salmonella no tífica
C. Treponema pallidum
B. Toxoplasmosis cerebral
D. Calymmatobacterium granulomatis
C. Criptococosis extrapulmonar
E. Haemophilus vaginalis
D. Púrpura trombocitopénica idiopática
80.Anomalía congénita asociada con oligodramnios:
E. Neumonía por Pneumocystis carinii
A. Displasia renal
74.El síndrome de Zollinger-Ellison ocurre a expensas de:
B. Hídrops fetalis
A. Células delta
C. Atresia esofágica
B. Células alfa
D. Espina bífida
C. Células no delta
E. Anencefalia
D. Células beta
81.La fecha de parto se calcula sumando …… días al primer
E. Células no beta día de la fecha probable de la última regla y se resta meses:

75.La complicación más frecuente de la varicela es: A. 10/3 B. 3/10 C. 7/3 D. 3/7 E. Ninguna anterior

A. Cerebelitis 82.La causa más frecuente de síndrome de Asherman es:

B. Laringitis A. Miomectomía

C. Infección cutánea B. Tuberculosis genital

D. Neumonía C. Legrado uterino post aborto

E. Síndrome de Reye D. Infección endouterina postparto

76. El germen más frecuente en la ITU intrahospitalaria E. Endometritis postaborto


es:
83.Recién nacido hijo de una madre con diagnóstico de sífilis
A. Pseudomona aeruginosa 3 semanas antes de dar a luz. Con título de VDRL 1/64; la
madre recibió tratamiento con penicilina benzatínica. El
B. Staphylococcus aureus recién nacido tiene un examen clínico normal y un VDRL
1/4¿Cuál sería el manejo más adecuado para el recién
C. Enterococo
nacido?
D. Klebsiella
A. Colocar una dosis de penicilina benzatínica al alta y
E. Escherichia coli control por consultorio externo.

77.En la tuberculosis miliar, ¿Por qué vía se diseminan los B. Hospitalizar y dar tratamiento con penicilina G
bacilos tuberculosos?
C. Hospitalizar y dar tratamiento con ampicilina
A. Linfática
D. Realizar VDRL en LCR y sólo si es positivo iniciar
B. Sanguínea tratamiento.

C. Endotraqueal E. Ningún tratamiento pues la mama recibió


tratamiento completo
D. Por contigüidad
84.Recién nacido con pobre esfuerzo respiratorio y
E. Digestiva frecuencia cardiaca <100 luego de 30 segundos de
administrar los pasos iniciales de reanimación, ¿Qué hacer?
78.¿Cuál es la diana del anticuerpo monoclonal Rituximab?
A. Masaje cardiaco
A. EGF2 (factor de crecimiento epidérmico tipo 2)
B. Intubar
B. IL-2 (interleucina)
C. Dar ventilación a presión positiva 89.Recién nacido a término de 1800 gr. De peso al nacer.
Madre con pre eclampsia moderada. ¿Cuál es la
D. Estimular complicación más probable en las siguientes 36 horas?
E. Usar adrenalina A. Sepsis
85.Neonato de 3 semanas de vida con ictericia, nació a B. Hiponatremia
termino con Peso 3100 gr. Grupo Rh madre es A positivo,
RN O positivo. Peso actual 3600gr recibe lactancia materna C. Apneas
exclusiva. Despistaje metabólico neonatal negativo.
Bilirrubina Total: 16 mg/dl Bilirrubina directa: 1.2 mg/dl. D. Dificultad respiratoria
¿Cuál es el diagnostico?
E. Hipoglicemia
A. Ictericia por leche materna
90.Niña de 3 años quien presenta fiebre a la que se agrega
B. Ictericia por incompatibilidad de grupo síntomas respiratorios altos , eritema en mejillas extendién-
dose a tronco y extremidades con posterior aspecto de
C. Ictericia por falta de aporte encaje. El diagnóstico más probable es:

D. Ictericia colestásica A. Sarampión.

E. Ictericia por hipotiroidismo B. Rubeola.

86.Recién nacido de 20 horas de vida sin factores de riesgo C. Eritema infeccioso.


prenatal, parto eutócico, se alimenta con lactancia materna
ex- clusiva. Madre O positivo RN A negativo. Al examen D. Roseola infantil.
ictericia hasta el abdomen. ¿Cuál es el diagnóstico más
E. Varicela.
probable?
91.Niña de 2 años quien acude con historia de 3 días de
A. Ictericia por hemólisis incompatibilidad O A
fiebre, vómitos a los que se agrega rinorrea, tos y lesiones
B. Ictericia por hemólisis incompatibilidad Rh ulce- rativas en paladar blando y úvula. Resto del examen
normal. El diagnóstico más probable es:
C. Ictericia por falta de aporte
A. Gingivoestomatitis herpética.
D. Ictericia fisiológica
B. Herpangina.
E. Ictericia por leche materna
C. Varicela.
87.Neonato de 10 días de vida, acude a su control, recibe
LME, tiene una perdida de peso del 15%, la madre le refiere D. Fiebre faringo conjuntival.
que tiene grietas en ambos pezones. Se le toma electrolitos
E. Faringitis aguda.
tiene Na 154 meq/l. ¿Cuál es la mejor conducta a seguir?
92.Niña de 3 años quien presenta desde hace un mes
A.Enseñarle la técnica de lactancia y complementar con
“descensos” así como prurito vaginal y anal a predominio
formula láctea con el suplementador de lactancia
nocturno. Cuál de las siguientes pruebas le solicitaría:
B.Enseñarle la técnica de lactancia y complementar con
A. Cultivo de secreción vaginal.
formula láctea con biberón
B. Test de Graham.
C.Suspender la lactancia hasta que los pezones de la madre
mejoren y alimentarlo con formula láctea C. Test de Tzanc.
D.Utilizar cubre pezón para que la madre pueda continuar D. Gram de secreción vaginal.
amamantando e incrementar frecuencia de lactadas
E. KOH.
E.Conseguir que un familiar amamante al bebe hasta que los
pezones de la madre mejoren 93.Paciente de 1 año quien acude con historia de fiebre, tos
y dolor abdominal de 3 días de evolución. Al examen
88.Recién nacido a término cuyo peso al nacer se encuentra murmu- llo vesicular disminuido y crépitos en base de
por debajo del percentil 10 de la curva de crecimiento intra- hemitórax derecho. El diagnóstico más probable es:
uterino de la relación Peso/edad gestacional ¿Cómo se
clasificaría según el percentil? A. Bronquiolitis

A. Pequeño para la edad gestacional B. Bronquitis aguda

B. Bajo peso al nacer C. Neumonia

C. Muy bajo peso al nacer D. Traqueobronquitis

D. Extremadamente bajo peso al nacer4 E. Atelectasia.

E. Adecuado para la edad gestacional 94.Paciente mujer de 1 año 3 meses quien acude con fiebre,
vómitos y diarrea con moco y sangre. Se agrega al cuadro
palidez marcada y oligoanuria. Su diagnóstico más probable C. Los recursos directamente recaudados.
es:
D. El paradigma hospitalario.
A. Disentería
E. La atención de tercer nivel
B. EDA con deshidratación sin shock.
100.La productividad, es decir, relación entre insumos y
C. Intususcepción. productos, es una medida de la ……………… de un
administra- dor o empleado en cuanto al aprovechamiento
D. Síndrome urémico hemolítico. de los recursos de la organización para producir bienes y
servicios.
E. Divertículo de Meckel.
A. Eficacia
95.Recién nacido a término Peso al nacer 3800 gr. A las 10
horas de vida presenta convulsiones, su glicemia es de 20 B. Organización
mg/dl, requiere dos bolos de Dextrosa al 10% y una
velocidad de infusión de glucosa (VIG) 15 mg x Kg x min. C. Eficiencia
para mantener glicemia normales. Antecedente de un
hermano con cuadro similar. ¿Cuál es el diagnóstico más D. Gestión
probable?
E. Institucionalidad
A. Error innato del metabolismo

B. Adenoma pancreático

C. Síndrome de Beckwith Wiedemann

D. Hijo de madre diabética

E. Hipopituitarismo

96.¿Cómo se denomina a la parte de la ventilación minuto


que realmente interviene en el intercambio gaseoso?:

A. Ventilación alveolar.

B. Espacio muerto.

C. Volumen respiratorio.

D. Capacidad ventilatoria.

E. Ventilación intrínseca.

97.¿Qué tipo de articulación es la radiocarpiana?

A. Enartrosis.

B. Condilea.

C. Artrodia.

D. Trocoide.

E. Troclear.

98.El estrato lúcido de la piel se observa en:

A. Cara. SIMULACRO 4B
B. Párpados. 1.¿Cuál de las siguientes vitaminas se comporta como
C. Palma y plantas. hormona esteroidea?

D. Cuero cabelludo. A. A

E. Tórax. B. B5

99.Un aspecto que jamás debe obviarse en Planificación de C. E


los servicios de salud para la Comunidad es: D. K
A. La opinión de la comunidad. E. D
B. La tecnología más moderna. 2.La prostaciclina producida a nivel del endotelio vascular
origina:
A. Síntesis del tromboxanos. 8.La formación del polígono de Willis contribuyen las
arterias:
B. Contracción de las arterias
A. Cerebrales anteriores y cerebelosas superiores
C. Activación de la agregación plaquetaria
B. Cerebrales medias y cerebelosas posteriores
D. Inhibición de la agregación plaquetaria
C. Cerebrales anteriores y cerebrales posteriores
E. Liberación de colesterol de las arterias.
D. Cerebrales medias y cerebrales posteriores
3.¿Cuál es el transportador de glucosa que permite la
absorción de fructosa por las células del intestino y riñón? E. Comunicante media y cerebrales posteriores

A. GLUT 5 9. La arteria circunfleja en el corazón es rama de la:

B. GLUT 1 A. Coronaria izquierda

C. GLUT 2 B. Septal anterior

D. GLUT 3 C. Marginal

E. GLUT 4 D. Septal posterior E.Mamaria interna

4.En el EKG, el intervalo QT registra el siguiente evento: 10.El esófago torácico se encuentra ubicado en el
mediastino:
A. Despolarización auricular y conducción al nódulo
sinusal A. Anterior y medio

B. Despolarización ventricular y despolarización B. Medio y posterior


auricular
C. Superior y anterior
C. Despolarización ventricular y repolarización
ventricular D. Superior y medio

D. Despolarización ventricular y repolarización E. Superior y posterior


auricular
11.En un estudio transversal sobre una muestra de sujetos
E. Despolarización ventricular y conducción al nódulo representativos de una comunidad, se ha observado una
sinusal pre- valencia de HTA del 20% (intervalo de confianza del
95%: 15- 25%). Cuál de las siguientes afirmaciones es
5.El efecto inotrópico positivo de la digoxina, a nivel del cierta?:
miocardio se debe a:
A. Se tiene un 95% de confianza de que entre un 15%
A. Bloqueo del canal de sodio y un 25% de los sujetos de la muestra son hipertensos.

B. Inhibición de la Calmodulina B. La prevalencia de HTA en la comunidad es del


20%.
C. Bloqueo del receptor alfa adrenérgico
C. Se tiene un 95% de confianza de que el intervalo
D. Inhibición de la enzima Na-KATPasa 15-25% incluye el verdadero valor de la prevalencia de HTA
en dicha comunidad.
E. Inhibición de la enzima fosfodiesterasa
D. El número de sujetos estudiados es insuficiente
6. El colon ascendente esta irrigado por la arteria:
para estimar la prevalencia de HTA en dicha comunidad.
A. Hemorroidal derecha
E. Es más probable que la prevalencia de HTA en
B. Mesentérica inferior dicha comunidad sea 25% que 15%.

C. Tronco celiaco 12.Se sabe que el 50% de la población fuma y que el 10%
que fuma y es hipertensa ¿Cuál es la probabilidad (%) de
D. Cólica izquierda que un fumador sea hipertenso?
E. Mesentérica superior A. 15
7.El conducto de Stenon drena a nivel de: B. 80
A. 1° Molar superior C. 40
B. 2° Premolar superior D. 20
C. 3° PM superior E. 25
D. 3° molar superior

E. 2° Molar superior
13.La matriz de programación se construye considerando B. Tasa de ataque.
una secuencia de las siguientes fases: 1) Cronograma 2)
Costos C. Tasa de Morbilidad.

3) Actividades y tareas 4) Responsable 5) Indicador 6) Meta D. Tasa de densidad.


y asignación de recursos
E. Tasa de prevalencia.
A. 1-2-3-4-5-6

B. 6-3-2-1-5-4
19.¿Cuál de las siguientes alternativas corresponde a nivel
C. 2-3-6-5-4-1 de prevención secundario?

D. 3-5-6-2-1-4 A. Inmunizaciones.

E. 3-6-2-5-1-4 B. Eliminación de factores de riesgo.

14.La tasa de incidencia de leucemia, en población expuesta C. Cloración de agua.


a radiación ionizante es de 4 en mil y la población no
D. Realización de cribado.
expues- ta es de 0,5 en mil. ¿Cuál es el riesgo atribuible al
factor de exposición?: E. Charlas educativas.
A. 8 B. 4,5 C. 3,5 D. 4,0 E. 0,5 20.La relación porcentual entre el número de atendidos y el
número de sujetos susceptibles a recibir la atención se deno-
15.Un centro de salud programó para el año 2009 realizar
mina:
2000 controles de crecimiento y desarrollo, para 1 500 niños
menores de 5 años, utilizando 800 horas enfermera. AI A. Rendimiento
finalizar el año atendió 200 consultantes nuevos, 300
reingre- santes y 500 continuadores, utilizando 600 horas B. Concentracion
enfermera. El porcentaje de avance de meta en atenciones
C. Consultantes
fue:
D. Cobertura
A. 50
E. Grado de ocupación
B. 85
21.Varón de 3 años de edad que encontrándose
C. 25
previamente bien, comienza de forma brusca con fiebre,
D. 75 dolor de gar- ganta, estridor, disnea, acumulo de
secreciones orales y emisión de saliva por la boca. Un
E. 10 hermano de 2 años, hace 72h.ha sido hospitalizado por una
meningitis bacteriana. El agente etiológico más frecuente de
16.La Atención Primaria de la salud considera en su
ambos cuadros clínicosserá con más probabilidad.
desarrollo:
A. Haemoephilus influenzae
A. Solo participación de la comunidad.
B. Estreptococo pneumoniae
B. Los recursos naturales y humanos de la comunidad
C. Klebsiella pneumoniae
C. La participación de la comunidad bajo tutela estatal
D. Neisseria meningitidis
D. La participación solo del estado.
E. Providencia rettgeri
E. La participación conjunta de la comunidad
organizada con el apoyo del estado y otros organismos 22.Un niño de 18 meses de edad, que presenta desde hace
12 h signos catarrales, es llevado a la emergencia por haber
17. Un test es tanto más sensible en razón de:
presentado hace 20 min movimientos tónico clónico
A. Su capacidad para clasificar como negativos a las generalizados de 3 minutos de duración. Niega
personas que no padecen la enfermedad’ antecedentes re- levantes. Al examen: T:39 C, congestión
orofaringea y examen neurológico normal. El diagnóstico
B. La probabilidad de tener la enfermedad cuando el más probable es:
test ha sido positivo.
A. Encefalitis viral
C. La prevalencia de la enfermedad investigada
B. Convulsión febril simple
D. Su capacidad para clasificar como positivos a los
individuos que realmente padecen la enfermedad. C. Meningoencefalitis

E. La incidencia de la enfermedad investigada D. Epilepsia

18. Tasa utilizada para medir brotes epidémicos es: E. Convulsión febril compleja

A. Tasa de incidencia.
23.Un paciente de 6 años con síndrome nefrótico presenta B. Valorar la posibilidad de iniciar lactancia
fiebre y dolor abdominal y distensión. No deposiciones
diarrei- cas. Otros sistemas evaluados sin alteraciones C. Fraccionar las tomas de leche
significativas. ¿Cuál es el diagnóstico más probable?
D. Tranquilizar a la madre y mantener la lactancia
A. Apendicitis Aguda
E. Enseñar a la madre técnicas de lactación
B. Linfadenitis Mesentérica
28.Un niño de 7 días de vida, presenta irritabilidad y lesiones
C. Perforación intestinal vesiculares en el antebrazo izquierdo y en la cara anterior
del hemitórax izquierdo. Es un RN a término que pesó 3000
D. Peritonitis Primaria grs y examen físico previamente normal. Señale el diag-
nóstico más probable:
E. Peritonitis Secundaria
A. Sepsis neonatal
24.Un paciente de 5 años, con historia de 5 días de
evolución que inició con rinorrea, estornudos. B. Rubéola congénita
Posteriormente fiebre, cefalea y leve enrojecimiento facial
que se extiende a tronco, extremidades y no descama. El C. Toxoplasmosis
hemograma muestra leve anemia, leucopenia y
D. Citomegalovirus
plaquetopenia. El Dx. más probable es:
E. Herpes neonatal
A. Eritema infeccioso
29.Un neonato de 28 días, es traído a consulta porque hace
B. Resfrío común
2 días presenta vómitos alimentarios frecuentes. Niega
C. Rubéola fiebre y diarrea. Al examen se encuentra sequedad de piel y
mucosas, palpándose una tumoración de 1,5 cm en
D. Proceso infeccioso probablemente viral hemiabdomen superior. ¿Cuál es su diagnóstico
presuntivo?:
E. Sarampión
A. Atresia intestinal
25.Paciente de 2 años y 6 meses, es llevada al centro de
salud por presentar desde hace 3 días, fiebre y dolor B. Hipertrofia del píloro
abdominal. Presenta también deposiciones disminuidas en
consistencia, frecuentes, con moco y sangre. El cuadro C. Atresia esofágica
clínico corres- ponde a una diarrea:
D. Megacolon congénito
A. Causada por parásitos
E. Estenosis intestinal
B. Aguda acuosa
30.Un recién nacido mediante fórceps no mueve el brazo
C. Aguda disentérica izquierdo, el que conserva en rotación interna anulado, con
el antebrazo en extensión y pronación; el reflejo de Moro es
D. Causada por virus asimétrico. El diagnóstico más probable es:

E. Persistente A. Fractura de clavícula izquierda.

26.Un lactante de 5 meses, sin antecedentes de importancia, B. Fractura de húmero izquierdo.


presenta desde hace 3 días, fiebre, rinorrea acuosa y estor-
nudos: Desde hace 12h se agrega tos y respiración rápida. C. Parálisis de Erb-Duchenne.
En la exploración presenta taquipnea, tiraje subcostal,
D. Parálisis de Klumpke.
alargamiento de la espiración, subcrepitantes y sibilantes
bilaterales. ¿Cuál es el diagnóstico más probable? E. Ninguno de los anteriores.
A. Neumonía 31.Una niña de 2 años acude a urgencias por cuadro de
disnea, estridor y tos perruna. En la exploración destaca
B. Crisis asmatica
rinorrea, orofaringe congestiva y fiebre de 38,5oC. A la
C. Fibrosis Quística auscultación pulmonar se aprecia tiempo inspiratorio
prolongado. El diagnóstico más probable será:
D. Bronquiolitis
A. Pertussis.
E. Cuerpo extraño bronquial
B. Bronquiolitis.
27.Un paciente de 5 meses de edad alimentado con
lactancia materna, la madre acude a consulta por pobre C. Laringotraqueitis aguda
ganancia de peso, a los 4 meses el peso era de 6800 gr. y el
D. Laringitis espasmódica.
peso actual es de 7400 gr La mejor sugerencia a la madre
es: E. Epiglotitis.
A. Agregar sucedáneo de leche materna 32.Señale el tratamiento empírico de la sepsis neonatal
A. Ampicilina + gentamicina A. Staphylococcus epidermidis.

B. Penicilina + gentamicina B. Staphylococcus aureus.

C. Cefotaxima mas vancomicina C. Streptococcus pyogenes.

D. Penicilina + cloranfenicol D. Streptococcus pneumoniae.

E. Gentaminica + clindamicina E. Streptococcus viridans.

33.La fontanela anterior generalmente se cierra: 39.Se evalúa a una paciente que presenta una altura uterina
de 25 cms. en una gestación de 34 semanas corroborada
A. A los 6 meses de edad. por ecografía del primer trimestre. La ecografía actual
menciona un peso estimado fetal de 1200 gramos, la
B. Entre 6 y 12 meses de edad.
circunferen- cia cefálica corresponde a 32 semanas, la
C. Entre 9 y 18 meses de edad. circunferencia abdominal a 27 semanas y la longitud de
femur a 30 semanas.¿Cuál de las siguientes causas podría
D. Entre 18 y 24 meses de edad. ser la causa de la restricción de crecimiento intrauterino
(RCIU) en este caso?
E. Entre 20 y 28 meses de edad.
A. Diabetes gestacional
34.La presencia de dificultad respiratoria en el recién nacido
prematuro, ¿Cuál es la causa probable más frecuente? B. Preeclampsia severa
A. Taquipnea transitoria. C. Síndrome de Down
B. Síndrome de aspiración meconial. D. Síndrome de Pateau
C. Membrana Hialina. E. Infección por toxoplasmosis
D. Neumonía. 40.Primigesta de 35 semanas acude a emergencia refiriendo
hace 30 minutos dolor abdominal y sangrado vaginal oscuro
E. Neumotórax
de 300cc. Al examen detecta PA 90/60 mmHg, pulso 120 x’,
35.Tiempo (en días) de incubación del sarampión: respiraciones 24 x’, afebril. Abdomen con altura uterina de
40 cm, contracciones uterinas cada 2 minutos, 50 segundos
A.5 de duración, intensidad ++++, latidos fetales 100 a 120 x’; en
el examen con especulo sangrado a través del cérvix, el cual
B. 10 a 14 se encuentra cerrado y largo. Se indica vía EV, hemograma
grupo y Rh, prueba cruzada y perfil de coagulación. ¿Cuál
C. 15 a 20
debe ser la siguiente conducta?
D. 21 a 28
A. Ecografía transvaginal
E. 30 a 45
B. Perfil biofísico

C. Inducción de parto con oxitocina


36.¿Cuál es la complicación más frecuente de la varicela?
D. Amniotomía y estimulación con oxitocina
A. Ataxia
E. Cesárea
B. Parálisis facial
41.Paciente de 45 años de edad, portadora de cáncer de
C. Convulsiones cérvix, estadio clínico IIIb. El tratamiento de elección es:

D. Pancreatitis A. Histerectomía total.

E. Sobreinfecciones de piel. B. Histerectomía radical.

37.Según el esquema de vacunación nacional en lactantes, C. Radioterapia.


la vacuna de hepatitis B debe administrarse:
D. Quimioterapia.
A. Dos dosis con intervalo de 6 meses
E. Hormonoterapia.
B. Una dosis y refuerzo al año
42.Paciente de 25 años, acude a emergencia por sangrado
C. Al nacer y a los 6 meses vaginal acompañado de coágulos. Al tacto vaginal: cuello
ute-rino con orificios externo e interno abiertos., ¿Cuál es el
D. A los 2,4 y 6 meses diagnóstico probable?
E. A los 3,6 y 9 meses A. Aborto incompleto
38.El agente etiológico que más frecuentemente produce B. Amenaza de aborto
artritis séptica en escolares es:
C. Metrorragia disfuncional C. Ceftriaxona

D. Mola hidatiforme D. Metronidazol

E. Embarazo ectópico E. Penicilina benzatínica

43.Multigesta a término, no trabajo de parto, que ingresa por 47.Multípara de 40 años de edad. Consulta por “sensación
fiebre, malestar general, escalofríos, taquicardia, dolor ute- de bulto en genitales” que se presenta cuando hace
rino y pérdida de líquido amniótico claro por vagina desde esfuerzos físicos. Al examen se observa protrusión de la
hace 24 horas. Al examen: latidos fetales: 152 por minuto, pared vaginal anterior y posterior que sobrepasa el introito.
cérvix blando, posición intermedia, incorporado 50%, ¿Cuál es el diagnóstico?:
dilatación dehiscente 1 cm, estación cefálica 2, pelvis
ginecoide compatible. Peso estimado fetal: 3.000 g. A. Uretrocele
Hemograma con leucocitosis y desviación izquierda. El
B. Histerocele
diagnóstico más pro- bable es... y su manejo más indicado
es...: C. Cistouretrocele
A. Corioamnionitis /antibiótico-terapia y cesárea D. Enterocele
B. Rotura prematura de membranas / antibioticoterapia y E. Cistorectocele
cesárea
48.Multípara de 45 años de edad, que refiere estreñimiento y
C. Corioamnionitis /antibioticoterapia e inducción de parto sangrado rectal ocasionado por las deposiciones. ¿Cuál es
su diagnóstico?:
D. Rotura prematura de membranas / antibioticoterapia e
inducción de parto A. Hemorroides
E. Corioamnionitis / antibioticoterapia y expectación B. Absceso perianal
44.Primigesta de 17 años, 06 meses de amenorrea, sin C. Fístula anal
control prenatal y que acude a consulta por Emergencia por
presentar desde hace 1 semana cefalea e hinchazón de las D. Estenosis anal
piernas, desde ayer lumbalgia y poliaquiuria, y desde hace 1
E. Proctitis
hora pérdida de flujo vaginal acuoso en regular cantidad con
olor a lejía. Al examen clínico: adelgazada, pálida y ansiosa, 49.Paciente de 28 años, hace 39 días fue sometida a
LOTEP, PA: 130/90mm Hg, AU de 32 cm, DU: legrado uterino; presenta hemoptisis, ginecorragia escasa.
(2/10)(++)(30”), LCF: dos focos 160 y 140 lat/min; Ex. Examen clínico PA: 120/70 mmHg, FC: 102 x minuto, útero
Ginecológico.: Se constata pérdida de LA claro en regular de 14 cm, cérvix normal. Dosaje de HCG-B: 32000 Ul/ml.
largo, OE y canal cervical cerrado. ¿Cuál sería el Hemog- lobina: 10.2 g/dL. ¿Cuál es el diagnóstico probable?
diagnóstico?
A. Reinicio del ciclo menstrual
A. Amenaza de parto pretérmino
B. Coriocarcinoma
B. Rotura prematura de membranas
C. Retención de restos placentarios
C. Hipertensión inducida por el embarazo
D. Endometritis
D. Trabajo de parto prematuro
E. Adenocarcinoma de endometrio
E. Infección del tracto urinario
50.Un aborto frustro, la complicación que se asocia con
45.Paciente de 52 años de edad, con síndrome climatérico y mayor frecuencia es:
cuadro de hemorragia uterina anormal. ¿Cuál es la conducta
inicial más importante?: A. Hiperemesis gravídica

A. Histerosalpingografía B. Sinequias intrauterinas

B. Ultrasonido transvaginal C. Coagulopatía por consumo

C. Histeroscopia D. Hipertensión.

D. Legrado uterino biópsico E. Endometritis

E. Laparoscopia F. 51.El puerperio inmediato comprende hasta:

46.El antibiótico de elección en una mujer de 32 años de A. Las primeras dos horas post parto
edad, con cervicitis mucopurulenta y PCR positivo para
Chla- mydia trachomatis es: B. Las primeras 12 horas post parto

A. Azitromicina C. Las primeras 48 horas post parto

B. Ciprofloxacina D. La primera semana post parto


E. Las primeras 24 horas post parto B. Solicitar ecografía

52.¿Qué maniobra obstétrica permite comprobar el C. Realizar TAC abdominal.


encajamiento de la presentación fetal?
D. Laparotomía urgente.
A. Tercera maniobra de Leopold
E. Lavado peritoneal diagnóstico.
B. Cuarta maniobra de Leopold
58.Mujer de 13 años con dolor en mesogastrio que se
C. Peloteo irradia, hacia fosa iliaca derecha, se acompaña de náuseas,
y vómi- tos. Al examen: temperatura de 38,4 C, abdomen
D. Primera maniobra de Leopold doloroso en hemiabdomen inferior derecho. Laboratorio:
Sedimento urinario: 6-8 hematíes x campo. Hemograma:
E. Segunda maniobra de Leopold
leucocitosis de 14 000 x mm3, FUR hace 30 días. Tiene
53.Los desgarros perineales de tercer grado comprometen: como presunción diagnóstica apendicitis aguda. ¿Cuál de
las siguientes patologías se considera en el diagnóstico
A. Mucosa rectal exponiendo la luz rectal diferencial?
B. Piel, mucosa vaginal, aponeurosis y músculos del A. Poliposis intestinal
cuerpo perineal
B. Vólvulo de ciego
C. Piel perineal y mucosa vaginal
C. TBC entero peritoneal
D. Piel, mucosa vaginal, cuerpo perineal y esfínter anal
D. Litiasis vesical
E. Horquilla, piel perineal y mucosa vaginal
E. Diverticulitis de Meckel
54.¿Cuál de los tipos de virus de papiloma humano (VPH)
está asociado al carcinoma del cuello uterino? 59.Mujer de 30 años que sufre quemadura de segundo
grado en la cabeza, la extremidad superior derecha y cara
A. 44 anterior del tórax y el abdomen. ¿Qué porcentaje de
superficie quemada presenta?:
B. 6
A. 36
C. 11
B. 45
D. 42
C. 18
E. 16
D. 27
55. La causa más frecuente de síndrome de Asherman
es: E. 30
A. Miomectomía 60.Un paciente de 25 años ha sufrido una caída de la
bicicleta golpeándose en el mentón, limitación dolorosa de la
B. Tuberculosis genital
aper- tura bucal con desviación hacia el lado derecho y
C. Legrado uterino diagnóstico maloclusión dentaria consistente en falta de contacto a nivel
de los molares del lado izquierdo. ¿Qué fractura facial
D. Infección endouterina postparto sospecharía y qué prueba solicitaría para diagnosticarla?:

E. Infección endouterina postaborto A. Cóndilo mandibular izquierdo/Tomografía axial


computarizada.
56.Jorge tiene 4 años y presenta desde hace 2 semanas
rinorrea purulenta unilateral, cual de los siguientes es el B. Fractura malar/Tomografía emisión positrones.
diag- nóstico probable.
C. Cóndilo mandibular derecho/Ortopantomografía.
A. Rinitis alérgica.
D. Cóndilo mandibular derecho/Resonancia magnética
B. Cuerpo extraño intranasal. nuclear.

C. Pólipo nasal. E. Fractura etmoides/Gammagrafía ósea

D. Etmoiditis supurativa. 61.Paciente de 50 años de edad que consulta por dolor en la


fosa renal, poliaquiuria, disuria y hematuria. En el análisis de
E. Atresia de coanas unilateral. orina se observa piuria y pH ácido con cultivos
repetidamente negativos ¿Cuál sería la primera posibilidad
57.Paciente de 23 años presenta una herida por arma
diag- nóstica, de entre las siguientes?
blanca penetrante en abdomen, con evisceración de varias
asas de intestino delgado, ¿cuál es la decisión más A. Pielonefritis aguda.
apropiada?:
B. Síndrome nefrítico.
A. Observación durante 24-48 h.
C. Tuberculosis genitourinaria.
D. Prostatitis aguda. B. Colangitis aguda.

E. Carcinoma renal de células claras. C. Hepatocarcinoma.

62.Paciente con quemaduras de tercer grado, presenta D. Pancreatitis aguda.


fiebre persistente y compromiso del estado general. De las
heridas se aisló Pseudomona aeuriginosa. ¿Cuál de los E. Síndrome hepatorrenal.
siguientes antibióticos es el más apropiado?:
67.De las siguientes, la prioridad en la atención de un
A. Cefaclor. politraumatizado es evaluar la lesión de:

B. Cefalotina. A. Piel

C. Cefoxitima. B. Vasos sanguíneos

D. Ceftazidima. C. Músculos

E. Cefuroxima. D. Huesos

63.Paciente de 30 años de edad presenta, tras sufrir un E. Nervios


accidente de moto, una fractura expuesta Grado III de tibia y
68.Una de las complicaciones inmediatas en la cirugía
peroné sin afección neurovascular. ¿Cuál sería la conducta
gástrica es:
a seguir?:
A. Diarrea coleriforme.
A. Limpieza de la herida y osteosíntesis inmediata con
placa y tornillos. B. Dumping.
B. Limpieza de la herida, estabilización provisional de C. Gastritis alcalina.
la fractura con tracción continua y yeso.
D. Hernia diafragmática.
C. Limpieza de la herida, estabilización provisional con
tracción, y posterior osteosíntesis con placa y tornillos. E. Infección broncopulmonar

D. Limpieza de la herida y estabilización de la fractura 69.La fístula biliodigestiva más frecuente es:
con fijador externo.
A. Bilio – biliar
E. Cierre de la herida y yeso con tracción.
B. Colecisto – colónica
64.Varón de 48 años, sometido a una intervención quirúrgica
C. Colecisto – duodenal
electiva bajo anestesia espinal, PA: 70/40 mmHg. No hay
evidencia de hemorragia ni sepsis. El tratamiento inicial D. Colecisto – gástrica
consiste en la administración de:
E. Colecisto – yeyunal
A. Adrenalina.
70.En el manejo de un paciente politraumatizado ¿Cuál de
B. Solución salina. los siguientes problemas es el que tiene prioridad?:
C. Corticoides. A. Examen neurológico inicial para TEC
D. Dopamina. B. Tratar el shock
E. Dextrano. C. Permeabilizar vías aéreas
65.¿Qué tipo de hernia protruye por el triángulo de la pared D. Tratar las fracturas expuestas
abdominal que está delimitado lateralmente por los vasos
epigástricos, medialmente por el borde lateral del músculo E. Observar pupilas.
recto anterior del abdomen y por el ligamento inguinal
inferiormente? 71.Un hombre de 55 años, fumador, obeso y dislipidemico,
presenta una PA de 170/110mmHg. Está medicado con ena-
A. Hemia crural. lapril 10 mg, hidroclorotiacida 25 mg y Amlodipino 5 mg/dia.
Hasta hace 6 meses sus presiones promedio eran de
B. Hemia inguinal indirecta. 130/85mm Hg. Entre los exámenes complementarios se
destaca una elevación de la creatina, respecto de los valores
C. Hernia pectínea.
de hace 5 meses. ¿Cuál de las siguientes afirmaciones es
D. Hernia epigástrica. MÁS probable en este paciente?:

E. Hernia inguinal directa. A. Tiene una hipertensión refractaria, probablemente


secundaria a una estenosis ateromatosa de la arteria renal.
66.Los escalofríos, fiebre e ictericia más dolor en
hipocondrio derecho es característico de: B. No es una hipertensión refractaria, previamente
debería agregarse un betabloqueante para diagnosticarla
A. Perforaciónduodenal.
C. Es altamente probable que presenta una difi- cultad respiratoria. Al examen: cuadriparesia hipotónica
feocromocitoma. e hiporreflexia. El diagnostico probable es:

D. Tiene una hipertensión refractaria, probablemente A. Polimiositis aguda.


secundaria a una displasia fibromuscular de una arteria
renal. B. Síndrome de Guillain- Barré.

E. Tiene una hipertensión refractaria, probablemente C. Polineuropatía carencial.


secundaria a una displasia fibromuscular de una arteria
D. Mielitis transversa.
renal.
E. Botulismo.
72.Paciente varón de 28 años que presenta 9 días de fiebre
y malestar general. No refiere antecedente de 76.Mujer de 76 años con obesidad y diabetes mellitus tipo 2
hospitalización ni enfermedades crónicas previas. Examen en tratamiento con Metformina. Acude a urgencias porque
físico T 40°C, resto de examen físico sin alteración. Lab: desde hace 3 días presenta fiebre de 38°C, polaquiuria,
leucocitosis, VSG, Trom- bocitosis, hemocultivos (-), PPD (-), disuria, intensa sed y disminución progresiva de su nivel de
Ziehl-Neelsen de esputo (-), aglutinaciones para S. typhi y conciencia. En la exploración física se aprecian signos de
Brucella spp. (-), resto de exámenes de laboratorio no deshidratación, presión arterial 95/54 mmHg y no hay signos
mostraron alteración. ¿Cuál de las siguientes definiciones se de focalidad neurológica. En la analítica destaca
ajusta MEJOR al presente caso?: leucocitosis, creatinina de 1.8 mg/dL ¿Cuál es el diagnóstico
más probable?
A. Fiebre de origen desconocido clásico
A. Coma hiperglucémico hiperosmolar no cetósico
B. Fiebre prolongada sin foco aparente
B. Insuficiencia cardíaca
C. Fiebre de origen desconocido en paciente
neutropénicos C. Hematoma subdural
D. Fiebre de origen desconocido en paciente con VIH D. Insuficiencia renal de causa obstructiva
E. Fiebre no prolongada sin foco aparente E. Cetoacidosis diabética con coma
73.Paciente mujer de 78 años con antecedente de DM que 77.Mujer de 35 años de edad, sin antecedentes de
fue intervenida por reemplazo completo de cadera, al tercer enfermedad digestiva previa ni ingesta de alcohol ni AINEs.
día PO presenta de manera súbita ansiedad, inquietud En el curso de un cuadro agudo de náuseas y vómitos
motora, alucinaciones visuales, insomnio, falsos intensos presenta sangre rutilante. El diagnóstico MÁS
reconocimientos y llanto inmotivado. Examen físico: probable es:
mucosas semisecas, entrevista: se mostró, somnolienta,
desorientada, hipoprosexia, con amnesia de hechos A. Síndrome de Mallory Weiss.
recientes y afecto que varía desde llanto inmotivado y
B. Gastritis erosiva aguda.
ansiedad extrema con intentos de autoa- gresividad. El
diagnóstico que explica las manifestaciones mentales es: C. Úlcera péptica sangrante.
A. Delirio. D. Neoplasia gástrica.
B. Demencia vascular. E. Várices esofágicas.
C. Depresión mayor + Psicosis. 78.Un hombre de 30 años presenta de forma brusca astenia
e ictericia, detectándose unas transaminasas (ALT y AST)
D. Episodio psicótico agudo.
superiores a 2000 UI/l. Los marcadores serológicos
E. Trastorno esquizoafectivo. muestran el siguiente patrón: anti-VHA IgM negativo, HBsAg
negativo, anti-HBc IgM positivo y anti-VHC negativo. ¿Cuál
74.Paciente de 70 años de edad, con anemia crónica es su diagnóstico?
severa. VCM 101fl y HCM 25pg. Considerando estos valores
¿Cómo clasifica a la anemia? A. Hepatitis crónica B.

A. Macrocitica hipocrómica. B. Hepatitis aguda no viral.

B. Microcítica hipocrómica. C. Hepatitis aguda B.

C. Normocítica normocrómica. D. Sobreinfección por virus D (delta).

D. Macrocitica normocrómica. E. Hepatitis aguda A y B.

E. Microcítica normocrómica crónica. 79.Una biopsia renal nos la informan como glomérulos
hiperplásicos por proliferación de células endoteliales y
75.Paciente de 25 años de edad. Hace dos semanas mesote- liales e infiltración por leucocitos
presento cuadro de influenza. Desde ayer parestesias en polimorfonucleares. Tras la tinción con colorantes tricrómico
miembros inferiores y debilidad progresiva que imposibilita de Masson se observan depósitos granulares de IgG y C3en
deambular. Hoy pérdida de fuerza en miembros superiores y la membrana basal. Con estos datos el diagnóstico más
probable es de:
A. Nefropatía por cambios mínimos E. Estudio HLA.

B. Glomeruloesclerosis segmentaria y focal 84.Varón de 50 años, con dolor lumbar persistente,


insuficiencia renal crónica sin factores de riesgo, con anemia
C. Glomerulonefritis membranosa modera- da, calcio sérico 11,5 mg/dL, proteínas totales 8,6
g/dL. La primera posibilidad diagnostica es:
D. Glomerulonefritis mesangial
A. Mieloma múltiple.
E. Glomerulonefritis aguda post infecciosa
B. Linfoma óseo.
80.Varón de 20 años, sufre accidente de tránsito en su moto,
se presenta monoplejía e hipoestesia del miembro superior C. Macroglobulinemia de Waldestrom
derecho ¿Cuál es la estructura nerviosa lesionada?: D. Osteoporosis primaria.
A. Plexo Braquial E. TBC renal.
B. Nervio mediano 85.Paciente varón de 30 años acude a consultorio por
presentar disfagia de larga data, ha progresado en las
C. Nervio musculo cutáneo
últimas se- manas hasta tener dificultades para ingerir
D. Plexo cervical líquidos, así mismo, manifiesta episodios esporádicos de
regurgitación no ácida de alimentos y ha tenido, el año
E. Nervio cubital pasado neumonía en 2 oportunidades. ¿Qué examen seria
de mayor rendimiento para llegar al diagnóstico?:
81.Un individuo que nunca ha sentido angustia por
exponerse al público, presenta una crisis de angustia en una A. Endoscopía digestiva alta.
conferen-cia y le aterroriza el pensar en tener otra
conferencia. Si presenta angustia únicamente frente a la B. Radiología esofagogástrica con bario.
exposición al público¿cuál es el diagnostico apropiado?
C. Estudio de vaciamiento esofágico con
A. Agorafobia sin crisis de angustia radioisótopos.

B. Trastorno de pánico D. Manometría esofágica.

C. Trastorno de angustia generalizada E. Phmetría de 24 hrs.

D. Fobia social 86.Varón de 25 años de edad, ganadero, procedente de


Puno, con antecedente de tratamiento antituberculoso hace
E. Trastorno de angustia con agorafobia 8 años. Consulta por hemoptisis, pérdida de peso, tos,
fiebre, esputo purulento, niega vómica, BK seriado negativo.
82.Hombre de 67 años, hipertenso y fumador que acude a
En la ra- diografía de tórax: Imagen cavitario con masa y
Emergencia por disnea de grandes esfuerzos en el último
menisco aéreo. Gérmenes grampositivos y gramnegativos
mes. La auscultación demuestra disminución del murmullo
en esputo.
vesicular sin otros hallazgos. La Rx de tórax y el ECG
realiza- dos son normales. Los valores en sangre de péptido ¿Cuál es el diagnóstico más probable?
natriurético tipo B son de 60 pg/mL. (valores normales
<100pg/ mL.). ¿Cuál de los siguientes diagnósticos es el A. Quiste hidatídico complicado
menos probable?:
B. Reactivación Tuberculosa
A. Disfunción ventricular sistólica.
C. Reinfección tuberculosa
B. Enfermedad pulmonar obstructiva crónica.
D. Absceso pulmonar
C. Asma.
E. Neoplasia infectada
D. Hipertensión pulmonar primaria.
87.Paciente mujer de 34 años, puérpera de 1 semana post
E. Tromboembolismo pulmonar. cesárea, que presenta súbitamente dolor torácico y disnea
con esputo hemoptoico. Al examen: PA 120/70 FC 110 FR
83.Paciente de 24 años, sin antecedentes de haber recibido 30 T 36°C. Tórax: MV pasa en ACP, no sibilantes. Rx tórax
quimioterapia ni radioterapia previamente, con leucopenia de Normal. AGA: pH: 7.46 pO2: 65 pCO2: 24 HCO3: 22 EKG:
1.100/ml, trombopenia de 22.000/ml y anemia de 6.8 g/dl de taquicardia sinusal, no desnivel ST, resto sin alteraciones.
Hb, sin blastos en sangre periférica y con aspirado de Hemograma, TP, TTP, plaquetas, CPK, MB: Sin
médula ósea muy hipocelular. Señale qué prueba de alteraciones. Luego de iniciar oxígeno, lo inmediato siguiente
confirmación está indicada: sería:
A. Biopsia de médula ósea. A. Iniciar anticoagulación con heparina.
B. Estudio citoquímico. B. Pulsos de corticoides, considerar ciclofosfamida.
C. Test de Ham. C. AAS, nitratos, analgesia y HBPM
D. Estudio citogénico. D. TAC helicoidal
E. Solicitar Dímero D y troponina T. A. Hombros

88.Paciente de 78 años de edad, afecto de EPOC muy B. Interfalángicas distales


grave, (FEV1, 25%), ex fumador, con tres hospitalizaciones
por exacerbación de EPOC durante el último año. Es C. Metatarsofalángicas
ingresado de Emergencia por disnea muy severa y febrícula.
D. Tobillos
No se ausculta murmullo vesicular y el esputo es verdoso y
abundante. La gasometría arterial (FIO2, 21%), muestra E. Metacarpofalángicas
PaO2: 36mmHg; PaCO2: 63 mmHg; pH 7,28; HCO3:31;
placa de tórax sin infiltrados. ¿Cuál sería, entre las 92.La coexistencia de gastritis atrófica y anemia macrocítica
siguientes, la pauta terapéutica más recomendable?: por malabsorción de vitamina B12 debida a falta de secre-
ción de factor intrínseco se denomina:
A. Combinar dos broncodilatadores de acción corta,
nebulizados, glucocorticoides sistémicos y un antibiótico de A. Anemia refractaria
amplio espectro, con empleo de oxigenoterapia de flujo
B. Anemia perniciosa
reducido
C. Anemia sideropénica
B. Combinar dos broncodilatadores de acción
prolongada, inhalados, glucocorticoides sistémicos y un D. Síndrome de malabsorción de vitamina B12
antibiótico de amplio espectro, con el empleo de ventilación
mecánica no invasiva E. Gastritis anemizante

C. Combinar dos broncodilatadores de acción corta, 93.El reflujo gastroesofágico crónico puede producir:
nebulizados, glucocorticoides inhalados, con el empleo de
ven- tilación mecánica no invasiva A. Candidiasis esofágica

D. Combinar dos broncodilatadores de acción corta, B. Hernia hiatal


nebulizados, glucocorticoides sistémicos y un antibiótico de
C. Infección por Helicobacter pylori de la mucosa
am- plio espectro, con el empleo de ventilación mecánica
gástrica
no invasiva.
D. Divertículos esofágicos
E. Combinar dos broncodilatadores de acción
prolongada, nebulizados, glucocorticoides inhalados y un E. Esófago de Barrett
antibiótico de amplio espectro, con empleo de
oxigenoterapia de flujo reducido. 94.La causa más frecuente de hemorragia digestiva alta es:

89.Un hombre de 62 años acude a Emergencia por A. Várices del polo superior del estómago
presentar de forma brusca mareo e inestabilidad. En la
B. Tumores
exploración se encuentra un nistagmo horizontal, un
síndrome de Horner derecho, una pérdida de la sensibilidad C. Traumatismo
dolorosa en la hemicara derecha y braquiocrural izquierda,
una ataxia de miembros derecho y disfagia. ¿Cuál sería D. Ulcera péptica
sospecha diag- nóstica?:
E. Prolapso de mucosa gástrica
A. Infarto de la arteria basilar.
95.En el diagnóstico de hepatitis viral B aguda se encuentra
B. Infarto de la protuberancia. uno de los siguientes:

C. Infarto de la arteria vertebral izquierda. A. Presencia de antígeno de superficie.

D. Infarto de la arteria cerebral posterior derecha. B. Presencia de Anti HBc Ig G

E. Infarto lateral bulbar derecho. C. Detección del RNA HBV por reacción de cadena de
polimerasa.
90.¿Cuál de los siguientes hallazgos es característico en los
pacientes con insuficiencia renal AGUDA, debido a necrosis D. Desaparición del anti – HBc.

tubular aguda tóxica?: E. Elevación del anti – HBc.

A. Excreción fraccionada de Na+ > 1 96.La causa más común de muerte debida a enfermedad de
Kawasaki en la fase inicial de la enfermedad es:
B. Osmolaridad urinaria> 500
A. Miocarditis
C. Concentración de Na+ en orina < 10
B. Rotura de un aneurisma
D. Densidad urinaria> 1,2
C. Ictus
E. Índice de insuficiencia renal < 1
D. Infarto de miocardio
91.De las siguientes articulaciones, la más frecuentemente
afectadas por la artritis reumatoidea son: E. Pericarditis
97.¿Cuáles de las siguientes manifestaciones son típicas de
la enfermedad de Parkinson?

A. Temblor de acción e hiperreflexia

B. Rigidez muscular y Babinski espontáneo

C. Mioclonías de torsión

D. Temblor de reposo y rigidez

E. Amnesia de fijación y flacidez

98.¿Cuál de los siguientes fármacos reduce la mortalidad en


los enfermos con insuficiencia cardiaca congestiva?

A. Digital.

B. Furosemida.

C. Enalapril.

D. Procainamida.

E. Aspirina.

99.En un paciente esplenectomizado, la infección más


frecuente se debe a:

A. Estreptococo pneumoniae

B. Neisseria meningitides

C. Hemophilus influenzae

D. Estafilococo aureus

E. Estafilococo epidermides
EXAMEN N° 11 –
100.¿Cuál es la causa más frecuente de MEC en adultos
jóvenes?: SIMULACRO 1B

A. S. Pneumoniae MEDICINA INTERNA

B. Neiseria meningitidis 1. Una paciente de 44 años, sometida a mastectomía


izquierda con vaciamiento ganglionar, consulta un año
C. Haenophylus influenzae después de la intervención por un cuadro febril con celulitis
en brazo izquierdo. ¿Cuál es el agente etiológico más
D. Listeria monocitógenes
probable?
E. coli
A. Salmonella enteritidis.

B. Streptococcus agalactiae.

C. Escherichia coli.

D. Streptococcus pyogenes.

E. Pasteurella multocida.

2. Un paciente de 28 años, aficionado a la caza, presenta


una lesión roja en muslo que se aclara en el centro a medida
que se extiende. Con respecto a la enfermedad de este
paciente, señale la respuesta correcta:

A. Se adquiere por contacto con herbívoros.

B. La presencia de una «mancha» negra en la piel es un


dato muy específico.

C. Puede complicarse con bloqueo auriculoventricular.

D. El tratamiento de elección es Doxiciclina,


independientemente de los órganos afectados.
E. Es probable que en los próximos días se asocie artritis de 7. Un sujeto sano tiene una ventilación minuto de 5 litros con
la cadera ipsilateral. una frecuencia de 16 respiraciones por minuto; si dismi-
nuye su frecuencia respiratoria a 8 respiraciones por minuto
3.¿Cuál es la forma habitual de adquirir la infección por y mantiene el mismo volumen ventilatorio por minuto (5
Coxiella burnetii? litros); señale que es lo que pasará con el valor del espacio
muerto fisiológico (VD):
A. Inhalación.
A. Aumenta.
B. Ingesta de carne contaminada.
B. Disminuye.
C. Picadura de garrapata.
C. Permanece igual.
D. Contacto con agua contaminada.
D. Variará de acuerdo a la nueva frecuencia.
E. De persona a persona.
E. Ninguno de los anteriores.
4. Paciente de 20 años, que hace 4 días comienza con dolor
brusco de oído derecho, seguido de supuración maloliente, 8.El soplo cardiaco en la miocardiopatía hipertrófica se
por lo que es tratado por su médico de cabecera con intensifica por, excepto:
ciprofloxacino en gotas óticas. La fiebre, que al inicio del
cuadro era de 38 °C, se eleva hasta 40 °C, con debilidad y A. Posición en cuclillas.
escalofríos. Al llegar a Urgencias, la TA es de 90/50, y el
paciente presenta tortícolis con dolor y enrojecimiento B. Ejercicio.
cutáneo paralelo al esternocleidomastoideo derecho, y tos
C. Isoproterenol.
intensa con dolor torácico. En la Rx de tórax se observan
infiltrados redondeados bilaterales. Señale la afirmación D. Digital.
correcta:
E. Maniobra de Valsalva.
A. La bacteria causante forma parte de la flora orofaríngea.
9.La causa más común de pericarditis constrictiva es:
B. Los infiltrados pulmonares son secundarios a endocarditis
tricuspídea. A. Fiebre reumática.

C. El dolor cervical nos indica la presencia de un B. Neoplasia.


adenoflemón.
C. Pericarditis aguda benigna.
D. En el hemocultivo se aislarán cocos positivos aerobios.
D. Tuberculosis.
E.El tratamiento de elección es cefotaxima i.v.
E. No determinada.
5.La respuesta dérmica al antígeno de la tuberculina es
10.La clonidina es un fármaco antihipertensivo de tipo:
positiva después de...... iniciada la infección:
A. Agonista adrenérgico.
A. 7 – 10 días.
B. Antagonista adrenérgico.
B. 6 – 12 meses.
C. Antagonista adrenérgico mixto.
C. 6 – 12 semanas.
D. Vasodilatador arterial.
D. 12 – 18 meses.
E. Bloqueador ganglionar.
E. 2 – 6 semanas.
11.Un paciente de 55 años ha sido diagnosticado de
6.Paciente de 40 años de edad, fumador desde los 18 años
miocardiopatía alcohólica e insuficiencia cardíaca
y sin antecedentes patológicos de interés. Consulta por
congestiva. Tiene una capacidad funcional IV/IV de la New
cuadro de 24 horas de evolución de escalofríos, fiebre de 39
York Heart Association (NYHA) e hipertensión esencial de
°C y tos con expectoración. La radiografía simple de tórax
180/110 mmHg.¿Cuál de los siguientes sería el tratamiento
pone de manifiesto un aumento de densidad con patrón
más apropiado para reducir la tensión arterial del paciente?
alveolar en el lóbulo superior derecho y el paciente es
diagnosticado de neumonía adquirida en la comunidad. A. Bloqueantes α-adrenérgicos.
¿Cuál es el agente etiológico más frecuente y por lo tanto de
obligada cobertura al elegir el tratamiento antibiótico B .Nitratos.
empírico?
C. Inhibidores de la enzima convertidora de la angiotensina.
A. Mycoplasma pneumoniae.
D. Bloqueantes β-adrenérgicos.
B. Chlamydia pneumoniae.
E. Antagonistas de los canales del calcio.
C. Streptococcus pneumoniae.
12.¿En cuál de las siguientes neoplasias gástricas en
D. Legionella pneumophila serogrupo 1. estadio incipiente, se puede considerar como primera opción
tera-péutica la erradicación de H. pylori con antibioterapia e
E. Haemophilus influenzae. inhibidores de la bomba de protones?
A. Adenocarcinorna gástrico tipo difuso. 18.En relación al aumento de la prevalencia de la obesidad y
la inactividad física en la población, se ha incrementado la
B. Adenocarcinorna gástrico de tipo intestinal. frecuencia de un trastorno denominado síndrome
metabólico. Señale cuál de las siguientes características NO
C. Linfoma MALT gástrico.
forma parte del síndrome metabólico:
D. Linfoma alto grado gástrico.
A. Hipertrigliceridemia.
E. Linfoma de Hodgkin gástrico.
B. Hipertensión arterial.
13.El hallazgo más importante en la Acalasia de esófago es:
C. Hiperglucemia en ayunas.
A. La estenosis del extremo distal del esófago.
D. Aumento del perímetro de la cintura abdominal
B. La dilatación esofágica marcada.
E. Aumento de la concentración plasmática de colesterol-
C. La ausencia de los plexo nerviosos - mioesofágicos. LDL.

D. La regurgitación alimentaria nocturna. 19.Uno de los siguientes signos NO es característico de la


enfermedad de Graves Basedow:
E. La presencia de disfagia que puede llevar a ser muy
intensa. A. Hipertiroidismo.

14. El carcinoma primario de hígado acompaña más a la: B. Macroglosia.

A. Hemocromatosis. C. Exoftalmos.

B. Cirrosis. D. Bocio difuso.

C. Gastritis hemorrágica. E. Mixedema pretibial.

D. Colecistitis calculosa.

E. Quiste hidático del hígado.

15.Las convulsiones febriles en la infancia son: 20.En la síntesis de hormonas esteroideas a partir del
colesterol, el primer paso regulatorio es la formación de:
A. Parciales mioclónicas.
A. Aldosterona.
B. Parciales motoras.
B. Cortisol.
C. Generalizadas mioclónicas.
C. 17-hidroxiprogesterona.
D. Espasmos infantiles.
D. Pregnenolona.
E. Generalizadas tónico - clónicas.
E. Tertosterona.
16.Sobre los síndromes miasténicos, señale lo correcto:
21.En el diagnóstico de polimiositis, la enzima que se
A. En la miastenia gravis, las anomalías tímicas son poco incrementa significativamente es:
frecuentes.
A. Transaminasa oxalacética
B. El síndrome de Lambert Eaton se asocia concáncer
prostático. B. Transaminasa pirúvica

C. En el botulismo se produce bloqueo postsináptico. C. Deshidrogenasa láctica

D. El hipertiroidismo no debe considerarse en el diagnóstico D. Creatinfosfoquinasa


diferencial.
E. Aldolasa
E. Pueden ser desencadenados por aminoglucósidos.
22. En una esclerosis sistémica progresiva (esclerodermia)
17.La esclerosis múltiple se asocia a: el peor pronóstico se asocia con:

A. Una distribución geográfica característica. A. Una extensa calcinosis cutánea.

B. Presencia de anticuerpos contra el sarampión en el B. La presencia de disfagia.


líquido cefalorraquídeo.
C. El número de articulaciones inflamadas.
C. Ciertos antígenos de histocompatibilidad (HLA).
D. La presencia de insuficiencia renal.
D. Elevación de inmunoglobulinas oligoclonales en el LCR.
E. Fenómeno de Raynaud severo con úlceras necróticas en
E. Todo lo anterior. dedos.

23.La esquizofrenia implica cambios patológicos a nivel de:


A. Hipotálamo. E. Miembros superiores

B. Sistema reticular. 29.Tipos de anemia que se observan generalmente en los


pacientes con paludismo agudo:
C. Lóbulo occipital.
A. Macrocítica.
D. Cerebelo.
B. Hipocrómica.
E. Lóbulo frontal, temporal y ganglios de la base.
C. Normocrómica.
24.Señale cuál de los siguientes tratamientos está indicado
para el trastorno por déficit de atención con hiperactividad: D. Drepanocítica.

A. Fluoxetina. E. Microcítica.

B. Haloperidol. 30. Los niveles de INR para control de anticoagulación oral


deben mantenerse entre 2 y 3, con la excepción de una de
C. Sertralina. lassiguientes indicaciones:
D. Metilfenidato. A. Trombosis venosa profunda.
E. Clozapina. B. Prótesis valvular cardíaca mecánica.
25.¿Cuál de los siguientes parámetros resulta más útil en el C. Fibrilación auricular en estenosis mitral.
diagnóstico diferencial de la necrosis tubular aguda por
fármacos? D. Disfunción severa de ventrículo izquierdo.

A. Creatinina plasmática. E. Tromboembolismo pulmonar.

B. Urea plasmática. CIRUGÍA

C. Sodio urinario. 31. ¿En cuál de los siguientes casos usaría ligadura elástica
como tratamiento?
D. Eosinofilia en plasma.
A. Hemorroides de primer grado.
E. Nivel de C3 en suero.
B. Hemorroides de segundo grado no trombosada.
26. Un niño de 4 años presenta insuficiencia renal aguda y
anemia marcada con abundantes esquistocitos en el frotis C. Hemorroides de tercer grado.
de sangre periférica. El diagnóstico probable es:
D. Pólipo rectal pediculado.
A. Glomerulonefritis aguda.
E. Pólipo simple de recto.
B. Fracaso renal agudo isquémico.
32. Las complicaciones de las hemorroides son:
C. Trombosis de las venas renales.
A. Tromboflebitis hemorroidal.
D. Síndrome hemolítico-urémico.
B. Trombosis hemorroidal.
E. Insuficiencia renal hemoglobinúrica.
C. Hematoma perianal.
27.Cuando se observa una inmunofluorescencia directa
positiva en la piel lesional y perilesional afectando a la D. Ninguna de las anteriores.
sustancia intercelular de la epidermis, podemos hacer el
E. Todas ellas.
diagnóstico de:
33. ¿Cuál de las siguientes derivaciones portosistémicas es
A. Penfi goide ampollar.
selectiva?
B. Dermatitis herpetiforme.
A. Porto-cava término-lateral.
C. Epidermólisis ampollar.
B. Porto-cava látero-lateral.
D. Pénfigo vulgar.
C. Espleno-rectal convencional.
28. El diagnóstico de ectima estreptocócico se debe
D. Interposición porto-cava.
sospechar cuando se presentan una o más úlceras
induradas princi- palmente en: E. Espleno-rectal distal.
A. Región facial 34.En relación con el carcinoma de vesícula biliar, señale lo
CORRECTO:
B. Región cervical
A. La frecuencia es menor en pacientes con “vesícula de
C. Miembros inferiores
porcelana”.
D. Región torácica
B. El cociente varón/mujer es igual a 2:1.
C. El adenocarcinoma no es el más frecuente. 40. ¿Cuál de los siguientes factores es el más común en la
producción de evisceraciones?
D. El 90% de casos se asocia a cálculos de vesícula.
A. Edad del paciente.
E. El tratamiento adyuvante con quimio o radioterapia es
eficaz. B. Anemia.

35. En el carcinoma de páncreas se presentan los siguientes C. Hipoproteinemia.


síntomas, EXCEPTO:
D. Distensión abdominal.
A. Dolor abdominal.
E. Diabetes mellitus.
B. Ictericia.
41. La complicación más seria después de una resección
C. Signo de Courvosier-Terrier. gástrica es:

D. Bloqueo A-V. A. Tromboflebitis.

E. Pérdida de peso. B. Escape del muñón duodenal.

36.¿Por qué mecanismo se da el prendido de un autoinjerto C. Hemorragia.


de piel?
D. Sepsis.
A. Tejido epitelial.
E. Atelectasia.
B. Incremento de bordes.
42.¿Cuál de las afirmaciones define el concepto de
C. Crecimiento por profundidad. politraumatismo?

D. Tejido de granulación. A. Traumatismo craneoencefálico y lesión visceral.

E. Conexiones vasculares. B. Lesiones viscerales y periféricas.

37. El estado físico (ASA) clasifica a los pacientes en función C. Dos o más lesiones periféricas.
de:
D. Lesiones traumáticas múltiples.
A. La gravedad de las enfermedades preoperatorias con la
mortalidad. E. Lesiones traumáticas múltiples que ponen en riesgo la
vida del paciente
B. La morbilidad y mortalidad intra operatoria.
43. En una lesión de la vesícula biliar por arma de fuego,
C. La gravedad de la enfermedad y la reserva fisiológica. ¿Cuál de los siguientes procedimientos es de elección?

D. La mortalidad anestésica secundaria. A. Colecistectomía.

E. La gravedad de la enfermedad y los accidentados B. Sutura de la vesícula.


anestésicos.
C. Colecistectomía + coledocotomía.
38. ¿Qué tipo de colección contiene un seroma?
D. Colecistostomía.
A. Serosa.
E. Colecistostomía y drenaje de Petzer
B. Linfática.
44.La enfermedad congénita de las vías biliares denominada
C. Intestinal. “Enfermedad de Caroli”, tiene la siguiente base anatómica:.

D. Purulenta. A. Dilatación quística del colédoco,.

E. Hemática. B. Atresia biliar extra hepática..

39. La característica más importante de la hernia C. Atresia biliar intrahepática..


incarcerada es:
D. Dilatación de los conductos biliares intrahepáticas..
A. Tener contenido visceral reductible.
E. Vesícula intrahepática.
B. Tener contenido irreductible con conservación de su
circulación. 45. Señale cuál síndrome NO se asocia con un aumento en
la incidencia de cáncer colorrectal:
C. Tener contenido esfacelado por bloqueo de su
circulación. A. Poliposis colónica familiar.

D. Tener contenido sólo de epiplón. B. Síndrome de Gardner.

E. Ninguna anterior. C. Síndrome de Turcot.

D. Síndrome de Lynch.
E. Síndrome de Peutz – Jeghers. E. Hígado.

GINECOLOGIA Y OBSTETRICIA 51. Mujer de 55 años con tumoración que protruye por
genitales externos. Al examen tiene una longitud de vagina
46. Nulípara de 20 años de edad, con antecedente de de 7. El punto Ba es +6. Según la clasificación de prolapso
infección de transmisión sexual y enfermedad inflamatoria de órganos pélvicos (POP-Q), ¿Cuál es el grado de distopia
pélvica a repetición. Conoce que su pareja tiene otras genital?
compañeras sexuales. Ella no desea concebir y quiere optar
por algún método de planificación familiar. ¿Cuál es el A. III
método de elección?
B. I
A. Píldoras anticonceptivas.
C. II
B. Métodos naturales.
D. V
C. Métodos de barrera.
E. IV
D. Dispositivos intrauterinos.
52. Los quistes de ovario con contenido de color
E. Anticoncepción parenteral. achocolatado se observan en:

47. En el diagnóstico diferencial del dolor pélvico agudo, la A. Endometriosis.


enfermedad o disfunción ginecológica MENOS probable de
considerar es: B. Quiste folicular.

A. Endometriosis. C. Cistoadenoma de ovario.

B. Embarazo ectópico roto. D. Todas las anteriores.

C. Enfermedad pélvica inflamatoria aguda. E. Ninguna de las anteriores.

D. Quiste ovárico funcional hemorrágico. 53. Entre las alteraciones que presenta la mujer geriátrica
debido al hipoestrogenismo se incluyen las siguientes,
E. Degeneración de leiomioma. EXCEP- TO:

48. Paciente de 22 años, con tiempo de enfermedad de 3 A. Vaginitis.


días. Examen ginecológico: vesículas en racimo sobre labio
mayor y adenopatías inguinales homolaterales dolorosas. B. Dispareunia.
¿Cuál es el diagnóstico PROBABLE?
C. Polaquiuria.
A. Chancro sifilítico.
D. Colpocitología con predominio de células superficiales.
B. Condiloma vulvar.
E. Osteoporosis.
C. Herpes genital.
54. ¿Cuál de los siguientes marcadores de
D. Absceso perineal. cromosomopatías del primer trimestre tiene MÁS valor?

E. Linfogranuloma venéreo. A. Gonadotropina coriónica.

49.El antibiótico de elección en una mujer de 32 años de B. La alfa-fetoproteína.


edad, con cervicitis mucopurulenta y PCR positivo para
C. La PAPP–A.
Chla- mydia trachomatis es:
D. La sonoluscencia nucal.
A. Azitromicina.
E. El acortamiento del femur.
B. Ciprofloxacina.
55. Paciente de 28 años, hace 39 días fue sometida a
C. Doxiciclina.
legrado uterino; presenta hemoptisis, ginecorragia escasa.
D. Metronidazol. Examen clínico PA: 120/70 mmHg, FC: 102 x minuto, útero
de 14 cm, cérvix normal. Dosaje de HCG-B: 32000 Ul/ml.
E. Penicilina benzatínica. Hemog- lobina: 10.2 g/dL. ¿Cuál es el diagnóstico
PROBABLE?
50.¿Hacia qué órgano se disemina con MÁS frecuencia el
carcinoma cervical? A. Reinicio del ciclo menstrual.

A. Hueso B. Coriocarcinoma.

B. Pulmones C. Retención de restos placentarios.

C. Vejiga. D. Endometritis.

D. Cuerpo uterino. E. Adenocarcinoma de endometrio.


56.Primigesta de 35 años de edad, con 37 semanas de E. Ninguna de las anteriores.
gestación, ingresa a la Emergencia con sangrado vaginal
rojo ruti- lante sin dolor; no obstante, la frecuencia cardíaca 61. Paciente gestante con diagnóstico de SIDA y sin
fetal se mantiene estable en 150 por minuto. ¿Qué tratamiento previo, llega a la Emergencia en periodo
procedimiento indicaría usted? expulsivo. El tratamiento retroviral elegido debe ser:

A. Examen digital del cérvix. A. Zidovudina en la madre y en el neonato.

B. Test no estresante. B. Lamivudina en la madre.

C. Inducción del trabajo de parto. C. Zidovudina y Lamivudina en la madre.

D. Ruptura de membranas. D. Zidovudina en la madre.

E. Examen de ultrasonografía. E. Lamivudina en el neonato.

57. Son criterios de diagnóstico clínico de corioamnionitis, 62. Puérpera de 7 días, regresa al hospital presentando
EXCEPTO: fiebre de 39,5 °C, cefalea, dolor de bajo vientre, útero
subinvolu- cionado con loquios purulentos y fétidos. ¿Cuál
A. Taquicardia materna mayor de 100 latidos por minuto. es el diagnóstico MÁS PROBABLE?

B. Dolor y contracciones uterinas cada 3 minutos. A. Infección urinaria aguda.

C. Taquicardia fetal mayor de 160 latidos por minuto. B. Endometritis puerperal.

D. Hipersensibilidad uterina. C. Vulvovaginitis mixta.

E. Fiebre materna mayor de 38 grados. D. Vaginosis bacteriana.

58. Primigesta de 39 semanas, al examen obstétrico revela E. Infección pélvica aguda.


abdomen elongado en sentido vertical, cabeza fetal a nivel
del fondo uterino y miembros fetales en flanco derecho PEDIATRIA
materno, con latidos fetales nítidos en cuadrante superior
63. En los primeros instantes del parto el recién nacido
izquier- do. Señale la situación, presentación, posición y
pierde más calor por:
probable variedad de posición:
A. Evaporación.
A. Longitudinal podálica izquierda y sacra izquierda anterior.
B. Radiación.
B. Longitudinal podálica izquierda y sacra izquierda
posterior. C. Conducción.
C. Longitudinal podálica derecha y sacra derecha anterior. D. Convección.
D. Longitudinal podálica derecha y sacra posterior. E. Por todos ellos.
E. Longitudinal sacro izquierda variedad anterior. 64. A las 32 semanas de gestación, en una sala de partos
con temperatura ambiental de 23,9ºC, nace un bebé que
59. ¿Cómo clasificaría a una gestación que antes del
pesa 1400 gms. Pocos minutos después del nacimiento,
embarazo tenia tensiones arteriales normales; que en la
este recién nacido tiende a manifestar los siguientes signos,
primera consulta, realizada en la semana 8 de edad
EXCEPTO:
gestacional, se le detecta una tensión arterial de 140/90
mmHg; y que en la semana 28 tiene una tensión arterial de A. Palidez.
170/110 mmHg, sin edemas, y con una proteinuria en orina
de 24 horas de 300 mg? B. Escalofríos.

A. Preeclampsia. C. Disminución de la temperatura corporal.

B. Preeclampsia grave. D. Aumento de la frecuencia respiratoria.

C. Hipertensión inducida por el embarazo. E. Acidosis metabólica.

D. Hipertensión crónica. 65. Lo más probable, ante la presencia de un área de


pigmentación obscura en zona del sacro y nalgas en un
E. Hipertensión crónica con preeclampsia sobreañadida. recién nacido:
60. El embarazo es afectado por la diabetes como sigue: A. Eritema tóxico.
A. Aumenta la incidencia de hidramnios. B. Melanoma maligno.
B. Aumenta la incidencia de toxemia. C. Mancha mongólica.
C. Aumenta la incidencia de malformaciones congénitas. D. Hemangioma plano.
D. Todas las alternativas son correctas. E. Meningomielocele oculto.
66. El tratamiento empírico inicial de la sepsis neonatal de C. 4 años.
inicio precoz es:
D. 6 años.
A. Meticilina y cefotaxima.
E. 5 años.
B. Ampicilina y cefotaxima.
72. Un niño que acude a control al mes de edad tiene las
C. Vancomicina y amikacina. siguientes medidas: talla 52cm, peso: 4kg y perímetro
cefálico: 37cm, correspondiéndoles dichas medidas al
D. Ampicilina y gentamicina. percentil 40. Si el niño recibe lactancia materna exclusiva y
es un niño sano, Ud. esperará en el siguiente control
E. Penicilina G y kanamicina.
mensual:
67.La documentación de sepsis en el recién nacido es a
A. Que el perímetro cefálico pase el percentil 80.
veces difícil. ¿Cuál de las siguientes localizaciones tiene un
bajo índice de positividad? B. Que sólo la talla persista en el percentil 40.
A. Líquido cefalorraquídeo. C. Que el peso suba al percentil 90.
B. Cultivos de aspirado endotraqueal. D. Que todas sus medidas continúen en el percentil 40.
C. Sangre venosa. E. Ninguna de las anteriores.
D. Urocultivo. 73. ¿Cuál de las siguientes malformaciones congénitas, es
la más frecuente en nuestro medio?
E. Cultivos de la arteria umbilical tomados al momento de la
inserción. A. Atresia intestinal.
68. La fenilcetonuria es producida por falta de: B. Hipertrofia pilórica.
A. Hidroxilasa de la fenilalanina. C. Ano imperforado.
B. Transaminasa de la fenilalanina- cc - cetoglutarato. D. Hernia hiatal.
C. Oxidasa del homogentisato. E. Ninguna de ellas.
D. Descarboxilasa de la DOPA. 74. ¿Cuál de las siguientes es la fórmula dental para los
dientes caducos (de leche)?
E. Sintetasa de la cistationina.
A. 2-1-2-.
69. ¿Qué afección se presenta con mayor frecuencia en
recién nacidos que han sido objeto de maniobras de B. 2-1-.
resucitación?
C. 3-2-1-.
A. Fractura de costillas.
D. 2-1-1-.
B. Neumotórax.
E. 2-1-.
C. Hemorragia cerebral.
75. De los siguientes antecedentes en el RN sometido a
D. Todas las anteriores. cuidados intensivos, ¿cuáles son la indicación para estudio
de la audición?
E. Ninguna de las anteriores.
1. Peso al nacer menor de 2.500 g.
70. El calostro se produce durante los primeros.....después
del parto: 2. Asfixia perinatal.
A. 7 días. 3. Hemorragia intracerebral.
B. 3 días. 4. Hiperbilirrubinemia indirecta mayor de 20 mg/dL.
C. 2 días. 5. Taquipnea transitoria del recién nacido.
D. 5 días. 6. Uso de aminoglucósidos.
E. 10 días. 7. Trisomía 21. A. 1,2,3,4,6.
71.Montar un triciclo, hacer torres de 10 cubos, decir su B. 1,2,4,5,6.
edad y sexo, y comunicarse con lenguaje gramatical, son
logros que normalmente deben adquirirse a los: C. 1,3,4,6,7.

A. 2 años. D. 2,3,4,5,6.

B. 3 años. E. 2,3,5,6,7.
76. Un lactante de 10 meses que presenta vómitos y D. Ulcerarse y provocar hemorragia.
diarreas, taquicardia, PA normal, mucosas secas, llenado
capilar me-nor de 2 segundos e irritabilidad. ¿Qué E. Todo lo anterior.
porcentaje de peso ha perdido este paciente?
SALUD PÚBLICA Y GESTION
A. 13-15%.
81. Analizó los determinantes de la Salud y construyó un
B. 3-5%. modelo que ha pasado a ser clásico en Salud Pública:

C. 1-2%. A. Rudolf Virchow

D. 6-9%. B. Edwin Chadwick

E. 10-12%. C. Sara Baker

77.Señale el enunciado CORRECTO con relación a las D. Marc Lalonde


infecciones del SNC en niños:
E. Michael Marmot
A. La etiología específica es la más frecuente
82. La Ley General de Salud (ley N° 26842) establece que la
B. Las causadas por Rickettsias son frecuentes. salud pública es responsabilidad:

C. Las de origen viral son las más frecuentes A. Primaria de las organizaciones de la sociedad civil

D. Las debidas a hongos y parásitos son más frecuentes que B. Compartida por la sociedad y el estado
las bacterianas
C. Primaria del estado
E. Las especies de Mycoplasma no producen infección
D. Compartida por el individuo y la sociedad
neurológica.
E. Compartida por el individuo, sociedad y estado
78. Nos traen a Urgencias a un escolar de 7 años de edad,
que estando totalmente bien, presenta fiebre de 40ºC, 83. La fluorización a través de enjuagues bucales en niños
acompaña- da de vómitos, cefaleas, pulso rápido y algo de 6 años, corresponde a una actividad de...
débil, con sensación de enfermedad grave. A la exploración
destacan amíg- dalas hipertróficas con exudado blanco- A. Control sanitario
amarillento, así como enantema en paladar blando. Además
B. Prevención secundaria
presenta adenitis satélite en ángulo mandibular izquierdo. A
las 14 horas aparece exantema puntiforme a la palpación C. Prevención terciaria
que respeta el ángulo nasogeniano. A los 3-4 días se inicia
descamación furfurácea. El tratamiento etiológico de este D. Promoción sanitaria
proceso es:
E. Prevención primaria
A. Macrólidos por 5 días.
84. Cuál es el proceso que proporciona información
B. Tratamiento sintomático. necesaria a las personas, familia y comunidad para mejorar
la salud y una mayor control sobre los determinantes
C. Penicilina oral durante 10 días. sociales?
D. Penicilina benzatina en 3 dosis consecutivas. A. Visitas domiciliarias.
E. Asociar corticoides al antibiótico elegido. B. Protocolos de salud.
79. En un niño con glomerulonefritis difusa aguda post- C. Participación social.
estreptocócica, ¿cuál de los siguientes hallazgos puede
observarse en la biopsia renal? D. Alianzas estratégicas.

A. Fibrosis periglomerular. E. Interacción comunitaria.

B. Engrosamiento de la membrana basal glomerular. 85. No es un objetivo de un sistema sanitario:

C. Esclerosis mesangial. A. Equidad de los usuarios

D. Medias lunas. B. Eficacia micro y macroeconómica

E. Proliferación endocapilar difusa C. Atención continuada

80. El divertículo de Meckel puede: D. Accesibilidad de los usuarios

A. Invaginarse y provocar intususcepción. E. Satisfacción de los usuarios

B. Secretar Ácido gástrico y perforarse. 86. La secuencia lógica del proceso administrativo es:

C. Provocar obstrucción del intestino delgado. A. Organización-dirección-control-planificación


B. Dirección-planificación-organización-control 92. Se desea estimar la prevalencia de una enfermedad que
se sospecha tiene un comportamiento estacional. Para
C. Planificación-organización-dirección –control obtener una muestra representativa se debe optar por el
muestreo:
D. Planificación-dirección-organización-control
A. Estratificado
E. Organización-planificación-dirección-control
B. Aleatorio simple con restitución
87. ¿Qué se desarrolla en la administración de un
establecimiento nivel I, al relacionar las actividades de sus C. Aleatorio simple sin restitución
diversas unidades para alcanzar las metas propuestas?
D. Sistemático
A. Organización
E. Por conglomerado
B. Negociación
93.En Epidemiología, ¿Cuáles son fuentes de información
C. Motivación primaria?
D. Planificación A. Observación, registros HIS, grupos focales
E. Coordinación B. Grupos focales, historias clínicas, registros HIS
88. En el análisis FODA de un hospital, “los permanentes C. Reportes de necropsias, grupos focales, observación
conflictos entre la dirección y el cuerpo médico” corresponde
a: D. Encuestas, observación, grupos focales

A. Debilidad E. Censo poblacional, grupos focales, registros HIS

B. Amenaza CIENCIAS BASICAS

C. Fortaleza 94. ¿Cuál de las siguientes sustancias inhibe la motilidad


intestinal?
D. Oportunidad
A. Noradrenalina.
E. Recurrencia
B. Insulina.
89. Un contraste de hipótesis:
C. Motilina.
A. Es un tipo de estadística descriptiva
D. Colecistocinina.
B. La hipótesis nula plantea la existencia de diferencias.
E. Serotonina.
C. La hipótesis alternativa plantea la no diferencia.
95. El músculo sartorio está inervado por el nervio:
D. La hipótesis nula y la alterativa pueden no ser
excluyentes. A. Glúteo mayor.

E. Se puede saber la probabilidad de equivocarse en la B. Glúteo menor.


afirmación.
C. Ciático poplíteo interno.
90.¿Cómo se denomina la tasa calculada a partir del número
de niños de edad superior a 28 días y menores de 1 año, D. Femoral.
que mueren en un año por cada mil recién nacidos vivos?
E. Femorocutáneo.
A. Mortalidad neonatal
96. Indicar los músculos que forman el canal del pulso:
B. Morbilidad neonatal
A. Supinador largo y palmar mayor.
C. Morbilidad postneonatal
B. Supinador largo y palmar menor.
D. Mortalidad postneonatal
C. Palmar mayor y palmar menor.
E. Mortalidad infantil
D. Abductor largo y extensor largo del pulgar.
91. Uno de los siguientes estadísticos NO es de dispersión:
E. Supinador largo y primer radial externo.
A. Desviación estándar.
97. La arteria meníngea media atraviesa el agujero:
B. Desviación media.
A. Esfenopalatino.
C. Cuartil.
B. Oval.
D. Rango intercuartilico.
C. Redondo mayor.
E. Coeficiente de variación.
D. Redondo menor.
E. Rasgado anterior. SIMULACRO 2B

98.¿Cuál es el origen embriológico de la corteza 1. En la autopsia de una mujer de 24 años se encuentra


suprarrenal? pleuritis, engrosamiento de la membrana de las paredes del
capilar glomerular, anillos concéntricos de colágeno
alrededor de las arteriolas esplénicas y excrecencias en la
parte inferior de la válvula mitral. Es más probable que el
A. Mesodermo.
análisis de sangre evidencie:
B. Endodermo.
A. Anticuerpos antinucleares.
C. Ectodermo.
B. Concentraciones elevadas de C3.
D. Neuroblasto.
C. Linfocitosis.
E. Células pluripotenciales.
D. Gammapatía monoclonal.
99. El epitelio de la vejiga urinaria es de tipo:
E. Anticuerpos antimembrana.
A. Transicional.
2. Paciente mujer de 59 años, acude a la consulta con dolor
B. Poliestratificado cornificado. crónico en cintura escapular y rigidez de cuello. También
refiere rigidez en región lumbar que le imposibilita agacharse
C. Poliestratificado no cornificado. a recoger objetos. Refiere estos síntomas desde hace 2 a 3
años. A la exploración se evidencia mucho dolor y rigidez a
D. Cilíndrico pseudoestratificado.
la palpación de musculatura de hombros y cervical pos-
E. Cilíndrico monoestratificado. terior, así como en la musculatura paravertebral lumbar.
Resto del examen físico normal. Al ahondar en la anamnesis
100. ¿Cuál es la característica de un electrocardiograma tiene dificultad para conciliar el sueño. VSG: 20. ¿Cuál es el
normal? diagnóstico y tratamiento de elección?

A. Onda R positiva en aVR. A. Fibromialgia - AINES y ansiolíticos.


B. Onda T negativa en derivación I.
C. Onda Q en derivación I. B. Espondilitis anquilosante - Corticoides a dosis bajas.
D. Onda T negativa, en derivación II.
C. Polimiositis - Azatioprina.
E. Onda P positiva en derivación II.
D. Polimialgia reumática - Corticoides a dosis altas.

E. Depresión atípica - Antidepresivos tricíclicos.

3. Paciente de 54 años que desde hace unos meses, y


después de iniciar una dieta, comienza a presentar un
cuadro caracterizado por astenia, anorexia con pérdida de
peso, apatía, anhedonia, abulia, sentimientos de minusvalía
y desesperanza, insomnio de despertar precoz y mejoría
vespertina. Antecedentes personales: episodio depresivo
hace 20 años, reactivo a la muerte de un familiar.
Antecedentes familiares de depresión. Se descarta patología
somática. El diagnóstico más probable es:

A. Depresión endógena.

B. Depresión reactiva.

C. Depresión crónica.

D. Depresión caracterial.

E. Depresión orgánica.

4. ¿Cuál de los siguientes es un síntoma característico de la


crisis de angustia?:

A. Euforia.

B. Irritabilidad.

C. Labilidad emocional.

D. Sensación de muerte.

E. Hiperactividad.
5. El incremento de eritropoyetina se relaciona con: 10. Varón de 34 años, procede de Lima, consulta por fiebre
de 39.5º. R: 22 por min, Hepato y esplenomegalia. Ictericia
A. Carcinoma gástrico. de piel y mucosas, laboratorio: Hb: 8 gr%. Leucocitos: 3,800,
Neutrófilos 60%. Abastonados: 14%. Linfocitos: 18%.
B. Adenocarcinoma de ovario.
Bilirrubinas: 6mg%. Directa: 3.5, TGP: 300, TGP: 350. F.
C. Carcinoma de riñón. Alcalina: 800 u. Aglutinaciones: = 1/80 H 1/160, Brucella (-).
El diagnostico probable es:
D. Carcinoma de endometrio.
A. Faciolasis Hepática.
E. Metástasis a médula ósea.
B. Leptospirosis.
6. Ante un enfermo con anemia megaloblástica que va a
iniciar tratamiento con vitamina B12 ¿Cuál de las siguientes C. Absceso Hepático-Amebiano.
recomendaciones es la más aconsejable?
D. Colangitis Ascendente.
A. Procurar tomarlo en ayunas.
E. Fiebre Tifoidea.
B. Asociar administración de Vitamina C.
11. Varón de 9 años, natural y procedente de zona endémica
C. Asociar administración de ácido fólico. de dengue en Piura, sin antecedentes de dengue, hace 7
días febril, con cefalea, artromialgias, náuseas y astenia.
D. La dosis elemental de Fe será 150 - 200 mg al día. ¿Qué examen debe solicitar para confirmar el diagnóstico?
E. Mantener tratamiento mayor o igual a 6 meses. A. PCR para serotipificación.
7. El mecanismo principal de la anemia de las enfermedades B. IFI.
crónicas es:
C. ELISA para dengue.
A. Bloqueo del hierro en los macrófagos sin poderlo utilizar
los precursores eritroides. D. IFI viral por hisopado nasofaringeo

B. Disminución de la absorción intestinal de hierro. E. Western Blot para dengue

C. Disminución de la producción de eritropoyetina.

D. Acortamiento de la vida media del hematíe. 12.Varón de 47 años, alcohólico, consulta por la
presentación brusca desde hace 72 horas de dolor intenso
E. Defectos adquiridos en la membrana de los hematíes. en el hemitórax derecho, refiere además fiebre, escalofríos y
sudoración desde hace 10 días, así como tos persistente
8. Una mujer de 56 años consulta por ptosis palpebral y
con abundante expectoración maloliente. Rx de pulmones:
debilidad muscular de la hemicara derecha, de aparición
consolidación redondeada con nivel hidroaéreo en el 1/3
brusca. También refiere tinitus. El examen muestra: parálisis
medio del hemi- tórax derecho. Se envía muestras de esputo
que compromete las partes superior e inferior de la hemicara
para examen directo, cultivo, ¿qué tratamiento instauraría
derecha, desviación de la comisura labial hacia la izquierda y
Ud.?:
signo de Bell con reflejo corneano ausente, ageusia en los
dos tercios anteriores de la lengua. ¿Cuál es el diagnóstico A. Claritromicina + Amikacina.
MÁS probable?:
B. Ceftriaxona + Clindamicina.
A. Parálisis facial central por accidente cerebrovascular
isquémico. C. Doxiciclina + Amoxicilina con Ácido clavulánico.

B. Parálisis facial (a frigore). D. INH + RFP + PZA + Etambutol.

C. Distrofia muscular oculofaríngea. E. Cotrimoxazol + Ciprofloxacino.

D. Masa ocupante cortical izquierda. 13. Paciente con derrame pleural hemático y masa en pared
torácica es altamente sospechoso de:
E. Ninguna de las anteriores.
A. Tuberculosis.
9. Paciente con diarrea sanguinolenta, con deshidratación
sin fiebre, leucocitos: 8:00, Hb: 7 gr. Plaquetas 20,000. DHL: B. Carcinoma broncogénico.
800 U. Subraye el Diagnóstico probable:
C. Histoplasmosis.
A. Shigella.
D. Mesotelioma.
B. Estafilococo.
E. Hidatidosis torácica.
C. Giardia lamblia.
14. El diagnostico de tromboembolismo pulmonar se
D. Clostridium dificile. confirma mediante:

E. Coli 0 157: H7. A. Radiografía de pulmones

B. Gammagrafía pulmonar
C. Electrocardiograma C. Tener unos valores de proteína C reactiva sérica
superiores a 120 mg/dl.
D. Estudio de gases arteriales
D. Presentar una puntuación APACHE II (Acute Physiology
E. Angio TC and Chronic Health Evaluation) superior a 8.
15. La causa más importante de bronquioectasias es: E. Presentar positivos 3 ó más criterios de Ransom.
A. Inflamación necrosante de origen infeccioso. 20. Paciente de 69 años, bebedor y fumador con anorexia y
disminución de peso, malestar abdominal, sensación de
B. Anormalidades hereditarias.
plenitud gástrica y cambios en el hábito intestinal. A
C. Anomalíascongénitas. exploración: tinción ictérica de piel, ganglio supraclavicular
izquierdo y axilar anterior izquierdo palpable,
D. Fibrosis quística. hiperpigmentación axilar y queratosis pruriginosa aguda.
Diagnóstico más probable:
E. Asma bronquial.
A. Adenocarcinoma gástrico precoz.
16. Varón de 49 años concurre a emergencia por disnea
severa, cianosis Presión arterial: 220/140. A. Respiratoria: B. Adenocarcinoma gástrico avanzado.
Cre- pitantes en 2/3 inferiores de ambos hemitórax, Rx. de
Pulmones: Sombras densas bilaterales en alas de mariposa. C. Carcinoma epidermoide gástrico avanzado.
La terapia indicada sería:
D. Leiomiosarcoma gástrico.
A. Ceftriaxona + Claritromicina.
E. Linfoma gástrico.
B. Prednisona.

C. Sufametozazole.

D. Ciclofosfamida + Prednisona.
21. Paciente de 35 años de edad con episodios de diarrea
E. Furosemida + Cedilanid. de presentación diurna y dolor hipogástrico que calma con la
defecación. No fiebre, heces sin moco ni sangre. El
17.Anciano de 76 años, es conducido a Emergencia por diagnóstico más probable es:
Lipotimia. Al examen: Paciente recuperado, pulso: 41 por
minu- to, regular.PA: 110/70. Esta sudoroso y pálido; Ud. A. Enfermedad inflamatoria del intestino.
dispone se realice: ECG y análisis mientras tanto; Ud.
B. Neoplasia de colon.
recomienda la siguiente terapia:
C. Colon irritable.
A. Glucosa al 10%, 1 litro 50 gotas por 1.
D. Giardiosis.
B. Cedilanid 0.4 mg 1 ampolla VEV.
E. Salmonelosis.
C. Sulfato de Atropina ½ mg. VEV.
22. De las siguientes pruebas, ¿Cuál se debe realizar para el
D. Atenelol 50 mg. VO.
diagnóstico de neurosífilis?:
E. Amiodarona V.O.
A. Examen de campo oscuro en LCR.
18. Una paciente de 32 años de edad presenta una
B. FTA absorbido en LCR.
auscultación cardíaca con un click mesosistólico y un soplo
mesotelesis- tólico que se incrementa de intensidad con la C. Inmovilización del Treponmeapallidum en LCR.
maniobra de Valsalva. El diagnóstico de presunción será:
D. Inmunofluorescencia directa en LCR.
A. Miocardiopatía hipertrófica.
E. VDRL en LCR.
B. Comunicación interauricular.
23. El síndrome anticolinérgico puede ser producido por:
C. Estenosis mitral.
A. Carbamatos.
D. Estenosis aórtica.
B. Gases nerviosos: Sarín.
E. Prolapso mitral
C. Neostigmina.
19. En un paciente diagnosticado de pancreatitis aguda, a
los dos días de su evolución, ¿cuál de los siguientes datos D. Flumazenilo.
carece de valor para pronosticar su severidad?:
E. Antiparkinsonianos.
A. Score tomográfico.
24. Varón de 42 años conducido a Emergencia por dolor
B. Valores de amilasa y/o lipasa sérica elevados más de 10 abdominal, despeños diarreicos y mareos. Desde hace 4
veces el límite normal. meses presenta debilidad, laxitus, a veces desvanecimiento
y episodios de oliguria. Ha perdido 10 kg de peso. Al
examen: frialdad de piel, vitíligo en labios y manos. PA: 29. Una reacción de hipersensibilidad que produce necrosis
80/60 mmHg. Pulso: 110 x’. T°: 38 °C. Hb: 7 g% leucocitos: de los desmosomas epidérmicos se ve en:
14.000, eosinófilos 10%, linfocitos 40%. Na: 118 mEq/L. Cl:
110 mEq/L. K: 6 mEq/L. HCO3 -: 14 mEq/L. CA: 107 mg%. A. Verruga Peruana.
Glicemia: 45 mg/dL. Creatinina: 3 mg/dL. Na urinario: 60
B. Verruga vulgar.
mEql/L. ¿Cuál es su posibilidad diagnóstica?:
C. Pénfigo vulgar.
A. Déficit de la enzima 11 –hidroxilasa.
D. Tuberculosis cutánea.
B. Gastroenteritis aguda.
E. Reacción a la tuberculina.
C. Insuficiencia suprarrenal.
30. Persona del sexo femenino de 38 años, murió el 10 de
D. Necrosis tubular aguda.
enero del 2006. Con antecedente de hospitalizaciones por
E. Secreción inapropiada de hormona antidiurética. hipere- mesis gravídica y aborto recurrente en los años
previos. Estando gestando, el 09 de enero del mismo año
25. Un varón con las siguientes características somáticas: fue atropella-da por un automóvil que le ocasionó fracturas
hipogonadismo hipogonadotrófico, anosmía, falo pequeño, múltiples en cráneo y extremidades inferiores. El día de su
fe- notipo masculino, hábito eunucoide y sin ginecomastia. fallecimiento presentó infarto agudo de miocardio. ¿Cuál es
¿A qué cuadro corresponde?: la causa básica de muerte?:

A. Síndrome de Klinefelter. A. Preeclampsia.

B. Síndrome de Noonan. B. Fracturas múltiples en cráneo.

C. Síndrome de Del Castillo. C. Accidente de tránsito.

D. Síndrome de Kallman-Morsier. D. Aborto.

E. Síndrome de Reifeinstein. E. Infarto del miocardio.

26. ¿Cuál es el examen auxiliar más útil para el diagnóstico 31. Los glucósidos digitálicos aumentan la contractilidad
de monoartritis tuberculosa de rodilla? miocárdica principalmente por uno de estos mecanismos:

A. Biopsia de membrana sinovial. A. Abertura de los canales de calcio.

B. Historia clínica. B. Liberación de calcio desde el retículo sarcoplásmico.

C. Rx de rodilla en dos posiciones. C. Estimulación de la ATPasa de la miosina.

D. Rx de tórax. D. Estimulación de la fosfolipasa C de la membrana.

E. Ninguna de las anteriores. E. Inhibición de la ATPasa de Na+ - k + de la membrana.

27. Mujer de 40 años que cursa con debilidad muscular 32. En la formación del polígono arterial de Willis,
proximal. Además presenta en los párpados y alrededor de contribuyen las arterias:
los mismos, lesiones eritematosas y liláceas, asociadas a
pápulas eritematosas en dorso de ambas manos. Estas A. Cerebrales anteriores y cerebelosas superiores.
lesiones son típicas de:
B. Cerebrales medias y cerebelosas posteriores.
A. Eccema de contacto.
C. Cerebrales anteriores y cerebrales posteriores.
B. Eccema atópico.
D. Cerebrales medias y cerebrales posteriores.
C. Dermatomiositis.
E. Comunicante media y cerebrales posteriores.
D. Lupus eritematoso sistémico.
33. Es un derivado de los conductos mesonéfricos:
E. Síndrome de Sjogren.
A. Tubos seminiferos.
28. Paciente con cuadro obstructivo pilórico por úlcera
B. Conducto uterino.
péptica, con cuadro antiguo de vómitos persistentes, indique
qué alteraciones metabólicas se producen en el paciente: C. Vesículas seminales.
A. Alcalosis hipoclorémica D. Glándulas de Cowper.
B. Hiperpotasemia y acidosis E. Testículos.
C. Acidosis hipoclorémica 34. En la formación de la “barrera hemato-encefálica” la
célula que participa es:
D. Hipercloremia y alcalosis
A. La de Schwann.
E. Hipernatremia sin variación de pH
B. La astroglia. B. Cefotaxima.

C. La neuroglía. C. Ceftazidima.

D. La ependimaria. D. Cefixima.

E. El astrocito. E. Cefazolina.

35. Un paciente de 15 años de edad presentó una hepatitis 40.El sistema reticular activador ascendente del tronco
viral aguda. La evolución clínica fue favorable y la biopsia cerebral se relaciona con:
he- pática posterior mostró una regeneración completa de
los hepatocitos dentro de la estructura lobulillar normal. A. El aprendizaje.
¿Cuál es el tejido que permitió mantener la arquitectura
B. El contenido de la conciencia.
normal del órgano?:
C. El mantenimiento del estado de alerta.
A. Fibras elásticas.
D. La coordinación de movimientos aprendidos.
B. Fibras colágenas.
E. Los comportamientos alimentarios.
C. Colágeno tipo I.

D. Glucoproteinas.
41.¿Qué tipo de gemelos da origen la partición del disco
E. Fibras reticulares.
embrionario más allá de los 13 días?:
36. La alteración en la flexión del 4º y 5º dedos por las
A. Monoamniótico - bicoriónico.
articulaciones interfalángicas distales al tratar de cerrar el
puño, y al tratar de flexionar la articulación de la muñeca la B. Monoamniótico - monocoriónico.
mano se desvía lateralmente hacia la cara radial. ¿Cuál será
la lesión probable?: C. Biamniótico – bicoriónico.

A. Nervio cubital. D. Biamniótico - monocoriónico.

B. Nervio radial. E. Monstruos dobles.

C. Nervio mediano. 42.La hemorragia vaginal, masa endocervical o vaginal y


dolor, son una triada característica de:
D. Nervio radial y nervio cubital.
A. Aborto en curso.
E. Nervio mediano y nervio radial.
B. Placenta acreta.
37. Un paciente de 40 años presenta la siguiente gasometría
arterial respirando aire ambiente: pH 7,30; pO2 56 mmHg; C. Placenta previa de inserción baja.
pCO2 46 mmHg ¿Cómo puede ser definida esta situación?:
D. Inversión uterina.
A. Alcalosis metabólica.
E. Alumbramiento incompleto.
B. lnsuficiencia respiratoria hipoxémica con alcalosis.
43.Paciente de 29 años, con antecedente de un parto
C. Acidosis con insuficiencia respiratoria hipoxémica e eutócico de 36 sem. hace 6 años, embarazo molar hace 4
hipercápnica. años y embarazo ectópico hace 11 meses. Actualmente
evidencia una gestación de 24 semanas, con un estudio
D. Acidosis metabólica con hiperventilación secundaria. ecográfico que revela una gestación gemelar compatible con
el tiempo de amenorrea. Señale su fórmula de paridad.
E. Alcalosis respiratoria crónica.
A. G: 4; P: 1-0-2-1.
38. Entre el grupo de fármacos antagonistas de los
receptores de angiotensina II (ARA-II), señale el que tiene B. G: 5; P: 0-1-1-1.
mayor afinidad por el receptor AT1
C. G: 5; P: 0-1-2-0.
A. Telmisartan.
D. G: 4; P: 0-1-2-1.
B. Valsartan.
E. G: 4; P: 1-1-1-1.
C. Losartan.
44.Paciente de 28 años que ha tenido un parto por cesárea
D. Irbesartan. hace 48 horas, presenta temperatura de 39ºC luego de un
tra- bajo de parto prolongado y múltiples tactos vaginales, el
E. Olmesartan.
diagnóstico más probable será:
39. Una de las siguientes no es una cefalosporina de tercera
A. Infección de herida operatoria.
generación. ¿De cuál se trata?:
B. Mastitis.
A. Ceftriaxona.
C. Tromboflebitis pélvica séptica. 49. Mujer de 56 años que refiere perdida de orina a grandes
esfuerzos. ¿Cuál es el tipo de incontinencia urinaria presenta
D. Endometritis. la paciente?:
E. Fiebre medicamentosa. A. Urgencia.
45.Paciente gestante con diagnóstico de SIDA y sin B. Esfuerzo.
tratamiento previo, llega a la emergencia en periodo
expulsivo. El tratamiento retroviral elegido debe ser: C. Rebosamiento.

A. Zidovudina en la madre y en el neonato. D. Mixta.

B. Lamivudina en la madre. E. Vejiga hiperactiva.

C. Zidovudina y lamivudina en la madre. 50.Mujer de 35 años de casada, que luego de legrado


uterino hace 6 meses, no presenta menstruación y sin
D. Zidovudina en la madre. actividad sexual hasta el momento de la consulta. Al
examen: Útero y anexos normales. El médico solicita una
E. Lamivudina en el neonato.
ecografía ¿Cuál es el síndrome a tener en cuenta como
causa de amenorrea secundaria?:

46.Mujer de 38 años, que presenta nódulo mamario indoloro, A. Asherman.


de bordes imprecisos. Mamografía: Imagen nodular con
B. Ovario poliquístico.
espículas en todos sus márgenes y finas
microcalcificaciones agrupadas en el interior. ¿Cuál es el C. Sheehan.
diagnóstico más probable?:
D. Kalman.
A. Fibroadenoma.
E. Turner.
B. Displasia mamaria.
51.Una mujer de 28 años con anticuerpos anticardiolipina y
C. Quiste. antecedente de tres abortos en el primer trimestre del em-
barazo es evaluada en la sexta semana de un cuarto
D. Carcinoma.
embarazo, el embarazo actual transcurre con normalidad,
E. Papiloma intraductal. nunca ha sido tratada por la positividad de los anticuerpos
anticardiolipina, ¿Cuál de las siguientes es la conducta más
47.Una muchacha de 15 años de edad, sexualmente activa, apropiada?
acude a la emergencia quejándose de dolor agudo en la
parte inferior del abdomen, que incluso le dificulta la A. Observación estrecha.
deambulación. Refiere que el dolor empezó dos días
B. Prednisona.
después de la terminación de su última menstruación, y que
padece de flujo vaginal cuyas características no precisa. En C. Aspirina.
su criterio, ¿cuál sería el microorganismo que con mayor
probabilidad está causando este cuadro de EIP?: D. Infusión intravenosa de inmunoglobulinas.

A. Neisseria gonorrhoeae. E. Heparina.

B. Bacteroides fragilis. 52. Paciente con 35 semanas de gestación, cesareada


anterior 2 veces y diagnóstico de PPT es cesareada
C. Actinomyces israelii. electivamente y se encuentra que tiene placenta percreta.
¿Cuál sería el mejor tratamiento?:
D. Staphylococcus aureus.
A. Cesárea segmentaria más ligadura de trompas.
E. Trichomona vaginalis.
B. Cesárea corporal más ligadura de trompas.
48. Una joven de 24 años está usando correctamente
anticonceptivos orales combinados de baja dosis (20 ug de C. Cesárea-histerectomia.
etinilestra- diol) desde hace 8 meses. Hace 2 meses que no
menstrua y desea cambiar de método para colocarse DIU D. Método de B Lynch.
¿Cuál sería la conducta correcta?
E. Taponamiento uterino.
A. Esperar que menstrúe.
53. Los desgarros perineales de tercer grado comprometen:
B. Dosar b-HCG.
A. Horquilla, piel perineal y mucosa vaginal.
C. Colocar DIU.
B. Piel, mucosa vaginal, cuerpo perineal y esfínter
D. Dar suplente de estrógenos. anal.

E. Realizar ecografía pélvica. C. Piel, mucosa vaginal, aponeurosis y músculos del


cuerpo perineal.
D. Mucosa rectal exponiendo la luz rectal. respiratoria de 22 por minuto; la auscultación del hemitórax
derecho demuestra disminución de los ruidos respiratorios y
E. Piel perineal y mucosa vaginal. la percusión del hemitórax demuestra timpanismo. ¿Cuál es
la mejor conducta a seguir?
54.Gestante nulípara de 38 semanas de embarazo por
amenorrea, con historia de pérdida de líquido, al examen se A. Tomografía computarizada de tórax.
apre- cia salida de líquido amniótico a través del cuello
uterino, y prueba de papel de nitrazina es positivo. Al B. Toracotomía urgente.
momento tiene 5 horas de ruptura de membranas y no hay
inicio de trabajo de parto. Cultivo de EGB negativo. ¿Cuál es C. Observación clínica.
el manejo más adecuado?
D. Radiología simple de tórax y según hallazgos
A. Programar cesárea. colocación de drenaje pleural.

B. Indicar cesárea de emergencia para evitar la E. Ingreso en unidad de cuidados intensivos.


infección materna y fetal.

C. Indicar antibiótico terapia y mantener expectante.

D. Iniciar inducción de parto.


59.Un paciente de 42 años de edad, de sexo masculino, con
E. Realizar ecografía y NST para tomar una decisión. antecedentes de asma bronquial, que es atropellado por
auto-móvil, ha sufrido traumatismo abdominal cerrado,
55.El parámetro ecográfico más útil para identificar el retardo presenta con más frecuencia:
del crecimiento intrauterino es:
A. Hematoma visible en pared abdominal.
A. El cociente circunferencia cefálica y abdominal
B. Lumbalgia severa.
B. El diámetro biparietal
C. Signos de peritonismo.
C. La circunferencia cefálica
D. Abdomen con poca reacción peritoneal.
D. La longitud del fémur
E. Signos de shock hipovolémico.
E. La longitud del húmero
60.Paciente varón de 40 años es ingresado a Emergencia
56.Segundigesta de 36 años, en sala de partos RN un debido a haber sufrido accidente automovilístico.
producto de 40 semanas de gestación y 2.960 gramos; el Politraumati- zado, cursa hipotenso, taquicárdico y con
periodo de dilatación fue de 1 hora y el expulsivo de 5 transtorno del nivel de conciencia, con pérdidas importantes
minutos, donde se utilizó oxitocina. La paciente en periodo de sangre. En un primer momento, la reposición de la
de alumbra- miento después de pujar en 2 ocasiones y no volemia se logra mejor a través de:
desprenderse la placenta, siente sensación de desmayo y
dolor abdominal. Sangrado vaginal. Constantes vitales A. Catéter de Swan-Ganz vía subclavia.
normales. Se realiza palpación abdominal con pérdida de
B. Disección de una vena de la flexura del codo.
fondo uterino. ¿Cuál sería su sospecha diagnostica?:
C. Disección de la vena safena.
A. Rotura uterina.
D. Catéter central vía yugular interna.
B. Obstrucción intestinal.
E. Dos cánulas venosas cortas percutáneas.
C. Inversión uterina
61.Uno de los siguientes datos es más característico de la
D. Placenta percreta.
hernia inguinal directa que de la indirecta:
E. Miomatosis uterina.
A. Aparición en la infancia.
57.Anomalía congénita asociada con oligohidramnios:
B. Forma alargada.
A. Atresia esofágica.
C. En la palpación digital del conducto inguinal, no se
B. Anencefalia. suele apreciar debilidad de la pared posterior.

C. Displasia renal. D. Las maniobras de reducción son difíciles.

D. Espina bífida. E. Ausencia de progresión hacia el escroto.

E. Hidrops fetalis. 62.Paciente con el diagnóstico de hemorroides trombosadas,


refiere mucho dolor en zona anal. El tratamiento quirúrgico
58.Un paciente de 38 años de edad acude trasladado a preferido es:
nuestro servicio de Urgencias tras sufrir herida por arma
blanca a nivel de hemitórax derecho octavo espacio A. Reblandecedores de heces y baños de asiento.
intercostal; a su ingreso presenta: tensión arterial de 120/60
B. Incisión de las hemorroides y evacuación del
mmHg; frecuencia cardíaca de 92 l/minuto y frecuencia
coágulo sanguíneo.
C. Incisión de las hemorroides más esfinterectomía. A. Mixto.

D. Ligadura de las hemorroides con bandas de B. Oxalato calcio.


caucho.
C. Ácido úrico.
E. Inyección de una solución esclerosante
D. Estruvita.
63.En el cáncer de colon izquierdo, la sintomatología del
paciente está fundamentalmente en relación con: E. Cistina.

A. La función absorbente del colon derecho

B. La naturaleza líquida de las heces

C. La existencia de un tumor úlcero-vegetante 69.Niño de 12 años quien va regularmente a la piscina,


cursa con otalgia intensa, signo del trago positivo y ocasional
D. El menor calibre luminal otorrea escasa muy líquida. ¿Cuál es el diagnóstico más
probable?
E. La presencia del síndrome anémico
A. Otitis media aguda.
64.Paciente con insuficiencia respiratoria moderada o
severa, se considera con riesgo quirúrgico (ASA) clase: B. Otitis media crónica.

A. II C. Otitis externa difusa.

B. I. D. Otocerumen.

C. III. E. Otitis externa circunscrita.

D. IV. 70. La adenopatía preauricular es más frecuente en las


conjuntivitis causadas por:
E. V.
A. Tóxicos
65.En la fractura de la cadera, el tratamiento inicial debe ser:
B. Irritantes
A. Bota de yeso antirrotatoria.
C. Atopia
B. Tracción esquelética supracondílea o de la
tuberosidad tibial. D. Chlamydia y Virus

C. Tracción cutánea. E. Bacterias

D. Pelvipedio. 71. Neonato de 20 días de edad sin factores de riesgo, nace


a termino con peso de 3000 gr Recibe lactancia materna
E. Tracción esquelética supramaleolar. exclu- siva. Es traído a Emergencia por vómitos frecuentes
que aumentan progresivamente desde hace 3 días. No
66.En caso de luxación de cadera de un hombre joven, con
diarrea, no fiebre. Peso actual: 3600 gr. El diagnóstico más
6 horas de evolución ¿Qué haría Ud.?
probable es:
A. Dar analgésicos y transferirlo.
A. Estenosis hipertrófica congénita de píloro
B .Reducir en forma incruenta y hacerle tracción continua.
B. Sepsis neonatal
C. Inmovilizarlo con un pelvipedio y transferirlo.
C. Meningoencefalitis
D. Reducir en forma cruenta y colocarle un pelvipedio.
D. Intolerancia a la lactosa
E. Reducir e inmovilizarlo.
E. Reflujo gastroesofágico
67.Las principales complicaciones del tratamiento de las
72. Mujer de 50 años con cuadro de varios años de
fracturas son: infección, pseudoartrosis, retardo de la
evolución de síndrome miccional irritativo y dolor
consoli- dación, consolidación viciosa, rigidez articular,
hipogástrico que cede con la micción. La ecografía renal y
artrosis secundaria y:
vesical, urografía, citología en orina, cultivos repetidos de
A. Síndrome Compartimental. orina, baciloscopía y exploración genital son negativas.
¿Cuál es la actitud más correcta?:
B. Artritis.
A. No realizar más exploraciones.
C. Síndrome de aplastamiento.
B. Practicar cistomanometría.
D. Daño tendinosos muscular.
C. Repetir la urografía intravenosa en 3 meses.
E. Atrofia ósea de Sudek.
D. Realizar cistoscopia.
68. El cálculo renal más frecuente es de tipo:
E. Realizar hidrodistensión vesical simple bajo fasciculacio-nes musculares y convulsiones. ¿Cuál es el
anestesia. diagnóstico probable?:

73.La dermatitis atópica usualmente aparece a la edad de: A. Intoxicación por organofosforados.

A. 2 semanas. B. Meningitis bacteriana.

B. 3 meses. C. Gastroenteritis con deshidratación severa.

C. 2 años. D. Intoxicación por salicilatos.

D. 6 años. E. Epilepsia.

E. Ninguna de las anteriores.

74.Niño de 10 meses con cuadro febril de 3 días de 78. La principal causa de paro cardíaco en niños es:
duración, sin otra sintomatología acompañante salvo
irritabilidad con los periodos de hipertemia. El cuarto día A. Asistolia ventricular.
presenta aparición de exantema en tronco y desaparición de
B. Insuficiencia respiratoria.
la fiebre. Respecto al cuadro clínico citado, ¿Cuál de las
siguientes afirmaciones es verdadera? C. Fibrilación ventricular.
A. El diagnóstico más probable es una infección por virus del D. Alteraciones hidroelectrolíticas.
sarampión.
E. Endocrinas.
B. El signo físico diagnóstico es la presencia de una
amigdalitis exudativa. 79. El lactante habitualmente comienza a mantenerse
sentado a la edad de:
C. El tratamiento indicado es amoxicilina oral.
A. 4 a 5 1/2 meses.
D.Se asocia a una infección por virus herpético humano tipo
6. B. 6 a 6 1/2 meses.

E.La duración del exantema suele ser de 4 semanas. C. 8 a 8 1/2 meses.

75. Se admite en urgencia a un preescolar previamente sano D. 9 a 10 1/2 meses.


de 15 meses con 3 horas de enfermedad caracterizada por
E. 11 a 12 meses.
rinorrea, fiebre alta, convulsión tónicoclónica generalizada y
un examen neurológico en el post - Ictal sin mayores al- 80.Recién nacido de 12 horas de vida, de parto eutócico,
teraciones. No tiene antecedentes de convulsiones previas y peso 3000 gramos, asintomático, con madre que presenta
no recibe medicación alguna. El diagnóstico más probable sero- logía positiva para hepatitis B. ¿Cuál es la conducta
es: más apropiada en este recién nacido?
A. Meningoencefalitis bacteriana. A. Administrar solo vacuna anti hepatitis B.
B. Epilepsia secundaria. B.Administrar inmunoglobulina más vacuna contra hepatitis
B.
C. Trastorno metabólico.
C. Serología para hepatitis B.
D. Convulsión febril.
D. Anticuerpos para hepatitis B.
E. Encefalomielitis.
E. Sólo vacuna contra hepatitis B.
76.Un infante de 10 meses que gozaba de buena salud,
repentinamente llora por “cólico abdominal”, con severos 81.Mujer de 11 años de edad, acude por presentar primer
episo- dios que ocurren en un período de 3 horas, algunos episodio de sangrado aparentemente vaginal de dos días de
acompañados de vómitos. El niño parece muy enfermo. Al duración, sin dolor y sin causa aparente. No refiere otras
examen hay signos de íleo, distensión, vómitos, taquicardia. molestias. Refiere hace seis meses presenta algunos días
El abdomen parece no ser doloroso. La mejor descripción de una secreción genital transparente sin ardor ni molestias
las heces en esta patología es: asociadas. Al examen se encuentra un desarrollo de
caracteres sexuales en estadio de Tanner IV. El diagnóstico
A. Agua de arroz.
probable es:
B. En jalea de grosella.
A. Menarquia.
C. En agua negra
B. Metrorragia de causa no determinada.
D. Pastosas.
C. Vulvovaginitis.
E. Alquitranadas.
D. Hematuria de causa no determinada.
77. Paciente de 12 años de edad, soporoso, quien presenta
E. Abuso sexual.
vómito, diarrea, sialorrea, miosis, broncoespasmo,
82.El Síndrome de dificultad respiratoria es la causa más 87.Una adolescente de 14 años tiene un exagerado miedo a
frecuente de ingreso de RN a la UCIN, siendo la etiología engordar y de hecho se ve más gorda de lo que los demás
más frecuente: aprecian. Ha mantenido su peso a pesar de que
frecuentemente se da atracones que no puede evitar.
A. Enfermedad de membrana hialina. Analíticamente encontramos hipokalemia. Probablemente,
estamos frente a un caso de:
B. Síndrome de aspiración de meconio.
A. Anorexia nerviosa.
C. Taquipnea transitoria del RN.
B. Depresión juvenil.
D. Persistencia de la circulación fetal.
C. Trastorno obsesivo-compulsivo.
E. Patología pulmonar congénita.
D. Trastorno narcisista de la personalidad.
83.Un lactante de 10 meses que presenta vómitos y
diarreas, taquicardia, presión arterial normal, mucosas E. Bulimia nerviosa.
secas, llenado capilar menor de 2 segundos e irritabilidad.
¿Qué porcentaje de peso ha perdido este paciente?: 88.Niña de 2 años presenta movimientos mioclónicos de
extremidades y sacudidas desordenadas de los ojos. En la
A. 13 – 15%. explo- ración se palpa masa abdominal en línea media y
flanco derecho. Se le realiza una TAC abdominal,
B. 3 – 5 %.
evidenciándose una masa de consistencia mixta y
C. 1 – 2 %. calcificaciones en su interior. Señale la afirmación correcta:

D. 10 – 12 %. A. Es el tumor neurológico más frecuente en la


infancia.
E. 6 – 9%.
B. La edad más frecuente al diagnóstico son los 3
84. Escolar de 6 años de edad de 20 k con diagnóstico de años.
crisis asmática acude a emergencia, donde Usted decide
nebuli- zarlo con salbutamol. ¿Cuántas gotas de salbutamol C. Cursan con síndrome WARG.
prescribiría?:
D. Las catecolaminas en orina estarán elevadas.
A. 12.
E. Se asocia con delección del cromosoma 11.
B. 16.
89.La causa más frecuente de obstrucción intestinal entre
C. 8. los 3 meses y 3 años de edad es:

D. 20. A. Enfermedad de Hirschsprung.

E. 4. B. Apendicitis.

85.Un niño de 2 años acude por presentar 5 días fiebre C. Bridas congénitas.
elevada, exantema maculo papular en tronco, inyección
D. Invaginación intestinal.
conjunti- val bilateral, enrojecimiento bucal, lengua
aframbuesada, faringe hiperémica sin secreción y una E. Malrotación intestinal.
adenopatía cervical izquierda de 4cm. ¿Cuál es el
diagnóstico más probable? 90.Paciente de 4 años con diagnóstico de meningoencefalitis
meningococica que asiste a una guardería infantil, tiene un
A. Síndrome de Kawasaki. hermano menor. ¿Cuál es la medida más recomendable?
B. Exantema súbito. A. Vacunar a todos los niños de la guardería.
C. Escarlatina. B. Administrar ampicilina al hermano menor.
D. Rubéola. C. Dar sulfas a todos los niños en contacto.
E. Mononucleosis infecciosa. D. Administrar gammaglobulina intramuscular.
86.En un niño mayor de 2 años de edad, asintomático, con E. Emplear rifampicina en el hermano y compañeros de la
examen de heces positivo para quistes de Entamoeba hysto- guardería.
litica, el tratamiento de elección es:
91.En un colegio primario se desea detectar casos de
A. Cloroquina enteroparasitosis. Se solicita a cada alumno una muestra de
heces. En- tre los negativos a esta primera muestra se
B. Metronidazol
solicita una segunda muestra. Al hacer este procedimiento
C. Dihidroemetina se está logrando:

D. Primaquina A. Aumentar la especificidad y sensibilidad.

E. Ninguna de las anteriores


B. Disminuir la sensibilidad y aumentar la D. El consentimiento informado previo a toda
especificidad. investigación.

C. Aumentar la sensibilidad y disminuir la E. Los procedimientos médicos en épocas de guerra.


especificidad.
96. En una comunidad hay abundante rabia animal y se
D. Disminuir la especificidad y sensibilidad. notifican algunos casos humanos todos los años. Con el fin
de fijar las bases para iniciar un programa de control, entre
E. La sensibilidad y especificidad se mantienen. las cuales se señalan las siguientes medidas: ¿Cuál a su
juicio, puede dar resultados más permanentes?:
92. En el análisis estratégico de una organización se
considera en el contexto interno:

A. Campaña para exterminar perros vagabundos.


A. El Plan Operativo Institucional (POI). B. Campaña masiva de educación sanitaria en la
población.
B. Mi cliente.
C. Campaña masiva para hacer tratamiento antirrábico
C. La competencia.
a todas las personas mordidas.
D. Otros productos en competencia.
D. Establecer un programa de vacunación canina
E. El marco político económico social. (80% de los perros estimados) en un año.

93. Dos grupos de investigadores plantean determinar la E. Sistema de inspección de carne de los mataderos.
prevalencia de gastritis por AINES en una misma población
97. Los elementos básicos en la Programación en salud son:
de pacientes que sufren de osteoartrosis. Ambos grupos
trabajarán con el mismo nivel de confianza, sin embargo el A. La población, las atenciones y los instrumentos.
pri- mer grupo plantea trabajar con un error relativo de
muestreo del 5% y el segundo con un error relativo de B. La política de salud, la población y la infraestructura
muestreo del 10%. Se puede afirmar entonces: física.

A. Los tamaños de las muestras serán similares, ya C. Los diferentes niveles organizacionales y los niveles
que el nivel de confianza es el mismo. de atención.

B. El primer grupo trabajará con una muestra más D. La doctrina sanitaria y los establecimientos de
pequeña. salud.

C. Los tamaños de las muestras serán similares, ya E. Todos los anteriores.


que la prevalencia de gastritis es alta.
98. En una institución de salud, el concurso público para
D. No se puede saber qué grupo tendrá una muestra cubrir vacantes requiere:
más grande.
A. Instrumentos de evaluación y equipo calificador.
E. El segundo grupo trabajará con una muestra más
pequeña. B. Análisis de puestos e inventario de recursos.

94. De 75 niños entre 6 y 10 años que tomaron su desayuno C. Inventario de recursos y equipo calificador.
escolar en un centro educativo de un centro poblado rural,
D. Cumplir con los requerimientos de las
40 presentaron un cuadro de náuseas, vómitos, dolor
características de los puestos.
abdominal y diarreas dentro de las 12 horas posteriores a la
ingesta del alimento servido. De los siguientes indicadores E. Instrumentos de evaluación y una conversación
epidemiológicos, el más apropiado calcular en este contexto cordial.
es:
99. Un exitoso convenio vigente entre un establecimiento de
A. Prevalencia. salud pública y una entidad privada debe ser considerado:
B. Letalidad. A. Oportunidad.
C. Tasa de ataque. B. Debilidad.
D. Densidad de incidencia. C. Fortaleza.
E. Incidencia acumulada. D. Amenaza.
95. El código de Nuremberg estableció normas para: E. Desafío.
A. Los procedimientos quirúrgicos en época de guerra. 100. En una distribución de valores de una variable
cuantitativa, el percentil 50 corresponde a:
B. Los procedimientos médicos en general.
A. Mediana.
C. Las publicaciones médicas.
B. Moda. EXAMEN N° 14.- SIMULACRO 7B

C. Media aritmética. 1. Al medir la presión arterial se clasifica al paciente como


hipertenso, normotenso e hipotenso. En este caso, se emplea
D. Media geométrica. una escala de medición:
A. Nominal.
E. Los percentiles no se corresponden con las B. De intervalo.
medidas de tendencia central. C. De razón.
D. Cuantitativa.
E. Ordinal.

2. ¿Cuál de las siguientes afirmaciones es FALSA?:


A. La media es una medida de tendencia central que se afecta poco
por los valores extremos.
B. El valor de la mediana se corresponde con el percentil 50.
C. La mediana es una medida de tendencia central que se afecta
poco por los valores extremos.
D. La moda no suele resultar de gran utilidad cuando manejamos
variables continuas.
E. El percentil 25 se corresponde con el primer cuartil.

3. El denominador de la tasa de mortalidad materna para un


periodo de tiempo y área determinada es:

A. Población total
B. Mujeres embarazadas
C. Mujeres de 15-49 años
D. Población femenina total
E. Recién nacidos vivos totales

4. ¿Qué variable de las enunciadas NO puede ser presentada en


una escala de medida ordinal?

A. Tabaquismo
B. Dolor
C. Clase social
D. Vía de administración de un fármaco
E. Glucemia basal

5. Si en un estudio aparecen 45 casos nuevos de una enfermedad


entre los 1000 individuos que iniciaron ¿Cuál de las siguientes
afirmaciones es CORRECTA?:

A. La prevalencia es de 45 por 1000


B. La incidencia es de 45 por 1000
C. El riesgo relativo es del 45 por 1000
D. El riesgo atribuible es del 45 por 1000
E. La fracción etiológica del riesgo es del 45 por 1000

6. Se llama epidemia cuando una enfermedad transmisible se


presenta en una localidad:

A. Todos los años con incremento estacional en primavera y


verano.
B. Si se presentan casos esporádicos todos los años.
C. Si se presentan uno o varios casos no esperados en un lugar
donde antes no se observaron casos durante varios años.
D. Si se observan numerosos casos con tasas elevadas durante
todos los años.
E. Si se observa una serie de casos en 2 continentes.

7. La capacidad de afectación a una población después de un brote


se denomina:

A. Tasa de ataque secundario


B. Tasa de ataque
C. Tasa de ataque global.
D. Tasa de transmisión.
E. Tasa de incidencia.
8. Estilos de vida saludables significa: D. Amiodarona.
E. Clopidrogel.
A. Inculpar a las víctimas; esto es, a los enfermos, por enfermar
B. Recomendar pautas saludables de conducta, acompañándolas 15. ¿Cuál de los siguientes NO es un criterio mayor en el diagnóstico
de la construcción de condiciones de vida saludable de la fiebre reumática
C. Incentivar un estilo de vida consumista, que siga las pautas de
las campañas comerciales televisivas A. Carditis.
D. Modificar nuestras rutinas de vida para hacer un espacio mayor B. Fiebre.
a los deportes C. Corea.
E. Cuidarse de vivir conforme a las costumbres anteriores y D. Nódulos subcutáneos.
asimilarse a los criterios de otros países E. Eritema marginado.

9. La prevención primaria tiene como propósito: 16. La causa MÁS frecuente de pericarditis aguda es:

A. Curar los pacientes y reducir consecuencias más graves de la A. LES.


enfermedad B. Insuficiencia renal aguda.
B. Hacer estudios para poner de manifiesto enfermedades o C. Tuberculosis.
defectos previamente desconocidos D. Pericarditis post infección viral.
C. Implementar todas las medidas disponibles para la detección E. Pericarditis estafilocócica.
precoz de sus causas y de los factores de riesgo
D. Limitar la incidencia de enfermedad mediante el control de sus 17. Paciente varón de 63 años con PA 80/50 mmHg, gasto cardiaco
causas y de los factores de riesgo y saturación de oxígeno venoso disminuido, resistencia Vascular
E. Reducir el progreso de las complicaciones de una enfermedad Sistémica y presión venosa central aumentada. ¿Cuál sería la
ya establecida PROBABLE etiología del shock que tiene la paciente?:

10. De acuerdo con las normas sectoriales, el establecimiento de A. Diarrea por V. choleare.
Salud que cuenta con servicio de emergencia, sala de B. Sepsis punto de partida urinario.
operaciones, las cuatro especialidades básicas y mediana C. IMA ST elevado anterolateral.
capacidad resolutiva, es categorizado como: D. Lesión medular.
E. Hemorragia aguda
A. I-3
B. I-4 18. La tuberculosis asociada a la infección por VIH se caracteriza
C. II-1 por:
D. II-2
E. III-1 A. Presentación subclinica de la enfermedad
B. Aparición característica en los estadios de inmunodepresión
11. ¿Cuál de las siguientes alternativas constituye una estrategia más severa
sanitaria nacional?: C. Menor frecuencia de afectación extrapulmonar y diseminada que
los seronegativos
A. Salud escolar D. Escaso rendimiento de los métodos microbiológicos de
B. Atención integral a la familia diagnóstico
C. Atención por etapas de la vida E. Mala respuesta al tratamiento antituberculoso
D. Atención integral al individuo
E. Salud de los pueblos indígenas 19. En el tratamiento antiTBC el fármaco que actúa como
bacteriostático en “bacilos en reposo” y como bactericida en fase
12. ¿Cuál de las siguientes se cataloga como angina inestable?: de multiplicación rápida es:

A. Angina que aparece con el primer esfuerzo del día. A. Etionamida


B. Angina al realizar cualquier tipo de actividad física. B. Estreptomicina
C. Angina de esfuerzo que no cede con nitroglicerina sublingual. C. Etambutol
D. Angina que se inició hace 4 meses. D. Pirazinamida
E. Angina al subir un tramo de escaleras. E. Isoniazida

20. Con respecto a los hallazgos en LCR correlacione:


13. El infarto de miocardio de pared posterior de ventrículo izquierdo
y área postero septal es debido generalmente a la obstrucción a. MEC TBC
de: b. MEC viral
c. MEC bacteriana
A. Tronco común de arterio coronaria izquierda. d. Todos
B. Rama descendente anterior de arteria coronaria izquierda. e. a y c
C. Rama circunfleja izquierda. f. ayb
D. Rama marginal izquierda. ( ) LCR Amarillo citrino
E. Arteria coronaria derecha. ( ) Tiene hipoglucorraquia
( ) Hay aumento de celularidad
14. La principal estrategia terapéutica actual para el IMA, que ( ) Predominio de mononucleares
presenta supra desnivel del ST en el ECG y que no cede tras la A. a,e,d,f
administración de nitroglicerina y aspirina es: B. a,e,d,b
C. a,b,c,d
A. Betabloqueantes EV. D. b,c,d,f
B. Trombolíticos EV o angioplastia primaria. E. b,e,c,d
C. Bloqueantes cálcicos.
exploración física se aprecian signos de deshidratación, presión
21. ¿Cuál es la opción incorrecta?: arterial 95/54 mmHg y no hay signos de focalidad neurológica.
En la analítica destaca leucocitosis, creatinina de 1.8 mg/dL
A. Chancro duro, Treponema pallidum; úlcera no doloroso ¿Cuál es el diagnóstico más probable?
B. Chancro blando, Haemophilus ducreyi; úlcera dolorosa
C. Linfogranuloma venéreo, Chlamydia trachomatis; úlcera no A. Coma hiperglucémico hiperosmolar no cetósico
dolorosa B. Insuficiencia cardíaca
D. Condiloma acuminado, virus del papiloma humano; verruga C. Hematoma subdural
húmeda D. Insuficiencia renal de causa obstructiva
E. Condiloma plano, Neisseria gonorrhoeae; secreción uretral E. Cetoacidosis diabética con coma
purulenta
28. Con respecto a la hipoglucemia marca lo falso:
22. ¿Qué síntoma no corresponde a la MANÍA?
A. Se define como la existencia de signos o síntomas de
A. Pensamiento inhibido hipoglucemia, niveles de glucosa bajo y mejoría de los síntomas
B. Agitación psicomotriz tras
C. Hiperactividad la elevación de glucosa.
D. Desenfreno B. La triada de Whipple es característica.
E. Verborrea C. Existe una primera fase en la que se dan síntomas adrenérgicos
y después otra con síntomas neuroglucopénicos.
23. Mujer de 47 años sin antecedentes psiquiátricos que es D. La presencia de cifras de glucosa > 70 mg/dL durante un
hospitalizada para estudio de metrorragias y a los 5 días de su episodio sintomático descartan hipoglucemia como cau- sante
ingreso es informada del diagnóstico de neoplasia uterina con del cuadro.
metástasis. Se consulta a Psiquiatría porque 24 horas después E. Los IECA y ARA - 2 no están asociados a hipoglucemia.
muestra tristeza y llanto frecuente, refiere ideas de muerte y
presenta insomnio. En la evaluación psiquiátrica no se recoge 29. El cambio más característico y precoz que se observa en la
ningún antecedente psiquiátrico y la exploración detecta elevada retinopatía diabética está representado por:
ansiedad y desesperanza en relación con las consecuencias de
su enfermedad neoplásica. El diagnóstico más probable es: A. Microaneurismas
B. Edema de papila
A. Trastorno esquizoafectivo. C. Hemorragias
B. Distimia. D. Neuritis óptica
C. Trastorno de ansiedad generalizada. E. Exudados
D. Trastorno por estrés postraumático.
E. Reacción normal frente al estrés 30. Cuando encontramos unas concentraciones séricas de T3
normal, T4 libre ligeramente disminuida y TSH elevada con
respuesta aumentada a la TRH, nos hallamos ante un:

24. Son indicaciones de Benzodiacepinas excepto: A. Hipotiroidismo primario.


B. Hipotiroidisrno hipofisario.
A. Delirium Tremens. C. Hipotiroidismo hipotalámico.
B. Síndrome de apnea del sueño. D. Funcionamiento tiroideo dentro del límite de la normalidad.
C. Depresión ansiosa. E. Hipertiroidismo.
D. Preanestesia.
E. Tétanos 31. ¿Cuál de los hallazgos descritos NO acompaña a la dislipemia
del diabético?:
25. Varón de 85 años de edad. Desde hace un mes presenta pérdida
de interés por lo que lo rodea, olvidos frecuentes, insomnio, A. LDL-colesterol más susceptible a la oxidación.
irritabilidad y pérdida de peso. Tiene un test mini-mental normal. B. Niveles bajos de HDL-colesterol.
¿Cuál es el diagnóstico más probable?: C. Niveles elevados de HDL-colesterol.
D. Niveles elevados de Triglicéridos plasmáticos.
A. Demencia vascular E. Presencia de LDL pequeñas y densas.
B. Demencia tipo Alzheimer
C. Síndrome confusional agudo 32. Paciente de 70 años de edad, con anemia crónica severa. VCM
D. Demencia frontal 105fl y HCM 20pg. Considerando estos valores ¿Cómo clasifica
E. Depresión a la anemia?

26. Con respecto a la obesidad, señale lo incorrecto: A. Macrocitica hipocrómica.


B. Microcítica hipocrómica.
A. Es un factor de riesgo cardiovascular C. Normocítica normocrómica.
B. Se asocia frecuentemente a diabetes tipo 2 D. Macrocitica normocrómica.
C. La obesidad periférica es la que se asocia a un mayor riesgo E. Microcítica normocrómica crónica.
cardiovascular
D. Se clasifica basándose en el Índice de Masa corporal 33. Señale la respuesta falsa respecto al metabolismo del hierro:
E. La medida del perímetro abdominal también es una intervención
de utilidad para clasificar la obesidad A. La mayor parte del hierro del organismo se encuentra en los
depósitos.
27. Mujer de 76 años con obesidad y diabetes mellitus tipo 2 en B. La mayor parte del hierro necesario para la eritropoyesis basal
tratamiento con Metformina. Acude a urgencias porque desde procede de la destrucción de los hematíes viejos.
hace 3 días presenta fiebre de 38ºC, polaquiuria, disuria, intensa C. La absorción intestinal es superior en forma de hierro hem de la
sed y disminución progresiva de su nivel de conciencia. En la dieta.
D. Las pérdidas de hierro en la mujer suelen ser el doble que en el E. Nefritis intersticial por drogas
hombre.
E. Las situaciones de eritropoyesis ineficaz incrementan la 40. En qué estadio de Clark estaría un melanoma que afecte de
absorción intestinal. forma incompleta a la dermis papilar:

34. Las gastritis agudas erosivas o ulcerdas pueden ser producidas A. I


por todas las siguientes causas menos una, indique B. II
cuál: C. III
D. IV
A. Shock séptico. E. E. V
B. Colonización de la mucosa por Helicobacter Pylori.
C. Ingesta de ácido acetil salicílico.
D. Traumatismo de cráneo grave. 41. De los siguientes hallazgos radiológicos, cual no corresponde a
E. Quemadura extensa. Criptococosis pulmonar:

35. Señale el enunciado CORRECTO respecto a Enfermedad A. Lesiones en masa circunscrita.


Ulcero Péptica: B. Infiltrado neumónico difuso con cavidades.
C. Milia.
A. La úlcera gástrica se perfora con más frecuencia que la D. Lesiones en perdigonazo.
duodenal. E. Nódulos solitarios subpleurales.
B. Un 10% de las úlceras duodenales recidivan.
C. Las úlceras duodenales suelen ser más grandes que las 42. Paciente mujer de 34 años, puérpera de 1 semana post cesárea,
gástricas. que presenta súbitamente dolor torácico y disnea con esputo
D. Las úlceras duodenales aparecen sobre todo en la primera hemoptoico. Al examen: PA 120/70 FC 110 FR 30 T 36ºC. Tórax:
porción del duodeno. MV pasa en ACP, no sibilantes. Rx tórax Normal. AGA: pH: 7.46
E. Las úlceras del fundus siempre son benignas. pO2: 65 pCO2: 24 HCO3: 22 mm de Hg. EKG: taquicardia
sinusal, no desnivel ST, resto sin alteraciones. Hemograma, TP,
36. Drogadicto de 30 años, acude a emergencia por presentar TTP, plaquetas, CPK, MB: Sin alteraciones. Luego de iniciar
astenia, malestar general y fiebre, posteriormente cursa con oxígeno, lo inmediato siguiente sería:
ictericia. Se sospecha de hepatitis viral B aguda. ¿Qué
marcadores serológicos confirman el diagnóstico? A. Iniciar anticoagulación con heparina.
B. Pulsos de corticoides, considerar ciclofosfamida.
A. RNA de HBV - anti HBs. C. AAS, nitratos, analgesia y HBPM
B. Anti HBe Ag - HBs Ag. D. TAC helicoidal
C. Ac anti HBe - Anti HBc. E. Solicitar troponina T.
D. HBs Ag + IgM HBc.
E. Ac anti HBs - HBs Ag 43. El tratamiento profiláctico de la crisis migrañosa involucra
fármacos que pertenecen a los siguientes grupos terapéu- ticos:
37. Paciente adulto mayor, procedente de la sierra con hemorragia
gastrointestinal, sometido a una endoscopia y colonos- copia, en A. Inhibidores de la receptación de la serotonina, antidepresivos
las cuales no se observan anormalidades. ¿Cuál sería la prueba tricíclicos, beta bloqueadores.
diagnóstica más adecuada?: B. Anticonvulsivantes, betas bloqueadores y ansiolíticos.

A. Solicitar parasitológico en heces. C. Antagonistas triciclicos, ansiolíticos y anticonvulsivantes.


B. Repetir estudios en 3 meses. D. Antidepresivos triciclicos, ansiolíticos y neurolépticos.
C. No es necesario más pruebas. E. Beta bloqueador, AINE y antidepresivos tricíclicos.

D. Coprológico funcional. 44. Paciente varón de 80 años de edad, con antecedentes de ACV
E. Capsula endoscópica. y hemiparesia derecha secuelar, actualmente no recibe
tratamiento, no se entiende lo que habla, no puede permanecer
38. Mujer de 45 años de edad, con Sepsis pp. gastrointestinal. Se de pie, PA 160/90, FC 140, pulso 100, Frecuencia resp. 24,
tiene los siguientes resultados: pH 7,26, p02 74mmHg, pC02 despierto, Glasgow AO 4, RV 4, RM 6, fuerza muscular izquierda
32mmHg, HC03 10mEq/L, Na 128mEq/L, K 3mEq/L, Cl 5/5, derecha 3/5, Babinski derecho positivo, que impresión
96mEq/L. El diagnóstico gasométrico es Acidosis me- tabólica diagnóstica es la más probable:
…………… con anión gap…………..:
A. ECV hemorrágico intraparenquimal derecho.
A. Compensada / alto. B. ECV isquémico capsular interno derecho.
B. Descompensada / alto. C. ECV isquémico protuberancial.
C. Descompensada / normal. D. Infarto lacunar cortical izquierdo.
D. Compensada / normal. E. ECV isquémico capsular interno izquierdo.
E. Descompensada / bajo.
45. ¿Cuál de los siguientes constituye un cambio respiratorio
39. Mujer de 15 años con edemas maleolares y periorbitarios, fisiológico durante el embarazo?:
efusión pleural, leucopenia, proteinuria de 4g/d, numerosos
hematíes y cilindros en el sedimento, C3 40mg/dl, C4 5mg/dl. La A. La capacidad vital disminuye.
nefropatía más probable es: B. La capacidad inspiratoria disminuye.
C. El volumen residual disminuye.
A. GMN rápidamente progresiva D. El volumen minuto respiratorio disminuye.
B. GMN membranosa E. El volumen pulmonar total no se modifica.
C. Nefropatía lupica
D. Amiloidosis renal
46. ¿Cuál de las siguientes afirmaciones respecto a la función piernas que no desaparecen con el reposo en cama, al que se
endocrina durante el embarazo es FALSA?: agrega el día de ayer dolor abdominal, lumbalgia y polaquiuria,
y desde hace 1 hora pérdida de flujo vaginal acuoso en regular
A. Los niveles de HCG aumentan durante el 1er trimestre. cantidad con olor a lejía. Al examen clínico: adelgazada, pálida
B. El lactógeno placentario tiene su máximo nivel alrededor de la y ansiosa, LOTEP, PA: 145/90mm Hg, AU de 32 cm, DU:
10ª semana. (2/10)(++)(30”), LCF: dos focos 160 y 140 lat/min; Ex.
C. Tras la muerte fetal, los niveles de progesterona pueden persistir Ginecológico.: Se constata pérdida de LA claro en regular
elevados durante semanas. cantidad, TV: cérvix posterior, blando, de 2 cm de largo, canal
D. La producción de estriol requiere la síntesis de DHEA-S en las cervical dehiscente 1 dedo. De las siguientes probabilidades
suprarrenales fetales, su 16-alfa-hidroxilación en el hígado fetal diagnósticas, marque la que no corresponde.
y su aromatización en la placenta.
E. El precursor de la progesterona es el colesterol materno. A. Amenaza de parto pretérmino
B. Rotura prematura de membranas
47. ¿Cuál de los siguientes marcadores de cromosomopatías del C. Hipertensión inducida por el embarazo
primer trimestre tiene más valor?: D. Trabajo de parto prematuro
E. Infección del tracto urinario
A. Gonadotropina coriónica.
B. La alfa-fetoproteína. 53. Decide inducir el parto en una gestante de 42 semanas,
C. La PAPP–A. presentación cefálica y test de Bishop de 9. Lo haría con:
D. La sonoluscencia nucal.
E. El acortamiento del fémur. A. Oxitocina, 1 ampolla intramuscular cada dos horas.
B. Oxitocina intravenosa en perfusión continua.
48. ¿Qué tiempo después de la enfermedad trofoblástica molar C. Gel de prostaglandinas E2 intracervical.
(mola hidatiforme) generalmente hace su aparición la D. Indometacina 25 mg/4 horas.
enfermedad trofoblástica maligna?: E. Tallos de laminaria intracervicales.

A. 4 a 6 años. 54. En una pelvis normal el conjugado diagonal o promonto -


B. 5 a 10 años. subpúbico del estrecho superior mide:
C. 36 meses.
D. 2 a 28 meses. A. 13,0 cm.
E. 6 a 24 meses. B. 10,5 cm.
C. 11,0 cm.
49. ¿Cuál es la complicación más grave del desprendimiento D. 9,5 cm.
prematuro de placenta?: E. 12,5 cm.

A. Déficit de gammaglobulina 55. Puérpera de 7 días, regresa al hospital presentando fiebre de


B. Déficit de vitamina K 39,5 C, cefalea, dolor de bajo vientre, útero subinvolu- cionado
C. Purpura trombocitopénica con loquios purulentos y fétidos. ¿Cuál es el diagnóstico más
D. CID probable?:
E. Hipocalcemia
A. Infección urinaria aguda
50. Multigesta a término, no trabajo de parto, que ingresa por fiebre, B. Endometritis puerperal
malestar general, escalofríos, taquicardia, dolor ute- rino y C. Vulvovaginitis mixta
pérdida de líquido amniótico claro por vagina desde hace 24 D. Vaginosis bacteriana
horas. Al examen: latidos fetales: 152 por minuto, cérvix blando, E. Infección pélvica aguda
posición intermedia, incorporado 50%, dilatación dehiscente 1
cm, estación cefálica 2, pelvis ginecoide compatible. Peso 56. Primigesta de 19 años con 33 semanas de gestación. PA:
estimado fetal: 3.000 g. Hemograma con leucocitosis y 170/110 mmHg, proteinuria: 5 g/L en orina de 24 horas,
desviación izquierda. El diagnóstico más pro- bable es... y su plaquetas < 100.000/mm3, deshidrogenase láctica > 600 Ul/L,
manejo más indicado es...: hemolisis microangiopática. ¿Cuál es el diagnóstico?:

A. Corioamnionitis /antibiótico-terapia y cesárea A. Hipertensión crónica


B. Rotura prematura de membranas / antibioticoterapia y cesárea B. Hipertensión crónica con síndrome HELLP
C. Corioamnionitis /antibioticoterapia e inducción de parto C. Pre-eclampsia severa y alteración renal
D. Rotura prematura de membranas / antibioticoterapia e inducción D. Hipertensión gestacional con CID
de parto E. Pre-eclampsia severa con síndrome HELLP
E. Corioamnionitis / antibioticoterapia y expectación

51. Una de las siguientes afirmaciones, respecto al síndrome de 57. En la infección congénita por virus de inclusión citomegálica,
transfusión feto-fetal, es FALSA. Señálela: este puede llegar al feto por:

A. Se produce una discordancia en los parámetros abdominales de A. Vía hematógena.


ambos gemelos. B. Canal del parto.
B. El feto transfusor puede llegar a quedar acardio. C. Vía ascendente.
C. Se debe a la presencia de anastomosis arteriovenosas. D. Todas las anteriores.
D. No hay ninguna posibilidad terapéutica intrauterina. E. Ninguna de las anteriores.
E. Es necesario que la placentación sea monocorial.
58. Paciente de 25 años, acude a emergencia por sangrado vaginal
52. G1 de 17 años, soltera con 06 meses de amenorrea, sin control acompañado de coágulos. Al tacto vaginal: cuello ute- rino con
prenatal y que acude a Emergencia por presentar desde hace 1 orificios externo e interno abiertos., ¿Cuál es el diagnóstico
semana dolor de cabeza fronto-temporal e hinchazón de las probable?
A. Cistoadenocarcinoma.
A. Aborto incompleto B. Ginadroblastoma.
B. Amenaza de aborto C. Quiste dermoide.
C. Metrorragia disfuncional D. Tumor de Brenner.
D. Mola hidatiforme E. Tumor del seno endodérmico.
E. Embarazo ectópico
65. ¿Cuál de los siguientes signos NO suele aparecer en la clínica
59. Gestante de 37 semanas, presenta hemorragia genital indolora del cáncer de mama?:
moderada, latidos cardiacos fetales de 150 por minutó,
regulares. ¿Cuál es la conducta a seguir? A. Mastodinia.
B. Telorragia.
A. Tacto Vaginal C. Tumoración.
B. Test no estresante D. Retracción del pezón.
C. Terminación del embarazo E. Adenopatía axilar.
D. Ruptura de membranas.
E. Estudio ecográfico 66. Mujer menopáusica con factores de riesgo positivos para cáncer
de endometrio. Acude con sangrado uterino impor- tante de
60. ¿Cuál es la complicación que se asocia al embarazo prolongado inicio súbito; la primera medida a practicarse en consulta externa
o post término? es:

A. Enfermedad de membrana hialina A. Ecografía transvaginal.


B. Oligoamnios B. Citología de Papanicolaou.
C. Enfermedad hipertensiva del Embarazo C. Histeroscopía.
D. Polihidramnios D. Dilatación y legrado.
E. Embarazo Múltiple E. Biopsia endometrial.

61. En el trabajo de parto normal, cuando la flexión se completa, el 67. ¿Cuál es el principal factor de riesgo para el cáncer de cuello
diámetro de la cabeza fetal que ingresa en el estrecho superior uterino?:
de la pelvis es:
A. Múltiples compañeros sexuales
A. Occipitofrontal
B. Suboccipitobregmático B. Inicio temprano de relaciones sexuales
C. Occipitomentoniano C. Bajo nivel socioeconómico
D. Infección por papiloma virus humano
D. Biparietal E. Antecedente familiar de cáncer de cérvix
E. Bitemporal
68. El antibiótico en una mujer de 32 años de edad, con cervicitis
62. Sobre infecciones TORCH, indique (V) verdadero y (F) falso, y mucopurulenta y PCR positivo para Chlamydia tra- chomatis es:
marque la combinación correcta:
A. Azitromicina
( ) La transmisión vertical de la toxoplasmosis ocurre con más B. Ciprofloxacina
frecuencia en el 1er trimestre, C. Cloxacilina
( ) Menos de la cuarta parte de los recién nacidos con toxoplasmosis D. Metronidazol
tienen una evidencia clínica de enfermedad al momento de E. Penicilina benzatinica
nacer,
( ) Casi todos los lactantes infectados de toxoplasmosis hacen 69. Paciente de 35 años de edad, acude por leucorrea y prurito
finalmente Corioretinitis, vulvovaginal. Al examen: leucorrea blanquecina grumosa, adhe-
( ) Es más probable que el feto se infecte con el Citomegalovirus si rida a paredes vaginales, sin mal olor, cérvix epitelizado, mucosa
hay infección materna durante la primera mitad del em- vaginal eritematosa. ¿Cuál es el diagnóstico más probable?:
barazo,
( ) La enfermedad por inclusión citomegálica se caracteriza por A. Vaginosis bacteriana
glaucoma, microftalmia, catarata y defectos del tabique inter- B. Tricomoniasis vaginal
ventricular. C. Candidiasis vaginal
A. VFVFF D. Vaginitis atrófica
B. FVFVV E. Vaginitis mixta
C. FVVVF
D. VFFVF 70. Mujer de 60 años de edad, con ictericia marcada y prurito, dolor
E. FFFVV tipo sordo en cuadrante superior derecho de un mes de evolu-
ción, así como anorexia y pérdida de peso de 10 Kg en 6 meses.
63. ¿Cuál es el tratamiento de elección en el carcinoma de cérvix En los exámenes ecográficos se encuentra dilatación de las vías
estadio Ib1 de la clasificación de la FIGO?: biliares intrahepáticas y en la colangiopancreatografía
retrógrada endoscópica se encuentra una estenosis focal. Su
A. Radium intracavitario y telecobaltoterapia. diagnóstico más probable es:
B. Histerectomía y doble anexectomía.
C. Conización cervical cuyos bordes excedan en más de un A. Carcinoma de las vías biliares.
centímetro los de la lesión. B. Coledocolitiasis.
D. Histerectomía radical más linfadenectomia. C. Síndrome post-colecistectomía.
E. Tratamiento antiblástico combinado. D. Colecistitis enfitematosa.
E. Íleo biliar.
64. ¿Cuál es el tumor de ovario más frecuente en la mujer joven?
71. Un paciente con acalasia ha sido tratado con dilataciones C. 2000 mL.
endoscópicas no teniendo ningún resultado. Su actitud te- D. 3000 mL.
rapéutica es: E. 500 mL.

A. By-pass esófagogástrico. 78. La escarlatina se asocia con todos los siguientes datos,
B. Miotomía de Héller. EXCEPTO:
C. Antagonistas del calcio + omeprazol.
D. Resección del esófago con plastias. A. Es debido a estreptococo betahemolítico del grupo A.
E. Continuar con dilataciones. B. La principal secuela es la fi ebre reumática, independiente del
foco de entrada (piel, faringe).
72. ¿Cuál de las siguientes hernias es más frecuente en la mujer? C. El exantema se palpa mejor que se ve, puesto que presenta
textura en forma de “carne de gallina.
A. Hernia inguinal. D. Es infrecuente en menores de 3 años.
B. Hernia de Littré. E. Son características las líneas de Pastia.
C. Hernia obturatriz.
D. Hernia de Bochdaleck. 79. La persistencia del ductus arterioso se asocia a todo lo siguiente,
E. Hernia femoral. EXCEPTO a:

73. Varón de 40 años, post-operado de peritonitis, en tratamiento A. Pulsos periféricos saltones.


antibiótico, al quinto día comienza con picos febriles y baja de B. Mayor frecuencia en prematuros.
peso. Al examen: abdomen distendido, escasos ruidos C. Soplo continuo en el área pulmonar.
hidroaéreos, herida operatoria cerrada no flogótica. Radiografía D. Puede ser efectivo el tratamiento con indometacina.
de tórax: hemidiafragma derecho elevado, derrame pleural con E. Las prostaglandinas son el mejor tratamiento médico para
obturación del seno costo diafragmático del mismo lado. ¿Cuál conseguir su cierre.
es el diagnóstico más probable?
80. Paciente de 26 años de edad, con diagnóstico de insuficiencia
A. Absceso retroperitoneal. renal crónica, en hemodiálisis. Acude a Emergencia por
B. Neumonía derecha. presentar: K 7,5 mEq/L, con alteraciones en el ECG. La acti-tud
C. Atelectasia pulmonar. inmediata sería:
D. Absceso pélvico. A. Administrar Gluconato de calcio EV.
E. Absceso subfrénico. B. Administrar bicarbonato de sodio.
C. Administrar dextrosa e insulina.
D. Beta-2 agonistas.
74. El tumor benigno más frecuente del estómago es: E. Administrar Furosemida.

A. Fibroma. 81. Es FALSO respecto a la HTA de origen renovascular:


B. Leiomioma.
C. Angioma. A. Es la causa de etiología tratable más frecuente de HTA.
D. Lipoma. B. Solo causa el 3% de las HTA.
E. Neurofibroma. C. Es más frecuente en mujeres jóvenes.
D. Debe hacernos sospechar una edad inusual.
75. Paciente varón de 35 años, que presenta, tras accidente de E. El tratamiento de elección es el Captopril.
tráfico, traumatismo abdominal contuso con shock hipo-
volémico, dolor y distensión abdominales. En la eco fast aparece 82. Un paciente de 15 años de edad consulta por poliuria y nicturia,
hemoperitoneo. El diagnóstico y tratamiento que le parece más cansancio fácil y astenia. Presenta dichos síntomas desde hace
indicado: años y tienden a intensificarse en veranos muy calurosos, en los
que se asocia hormigueo lingual y peri- bucal. A la EF destaca
A. Rotura esplénica - Laparotomía urgente. TA 100/50 mmHg. El ionograma muestra Na 135 mEq/l, K 2
B. Rotura hepática - Laparoscopia exploradora. mEq/l, Cl 105 mEq/l, pH 7,45, bicarbonato 30 mEq/l. La
C. Laceración hepática - Vigilancia en UCI. determinación de renina y al-dosterona muestra cifras elevadas,
D. Rotura diafragmática - Reparación quirúrgica urgente. tanto basales como tras estímulo. De los procesos que siguen,
E. Rotura de vena cava - Embolización. ¿cuál es compatible con el cuadro clínico descrito?
A. Hiperaldosteronismo primario.
76. En un paciente de 78 años, intervenido con el diagnóstico de B. Estenosis de arteria renal.
peritonitis aguda, se halla en el acto operatorio una neo- plasia C. Síndrome de Liddle.
de sigmoides perforado. ¿Cuál sería la actitud terapéutica a D. Enfermedad de Addison.
seguir? E. Síndrome de Bartter

A. Colostomía transversa. 83. En los neonatos, la ictericia de piel y mucosas no aparecen


B. Colostomía transversa + sutura de la perforación. usualmente hasta que la bilirrubina sérica excede de:
C. Colostomía transversa + drenaje de la perforación.
D. Resección de la neoplasia + colostomía y ano mucoso. A. 2 mg/dL
E. Resección de la neoplasia + anastomosis primaria. B. 5 mg/dL
C. 8 mg/dL
77. Varón de 32 años, 70 kg de peso, con quemadura del tercer D. 10 mg/dL
grado y 20% de superficie corporal; según la fórmula de E. 15 mg/Dl
Parkland requiere 2800 mL de suero fisiológico. En las primeras
8 horas se deben de pasar: 84. Un recién nacido, con peso al nacer de 4300 gramos, hijo de
madre con diabetes gestacional, doble circular ajustada a cuello;
A. 1400 mL. se muestra muy inquieto a los 30 minutos de vida; quince
B. 1000 mL.
minutos más tarde, sufre una convulsión tónico clónica. El E. Onda P negativa en derivación II
diagnóstico más probable es : 93. Las células de Reed Sternberg son características de:

A. Hipoglucemia. A. El linfoma no Hodgkin.


B. Hipocalcemia. B. El linfoma de Hodgkin.
C. Hiponatremia. C. Leucemia.
D. Hiperviscosidad. D. Leucemia mieloblástica.
E. Encefalopatía hipoxico isquémica E. Papiloma ductal.

85. El tratamiento empírico inicial de la sepsis neonatal de inicio 94. La arteria meníngea media atraviesa el agujero:
precoz es: A. Esfenopalatino.
B. Oval.
A. Meticilina y cefotaxima C. Redondo mayor.
B. Ampicilina y cefotaxima D. Redondo menor.
C. Vancomicina y Amikacina E. Rasgado anterior.
D. Ampicilina y Gentamicina
E. Penicilina G y kanamicina 95. El conducto de Stenon drena al:
86. El “síndrome de hombre rojo” es un efecto adverso de la A. Glándula submandibular.
vancomicina, que se presenta por... B. Glándula sublingual.
A. Estimular poliglobulia C. Glándula parótida.
B. Generar taquicardia D. Glándula lacrimal.
C. Estimulación de centros pirógenos E. Páncreas.
D. Liberación de histamina por mastocitos
E. Extravasación de hematíes 96. En el síndrome de Guillain Barre, la alteración más frecuente del
líquido cefalorraquídeo es:
87. Entre los efectos colaterales del litio NO se incluye A. Proteínas elevadas.
A. Aumento de peso B. Glucosa disminuida.
B. Gastritis C. Cloruro disminuido.
C. Polidipsia D. Cloruro aumentado.
D. Leucocitosis E. Magnesio disminuido.
E. Convulsiones
97. ¿Cuáles son los tumores primarios que más frecuentemente
88. ¿Cuál es la especie predominante en la flora colónica? producen metástasis cerebrales?

A. Bifidobacterium A. Cáncer de cuello uterino y de próstata.


B. Bacteroides B. Linfoma y carcinoma gástrico.
C. S. aureus C. Cáncer de pulmón y de mama.
D. E. coli D. Cáncer gástrico y de páncreas.
E. Enterococo E. Cáncer gástrico y de colon.

89. Las exotoxinas son: 98. Paciente de 80 años que se presenta en ambas extremidades
ampollas grandes, tensas, algunas con contenido sanguí- neo,
A. Estables al calor sobre una base urticariana y con moderado prurito. No hay
B. Menos potentes que las endotoxinas compromiso del estado general y la histopatología muestra
C. Parte de la pared celular de la bacteria lesiones subdérmicas con acumulo de eosinófilos en la dermis
D. Proteínas en naturaleza papilar. La inmunofluorescencia muestra depó- sito lineal de IgG
E. Lípopolisacáridos en naturaleza y de C3. ¿Cuál es la dermatopatia más probable?

90. ¿Cuál es el tipo de estreptococo que produce estreptolisina O? A. Dermatitis herpetiforme.


B. Pénfigo cicatricial.
A. Faecalis C. Pénfigo seborreico.
B. Bovis D. Pénfigo vulgar.
C. Pyogenes E. Penfigoide ampollar.
D. Agalactiae
E. E. Pneumoniae 99. El test estadístico que permite probar diferencias entre dos
medias en muestra pequeñas es:
91. ¿Cuál de las siguientes funciones NO se relaciona con las
células de Sertoli? A. Chi cuadrado.
B. “t” de Student.
A. Conservan la espermatogénesis C. Correlación.
B. Producen factor estimulante de los conductos de Muller D. Análisis de varianza.
C. Sostienen la barrera hemato-testicular E. Regresión.
D. Producen la inhibina
E. Aromatizan la testosterona 100. En un paciente de 62 años, con el antecedente de haber
trabajado durante cerca de 40 años como ebanista, que de-
92. ¿Cuál es la característica de un electrocardiograma normal? sarrolla progresivamente en el curso de 4 meses un cuadro de
obstrucción nasal unilateral, rinorrea sanguinolenta y cierto
A. Onda R positiva grado de proptosis con desviación del globo ocular hacia afuera
B. Onda T negativa en derivación I y abajo. ¿Cuál es el diagnóstico más probable?:
C. Onda Q ancha y profunda
D. Onda T negativa, en derivación II A. Carcinoma epidermoide de seno maxilar.
B. Neuroestesioblastoma olfatorio.
C. Poliposis nasoetmoidal. NO ÚLCERA 170 9680
D. Adenocarcinoma de etmoides.
E. E.Mucocele frontoetmoida De las siguientes cifras ¿Cuál es la más cercana a la sensibilidad del
diagnóstico clínico de la úlcera?
A. 3%.
B. 13%.
C. 87%.
D. 98%.
E. 100%.

3. La atención primaria de la salud se considera en su desarrollo:

A. Sólo participación de la comunidad.


B. Los recursos naturales y humanos de la comunidad.
C. La participación conjunta de la comunidad organizada con el
apoyo del estado y de otros organismos.
D. La participación de la comunidad bajo tutela estatal.
E. Ninguna anterior.

4. La lesión histopatología característica de la glomerulonefritis


rápidamente progresiva es:

A. Proliferación mesangial.
B. Engrosamiento de la membrana basal.
C. Compromiso glomerular focal y segmentario.
D. Proliferación epitelial extracapilar.
E. Proliferación endotelial.

5. ¿Qué haría si un RN no presenta movimientos respiratorios


luego de aspirarlo, colocarlo en posición adecuada y es-
timularlo táctilmente

A. Obtener puntaje de APGAR.


B. Iniciar ventilación con presión positiva con ambú.
C. Evaluar color de piel.
SIMULACRO 8B D. Ofrecer oxigeno flujo libre.
E. Evaluar la frecuencia cardiaca.

1. Respecto a los trastornos de ansiedad señale la respuesta 6. En el manejo de Pielonefritis aguda en una gestante de 37
FALSA: semanas se indica:

A. Los pacientes que sufren un trastorno obsesivo compulsivo A. Hidratación mínima 3 litros por vía oral
tienen ideas obsesivas irracionales e intrusivas. Las más fre- B. Disminuir la temperatura superior a 38 Celsius con Aspirina de
cuentes son las obsesiones de limpieza. 500mg VO
B. Son los trastornos más comunes en la población general. Son C. Solo Ampicilina 1 gramo EV cada 6 horas
más frecuentes en mujeres y su incidencia disminuye con la D. Todas las anteriores
edad. E. Ninguna de las anteriores
C. El abuso de alcohol es una complicación grave y frecuente.
D. El trastorno por estrés postraumático suele aparecer en los seis 7. Varón soltero de conducta de aislamiento social, negligencia en
primeros meses tras el suceso traumático. su aseo personal, 21 años de edad, quien desde hace 8 meses
E. El trastorno de ansiedad generalizada se caracteriza por la escucha voces que lo insultan, refiere que es controlado por la
presencia de crisis de angustia, recurrentes e inesperadas y antena parabólica de su barrio. El diagnóstico MÁS probable es:
puede cursar con o sin agorafobia.
A. Trastorno delirante persistente.
2. Está usted comparando el diagnóstico clínico de úlcera B. Psicosis reactiva breve.
gastroduodenal y su hallazgo en la autopsia en una serie de C. Trastorno esquizoafectivo.
10.000 pacientes. Al comenzar a analizar los datos se construye D. Trastorno esquizofrenoide.
la siguiente tabla: E. Esquizofrenia.

8. ¿Cuál es el tumor testicular más frecuente?


AUTOPSIA
A. Teratoma.
ULCERA NO ULCERA B. Linfoma.
C. Gonadoblastoma.
DIAGNÓSTI D. Seminoma.
CO E. Neuroblastoma.

ULCERA 30 20
9. Gestante de 11 semanas, con náuseas y vómitos esporádicos individuos seleccionados constituyen la muestra de esta
desde hace una semana. ¿Cuál es la recomendación MÁS población. Esta descripción corresponde a un:
pertinente? A. Muestreo por conglomerado
B. Muestreo al azar simple
A. Tiamina EV. C. Muestreo sistemático
B. Terapia psicológica. D. Muestreo estratificado
C. Dimenhidrinato EV. E. Muestreo por racimos
D. Metoclopramida oral. 17. En un estudio epidemiológico de casos controles, el grupo de
E. Observación. control está integrado por:

10. La tríada clásica de las manifestaciones clínicas ocasionadas A. Los sanos no expuestos a la causa
por el incremento de la presión intracraneal incluye las B. Los sanos expuestos y no expuestos a la causa
siguientes, EXCEPTO: C. Los enfermos expuestos a la causa
D. Los sanos expuestos a la causa
A. Cefalea. E. Los enfermos no expuestos a la causa
B. Vómitos.
C. Midriasis. 18. El concepto de supervisión MÁS aceptado en la ejecución de los
D. Edema de papila. programas de salud es el siguiente:
E. Todas. A. Proceso de control de la ejecución y desempeño técnico,
realizado por un superior jerárquico.
11. Ante un paciente que desarrolla un bloqueo AV de forma aguda, B. Proceso de guiar, educar y corregir las actividades, realizada por
el primer proceso a descartar será: un experto.
C. Proceso de evaluación con la finalidad de mejorar las normas y
A. Isquemia. procedimientos.
B. Tumores. D. Evaluación del desempeño técnico con fin de sancionar o
C. Traumatismos. premiar los subordinados.
D. Endocarditis aguda. E. Previene el conformismo.
E. Sarcoidosis.
19. El precursor metabólico inmediato de aceto-acetato es:
12. La Neoplasia cerebral MÁS frecuente en el adulto es:
A. Acetoacetil-coA
A. Glioblastoma multiforme. B. Acetil CoA-Acetato
B. Ependimoma. C. Succinil-coA
C. Metástasis. D. Beta-hidroxibutiril-coA
D. Astrocitoma. E. Hidroximetilglutamil-coA
E. Meningioma.
20. ¿Cuál de los siguientes átomos, es MENOS abundante en el ser
13. La causa más frecuente de epistaxis en niños es: humano?:

A. Telangiectasia. A. Sodio
B. Trauma. B. Oxigeno
C. Sinusitis. C. Carbono
D. Pólipos. D. Nitrogeno
E. Trombocitopenia. E. Hidrogeno

14. Señale lo FALSO con respecto a la Enfermedad litiasis 21. Realizan el transporte transepitelial de antígenos y patógenos al
Vesicular. sistema inmuno-mucoso intestinal:

A. El paciente con cólico biliar cede frecuentemente con A. Macrófagos


antiespasmódicos en menos de 6 horas. B. Células de paneth
B. La mayoría de los cálculos son de pigmento biliar. C. Enterocitos de las vellosidades
C. La pared de la vesícula por ecografía normalmente mide menos D. Células M
de 3 mm. E. Linfocitos intraepiteliales
D. La causa más frecuente de colecistitis aguda es obstrucción por
litiasis. 22. Cuál de los analgésicos es el MÁS indicado para administrar en
E. Los cálculos vesiculares pueden pasar a la vía biliar hasta en un el trabajo de parto:
20%.
15. La rubeola en el niño suele tener un buen pronóstico pero a A. Petidina
veces pueden surgir algunas complicaciones, señale cuales son B. Morfina
las probables complicaciones que pueden surgir: C. Diamorfina
D. Fenazocina
A. Artritis. E. Ninguno de los anteriores.
B. Púrpura trombocitopenica.
C. Encefalitis. 23. Respecto al ciclo ovárico, ¿qué hormonas son interdependientes
D. Solo A y C y actúan sinérgicamente preparando al folículo?:
E. Todas las anteriores.
A. GnRH y progesterona.
16. Al realizar un muestreo se divide a los individuos en subgrupos B. FSH y progesterona.
en base de una característica relevante y, en seguida, se C. FSH y LH.
escogen muestras al azar en cada subgrupo. El total de D. LH y progesterona.
E. GnRH y FSH.
A. Cefalea
24. Señale lo INCORRECTO en relación al cáncer de vulva: B. Vértigos
C. Trastornos visuales
A. El cáncer de vulva corresponde al 3 - 5% de todas las neoplasias D. Anorexia
femeninas. E. Vómitos
B. El cáncer de vulva se está presentando con mayor frecuencia
debido a que las mujeres superan los 70 años. 33. La complicación MÁS grave de la difteria es:
C. El virus PVH es sospechoso en la etiología del cáncer de vulva.
D. En más del 50% de las pacientes con cáncer de vulva hay A. Hepatitis viral
historia de prurito de larga evolución. B. Bronconeumonía
E. El 70% de cánceres de la vulva se origina alrededor del clítoris C. Obstrucción de vías aéreas superiores
D. Miocarditis
25. Tratamiento específico para tricomoniasis vaginal es: E. Endocarditis

A. Penicilina. 34. En cuál de las siguientes muestras existe la probabilidad de


B. Clindamicina. aislar más fácilmente S. typhi durante la primera semana de la
C. Metronidazol. enfermedad:
D. Fluconazol.
E. Clotrimazol. A. Sangre
B. Urocultivo
26. El aporte de oxígeno al feto puede disminuir por: C. Coprocultivo
D. LCR
A. Complicaciones hipertensivas del embarazo. E. Liquido sinovial de las metástasis sépticas
B. Contracciones uterinas prolongadas.
C. Anemia materna crónica y disfunción cardiaca fetal. 35. ¿Cuál de los siguientes datos apoya el diagnóstico de un
D. Ninguna anterior. síndrome de Guillain Barré?:
E. Todas las anteriores.
A. Hiperreflexia.
27. Se habla de hemorragia posparto cuando ésta supera los……. B. Tetraparesia flácida.
en un parto vaginal: C. En el LCR es típico el hallazgo de celularidad alta sin proteínas.
D. Es excepcional el antecedente de infección respiratoria o
A. 200 mL. gastrointestinal.
B. 400 mL. E. Una prueba complementaria útil es la electromiografía.
C. 500 mL.
D. 600 mL. 36. ¿Cuál es el método MÁS sensible para el diagnóstico de la
E. 9000 mL. embolia pulmonar?:

28. Factores que influyen en la incidencia de preeclampsia: A. Resonancia magnética


B. Tomografía axial computarizada
A. Obesidad C. Gammagrafía de perfusión pulmonar
B. Paridad. D. Angiografía pulmonar
C. Raza. E. Ecocardiografía
D. Edad.
E. Todas las alternativas son correctas. 37. Niño con anemia aplásica, malformaciones renales, hipoplasia
de pulgares a hiperpigmentación cutánea. Su diagnós- tico es:
29. El divertículo del Meckel es un remanente del:
A. Síndrome de Kostman.
A. Conducto de Stenon B. Síndrome de Bloom.
B. Conducto de Wolff C. Síndrome de Fanconi.
C. Conducto de Müller D. Síndrome de Blackfan-Diamond.
D. Conducto de Wirsung E. Embriopatía por inhalación materna de benceno.
E. Conducto vitelino 38. ¿Qué germen causa complicaciones infecciosas con más
frecuencia en el EPOC?:
30. La epiglotis aguda es primariamente causada por:
A. H. influenzae.
A. Neumococo. B. A. fumigatus.
B. Estafilococo. C. S. pneumoniae.
C. Hemophilus influenzae tipo B D. E. Coli
D. Rinovirus. E. E. Pseudomonas aureginosa
E. Adenovirus.
31. En el recién nacido, la salivación excesiva está asociada a: 39. Paciente de 20 años acude a emergencia por palpitaciones
intensas de inicio brusco, regulares. Tiene antecedentes de
A. Hernia diafragmática congénita episodios similares de inicio y terminación brusca. La primera
B. Atresia esofágica alternativa diagnóstica es:
C. Estenosis hipertrófica A. Extrasistolia supraventricular
D. Estenosis duodenal B. Taquicardia sinusal
E. Acalasia. C. Taquicardia paroxística supraventricular
D. Bradicardia sinusal
32. La principal manifestación clínica de toxicidad por digital en un E. Episodio de bloqueo AV
lactante seria:
40. ¿Cuál de los siguientes hallazgos endoscópicos es MÁS
característico encontrar en la colitis ulcerosa?: 48. En el planeamiento estratégico en salud, los problemas
epidemiológicos son componentes del análisis de:
A. Úlceras serpinginosas.
B. Presencia de pseudopólipos. A. Interno.
C. Presencia de mucosa normal con áreas de inflamación (lesiones B. Entorno.
alternantes). C. Proceso.
D. Ulceraciones añosas. D. Contexto administrativo.
E. Estenosis del colon. E. Contexto estratégico.

41. RN producto de un parto traumático presenta a la exploración 49. 23. ¿Cuál de los siguientes cálculos urinarios son radiolúcidos?:
física: reflejo de Moro asimétrico, brazo izquierdo en aducción y
rotación interna con el antebrazo en pronación y extensión. A. Cálculos de oxalato de calcio.
¿Qué tipo de lesión presenta?: B. Cálculos ácido úrico.
C. Cálculos de cistina.
A. Fractura de clavícula izquierda D. Cálculos de fosfato triple.
B. Fractura de humero izquierdo E. Cálculos mixtos
C. Lesión de nervios C5 y C6
D. Lesión de nervios C7, C8 y D1 50. En presencia de la normalidad de cuál de los siguientes
E. Lesión raquídea con hemiparesia izquierda exámenes se puede descartar el embolismo pulmonar:

42. Uno de estos síntomas NO es típico de la laringitis aguda A. Radiografía de tórax.


catarral: B. Electrocardiograma.
A. Disfonía. C. Scan de perfusión pulmonar.
B. Disfagia. D. Scan tomográfico.
C. Dolor a la fonación E. Imágenes de resonancia magnética.
D. Dolor cervical difuso.
E. Tos irritativa. 51. La neumonía atípica primaria es una complicación de:

43. La causa MÁS frecuente en el niño de una uveítis crónica es: A. Sarampión.
B. Varicela.
A. Herpes. C. Resfrío común.
B. Artritis reumatoide juvenil. D. Ninguna de ellas.
C. Micosis. E. Hay más de una respuesta correcta.
D. Toxocariasis.
E. Citomegalovirus. 52. Señale la MEJOR afirmación referente a la Loperamida:

44. EI dolor en una peritonitis aguda difusa es: A. Actúa principalmente en receptores opioides centrales.
B. No debe utilizarse en menores de 6 años.
A. Agudo y constante. C. En adultos se puede ingerir hasta 4 mg al día.
B. Cólico y permanente. D. Tiene una vida media de 3 horas.
C. Punzante y autolimitado. E. Coadyuvante en el tratamiento de la diarrea crónica.
D. Agudo y cólico.
E. Todas las opciones son posibles. 53. La hemorragia uterina anormal en la mujer perimenopáusica,
45. El hiato de Winslow, que comunica la transcavidad de los nos obliga a descartar primero:
epiplones, está limitado en su lado inferior por:
A. Amenaza de aborto.
A. Vena cava inferior B. Eritroplasia severa sangrante.
B. Lóbulo caudado del hígado C. Cáncer de endometrio.
C. Primera porción del duodeno D. Pólipo cervical sangrante.
D. Conducto hepático común E. Mioma cervical abortado.
E. Arteria aorta abdominal
54. Una Pancreatitis aguda es de mal pronóstico si se presenta:
46. El Mieloma múltiple se asocia por lo común a todo lo siguiente,
EXCEPTO: A. Bilirrubinas totales de más de 5 mg.
B. Hipocalcemia.
A. Dolor óseo. C. Dolor en faja.
B. Lesiones óseas osteoblásticas. D. Ninguna de las anteriores.
C. Proteinuria de Bance Jones. E. Todas las anteriores.
D. Ocurrencia después de la edad de 40 años en la mayoría de los
casos. 55. Los efectos colaterales indeseables de las Fenotiazinas incluyen
E. Células de mieloma en la médula ósea. a todos los siguientes, EXCEPTO:

47. Clínicamente, el marcador MÁS sensible del incremento de la A. Pérdida sensorial.


producción de andrógenos es: B. Síntomas parecidos al parkinsonismo.
C. Movimientos distónicos.
A. El incremento en la grasa corporal. D. Episodios hipotensivos.
B. El incremento de la libido. E. Convulsiones.
C. El hirsutismo.
D. La irregularidad menstrual. 56. Qué recomendaría Ud. para un pacientes de 18 años de edad,
E. La clitoromegalia. que viene a la consulta por dolor abdominal difuso de 12 meses
de evolución que mejora con la defecación, alteración del rito C. Ambos procesos son mediados por linfocitos.
intestinal (periodos de diarrea y estreñimiento), Al examen: T° D. El fenómeno de hipersensibilidad a la tuberculina puede durar
37.2°C. Hay dolorabilidad difusa del abdomen, no hay rebote, años.
ruidos intestinales normales, Lab: 9.200 leucocitos sin E. Todo lo anterior es verdadero.
desviación izquierda y E. orina: Negativo.
64. Todo lo siguientes es verdadero en los cuadros de
A. Laparotomía exploradora. Faringoamigdalitis, EXCEPTO:
B. Laparoscopia diagnostica.
C. Antiespasmódicos + antibióticos y alta. A. Los virus constituyen el 70% de la patología del tracto
D. Enema evacuante, repetirlo si es necesario. respiratorio superior.
E. Explicar al paciente la benignidad del proceso. B. La faringitis bacteriana se presenta con mayor frecuencia en el
niño menor de 2 años.
57. La Meningitis supurada es de MAL pronóstico: C. La linfadenopatia cervical anterior no distingue los procesos
virales de los bacterianos.
A. Cuando se inicia con convulsiones. D. La presencia de un moteado petequial en el paladar blando
B. Cuando se presenta en el lactante. asociado a exudado difuso en las amígdalas puede sugerirnos
C. Cuando presenta fiebre alta. infección bacteriana.
D. Cuando se retrasa el inicio del tratamiento. E. Ninguno de los anteriores.
E. Ninguna de las anteriores.
65. El MEJOR tratamiento de la Fisura anal es:
58. La operación de elección en el Megacolon congénito es:
A. Extirpación de la fisura y dilatación manual del esfínter.
A. Colostomía parcial. B. Esfinterotomia y extirpación de la fisura.
B. Enterostomía. C. Tratamiento médico únicamente a base de estreptomicina e
hidroterapia local.
C. Colostomía total. D. Colectomía.
D. Gastrostomía. E. Ninguno de los anteriores.
E. Ninguna de las anteriores.
66. La pelvis femenina que tiene el menor diámetro antero posterior
59. Con respecto a la Pielonefritis aguda que complica el embarazo en estrecho inferior:
y el puerperio, marque lo INCORRECTO:
A. Ginecoide.
A. Afecta aproximadamente a 2% de las pacientes. B. Antropoide.
B. Se asocia a bacteriuria asintomatica. C. Androide.
C. Los síntomas incluyen anorexia, náuseas y vómitos. D. Platipeloide.
D. El microorganismo causal predominante es la E.coli. E. Ninguna de las anteriores.
E. Hay una deficiencia concomitante de la autoinmunidad.
67. Son eslabones de la cadena epidemiológica, marque las
60. La vigilancia epidemiológica de las infecciones intrahospitalarias alternativas CORRECTAS:
por factores de riesgo presenta la siguiente carac- terística:
1. Fuente.
A. No focalizada. 2. Mecanismo de transmisión.
B. Esporádica. 3. Huésped.
C. Pasiva. 4. Estado inmunológico.
D. Especializada. 5. Contagiosidad. A. 1,3,4.
E. Selectiva. B. 1,2,4.
C. 2,3,5.
61. Qué actitud debe usted tomar ante una paciente de 22 años con D. 1,3,5.
síntomas de shock, dolor pélvico, amenorrea de 2 meses y E. 1,2,3.
hemorragia vaginal:
68. El tratamiento de la Hemoptisis masiva traumática es:
A. Control del pulso.
B. Medir la tensión arterial. A. Instalación de la presión venosa central.
C. Instaurar una vía venosa. B. Aspiración traqueo – bronquial.
D. Reposición de líquidos. C. Transfusión sanguínea.
E. Todas las anteriores. D. Administración de antibióticos.
E. Exploración quirúrgica.
62. La amenorrea en la Tuberculosis genital se debe a:
69. Entre los síntomas histriónicos que mencionamos a continuación
A. Disfunción ovárica. hay uno que no corresponde a la forma clínica con- versiva de
B. Desnutrición. la enfermedad, señale cual es:
C. Inhibición hipofisaria.
D. Endometritis caseosa. A. Movimientos anormales atípicos.
E. Anemia crónica. B. Parálisis sin base orgánica demostrable y con patrón
neurológico atípico.
63. Con respecto a la TBC señale lo CORRECTO: C. Amnesia psicógena.
D. Amaurosis o ceguera sin base orgánica y con origen psíquico.
A. La respuesta a la tuberculina es un proceso independiente de la E. Anestesia de un miembro con patrón neurológico atípico y sin
capacidad de defensa. base orgánica demostrable.
B. La respuesta a la tuberculina y la capacidad de defensa se dan
separadamente pero ocurren casi simultáneamente.
70. Si en un banquete de 100 personas, 20 contraen una
toxiinfección alimentaria, ¿Cuál es la probabilidad de la toxiin- E. El receptor Nicotínico/Aumento de los niveles de AMPc
fección en ese banquete?
78. Con respecto a la vacuna triple (DPT), señale lo falso:
A. 20/100
B. 80/100 A. Debe conservarse en refrigeración entre 4 y 8° C
C. 100/20 B. Se aplica por vía IM en una dosis de 0,5 mL
D. 80/20 C. Se puede administrar desde los 3 meses hasta los 72 meses de
E. 100/80 edad
71. El Síndrome de Meiggs está caracterizado por: D. La duración de la inmunidad conferida es de 10 años
E. Puede causar reacción febril en las primeras 24 – 48 horas
A. Tumor sólido benigno de ovario.
B. Hidrotórax. 79. ¿Cuál de las siguientes enfermedades puede erradicarse sólo
C. Ascitis. por vacunación?
D. Todos.
E. Ninguna de las anteriores. A. Difteria
B. Coqueluche
72. El beri-beri tipo húmedo cursa con: C. TBC
D. Poliomielitis
A. Rinorrea. E. Sarampión
B. Polaquiuria.
C. Lesiones dérmicas con exudado. 80. Estructuras que forman el plexo de Kiesselbach, EXCEPTO:
D. Incremento de la transpiración.
E. Edema. A. Esfenopalatina.
B. Palatina mayor.
73. El volumen de expulsión del ventrículo izquierdo sano es C. Etmoidal anterior.
afectado por los siguientes factores MENOS uno: D. Etmoidal posterior.
E. Palatina menor.
A. Precarga.
B. Contractilidad. 81. Respecto a la fisiología del oído interno, señale lo
C. Postcarga. INCORRECTO:
D. Ley de Frank Starling.
E. Viscosidad sanguínea. A. El órgano de Corti se dispone sobre la membrana basilar.
B. La endolinfa tiene una composición iónica similar al medio
74. El diagnóstico de Artritis reumatoide juvenil se hace intracelular.
fundamentalmente por: C. Las rampas vestibular y timpánica no presentan comunicación
libre ni entre sí ni con el oído medio.
A. Presencia de factor reumatoide. D. La mayoría de las fibras nerviosas del nervio coclear conecta
B. Electroforesis de proteínas. con las células ciliadas internas.
C. Por descarte clínico. E. La perilinfa tiene una composición iónica similar al medio
D. Por transaminasas altas. extracelular.
E. ANA, ANCA y Anti-Sm.
82. Las pruebas no paramétricas:
75. El exceso de cuál de los siguientes vitaminas causaría
hiperostosis frontales: A. Únicamente se utilizan para comparar distribuciones de
variables continuas.
A. Vitamina A. B. Requieren la comprobación del requisito de normalidad.
B. Complejo de vitamina B. C. Originan unos valores de error alfa similares a los que se
C. Vitamina C. calculan mediante pruebas paramétricas.
D. Vitamina D. D. Deben utilizarse siempre que manejemos muestras de gran
E. Ácido Fólico. tamaño.
E. No realizan asunciones sobre el tipo de distribución de la
76. La desviación de la atención hacia otros puntos del variable
pensamiento, se denomina…. Y la afectividad inadecuada
respecto al contexto en que se produce, se denomina………: 83. Uno de los siguientes NO es un test no paramétrico:

A. Hiperprosexia/Aprosodia A. U de Mann-Whitney
B. Hipoprosexia/Paratimia B. Test de Kruskal-Wallis
C. Paraprosexia/Alexitimia C. Test de Wilcoxon
D. Paratimia/Afasia D. ANOVA
E. Disprosexia/Distimia E. Coeficiente de correlación de Spearman

77. Las benzodiacepinas alostéricamente refuerzan las acciones de


…. basándose su modo fisiológico en….. 84. Ante una amenorrea secundaria, una vez descartada la
gestación, si la respuesta a la progesterona es positiva, con
A. El receptor dopaminérgico/Aumento de la velocidad de niveles de prolactina normal, el diagnóstico más probable será:
conducción nerviosa
B. El receptor acetilcolinérgico/Disminución de la velocidad de A. Hipoplasia uterina
conducción nerviosa
C. El receptor GABAminérgico/Apertura de los canales de cloro B. Menopausia precoz
D. El receptor Muscarínico/Disminución de los niveles de AMPc C. Fallo de ovulación
D. Síndrome de ovario resistente
E. Amenorrea de origen hipofisiario 92. Con relación a la definición de proyectos y programas, señale la
correcta:
85. Cuál de las siguientes afirmaciones es falsa:
( ) Un proyecto tiene duración indefinida
A. En una paciente perimenopáusica y deseos de descendencia ( ) Un programa o proyecto responde a un problema que se intenta
cumplidos, que no responde al tratamiento con gestágenos es solucionar
candidata a una ablación endometrial o histerectomía ( ) Un proyecto debe señalar solo objetivos y no precisar actividades
B. Durante los 18 meses posteriores a la menarquia son frecuentas ni acciones ( ) Un programa es un conjunto de proyectos
los ciclos anovulatorios, a intérvalos irregulares. A. FVFV
C. Debido a lo comentado en el apartado anterior, son frecuentes B. FFFV
el flujo menstrual irregular entre las adolescentes jóvenes C. VVVF
D. Cuando el sangrado menstrual excesivo, comienza con la D. VVVV
menarquia y como consecuencia del sangrado produce una E. FFFF
anemia severa, hay que descartar un trastorno de la coagulación
(deficiencia del factor de Von Willebrand) 93. Cuál es la finalidad de la administración entendida como una
E. En la valoración de la HUD es imprescindible la realización de ciencia:
un estudio hormonal.
A. Explicar el comportamiento y la conducción de la organización
86. El agente causal del impétigo ampollar en lactante y niños B. Uso de técnicas para la conducción de la organización
pequeños es: C. Comprender el comportamiento del personal
D. Explicar el porqué del éxito de la organización
A. Aeromona hydrophila E. Explicar el funcionamiento y evolución de la organización
B. Estafilococo aureus coagulasa positivo
C. Estreptococo piógeno 94. La secuencia lógica del proceso administrativo es:
D. Estreptococo pneumoniae
E. Legionella A. Organización-dirección-control-planificación
B. Dirección-planificación-organización-control
87. ¿Cuál de los siguientes cuadros NO está producido por el C. Planificación-organización-dirección –control
Streptococcus pyogenes? D. Planificación-dirección-organización-control
E. Organización-planificación-dirección-control
A. Ectima
B. Erisipela 95. Respecto a trastornos de la refracción, señale la respuesta
C. Impétigo ampolloso INCORRECTA:
D. Fasceitis necrotizante
E. Celulitis A. La presbicia se corrige con lentes positivas.
B. Las ametropías esféricas se corrigen con lentes cónicas.
88. ¿En cuál de los siguientes procesos NO se encuentra implicado C. En las lentes negativas, el movimiento de la imagen acompaña
el Staphylococcus aureus? al movimiento de cristal.
D. En la hipermetropía la imagen de la esquiascopÍa acompaña el
A. Síndrome de la piel escaldada movimiento del retinoscopio.
B. Síndrome del shock tóxico E. El tratamiento del astigmatismo en la actualidad es la cirugía
C. Escarlatina láser (LASIK, etc.).
D. Impétigo
E. Eritrasma
96. El reflejo de acomodación consta de los siguientes
89. En la sala de partos se advierte que un RN presenta componentes:
movimientos respiratorios, pero cuando cierra la boca no entra
aire en los pulmones. El diagnóstico más probable es: A. Convergencia ocular, contracción pupilar y acomodación.
A. Narcosis B. Convergencia ocular, dilatación pupilar y aumento de la
B. Hernia diafragmática concavidad de la superficie retiniana
C. Atresia de coanas C. Contracción pupilar, aumento del diámetro AP del cristalino y
D. Hipoplasia pulmonar aplanamiento retiniano.
E. Cardiopatía congénita D. Miosis, enoftalmos y reducción de la presión de la cámara
90. La hemorragia intracraneal neonatal es más frecuentemente anterior.
encontrada en los siguientes casos, EXCEPTO: E. Contracción pupilar y contracción del musculo ciliar.

A. Prematuros 97. La vena porta generalmente se forma por la confluencia de las


B. Neonatos nacidos por cesárea venas:
C. Neonatos con presentación podálica
D. Neonatos nacidos con ayuda mecánica, excepto el fórceps bajo A. Mesentérica superior y esplénica
E. Neonatos que han sufrido asfixia B. Mesentérica superior y mesentérica inferior
C. Mesentérica superior y coronaria estomáquica
91. ¿Cuál es el principal tratamiento psicofarmacológico del D. Mesentérica inferior y esplénica
trastorno obsesivo compulsivo? E. Esplénica y coronaria estomáquica

A. Antipsicóticos 98. ¿Cómo se llama al ligamento al cuál se tiene que fijar la malla
B. Benzodiacepinas cuando se quiere reparar una hernia crural o al cual se fija el
C. Antidepresivos inhibidores de la recaptación de noradrenalina tendón conjunto en la técnica de Mc Vay?
D. Antidepresivos inhibidores de la recaptación de serotonina
E. Estimulantes A. Inguinal o arco crural
B. Lagunar o de Gimbernat A. Hijo de madre drogadicta.
C. Iliopúbico B. Enfermedad de Minamata.
D. De Hesselbach C. Enfermedad fetal por incompatibilidad Rh.
E. De Cooper D. Hijo de madre diabética.
E. Fetopatía infecciosa.
99. El tendón conjunto está formado por las porciones
aponeuróticas de los músculos: 3. ¿Cuál es la causa más frecuente de abdomen agudo en el
lactante de más de 3 meses?
A. Oblicuo menor y transverso A. Bridas.
B. Oblicuos mayor y menor B. Apendicitis aguda.
C. Oblicuos mayor y vaina del recto C. Enterocolitis necrotizante.
D. Psoasilíaco y pectíneo D. Invaginación intestinal.
E. Oblicuo mayor, oblicuomenor y transverso E. Peritonitis primaria.

100. El ectropión puede ocasionar: 4. ¿Cuál es el indicador bioquímico más utilizado para detectar un
tumor carcinoide?
A. Blefaritis A. Ácido 5-hidroxiindolacético.
B. Triquiasis B. Histamina.
C. Blefaroespasmo C. Serotonina.
D. Distiquiasis D. Bradicinina.
E. Queratopía por exposición E. Ácido orótico.

5. Único fármaco disponible para erradicar la fase hepática de los


plasmodios:
A. cloroquina
B. primaquina
C. quinina
D. mefloquina
E. hidroxicloroquina.

6. Para una paciente primípara con dilatación de 2 cm durante 6


horas escoja una de las siguientes fases del trabajo de parto:
A. Fase latente prolongada.
B. Fase de desaceleración prolongada.
C. Fase activa prolongada.
D. Cese del trabajo del parto.
E. Trabajo del parto normal.

7. La relación entre la disponibilidad de los servicios y las


necesidades de la población se denomina:
A. Accesibilidad.
B. Efectividad.
C. Continuidad.
D. Adecuación.
E. Capacitación.

8. La arteria pélvica de la cual nace la arteria uterina es la:


A. Aorta
B. Hipogástrica
C. Iliaca externa
D. Ovárica
E. Iliaca interna

9. En el síndrome de Turner, una de las siguientes es verdadera:


A. Suele existir retraso menstrual importante
EXAMEN N°15 - SIMULACRO 9B B. El riesgo de sufrir este sindrome en el siguiente hijo estaría
aumentado
1. Cuando en el transcurso de una pancreatitis crónica, el dolor C. La fertilidad de estas niñas esta conservada
abdominal es continuo, sugiere: D. La cardiopatía más asociada es el ductus arterioso persistente
E. Puede existir linfedema de manos y pies
A. Existe una pancreatitis crónica muy avanzada.
B. Se ha complicado con alteraciones gastroduodenales. 10. Un neonato presenta ictericia, dificultad respiratoria, Tº
C. Englobamiento de filetes nerviosos por la fibrosis pancreática. inestable, distensión abdominal y hepatomegalia. Diagnosti- co
D. Está en fase de insuficiencia pancreática. de presunción:
E. Se ha complicado con esteatosis hepatica. A. Kernicterus.
B. Meningitis neonatal.
2. Si un neonato presenta un aspecto macrosómico, con facies C. Enfermedad granulomatosa crónica.
cushingoide, hipotónico y con hepatoesplenomegalia, D. Lupus neonatorum.
cardiomegalia, distress respiratorio, trombosis de vena renal, E. Sepsis neonatal.
hipoglucemia, hipocalcemia, hiperbilirrubinemia y aci- dosis,
sospecharemos que se trata de:
11. Un niño de 5 años de edad presenta síntomas generales tipo E. Transaminasa glutámica - oxaloacética sérica mayor de 250.
cefalea, anorexia, vómitos y hematuria desde hace días. Dolor
en el flanco a la palpación y edema en las piernas; aumento de 18. Una mujer embarazada de 10 semanas presenta hemorragia
creatinina sérica, disminuidos C3 y CH50 y en orina cilindros vaginal, hiperemesis, dolor abdominal, síntomas de hi-
hemáticos, leucocitos y proteinuria. Historia de infección pertiroidismo y un útero de mayor tamaño que lo que le
faríngea hace 14 dias. Diagnostico mas pro- bable: corresponde al tiempo de amenorrea. Diagnóstico clínico de
A. GNF por cambios mínimos. sospecha:
B. GNF con proliferación extracapilar. A. Amenaza de aborto.
C. GNF con proliferación endocapilar difusa. B. Aborto en curso en mujer con bocio multinodular tóxico.
D. Síndrome de Schönlein-Henoch. C. Mola hidatiforme.
E. Enfermedad de Berger. D. Coriocarcinoma.
E. Embarazo ectópico.
12. Varón de 70 años con antecedentes de demencia senil,
estreñimiento crónico y abuso de laxantes presenta náuseas y 19. Respecto a las hernias inguinales, marque lo correcto:
dolor abdominal. El abdomen está distendido y doloroso a la A. Las hernias indirectas protruyen por el anillo inguinal profundo
palpación, sobre todo en el flanco izquierdo, con per- cusión B. Las hernias directas suelen ser congénitas
timpánica y ruidos “metálicos”. Rx abdominal: gran distensión C. Las hernias indirectas protuyen por el triángulo inguinal
del colon, que se incurva produciendo una imagen en “grano de D. Las hernias directas ocurren por persistencia del conducto
café”. Diagnóstico más probable: peritoneo vaginal
A. Síndrome de Ogilvie. E. Las hernias indirectas son mediales a los vasos epigástricos
B. Diverticulitis aguda.
C. Vólvulo de ciego. 20. Pescador de Tumbes, aficionado a comer cangrejo de río,
D. Vólvulo de sigma. consulta por tos, hemoptisis y disminución de peso. Su ra-
E. Isquemia mesentérica. diografía de tórax muestra una cavidad en ápice derecho. Su
primera sospecha diagnóstica es:
13. La maniobra de Rojas se utiliza para: A. Quiste hidatídico
B. Histoplasmosis
A. Girar la cabeza a mento anterior en el parto de cara. C. Amebiosis
B. Extraer los hombros en el parto de nalgas. D. Paragonimiosis
C. Extraer la cabeza en el parto de nalgas. E. Aspergillosis .
D. En el parto en transversa.
E. En el parto de hombro. 21. Si pretendemos evaluar las consecuencias sanitarias en
términos monetarios de la aplicación de un programa de pre-
14. ¿Cuándo estaría indicado el tratamiento médico de la vención, el análisis indicado consistirá en una relación entre el:
toxoplasmosis en una gestante? A. Costo y beneficio del programa.
A. En todos los casos de anticuerpos negativos a título preventivo. B. Costo y eficacia de la intervención.
B. En los casos de títulos de anticuerpos IgG positivos. C. Costo y efectividad de la intervención.
C. En los casos de títulos elevados de IgG. D. Costo y eficiencia de la intervención.
D. En los casos de títulos de IgG altamente positivos o ascendentes E. Costo y rendimiento del programa.
con IgM positiva.
E. En toda gestante independientemente de los títulos de 22. Paciente que presenta: esclerodactilia, telagiectasia, calcinosis,
anticuerpo. Raynaud, y alteración del esófago,
A. Esclerodermia
15. Paciente presenta polidipsia y poliuria. En una prueba de B. Dermatomiositis
deshidratación la osmolaridad urinaria se halló por debajo de la C. CREST
plasmática, y con la posterior inyección de 5 U de vasopresina D. HELLP
acuosa, no se observa aumento de la osmolaridad urinaria. Tras E. Overlap síndrome.
ello se concluye la existencia de:
23. De primera elección en el control del dolor de la osteoartritis es:
A. Función hipofisaria normal. A. Etoricoxib
B. Diabetes insípida neurogénica total. B. Paracetamol
C. Diabetes insípida neurogénica parcial. C. Codeína más paracetamol
D. Diabetes insípida nefrogénica. D. Oxicodona
E. Secreción inadecuada de ADH. E. Infiltraciones de lidocaína y corticoides

16. Niño de 3 años diagnosticado de meningitis supurada a germen 24. Síndrome miasténico de Lambert Eaton es para neoplásico
desconocido. El tratamiento antibiótico empírico inicial lo frecuente de:
haríamos con: A. Cáncer de células grandes
B. Carcinoma epidermoide de pulmón
A. Ampicilina más Aminoglucósido. C. Adenocarcinoma de pulmón
B. Penicilina G sódica. D. Cáncer de células pequeñas
C. Cefaclor. E. Carcinoma adenoescamoso
D. Aminoglucósidos.
E. Ceftriaxona. 25. El tratamiento endoscópico de la Hemorragia Digestiva Alta está
indicado en
17. Los signos de Ranson de la pancreatitis aguda incluyen todos A. Forrest IA, IB y IIA
los siguientes, EXCEPTO: B. Forrest I y II
A. Edad mayor de 55. C. Sangrado masivo
B. Bilirrubina mayor de 2.0. D. Hematemesis incoercible
C. Recuento leucocitario mayor de 16,000. E. Forrest IA y B
D. Glucosa mayor de 200.
26. El motivo por el cual la sulfonilureas deben ser usadas con E. Anemia ferropénica.
cuidado es:
A. Pueden causar acidosis láctica 34. En un niño afectado de celiaquia está prohibido darle:
B. Hiperkalemia A. Arroz
C. Agitación B. Zanahoria
D. Hipoglicemia C. Pan
E. Daño hepático. D. Papa
E. Maíz
27. Señale el movimiento alterado tras sufrir una fractura-luxación
de Monteggia: 35. Tratamiento de Leishmaniasis:
A. Supinación del antebrazo.
B. Flexoextensión de la muñeca. A. Estibogluconato de sodio
C. Aproximación de los dedos. B. Artesunato
D. Separación de los dedos. C. Nifurtimox
E. Separación del miembro superior. D. Albendazol
E. Praziquantel
28. Paciente varón de 50 años, fumador, con Hipertensión arterial
controlada, programado para cura quirúrgica de her- nia 36. El tratamiento más adecuado para una mujer gestante de 30
inguinal. Su riesgo ASA es: semanas con membranas rotas y una corioamnionitis, sería:

A. E A. Inducción del parto.


B. II B. Tetraciclina en infusión lenta i.v.
C. I C. Administrar prednisona para la maduración pulmonar fetal y
D. IV parto vaginal en 24 horas.
E. III D. Cesárea solo si el índice lecitina / esfingomielina es igual o
mayor a 2.
29. Politraumatizado de 25 años de edad, hemodinámicamente E. Cesárea inmediata.
estable, en la Rx de tórax se evidencia neumotórax de
aproximadamente 40% en el hemitórax derecho. Cuál es la 37. ¿Cómo se denomina al concepto “objetivos alcanzados en
conducta a seguir: relación con los recursos utilizados”?
A. Toracotomía A. Costo – efectividad.
B. TAC torácica B. Eficiencia.
C. Observación C. Costo – utilidad.
D. Colocación de tubo de drenaje D. Cobertura.
E. Ecofast torácico E. Efectividad.

30. Llega a consulta un lactante que presenta deposiciones liquida 38. La paranoia se caracteriza por presentar una de las siguientes
en frecuencia de 7 por dia sin sangre sin moco asociada a manifestaciones:
vómitos esporádicos quien llega a tener signos de A. Ideas delirantes de identidad.
deshidratación leve a moderada. Si usted iniciara rehidratación, B. Ideas delirantes de carácter interpretativo.
¿Cuál es el volumen en ml/Kg/4h aproximado a reponer? C. Alucinaciones auditivas.
D. Autismo.
A. 20-50 E. Por toda la sintomatología citada.
B. 30-40
C. 30-80 39. Mujer de 52 años con diagnóstico de úlcera pilórica, presenta
D. 10-90 vómitos a repetición de una semana de evolución. In- gresa a
E. 100. emergencia deshidratada, PA 100/58 mmHg. Laboratorio: Na:
140 mmol/L, K: 2.8 mmol/L, Cl: 86 mmol/L, Bicarbonato: 42
31. ¿Cuál de los siguientes antibióticos es el que puede generar un mmol/L, pH: 7.53, pCO2: 53 mmHg y creatinina: 2,9 mg/dL. En
mayor número de interacciones medicamentosas? orina: Na: 2 mmol/L, Cl: 21 mmol/L.¿Cuál es el diagnóstico más
A. Ciprofloxacino. probable?
B. Ceftazidima.
C. Azitromicina. A. Acidosis hipoclorémica.
D. Rifampicina. B. Alcalosis metabólica.
E. Gentamicina. C. Alcalosis mixta.
D. Acidosis metabólica con Anión-Gap normal.
32. ¿Cuál es el tipo de cáncer de pulmón más frecuente? E. Bicarbonaturia paradójica.
A. Carcinoma epidermoide.
B. Carcinoma de células pequeñas. 40. ¿En qué personas se puede considerar positiva una reacción de
C. Carcinoma de células grandes. 6 mm a la prueba de tuberculina, que sugiere posible- mente una
D. Adenocarcinoma. TBC?
E. Linfoma. A. Niño de 3 años vacunado que recibió al nacer la vacuna
BCG
33. Un varón de 43 años presenta astenia y palidez mucocutánea. B. Niño de 8 años con una reacción de 0 mm el año anterior
En la analítica se muestra sideremia de 40 g/dl, transfe-rrina de C. Niño de 2 años con un padre asintomático que trabaja como
450 μg/dl, y ferritina de 15 ng/ml. Lo más probable es que el funcionario de prisiones.
enfermo padezca: D. Niño de 3 años, cuya madre muestra bacilos ácido-resistentes
A. Anemia sideroblástica. en la tinción del esputo
B. Beta-talasemia minor. E. Niño de 10 años nacido en el Cerro San Cosme
C. Beta-talasemia mayor.
D. Anemia secundaria a trastornos crónicos
41. Un niño de 9 años presenta un dolor inguinal derecho, cojera y B. Enfermedad de Hirschsprung.
limitación de la movilidad de esa pierna. En la explo- ración se C. Hipotiroidismo.
halla una contractura en flexión y limitación en abducción y D. Tumor de la médula espinal.
rotación interna. Sufrió un proceso faríngeo hace pocos días. E. Botulismo.
Temperatura, recuento leucocitario y VSG normales. Las placas
no muestran hallazgos patológicos. Diagnóstico más probable: 48. La deficiencia por vitamina B12 es propensa que ocurra en un
niño con:
A. Artritis séptica de cadera. A. Reseccion del yeyuno.
B. Epifisiólisis femoral proximal. B. Resección del ileon.
C. Enfermedad de Perthes. C. Resección del colon.
D. Sinovitis transitoria de cadera. D. Una colostomia.
E. Contractura de aductores tras algún esfuerzo. E. Una gastroyeyunostomia.

42. Un neonato presenta un pulso de 130 lpm, cianosis de manos y


pies, llanto activo, movilidad espontánea, tos al poner una sonda 49. Un administrador de supermercado de 64 años tuvo una
en sus fosas nasales. Su puntuación de Apgar es: operación por elección debida a una úlcera duodenal. No ha
regresado a trabajar porque tiene diarrea con más de 20
A. 7. evacuaciones intestinales por día. La medicación no sido
B. 9. efectiva. Los detalles exactos de su operación no se pueden
C. 8. determinar. ¿Cuál es la operación más probable que se le
D. 10. realizó?
E. 4.
A. Antrectomía y anastomosis de Billroth I.
43. De los siguientes signos peritoneales, ¿cuál indica que el B. Cirugía gástrica combinada con colecistectomía.
peritoneo está inflamado? C. Vagotomía troncular.
D. Vagotomía muy selectiva.
A. Signo de Murphy E. Vagotomía selectiva.
B. Signo de rebote
C. Signo del obturador 50. Una madre con tipo de sangre AB tiene un hijo con tipo AB. A
D. Signo del psoas o iliopsoas ella le gustaría establecer la paternidad a través del tipo
E. Signo de Rovsing sanguíneo. ¿Cuál tipo de sangre excluye a un varón de ser el
padre biológico?
44. Una mujer de 22 años de edad y su esposo reciben asesoría
después de un aborto en el primer trimestre. Ella no tiene A. AA.
problemas médicos significativos. La exploración física no ofrece B. BB.
datos de importancia. Al orientarla, el médico le explica en C. BO.
términos simples que la causa más probable de su aborto es D. AO.
aneuploidia. ¿Cuál de las siguientes opciones es la aneuploida E. OO.
más frecuente causal de aborto?
A. Trisomía 18. 51. La frecuencia cardiaca normal habitual de un recién nacido en
B. 45, X0. reposo es:
C. Triploidia.
D. Translocación desequilibrada. A. 120-160 lat/min.
E. Trisomía 21. B. 140-180 lat/min.
C. 100-130 lat/min
45. ¿Cuál de las siguientes medidas informa de una manera más D. 90-120 lat/min.
completa sobre el grado de utilización del recurso-cama de un E. 100- 110 lat/min.
hospital?
52. El grado de afectación fetal en la isoinmunización Rh se aprecia
A. Índice de rotación enfermo-cama. con mayor exactitud por medio de:
B. Tasa de frecuentación.
C. Estancia media. A. Test de Coombs indirecto.
D. Índice de ocupación. B. Test de Coombs directo.
E. Tasa de atracción. C. Niveles de b-HCG en líquido amniótico.
D. Medición de los niveles de bilirrubina en los líquidos amniótico.
46. ¿Cuál de los siguientes puede causar anemia hipocrómica E. Test de Coombs y b-HCG.
microcítica?
53. El criterio que distingue si las compras o pagos empleados en la
A. Deficiencia de folatos. actividad sanitaria corresponden a la organización y
B. Deficiencia de vitamina B12. funcionamiento del sistema de salud o si son ajenos a este y
C. Talasemia. fundamentalmente repercuten en el usuario, permite diferenciar
D. Deficiencia de vitamina B6. entre:
E. Pérdida aguda de sangre.
A. Costos tangibles e intangibles.
47. Niño de 6 meses, antes sano, hijo de apicultor, desarrolla B. Costos directos e indirectos.
gradual letargo, mala alimentación, estreñimiento y debi- lidad C. Costos sanitarios y no sanitarios.
generalizada. En fechas recientes está consumiendo una D. Costos fijos y variables.
fórmula casera con leche evaporada, agua y miel. La explicación E. Costos presupuestarios y financieros.
más probable para el cuadro es:

A. Intoxicación por sodio.


54. La secreción de leche por el pezón (galactorrea) es normal etapa 1 de Tanner de desarrollo mamario y vello púbico. PA
durante el puerperio. Si una mujer que no está embarazada 142/94. Exploración pélvica normal para una mujer prepúber.
presenta galactorrea, se debe pensar con mayor razón en: Diagnostico mas probable:

A. Hipotiroidismo A. Deficiencia de 17 alfa – hidroxilasa.


B. Insuficiencia renal B. Síndrome de feminización testicular.
C. Prolactinoma C. Deficiencia de la 21-hidroxilasa.
D. Cirrosis D. Hermafroditismo verdadero.
E. Hipogonadismo primario E. Agenesia de los conductos de Müeller.

55. En niños y adolescentes las lesiones maculopapulares ovales 62. En relación con la etiología del RCIU II, señale factor más
orientadas con el eje largo a lo largo de las líneas de ten- sión frecuente
cutánea sugieren:
A. Enfermedad hipertensiva
A. Miliaria rubra. B. Rubéola
B. Verruga vulgar. C. Alcoholismo
C. Condiloma acuminado. D. Drogadicción
D. Molusco contagioso. E. Tabaquismo
E. Pitiriasis rosada.
63. En relación a la fisiopatología de la ICC señale lo verdadero
56. Varón de 65 años, sufre 4 días con dolor en la porción inferior
del abdomen, cada vez más intenso y con estreñimien- to. Tº A. La precarga se reduce con espironolactina
38.5 ºC con dolor a la palpación en la porción inferior del B. La poscarga se reduce con furosemida
abdomen que es más intenso en la línea media y en el cuadrante C. La angiotensina II produce remodelado cardiaco y se inhibe con
inferior izquierdo con masa palpable. Leucocitosis moderada y lisinopril
en la Rx del abdomen se observa íleo. Diagnóstico más D. La aldosterona aumentada produce hiperkalemia
probable: E. La ADH aumentada se controla con somatostatina

A. Apendicitis aguda. 64. Un niño de 10 años de edad se queja de malestar abdominal


B. Diverticulitis de sigmoides. episódico. Su madre indica que estas crisis se acompañan de
C. Pancreatitis aguda. periodos de mirada fija, seguidos por un periodo breve de
D. Colecistitis aguda. letargo. ¿Cuál de los que siguen es la causa más probable de
E. Vólvulo de ciego. los síntomas?
A. Convulsiones psicomotoras.
57. Las fracturas del 1/3 superior del cúbito se suelen asociar a: B. Migraña.
C. Epilepsia de pequeño mal.
A. Luxación radio-cubital inferior. D. Reacción de conversión.
B. Luxación de la cabeza del radio. E. Ninguno de los trastornos señalados.
C. Fractura de la cabeza del radio
D. Arrancamiento de la apófisis coronoides del cúbito. 65. Una niña de 9 años de edad tiene una historia de sibilancias
E. Fractura de la metáfisis inferior del radio. intermitentes de varios años; no ha recibido medicamen- tos
hace algún tiempo. Se halla febril, agitada y con cianosis
58. El término menopausia significa la desaparición definitiva de la peribucal. Tirajes intercostales y supraesternales; los ruidos
menstruación. Respecto a esta entidad, señale lo FALSO: respiratorios están disminuidos y hay sibilancias bilaterales. La
intervención inicial más apropiada es:
A. Hay aumento en la producción de progesterona.
B. Tiene lugar entre los 45 y los 55 años. A. Administración de aminofilina intravenosa.
C. La estrona es el estrógeno más importante. B. Solicitar radiografía torácica.
D. Hay atrofia genital generalizada. C. Prescribir la nebulización de cromoglicato sódico.
E. Hay disminución de la secreción de la hormona paratiroidea. D. Pedir biometría hemática completa y hemocultivo.
E. Prescribir salbutamol nebulizado.
59. Según un estudio, la mediana de sobrevida de pacientes
sometidos a una intervención quirúrgica es de 5 años. Ello quiere 66. Si en una muestra de tamaño 10 se eliminan el valor máximo y
decir que: el mínimo. ¿Qué ocurre con la media y la mediana?
A. El valor esperado del tiempo de supervivencia es de 5 años. A. La media es la misma y la mediana puede cambiar.
B. La mitad de los pacientes sobreviven más de 5 años. B. Ninguna cambia.
C. No hay ningún paciente que sobreviva menos de 5 años. C. La media puede cambiar pero no la mediana.
D. La mitad de los pacientes sobreviven aproximadamente 5 años. D. Las dos pueden cambiar.
E. 5 años es el tiempo de supervivencia más probable. E. Si la media cambia también cambia la mediana.

60. ¿Por qué mecanismo principal el sindrome del distrés 67. Varón de 25 años; en la calle grita y gesticula solo. Nos presenta
respiratorio agudo provoca hipoxia hipoxémica? a su amigo con el que pasa todo el día, y señala un sillón vacío.
Decimos que no hay nadie y responde enfadado que no le
A. Por alteración del metabolismo celular. hagamos bromas. Tipo de alteración de su per- cepción:
B. Por alteración de la curva de saturación de la oxihemoglobina.
C. Por reducción del gasto cardíaco. A. Ilusión.
D. Por cortocircuito. B. Pseudoalucinación.
E. Por hipoventilación pulmonar. C. Alucinosis.
D. Alucinación.
61. Mujer de 18 años refiere que nunca menstruó; no hubo E. Heautoscopia.
desarrollo mamario ni crecimiento de vello púbico. Se halla en la
68. La denominación sinclitismo y asinclitismo se refiere a la relación B. El microlegrado
entre: C. La citología endocavitaria
D. La histeroscopia
A. La sutura interparietal y las tuberosidades izquierdas. E. El legrado fraccionado
B. La sutura metódica y el promontorio.
C. La sutura occipital y el promontorio. 76. Se realiza un estudio para valorar la eficacia de la vacuna
D. La sutura sagital con el promontorio y el pubis. antineumocócica. Se eligen 10000 ancianos. Aleatoriamente un
E. La sutura sagital y las espinas ciáticas. grupo recibió la vacuna y otro un placebo y se valoraron los
resultados. ¿Cómo se denomina este tipo de estudio?
69. ¿Cuál lesión mamaria tiene el mejor pronóstico?
A. Ensayo ecológico.
A. Carcinoma papilar infiltrativo B. Ensayo clínico sin cruzar.
B. Carcinoma ductal infiltrativo con fibrosis productiva C. Ensayo clínico cruzado.
C. Carcinoma coloide D. Ensayo de campo.
D. Carcinoma tubular E. Ensayo comunitario de intervención.
E. Carcinoma medular
77. La hemorragia uterina disfuncional se asocia principalmente a:
70. En una prueba diagnóstica, la especificidad es 85%. ¿Cuál de A. Deficiencia de cuerpo lúteo.
estas afirmaciones es correcta? B. Congestión miometrial.
C. Alteración de prostaglandinas.
A. El valor predictivo negativo de la prueba es del 15%. D. Alteraciónvascular.
B. El valor predictivo positivo de la prueba es del 15%. E. Ciclos anovulatorios.
C. La probabilidad de un resultado falso positivo es del 15%.
D. La probabilidad de un resultado falso negativo es del 15%. 78. La posible relación entre dos variables cualitativa se establece
E. La sensibilidad de la prueba es del 155. mediante

A. Chi cuadrado
71. Mujer de 34 años, nuligesta, con historia de infección por virus B. T student
del papiloma humano (VPH) hace 9 años y que se trató con C. ANOVA
crioterapia. Consulta por biopsia extraclínica tomada por pinza D. Kruskall Wallis
sacabocado que informa de displasia severa (CIN E. Poisson.
III) respecto al VPH. ¿Cuál es su actitud?
79. Paciente de 29 años que acude a emergencia por haber sido
A. Histerectomía total. víctima de violación sexual. Se encuentra con 16 semanas de
B. Histerectomía radical (Wertheim). gestación. El manejo preventivo contra ETS que se recomienda
C. Conización cervical. es:
D. Destrucción cervical (láser, crioterapia…).
E. Aplicación de 5-fluoracilo. A. Ceftriaxona + penicilina benzatínica + azitromicina
B. Ceftriaxona + penicilina benzatínica + ciprofloxacino
72. La hipoglucemia del recién nacido, hijo de madre diabética, se C. Ceftriaxona + penicilina benzatínica + doxiciclina
presenta en: D. Ciprofloxacino + penicilina benzatínica + azitromicina
E. Ciprofloxacino + penicilina benzatínica + doxiciclina
A. Las primeras 6 horas
B. Las primeras 24 horas 80. RN de parto vaginal con expulsivo de 90 minutos, con 4,2 kg de
C. La primera semana peso. Antecedente materno: 2 RN macrosómicos. Desde el 2do
D. El primer mes día de vida, tumoración parietal derecha de 8 x 10 cm., que no
E. El primer año atraviesa la línea media. ¿Cuál es el diag- nostico probable?

73. Una artritis infecciosa en que se ven cocos gram negativos en el A. Hemorragia subgaleal.
líquido articular obtenido tras punción, debe tratarse B. Caput succedaneum.
empíricamente con: C. Moldeamiento.
D. Cefalohematoma.
A. Cloxacilina E. Fractura craneal.
B. Vancomicina
C. Cloxacilina + Gentamicina 81. ¿Cuál es la maniobra de Leopold que determina el cálculo de la
D. Amoxicilina + Clavulánico altura uterina?
E. Ceftriaxona
A. Tercera
74. ¿Cuál afirmación acerca de la hipertensión es correcta? B. Primera
C. Segunda
A. La coartacion de la aorta es una causa rara de HT en la infancia. D. Cuarta
B. La HT esencial es la causa más común entre los adolescentes. E. Quinta.
C. La incidencia en los adolescentes blancos es dos veces que los
adolescentes negros. 82. Llega a consulta un niño de 4 años con lesiones en rostro en
D. La mayoria de pacientes pediatricos con HT son sintomaticos. imagen de bofetada que luego desarrolla una lesion ma-
E. Un adolescente con HT no debe participar en deportes. culopapular eritematosa que luego no descama. ¿cuál es el
diagnostico?
75. ¿Cuál es el procedimiento más seguro para el diagnóstico del
cáncer de endometrio? A. Eritema infeccioso
B. Roséola
A. La ecografía C. Rubeola
D. Varicela en un tumor determinado. El concepto de ganglio centinela en el
E. Escarlatina cáncer de mama es:
A. El primer ganglio linfático aumentado de tamaño en un
83. Sobre tratamiento de HTA señale lo correcto territorio que drena un tumor primario.
B. El primer ganglio linfático con metástasis que aparece en un
A. Paciente con HTA con DM2 es de elección carvedilol paciente con un carcinoma primario.
B. Paciente con HTA e IRC terminal con K > 5.5 indicamos enalapril C. El primer ganglio en un territorio linfático que recibe el flujo
C. Paciente con HTA y migraña indicamos tiazidas linfático de un tumor primario.
D. Paciente con HTA y angina de prinzmetal indicamos amlodipino D. Se refiere a todos los ganglios del territorio axilar en pacientes
E. Paciente con HTA y osteoporosis indicamos atenolol con carcinoma de mama.
E. Se refiere a todos los ganglios linfáticos más próximos en el flujo
84. ¿Cuál es el antibiótico de elección en pielonefritis aguda de una linfático de drenaje de un carcinoma primario.
gestante cuyo resultado de urocultivo mencionan E. coli BLEE
positivo? 90. Hombre, de 65 años de edad, con antecedentes de demencia
en varios familiares próximos, comienza en el curso de varios
A. Meropenem meses con movimientos anormales de las extremidades e
B. Ceftriaxona incapacidad de realizar sus tareas habituales. El EMG y el EEG
C. Amoxicilina son normales, pero la TAC define una característica atrofia de
D. Piperacilina mas tazobactam los núcleos caudados. Posible diagnostico:
E. Ampicilina más sulbactam
A. Enf. de Creutzfeldt-Jakob.
85. Varón de 80 años, acude a emergencia por presentar síncope B. Sd. de Devic
mientras subía la escalera. E. físico: soplo sistólico eyec- tivo C. Enf. de Parkinson.
III/VI. EKG: ritmo sinusal normal con signos de hipertrofia del D. Parálisis supranuclear progresiva.
ventrículo izquierdo. ¿Cuál es el diagnóstico más probable? E. Enf. de Huntington.

A. Estenosis aórtica. 91. Las alteraciones de crecimiento luego de una lesión epifisaria en
B. Insuficiencia aórtica. tratamiento ortopédico adecuado es mayor en el tipo de Salter:
C. Insuficiencia mitral.
D. Estenosis mitral. A. Tipo I.
E. Comunicación interventricular. B. Tipo II.
C. Tipo III.
86. Varón de 19 años que refiere aumento de volumen en el D. Tipo IV.
hemiescroto derecho transiluminacion positiva, deberíamos E. Tipo V.
descartar
92. Mujer de 23 años, presenta dolor en zona de genitales que le
A. Hernia inguinoescrotal. impide la deambulación. Examen: tumoración de 4 x 3 cm,
B. Hidrocele localizado en labio mayor derecho con signos de flogosis. ¿Cuál
C. Quiste del epidídimo. es el diagnóstico más frecuente?
D. tumor testicular tipo seminoma
E. Tumor testicular tipo no seminoma A. Quiste de glándula de Skene.
B. Absceso de Gartner.
87. Paciente con dolor de garganta además de fiebre, observamos C. Absceso de quiste de Bartholino.
exudado en la orofaringe mencione la complicación no D. Quiste de Naboth.
supurativa más frecuente E. Forúnculo infectado.

A. Sinusitis aguda 93. El dedo meñique montado sobre el anular y el índice montado
B. Fiebre reumática sobre el medio es un signo clínico que se observa espe-
C. Escarlatina cialmente en el síndrome de:
D. Absceso periamigdaliano
A. DiGeorge.
E. Absceso retrofaringeo B. Lejeune.
C. Edwards.
88. Paciente de 80años de edad, que presenta en ambas D. Patau.
extremidades ampollas grandes, tensas, algunas con contenido E. Down.
sanguíneo, sobre una base urticariana y con moderado prurito.
No hay compromiso del estado general y la histopato- logía 94. ¿Cuál de los siguientes datos conlleva mayor índice de
muestra lesiones subdermicas con acumulo de eosinofilos en la sospecha de neumonía por legionella?
dermis papilar. La inmunofluorescencia muestra depósito lineal
de IgG y de C3. ¿Cuál es la dermatopatia más probable? A. Leucopenia.
B. Hiponatremia.
A. Dermatitis herpetiforme C. Hipopotasemia.
B. Penfigo cicatricial D. Leucocituria.
C. Penfigo seborreico E. Leucocitosis.
D. Penfigo vulgar
E. Penfigoide ampollar 95. A pocas horas de nacer, el bebé de madre diabética tenia ritmo
respiratorio creciente y respiración dificultosa. El bebé se puso
89. En la decisión terapéutica de un paciente neoplásico es cianotico y murió. Examen histológico de la autopsia: alveolos
fundamental determinar el estadio tumoral. En los últimos años colapsados recubiertos de material eosinofilo.
se ha extendido la práctica de biopsia del denominado “ganglio ¿Cuál es el diagnostico?
centinela” para conocer la existencia de disemi- nación linfática
A. Enfisema congenito.
B. Síndrome de dificultad respiratoria
C. Fibrosis quistica
D. Fistula traqueoesofagica
E. Carcinoma pulmonar.

96. Un varón de 23 años presenta un nódulo rojizo, indoloro, más


visible en el lado conjuntival del párpado. Diagnóstico más
probable:

A. Chalazión.
B. Orzuelo externo.
C. Orzuelo interno.
D. Blefaritis simple.
E. Blefaritis ulcerosa.

97. Varón de 62 años, hipertenso, presenta dolor torácico


retroesternal y diaforesis. EKG: supradesnivel del segmento ST,
ondas T negativas de V1 a V4. ¿Cuál es el mejor marcador
cardiaco que confirma el diagnóstico de infarto miocardico
agudo?

A. Troponina I.
B. LDH.
C. Mioglobina.
D. CPK - MB.
E. Péptido natriurético atrial.

98. Un neonato de 3 días, nacido de parto domiciliario, es traído por


presentar dificultad respiratoria. Pesa 3000 gr, y mide 52 cm.
Examen clínico: adelgazado, escaso paniculo adiposo, piel
seca, cordón umbilical impregnado de meconio. Di- ficultad
respiratoria con Silverman Anderson de 7. ¿Cuál es el SIMULACRO 10B
diagnóstico más probable?

A. Neumonía aspirativa. 1. El triángulo de la auscultación está delimitado por el dorsal


B. Taquipnea transitoria. ancho, el............................ y el........................:
C. Neumonía connatal. A. Serrato mayor / Trapecio
D. Alcalosis respiratoria. B. Trapecio / Borde medial de la escápula
E. Enfermedad de membrana hialina. C. Romboides mayor / Romboide menor
D. Redondo mayor / Romboide mayor
99. ¿Cuál de las siguientes lesiones NO se considera una lesión E. Redondo menor / Trapecio
primaria de piel?
2. Paciente femenina de 52 años de edad que refiere dolor en el
A. Pápula. hombro izquierdo. Al examen físico se evidencia leve chasquido
B. Mácula. a la exploración activa del mismo, sin limitación de los arcos de
C. Cicatriz. movilidad. Se solicita exámenes y se evidencia engrosamiento
D. Roncha. en los tendones del manguito rotador izquierdo, diga usted: el
E. Pústula. principal músculo rotador del brazo hacia afuera sería el:
A. Infraespinoso
100. Acerca de los canales sinápticos en la placa terminal del B. Supraespinoso.
músculo esquelético es cierto: C. Redondo menor.
A. Son altamente selectivas para Na+ D. Redondo mayor.
B. Se abren cuando la membrana celular despolariza E. Ninguna de las anteriores.
C. Se activan por acetilcolina (ACh)
D. Se inhiben por atropina 3. El ligamento umbilical medial es vestigio embriológico de:
E. Son responsables del periodo refractario relativo
A. Arteria umbilical
B. Vena umbilical
C. Arterias umbilicales
D. Venas cardinales
E. Conducto venoso

4. ¿A qué edad del feto humano se encuentran formados la


mayoría de órganos internos?

A. 1 mes
B. 2 meses
C. 3 meses
D. 4 meses
E. 5 meses
B. Pólipo cervical sangrante
5. En el sistema respiratorio, respecto a las células de Clara todas C. Eritroplasia severa sangrante
las frases son verdaderas. EXCEPTO: D. Mioma cervical abortado
E. Amenaza de aborto
A. Se encuentra en el bronquio terminal
B. Son células que no se regeneran 13. ¿Cómo se encuentran los niveles de hormona
C. Presentan gránulos secretorios folículoestimulante (FSH) en una mujer con menopausia
D. Regulan el transporte de cloro fisiológica?:
E. Producen sustancias tensoactiva
A. Elevados, porque también lo están andrógenos ováricos. –
6. Respecto a la estructura de los miocitos todo lo siguiente es
cierto, excepto: B. Elevados, porque la retroacción negativa de las hormonas
esteroides ováricas no existe. –
A. La sarcómera está delimitada por dos líneas oscuras C. Bajos, porque el hipotálamo funciona inadecuadamente. –
adyacentes, que son las líneas Z. D. Muy altos, porque la prolactina no inhibe el centro cíclico
B. Las bandas I contienen filamentos finos de miosina. hipotalámico. –
C. El sarcomera presenta invaginaciones a lo largo de las líneas Z E. Bajos, porque la hipófisis deja de responder al hipotálamo
de las sarcómeras.
D. En el centro de la sarcómero se halla una banda oscura de 14. El tratamiento de elección en Vaginosis Bacteriana es:
anchura constante, la banda A flanqueada por dos bandas claras A. Metronidazol
de anchura variable, las bandas I. B. Amoxicilina
E. La miosina se localiza en las bandas A. C. Azitromicina
D. Ciprofloxacina
7. El volumen minuto está dado por: E. Cloranfenicol

A. Frecuencia respiratoria por la presión al final de la inspiración 15. Se produce degranulacion de los mastocitos y de los eosinófilos
B. Frecuencia respiratoria por el flujo inspiratorio cuando el antígeno especifico reacciona con los anti- cuerpos de
C. Flujo inspiratorio por el volumen tidal tipo:
D. Volumen tidal por la frecuencia respiratoria
E. Volumen tidal por presión de inspiración A. IgG.
B. IgM.
8. Contribuye el 75% de la osmolaridad C. IgA.
D. IgE.
A. Glucosa E. IgD.
B. Potasio
C. Proteínas 16. Señale verdadero o falso:
D. Cloruro de Sodio
E. Compuestos nitrogenados no proteicos ( ) Ca -125 puede encontrarse alto en procesos inflamatorios pélvicos
( ) Beta HCG se usa como marcador en el estudio de tumoraciones
ováricas de origen epitelial
9. En líneas generales, en el manejo de los síntomas leves sin ( ) Informe ecográfico: presencia de tumoraciones quísticas en ambos
daño de órganos vitales, de las enfermedades reumatológi- cas, ovarios y presencia de ascitis es sospechoso de cáncer ( )
se utiliza: Tumoración anexial mayor de 7 cm en menopáusicas
A. AINE. generalmente es benigno
B. Corticoides intra-articulares. A. VFVF
C. Inmunosupresores. B. FVFV
D. AINE +/- corticoides a dosis bajas (<15 mg/día). C. VVVF
E. Corticoides a dosis altas (1mg/kg/día) +/- Inmunosupresores. D. VVVF
E. VFVV
10. Señale el fármaco antimuscarínico broncodilatador que se
administra por vía inhalatoria: 17. Una mujer en su cuarto mes de embarazo comienza a sangrar y
el médico la detecta un útero más grande que el es- perado para
A. Metaproterenol su edad gestacional. Decide interrumpir el embarazo y la
B. Albuterol practica un legrado. El patólogo al examinar la muestra observa
C. Ipatropio numerosos quistes avasculares con proliferación significativa del
D. Ritodrina trofoblasto, no hay feto y no se detecta inva-sión del miometrio.
E. Fenoterol ¿Cuál será el diagnóstico?:

11. Paciente adolescente con NIC I más PVH cérvico vaginal ¿Cuál A. Mola invasiva.
de las siguientes seria la conducta adecuada?: B. Mola hidatiforme parcial.
C. Mola hidatiforme completa.
A. Cono frio D. Tumor trofoblástico del lecho placentario.
B. Vacunación inmediata contra cepas 16, 18, 6 y 11 E. Coriocarcinoma.
C. Histerectomía ampliada a vagina
D. Aplicación de podofilina o ácido tricloro acético más interferon 18. Mujer de 30 años con antecedentes de aborto recurrentes,
E. Observación y control periódico consulta por fatiga, disnea, fiebre, examen: Petequias en brazos
y piernas, miembros inferiores aumento de volumen hasta rodilla
12. La hemorragia uterina anormal en la mujer perimenopáusica, que son dolorosa a la presión. Abdomen no visceromegalia, Hb:
nos obliga a descartar en primer lugar: 7 gr/dl, ANA (-) reticulocitos: 7 % Coombs (+). Se pide una
prueba de análisis. ¿Cuál sería el diagnos-tico según la
A. Cáncer de endometrio positividad de análisis?:
A. LES 25. ¿Cuál de las siguientes vacunas puede administrarse a la madre
B. Síndrome Evans durante el embarazo?:
C. Síndrome sobreposición LES + Crioglobulinemias
D. Síndrome antifosfolipidico A. Sarampión.
E. Artritis reumatoidea B. Poliomielitis.
C. Tétanos.
19. ¿Cuál es la localización más frecuente del embarazo ectópico D. Rubéola.
tubárico?: E. Tos ferina.

A. Cornual 26. La mastitis aguda ocurre frecuentemente en:


B. Fímbrica
C. Ístmica A. El nacimiento.
D. Ampular B. La pubertad.
E. Intersticial C. El embarazo.
D. La lactancia.
20. Gestante con 37 semanas de embarazo por amenorrea E. La menopausia.
confiable y ecografía de primer trimestre, hospitalizada ac-
tualmen-te con diagnóstico de RCIU, las pruebas de bienestar 27. La infección vertical por VIH es más frecuente por transmisión:
fetal están dentro de lo normal. ¿Cuál de los siguientes cuidados
NO es pertinente en su caso? A. Intraparto
B. Intrauterina
A. Programar cesárea electiva para concluir la gestación de C. Posparto
inmediato. D. Carioamniótica
B. Durante el trabajo de parto indicar monitoreo electrónico fetal E. Extrauterina
continuo.
C. Aspiración de secreciones nasofaringeas a la salida de la 28. Diarrea de 4 semanas de evolución, después de varias ingestas
cabeza del RN. alimenticias en zona rural, no fiebre, leucocitos 6,000, Hb: 12 gr.
D. Presencia física del neonatólogo durante el periodo expulsivo. En heces: No leucocitos, ni hematíes. Subraye el Diagnostico
E. Valorar la madurez cervical para evaluar la posibilidad de inducir probable:
el parto, sino existe sufrimiento fetal agudo (SFA) y las
condiciones obstétricas son favorables. A. Shigella
B. Estafilococo
21. Una mujer de 26 años presenta en el tercer trimestre del C. Giardia – Lamblia
embarazo ictericia y prurito. Analíticamente la bilirrubina es de 5 D. Clostridium Dificile
mg%, fosfatasa alcalina de 800 UI/l, colesterol de 460 mg%, E. E.Coli 0 157: H7.
GOT de 80 y GPT de 120 UI/l. En un embarazo previo había
presentado prurito. Tras el parto la paciente se recuperó. ¿Cuál 29. ¿Cuál de los siguientes agentes no causa úlcera genital?
es el diagnóstico de sospecha?:
A. Neisseria gonorrheae
A. Hígado graso del embarazo. B. Herpes simple
B. Hepatitis aguda vírica. C. Treponema pallidum
C. Colestasis intrahepática del embarazo. D. Haemophilus ducreyi
D. Hepatitis tóxica. E. Chalmydia trachomatis
E. Coledocolitiasis
30. Un agricultor de 25 años de edad ingresa a Emergencia con
22. La lesión hepática característica de la eclampsia es: diaforesis profusa, dolor abdominal intenso, edema fácil
periorbitario severo y mialgias severas. Informa que fue mordido
A. Necrosis hemorrágica del área periportal. por un “animal ponzoñoso” mientras trabajaba en el campo ¿A
B. Necrosis central de los lobulillos hepáticos. qué cuadro clínico corresponde?
C. Edema de las células hepáticas.
D. Necrosis focal de todas las áreas. A. Flevotomus verrucarum
E. Ninguna de las anteriores. B. Loxocelismo
C. Leptospirosis
23. La aparición de un patrón de desaceleración variable o DIP III D. Latrodectismo
en la monitorización tocográfica de una embarazada es E. Lutzomia.
indicativo de:
A. Abruptio placentae. 31. Un varón de 65 años acude con una disnea progresiva. Aparte
B. Compresión del cordón umbilical. de los antecedentes de tabaquismo intenso, no refiere ningún
C. Enfermedad hemolítica perinatal. otro dato digno de mención. El murmullo vesicular se encuentra
D. Rotura uterina. abolido en los dos tercios superiores del cam- po pulmonar
E. Sufrimiento fetal grave. izquierdo. En la percusión de la pared torácica del mismo lado
se aprecia una resonancia inferior a lo normal. Al colocar usted
la mano en el hemitórax izquierdo y pedir al enfermo que diga
24. ¿Cuál de las siguientes es indicación de parto vaginal? “treinta y tres”, no aprecia la transmisión del sonido a la mano.
La tráquea parece desviada a la izquierda. ¿Cuál de los
A. Parto con dos cesáreas anteriores. siguientes es el diagnóstico más probable?:
B. Presentación de cara, variedad mentoanterior.
C. Gemelar, biamniótica, primero en podálica. A. Neumonía bacteriana
D. Gemelar, monoamniótica, primero en cefálica. B. Neumonía vírica
E. Presentación de frente. C. Obstrucción bronquial
D. Derrame pleural Actualmente acude al servicio de urgencias por dolor epigástrico
E. Neumotórax continúo irradiado a ambos hipocondrios y espalda que empeora
con la ingesta, náuseas y vómitos. En la radiografía simple de
32. Un enfermo ha sufrido un ataque severo de asma, motivo por el abdomen se observan microcalcificaciones en el área
cual ha recibido dosis elevadas de esteroides y bron- pancreática. ¿Qué prueba de las siguientes seria de indicación
codilatadores. ¿Cuál es el mejor criterio, de los que a para confirmar el diagnóstico de pancreatitis cróni-ca?
continuación se mencionan, para poder reducir la dosis del
esteroi-de utilizado sin riesgo de recaída?: A. TAC de abdomen.
A. Normalización de la gasometría. B. Determinación de grasas en heces.
B. La desaparición del pulso paradójico. C. Prueba de la secretina-colecistoquinina
C. La disminución de la taquipnea. D. Colangiopancreatografía retrógrada endoscópica.
D. La desaparición de las sibilancias. E. Ecografía
E. La normalización del flujo pico.
39. En la prueba del aliento para diagnóstico de Helicobacter pylori,
33. Varón de 50 años que en el décimo día tras cirugía abdominal, consideramos:
desarrolla súbitamente dolor torácico de tipo pleurí- tico, disnea
y taquipnea. PaO2 con aire ambiente <50 mmHg ¿Qué A. Se recomienda el test de Urea marcado con C14
exploración será más sensible y específica para el proceso que B. Se recomienda el test de Urea marcado con C13.
debemos sospechar?: C. Debe consumir antiácidos los días previos al test del aliento
A. ECG. D. El C13 es altamente radioactivo
B. Radiografía de tórax. E. No es una prueba útil para el seguimiento post tratamiento en
C. Nueva gasometría con FiO2del 40%. ulceras gastricas
D. Gammagrafía pulmonar de perfusión.
E. Ecografía doppler de extremidades inferiores. 40. De los siguientes enunciados, ¿Cuáles son correctos?:

34. Un ingeniero de 40 años de edad presenta agrandamiento de 1. La hemorragia por úlcera duodenal es cuatro veces más
ganglios hiliares, hepatoesplenomegalia, hipercalcemia y frecuente que la debida a úlcera gástrica.
enfermedad pulmonar intersticial. El diagnóstico más probable 2. El síndrome de Mallory Weiss es la laceración de la mucosa a la
es: altura de la unión gastroduodenal.
A. Hiperparatiroidismo. 3. Hematemesis es la pérdida de sangre originada desde la
B. Leucemia. orofaringe hasta el ángulo de Treitz.
C. Histoplasmosis. 4. Puede presentarse hemorragia por la administración de AINEs,
D. Sarcoidosis. alcohol y corticoides.
E. Tuberculosis. 5. El signo endoscópico de Forrest III indica sangrado activo.
6. En una hemorragia digestiva alta el primer examen auxiliar
solicitado debe ser una radiografía de esófago, estómago y
35. En la angina de Prinzmetal, ¿cuál de las siguientes alternativas duodeno a doble contraste.
es cierta? A. 1, 5, 6
B. 2, 3, 4
A. Presencia de onda Q en el electrocardiograma. C. 1, 2, 3
B. Es precipitada por el esfuerzo. D. 4, 5, 6
C. Usualmente resulta en infarto de miocardio. E. 1, 3, 4
D. El espasmo coronario produce esta variante.
E. Requiere uso de trombolíticos. 41. Paciente con hipoestesia hemifacial izquierda y hipoestesia
hemicorporal derecha. Además presenta síndrome verti- ginoso,
36. Cuál sería el mejor ejemplo de shock neurogénico: disartria y disfagia. En la etiología se demuestra oclusión de la
arteria cerebelosa posteroinferior. Señale la locali-zación más
A. Traumatismo raquimedular con flacidez y ausencia de los probable de su lesión:
reflejos osteotendinosos
B. Traumatismo raquimedular con pérdida del tono vasomotor y A. Corteza parietal izquierda.
supresión de la inervación simpática B. Tálamo izquierdo.
C. Traumatismo raquimedular con hipotensión, atonía gástrica y C. Mesencéfalo izquierdo.
anestesia intralesional D. Bulbo izquierdo.
D. Traumatismo raquimedular con pérdida de los reflejos E. Ganglio de Gasser izquierdo.
osteotendinosos y cutáneos intralesionales
E. Ninguna de las anteriores. 42. Un hombre de 25 años es conducido a urgencias porque “ lo
persiguen unos asesinos que van a matarlo”. En la explo- ración
37. Es FALSO en Fiebre Reumática, lo siguiente: física se observan pupilas dilatadas, T° 37.8°C, FC: 110x’ y PA:
160/95x’, sin otros hallazgos. La familia afirma que tiene historia
A. En su patogenia interviene la triada: Huésped susceptible, medio de abuso de drogas. ¿Qué droga es probablemente responsable
ambiente y agente etiológico. del cuadro actual?
B. Probablemente existan genes de susceptibilidad para contraer
la enfermedad. A. Alcohol.
B. Cocaína.
C. Las cepas dérmicas siempre producen un cuadro más severo. C. Heroína.
D. La válvula más afectada es la mitral. D. Diazepam.
E. Existe una respuesta celular y humoral contra la proteína “M” de E. Fenobarbital.
la bacteria y algunas estructuras cardiacas.
43. En un paciente con un tumor cerebral infratentorial, pensaría en
38. Varón de 45 años, con antecedentes de alcoholismo desde los todos los siguientes, excepto:
20 años y dolor abdominal intermitente de hace 5 años.
A. Meduloblastoma. C. Lóbulo occipital
B. Hemangioblastoma. D. Cerebelo
C. Craneofaringioma. E. Lóbulo frontal, temporal y ganglios de la base
D. Astrocitoma.
E. Colesteatoma. 50. ¿Qué es la anhedonia?

A. Un sentimiento exaltado de la vida


44. La causa más frecuente de hipoglucemia en un paciente B. Una depresión reactiva
hospitalizado es: C. Una disminución mímica
D. Imposibilidad de vivir situaciones placenteras
A. Tumores. E. Una percepción delirante
B. Desnutrición.
C. Hepatopatía. 51. Varón de 26 años de edad, quien desde hace seis meses
D. Fármacos. presenta cefaleas, debilidad, hiporexia, náuseas, disminución de
E. Septicemia. peso. Al examen físico: PA 170/110mmHg, FC 96 por minuto,
palidez, edema en miembros inferiores, hemoglobina 8.2mg%,
45. Varón de 58 años, acude a Emergencia por vómitos, dolor creatinina 4.5mg/dl, urea 100. Ecografía renal: riñones
abdominal, diarreas (3 diarias). Historia previa de pérdida de pequeños. ¿Cuál es el diagnóstico más probable?
peso de 10 Kg en 3 meses, fatiga marcada, episodios de
lipotimia. Al examen: somnolencia, PA: 80/50, pulso: 120 por A. Síndrome nefrótico.
minuto, manchas de tipo vitíligo en manos y labios. Análisis: B. Insuficiencia renal aguda.
hemoglobina 8 g%, leucocitos: 14.000 uL, glicemia: 50 mg%, C. Insuficiencia renal crónica.
creatinina: 2,5 mg%, urea: 60 mg%, Na: 112 mEq/L, K: 6 mEq/L, D. Nefritis intersticial.
Célula: 108 mEq /L, bicarbonato: 12 mEq/L, Na uri-nario: 60 E. Síndrome nefrítico.
mEq/L. ¿Cuál de las siguientes es su posibilidad diagnóstica?:
52. La manifestación clínica inicial del síndrome nefrótico es:
A. Insuficiencia suprarrenal
B. Hiponatremia con hipovolemia por pérdidas digestivas A. Proteinuria
C. Hiponatremia por secreción inapropiada de hormona B. Hipoproteinemia
antidiurética C. Edema
D. Hiponatremia dilucional por insuficiencia renal D. Hiperlipidemia
E. Hiperpotasemia por necrosis tubular aguda E. Hipertensión arterial

46. Varón de 38 años de edad, alcohólico, en coma, con herida 53. ¿Qué tratamiento instauraría en una micosis fungoide con
craneal. Radiografía de cráneo: trazo de fractura. Diuresis: 170 lesiones eccematosas limitadas al tronco?
mL/h; Na: 170 mEq/L, K: 4 mEq/L; Bicarbonato: 18 mEq/L;
Osmolalidad en plasma: 360 mOsm/Kg. H2O en orina: 70 A. Corticoides tópicos o bexaroteno
mOsm/Kg; Urea: 45 mg%; Creatinina: 1,8 mg%. Diagnóstico B. UVB.
probable: C. Radioterapia.
D. Acitretino.
A. Polidipsia compulsiva. E. Interferón alfa.
B. Secreción inapropiada de hormona antidiurética.
C. Panhipopituarismo con insuficiencia suprarrenal. 54. Paciente varón de 31 años de edad, con un tiempo de
D. Diabetes insípida nefrogénica. enfermedad de hace 3 años, de inicio insidioso y curso progresi-
E. Diabetes insípida hipotalámica. vo; caracterizado por la aparición de lesiones dérmicas
eritematosas y descamativas en el cuero cabelludo, en los codos
47. ¿Cuál es el microorganismo más frecuentemente aislado en la y rodillas. Posteriormente se generaliza al dorso y región sacra.
artritis no gonocócica? Al examen clínico preferencial de la piel se evidencia lesio-nes
tipo placa eritematosas y descamativas en cuero cabelludo,
A. Estafilococo aureus tronco y extremidades, de diferentes tamaños y cuando se
B. Salmonella sp remueven las escamas por rascado suave, se ven puntos
C. Hemophilus influenzae sangrantes pequeños. ¿Cuál es el diagnóstico clínico y cuál es
D. Neumococo la alteración histológica?:
E. Candida sp
A. Pénfigo – Enfermedad autoinmune.
48. Si al practicar una punción articular en una rodilla, se extrae un B. Dermatitis crónica – Reacción inflamatoria.
líquido viscoso, transparente y que, en el recuento, tiene menos C. Dermatitis alérgica – Hipersensibilidad.
de 2.000 leucocitos/mm3 cabrá pensar que el enfermo tiene con D. Dermatitis espongiotica – infección.
toda probabilidad una: E. Psoriasis – proliferación incrementada de queratinocitos.

A. 55. Varón de 66 años, con dolor lumbar persistente, anemia de 8 gr.


B. Artrosis. De Hb, creatinina de 4 mg%, Ca++ P: 14 mg%. Pro- teínas
C. Artritis reactiva. totales: 11 gr %. Diagnostico probable:
D. Artritis reumatoide. A. Metástasis ósea de Ca. de Próstata
E. Artritis séptica. B. Hiperparatiroidismo primario
F. Espondiloartropatía. C. Linfoma óseo
D. Mieloma múltiple
49. La esquizofrenia implica cambios patológicos a nivel del: E. Insuficiencia renal crónica

A. Hipotálamo 56. La asfixia traumática, es un tipo de asfixia mecánica que en la


B. Sistema reticular gran mayoría de casos ocurre por muerte:
Grupo RN O+, Coombs indirecto negativo. La causa probable de
A. Homicida la ictericia es:
B. Suicida
C. Repentina A. Incompatibilidad de grupo sanguíneo OA
D. Accidental o laboral B. Por el Cefalohematoma
E. Natural C. Fisiológica
D. Por la leche materna
57. Con respecto a la investigación en seres humanos, marque lo E. Por una Sepsis neonatal
incorrecto:
63. Recién nacido pretérmino de 30 sem. de edad gestacional, 1200
A. El informe Belmont cuestiona la autonomía para el gr. de peso, madre con corioamnionitis, Apgar 7 al 1’, 8 a los 5’,
consentimiento informado no requirió reanimación neonatal, pero inicia Distress
B. Debe guiarse por los principios de Beneficencia, No respiratorio con Test de Silverman de 5 de puntaje.
maleficencia, autonomía y Justicia ¿Cuál es el manejo del recién nacido en relación a la probable
C. La vulnerabilidad del participante requiere mayor protección de infección?
sus derechos
D. Debe valorarse el riesgo beneficio de la intervención antes de su A. Manejo expectante sin antibióticos (ATB) hasta que el recién
aplicación nacido manifieste signos de infección.
E. El seguimiento e información al participante es constante B. Uso de ATB de inicio con Ampicilina y Aminoglucósido
durante todas las fases de la investigación C. Uso de ATB siempre y cuando el hemograma y el PCR sean
anormales
58. Referente a los defectos congénitos de pared, marque lo D. Uso de ATB de inicio con Vancomicina y Cefalosporina de 3a.
incorrecto: generación.
E. Uso de ATB si la Rx de tórax es compatible con Neumonía.
A. El defecto en las gastrosquisis está a la izquierda del cordón
umbilical 64. Un recién nacido deprimido al nacer, no respira después de las
B. El onfalocele está cubierto por un saco de peritoneo medidas iniciales de reanimación, por lo que recibe ventilación a
C. La gastrosquisis presenta saco de peritoneo presión positiva por 30 segundos, al cabo de los cuales tiene una
D. EL hígado puede herniarse en el onfalocele frecuencia cardiaca de 4 latidos en 6 se- gundos y aún no
E. Las asas intestinales suelen estar edematizadas en la respira. La siguiente acción a realizar sería:
gastrosquisis
A. Continuar la ventilación a presión positiva sola, durante 30
segundos más
59. Niño de 12 meses de edad que acude al establecimiento de B. Intubar la tráquea y continuar con la ventilación bolsa-tubo
salud y al que le han colocado regularmente sus vacunas endotraqueal
¿Cuáles son las vacunas que debería recibir a esta edad?: C. Suspender la ventilación a presión positiva e iniciar el masaje
cardiaco
A. AMA + BCG D. Administrar adrenalina al 1:10 000 por la vena umbilical o
B. 2da Neumococo + SRP intratraqueal
C. 1º Penta + 1º APO E. Continuar la ventilación a presión positiva e iniciar el masaje
D. 3era Neumococo + SRP cardiaco
E. BCG + APO
65. Un lactante de 2 meses presenta desde hace 1 semana vómitos
60. La leche materna es muy importante en la nutrición del lactante, mucosos y crisis de tos con cianosis facial 5-6 veces al día y
señale Ud., su contenido proteico: auscultación cardiopulmonar normal. Señale el diagnóstico más
probable entre los propuestos:
A. 1-1.5 gr %
B. 3- 4.5 gr.% A. Rinofaringitis.
C. 2- 3.5 gr.% B. Bronquiolitis.
D. 4-5 gr.% C. Epiglotitis.
E. 2-2.5 gr.% D. Tos ferina.
E. Bronconeumonía.
61. Un prematuro de 900gr de peso y 27 semanas de gestación
sufre Distress respiratorio desde el nacimiento. A las 36 horas 66. Paciente de 2 años y 6 meses, es llevada al centro de salud por
de vida, presenta hipotensión, bradicardia, cianosis y tensión presentar dolor abdominal, deposiciones frecuentes de poco
aumentada de la fontanela. ¿Qué exploración diagnóstica le volumen, con moco y sangre; además fiebre de 7 días de
parece más oportuna, entre las siguientes? duración. El cuadro clínico corresponde a una diarrea:

A. Análisis de sangre con serie roja y plaquetas A. Causada por parásitos


B. Exploración cerebral con ultrasonidos B. Aguda acuosa
C. Estudio del tiempo de protombina y del tiempo parcial de C. Aguda disentérica
tromboplastina
D. Punción lumbar y cultivo de LCR D. Causada por virus
E. Ecocardiograma E. Persistente

62. Neonato de 72 hr de edad nacido de parto eutócico, a término, 67. Lactante de 5 meses que presenta desde 3 días antes, fiebre,
con peso de 3500 gr. Recibe lactancia materna exclusi- va. Es rinorrea acuosa y estornudos, comenzando el día de la consulta
traído a Emergencia por ictericia. Al examen se le encuentra: con tos y dificultad respiratoria. En la exploración presenta
cefalohematoma leve e ictericia hasta abdomen. Peso actual: taquipnea, tiraje subcostal, alargamiento de la espi-ración y
3300 gr. Exámenes auxiliares: BT: 15 mgr, BD: 0.7 mgr, crepitantes y sibilantes diseminados. ¿Cuál es el diagnóstico
Hematocrito 52%, Reticulocitos 5%, Grupo de la madre O+ , más probable de entre los siguientes?:
74. Niño de 6 años de edad, acude por náuseas y vómitos de dos
A. Neumonía días de evolución. Presenta fiebre y rinorrea por lo cual recibió
B. Asma antipiréticos, descongestionante nasal y vitamina C.
C. Fibrosis Quística Posteriormente cursa con ictericia, bilirrubinas y transa- minasas
D. Bronquiolitis muy elevadas. Usted podría sospechar intoxicación por:
E. Cuerpo extraño bronquial
A. Vitamina C
68. Con respecto al tumor de Wilms, es FALSO que: B. Salicilatos
C. Loratadina
A. Puede cursar con poliglobulia. D. Clorfenamina
B. En la UIV se observa una masa dependiente del tejido renal. E. Acetaminofeno
C. La quimioterapia y la cirugía se indica, aun en presencia de
metástasis pulmonares. 75. Respecto a la mononucleosis infecciosa, es FALSO que:
D. Posee gran tendencia a presentar síndromes paraneoplasicos
como el opsoclono-mioclono A. Hasta en el 80% hay aumento de transaminasas.
E. Usualmente son unilaterales. B. En la analítica aparece linfopenia
C. Títulos elevados de anticuerpos frente al virus de Epstein- Barr.
69. Niño de cuatro años de edad, cuya madre BK positiva recibe D. Existe un síndrome linfoproliferativo en varones que han sufrido
tratamiento para TBC sensible. Tiene PPD 12 mm, BK negativo la infección por este virus.
y la radiografía muestra infiltrado parenquimal en base derecha. E. La complicación más temida es la rotura esplénica.
¿Cuál es el diagnóstico y el tratamiento más indicado?
76. Una limitación importante de la ultrasonografía en la valoración
A. TBC pulmonar y esquema para TBC sensible inicial del paciente con traumatismo abdominal es:
B. Contacto TBC y quimioprofilaxis
C. Primo infección y quimioprofilaxis A. Inestabilidad hemodinámica
D. TBC pulmonar y esquema para TBC resistente B. Obesidad
E. Contacto TBC y prueba terapéutica C. Lesiones extra abdominales asociadas
D. TEC con Glasgow bajo
70. Un recién nacido presenta una deformidad rígida de ambos pies E. Desorientado
con sus plantas enfrentadas, los bordes laterales orientados
hacia distal y los bordes mediales hacia craneal, y ambos 77. En relación a un paciente masculino de 35 años, con colelitiasis
antepiés más próximos a la línea media y más dista-les que los asintomático, marque lo VERDADERO:
retropiés. ¿Cuál es la denominación más adecuada?:
A. Debe tratarse con medicación que disuelva los cálculos.
A. Pies planos. B. Realizar la colecistectomía solo si la técnica laparoscópica es
B. Pies cavos. posible.
C. Pies valgos. C. El paciente tiene probabilidad menor del 10% de ser sintomático
D. Astrágalo vertical congénito. en los siguientes 5 años.
E. Pies zambos. D. El riesgo de cáncer de vesícula por detección de litiasis a tan
temprana edad justifica la colecistectomía electiva.
71. En una persona que ingiere dieta con alto contenido de glúcidos E. Debe realizarse colecistectomía solo si tiene comorbilidad como
se observará: Diabetes Mellitus.

A. Mayor síntesis de ácido linoleico. 78. ¿Cuál de los siguientes analgésicos que se mencionan a
B. Disminución de la síntesis de ácidos grasos. continuación es el más adecuado para calmar el dolor en Pan-
C. Mayor síntesis de triglicéridos. creatitis Aguda?
D. Hiperquilomicronemia
E. Mayor síntesis de colesterol y cetosis. A. Meperidina
B. Tramadol
72. Paciente de 2 meses de edad acude por presentar convulsiones C. Metamizol
tónico clónicas generalizadas de 2 minutos de dura- ción, D. Ketorolaco
durante la evaluación no se determina fiebre. Los exámenes E. Morfina
bioquímicos y la punción lumbar son normales, los estudios de
imágenes no muestran mayor alteración. Durante la 79. Abierto al trayecto inguinal y al levantarse el cordón espermático
hospitalización en los días siguientes se presentaron convulsio- /ligamento redondo, se encuentran las siguientes estructuras:
nes de corta duración. El diagnóstico probable será:
A. Tendón en conjunto.
A. Convulsión neonatal B. Vasos epigástricos profundos.
B Convulsión febril C. Ligamento de Hesselbach.
C. Epilepsia D. Fascia transversalis.
D. Convulsión asociada a fiebre E. Todos ellos.
E. Convulsión febril compleja
80. La causa más frecuente de abdomen agudo por oclusión
73. ¿Cuál es la causa más conocida de pancreatitis aguda en niños? mecánica de intestino delgado es una de las siguientes:

A. Infección por Mycoplasma A. Plastrón apendicular.


B. Administración de sulfamidas B. Hernia inguinal incarcerada.
C. Administración de tiazidas C. Hernia crural incarcerada.
D. Parotiditis D. Adherencia o bridas de intestino delgado consecuencia de
E. Fibrosis quística cirugías previas.
E. Neoplasia de colon derecho
D. Rotura dural.
81. Un paciente que presenta ictericia progresiva con estudio de E. Fístula laberíntica.
laboratorio de tipo obstructivo, y que la colangioresonan- cia
muestra dilatación de las vías biliares intrahepáticas pero no de 88. Acude a Urgencias un paciente con intenso picor y lagrimeo en
la vía biliar principal, el diagnóstico más cercano es: ambos ojos, hiperemia conjuntival, fotofobia, exudado más o
menos viscoso y formación papilar en la conjuntiva tarsal. ¿Cuál
A. Tumor de Klastkin. es el diagnóstico probable?:
B. Estado inicial de cáncer de cabeza de páncreas.
C. Tumor de los islotes gastrónoma. A. Chalazión.
D. Tumor periampular. B. Conjuntivitis bacteriana.
E. Cálculo enclavado en la ampolla de Vater. C. Conjuntivitis vírica.
D. Queratoconjuntivitis seca.
82. ¿Cuál de las siguientes características NO es propia de las E. Conjuntivitis alérgica.
quemaduras de tercer grado?
89. La causa más común de hemorragia subaracnoidea espontánea
A. La profundidad de la quemadura llega a la subdermis. es:
B. Su color suele ser blanco nacarado.
C. Tienen un gran número de flictenas. A. A.Discrasia sanguínea
D. Son propias las escaras. B. Malformación arterio-venosa
E. No tienen dolor ni sensibilidad táctil. C. Aneurisma cerebral
D. Vasculitis intracraneal
E. Traumatismo cráneo-encefálico
83. Varón de 45 años que acude a emergencia con diagnóstico de
fractura de cuerpo de la escapula, sin desplazamiento, el 90. El hallazgo clínico de pulso paradójico, ingurgitación yugular e
tratamiento indicado es: hipotensión arterial es diagnóstico de:

A. Reducción cruenta + osteosíntesis. A. Embolia pulmonar aguda


B. Reducción Incruenta + tracción ósea. B. Fibrilación auricular
C. Inmovilización y a los 7 días, realización de ejercicios C. Crisis hipertensiva
pendulares. D. Taquicardia paroxística
D. Tracción al cenit de miembro afectado. E. Taponamiento cardíaco
E. Reposo absoluto por 4 semanas.

84. Niño de 9 años traído a la Urgencia tras caída de un columpio. 91. Después de la explosión de las bombas atómicas en Hiroshima
Presenta su antebrazo con intenso dolor espontáneo, y Nagasaki en 1946 se empezaron a reportar más fre-
impotencia funcional absoluta y gran deformidad de vértice volar cuentemente casos de leucemia en personas que estuvieron
en tercio medio. ¿Cuál será el diagnóstico más proba-ble?: expuestas a la radiación. Por este motivo se diseñó un es- tudio
a fin de determinar la incidencia de esta enfermedad en ambas
A. Fractura diafisaria de cúbito y radio. poblaciones. Para ello entre los años 1946 y 1964 se registraron
B. Epifisiolisis de muñeca. todos los casos de leucemia que presentaron los sobrevivientes
C. Fractura de Smith. expuestos a la explosión de la bomba. Al final del estudio se
D. Fractura-luxación de Monteggia. determinó que las personas que estuvieron entre 0 a 1500
E. Fractura de Colles. metros del epicentro de la explosión, tuvie-ron mayor riesgo de
padecer leucemia que aquellas que estuvieron entre 1501 a
85. Paciente de 12 años de edad, llega a Emergencia con historia 10000 metros. ¿Qué tipo de muestreo se ha realizado?:
de fractura de antebrazo izquierdo. Atendido hace 2 días en
traumatología, en donde le colocaron yeso braquiopalmar. A. Muestreo aleatorio simple.
Actualmente se queja de dolor y al examen se encuentra la B. Muestreo aleatorio sistemático.
mano izquierda edematizada. ¿Cuál es el diagnóstico?: C. Muestreo por conveniencia.
D. Muestreo por conglomerados.
A. Desplazamiento de la fractura. E. Se estudió a toda la población objetivo.
B. Síndrome compartimental.
C. Desgarro de la vena cefálica. 92. El cadmio es un metal pesado que puede causar daño renal. El
D. Compresión del nervio mediano. signo inicial es la proteinuria tubular. En un estudio a 944
E. Desgarro de la arteria interósea. personas se determinó cadmio en orina y simultáneamente
proteinuria tubular. En los resultados se mostró que la mayo-ría
86. Paciente sometido a resección extensa del Íleon terminal, puede de los casos de daño tubular renal temprano presentó niveles
presentar con más frecuencia un tipo de litiasis uri- naria: elevados de cadmio. ¿A qué diseño corresponde el estudio?:

A. Úrica. A. Transversal.
B. Oxálica. B. Cuasi-experimento.
C. Cistínica. C. Ecológico.
D. Fosfato-amónico-magnésico. D. Cohortes.
E. La única litiasis con esa etiología es la vesical. E. Casos y controles

87. Después de un traumatismo craneoencefálico, la otorragia 93. ¿Cuál de los siguientes criterios NO se aplica para establecer
sugiere:: una posible relación causa-efecto entre un potencial factor de
riesgo y una enfermedad?:
A. Fractura temporal longitudinal.
B. Fractura temporal transversa. A. Existencia de una asociación estadísticamente significativa.
C. Hemotímpano. B. Que la presunta causa preceda al efecto.
C. Que el lapso exposición – comienzo de la enfermedad sea
compatible con su historia natural.
D. Plausibilidad biológica.
E. Disminución del riesgo después de la reducción de la exposición.
94. De las siguientes afirmaciones sobre el tabaco, sus riesgos y
consumo, encontrar la FALSA:

A. Los cánceres de esófago son más frecuentes en alcohólicos que


en consumidores de tabaco.
B. Es un factor de riesgo para Cáncer de cérvix.
C. A los 15 años de abandonar el hábito, las razones de muerte de
los ex fumadores son muy próximas a las de los no fumadores.
D. El más importante factor de riesgo de la enfermedad arterial
periférica es el tabaco.
E. Hasta el 90% de los casos de EPOC son atribuibles al tabaco.
95. En relación al paquete de Atención Integral de Salud del Adulto
basado en familia y comunidad; la evaluación integral para la
identificación de factores de riesgo del adulto mujer y del adulto
varón en general, se diferencian en:

A. Riesgo de enfermedades no transmisibles.


B. Riesgo de enfermedades transmisibles.
C. Factores de riesgo en salud mental.
D. Problemas psicosociales y estilos de vida saludables.
E. Identificación de síndrome de maltrato y violencia.

96. Las funciones del hospital son:

A. Promoción.
B. Protección.
C. Recuperación.
D. Rehabilitacion.
E. Todas las anteriores.

97. Uno de los siguientes trastornos no es un síndrome


paraneoplásico: –

A. Dermatomiositis.
B. Síndrome de Eaton Lambert.
C. Encefalitis límbica.
D. Enfermedad del central core.
E. Polineuropatía sensitiva pura.

98. El CENTRO DE SALUD está mejor representado cuando se le


define como:

A. Un sistema sociotécnico.
B. Un sistema tecnológico.
C. Una red de servicios complementarios.
D. Un modelo de atención ambulatoria de la salud.
E. Un sistema social.

99. En qué documento de una institución de salud, se especifican


las funciones de un puesto de trabajo:

A. Norma de las características del puesto.


B. Reglamento de organización y funciones de trabajo.
C. Manual de procedimientos administrativos.
D. Manual de organización y funciones.
E. Guías de practica clínica.

100. Señale con una la respuesta que expresa los rasgos


característicos de la Cultura Organizacional:

A. La formulación de la Visión y Misión.


B. Los valores y costumbres individuales de los trabajadores.
C. Los valores, creencias y principios básicos compartidos por lo
miembros de una organización.
D. Las formas como se hacen las cosas dentro de la institución.
E. Trata de solucionar todos los problemas del establecimiento

También podría gustarte